You are on page 1of 261

ក គណិត វទ

កទ &

ក ប ង
ៃហ ហុន
ិ ៖ វទ ល័យប៊ន
ុ នី ហ៊ន
ុ ែសន រម
ៃហ ចរ ៖ វទ ល័យសុខ នសំេ ង
យ៉តពន'ក ៖ និស)ិត*+ទ
ំ ៤
ី បរ-.ប/តគណិតវទ
កព ទ យ ៃហ ហុន

នលក់ដុំ និង យេ តូបេលខ ៥០ ១៥ ៤៣


ផ រអូរឬស ី (#ង $សី$សស់)
ទំ'ក់ទន
ំ ងទូរសព)
សព)េលខ ៖ ០១៧ ៥២៧ ៤៩៣

ប មកម ធ
! ! ក#ថ ប ក %ង

ក ទ
ចក ថង គ
'ម%យក( ន* មច,ន យ- .ន/ ជ%យ
1ម ជង ព 2ក 3
4ក កឹម ពន'ក (6យក ទ7
! យប8ន9: ន ;8ន ន-< ម)
4ក ងួន វ4ន (6យក ង ទ7
! យប8ន9: ន ;8ន ន-< ម)
4ក ស៊ូ ែសន ( ប>ន ក?មប ច@ក ទ .ប8
! ន9: ន ;8ន ន-< ម)
4ក ែប៉ន សំអល
ុ (-B-CD ទ7
! យប8
យប8ន9: ន ;8ន ន-< ម)
4ក អ៊ូ 7ត (-B-CD ទ7
! យប8ន9: ន ;8ន ន-< ម)
4ក ែប៉ន 8ន់បូ (-B-CD ទ7
! យប8ន9: ន ;8ន ន,ទ
ន,ទ)
4ក ឃិន េឡវ (-B-CD ទ7
! យប8ន9: ន ;8ន ន-< ម)
4ក ហួរៃហ ភក=ី (-B-CD ទ7
! យប8ន9: ន ;8ន ន-< ម)
4ក ហុម
ឹ េជៀន (-B-CD ទ7
! យប8ន9: ន ;8ន ន-< ម)
4ក ែប៉ន រទ@ី (-B-CD ទ7
! យប8ន9: ន ;8ន ន-< ម)
4ក ុ (-B-CD .ប8
សំ សំអល ! ន9: ន ;8ន ន-< ម)
កEF /សុម
ឹ ែ/សនAរB (-B-CD .ប8
! ន9: ន ;8ន ន-
ន-< ម)
4ក ឈិត ន់ 1ភ H
! នង
! កន ប? ៃហហួ មុនស
ី ុ

យ ! I លន់ ង
ំ , ផុន សុកល , ែកម ច័នរ# ស% &ី

សូមេ រពជូនពរឲ េ ក ង
ំ អស់ ងេលើជួ
ជប ួ ែតេសចកី

សុខ"គប់េពលេវ េហើយសូមទទួលនូវ(រេ រពដឹងគុណអំពេ


ី យើង

ខ,-ំ ។ /ល់កហ
ំ ស
ុ ែដលេកើត0នេ1ក2ង
- េសៀវេ4េនះ ង
ំ អក6/វ7រទ
ុ 8


ំ បេច9កេទស គឺមន
ិ ែមន<កំហស
ុ របស់េ ក ង
ំ អស់ ងេលើ
ងេលើ

េនះេទ , គឺ<កំហស
ុ >?ល់របស់េយើងខ,-ំ អ2កេរៀបេរៀង និងអ2ក@យ

អតAបទ ។ េហើយក៏សម
ូ អភ័យេ សចំ
សចំេEះកំហស
ុ ឆGង ង
ំ អស់េHះ ។
េ(កឪពុក ៃហ ហួ +យុ ៦២ /0ំ

អ0ក23យ ហួត េទ4ង +យុ ៦២ /0ំ

សូមេ6រពជូនពរេ(កអ0ក2នគុណដ៏មហិ2

:ំងពីរ ឲ<េ(កជួប>បទះែតនឹងេសចក3 ស
ី ខ

>គប់េពលេវ( ។ ឲ<េ(ក 2ន+យុយឺនយូរ

េដDមEីFមGបដ
់ >៏ តFក់ Fទីេ6រព របស់កូនេH

េHទូត តេរ4ងេI ។

េសើឲ រក ននូវេសៀវេ
ទស
“ទស វដីគណិតវទ ៃនកម និង រអប់រ”
េចញ នដល់េលខ ៥ េហើយ
េសៀវេGHងេលើេនះនឹងជួយJ/បេ ជន៍ដល់
អLកសិកMគណិតវទ ពុH
ំ ន។
មក
េសៀវេ “កំែណលំJត់គណិត វទ K0ក់ទ៩
ី ”
ែដលបនៗកំ
ែដលបនៗកំពង
ុ ន់េ ៃដេនះ តវ នចង កងេឡើងកង
! េ"ល

បំណង$ជំ
ង$ជំនយ
ួ ()រ* តីស -ប់បេង.ន
ើ ចំេណះដឹងដល់បនៗ េ0យ-ន

ប12ញ
* ជូនអំពី ៖ លំ7ត់ និងដំេ9ះ (យ7ត់ គប់េមេរៀន ។

បនៗ$ទី>ប់?ន េសៀវេ េនះមិនែមនស -ប់េបើកចម@ង

េAះេទ គឺ "ន់ែត$ឯក(រេDងមួយកង
! ចំេ9មឯក(រ$េ ចើន

ស -ប់េផFGងHIត
* ់ និង េ បៀបេធៀបែតប៉េ
ុ 9Lះ* ។ ដំេ9ះ (យ

លំ7ត់មយ
ួ ចំនន
ួ បនៗ?ចេឆ@
បនៗ?ចេឆ@យ
ើ េផ*Nងពីេនះក៏ ន ឬ ក៏?ចបែនQម

ចំណច
R មួយចំនន
ួ ែដលបនយល់
ែដលបនយល់Sល បេសើរ ។

រែដលបនៗ នេសៀវេ កំែណគណិតវទ"#$ក់ទ៩


ី េនះេលើសពីមយ
ួ ន

ន័យ# បន ចេ/បៀបេធៀបដំេ1ះ/2យដ៏សម4រ ែបបរបស់អកនិពន6េផ8ងៗ:$ េហើយ

ចឲ នកំហស
ុ តិចតួចបំផត
ុ ។
េយើងខU!ំ នខិតខំេរៀបចំVង
ំ ពីអកW>វXរទ
ុ ឱ
Y [* \េផFGងHIត
* េ
់ ]

នឹងវចAនុ កមរបស់សេម= ចសងO ជ ជួន Mត ែតកំហស


សេម=ចសងO ុ ឆ_ងច*`ស់

$េ -នេជៀសមិនរួច ។ េហតុេនះេយើ
នះេយើងសូមអភ័យេcសទុក$មុន

ចំេdះ េeក គ/អក គ នូវ>ល់កហ


ំ ស
ុ ឆ_ងcំងអកW>វXរទ
ុ Y និង

បេចgកេទស ។

ែកវ , ៃថ#ទី ៣១ ឧស' ២០១២

តំ7ងអL
ងអLកេរៀបេរៀង
ៃហ ហុន

( Q 8
R រS គណិតវទ -រូបវទ )
ទូរសពV ៖ ០១២ ៣៤៧ ៦២៤
អីែម៉ល ៖ pahen_hay@yahoo.com
ប ! "#បទ

ទព
េមេរៀនទី១ ៖ ចំនួនអសនិ0ន ១
េមេរៀនទី២ ៖ ស112ត ១៩
េមេរៀនទី
េមេរៀនទី៣ ៖ កេន4មពីជគណិត ២៤
េមេរៀនទី៤ ៖ សមី រដឺេ2កទី១1នមួយអ:;ត ៣៥
េមេរៀនទី៥ ៖ វសមី រដឺេ2កទី១1នមួយអ:;ត ៥១
េមេរៀនទី៦ ៖ បំែណងែចកេ2បកង់ ៦៥
េមេរៀនទី៧ ៖ មធ@មសAត
ិ ិ ៧៧
េមេរៀនទី៨ ៖ 2បCDប ៨៧
េមេរៀនទី៩ ៖ ច1EយរFងពីរចំណGច ៩៨
េមេរៀនទី១០
១០ ៖ សមី រៃនប ត់
H ១១៧
េមេរៀនទី១១
១១ ៖ 2បព័នស
J មី រដឺេ2កទី១ 1នពីរអ:;ត ១៣៧
េមេរៀនទី១២
១២ ៖ 2ទឹសបទពី
ីបទពី គ័រ ១៦៣
េមេរៀនទី១៣
១៣ ៖ រងLង់ និងប ត់
H ១៧១
េមេរៀនទី១៤
១៤ ៖ លកNណៈមុៃំ នរងLង់ ១៨៣
េមេរៀនទី១៥
១៥ ៖ 2ទឹសប
ី ទ ែលល ១៩៣
េមេរៀនទី១៦
១៦ ៖ 2តីេ ណដូចRS ២០៤
េមេរៀនទី១៧
១៧ ៖ ពហុេ ណ ២២៤
េមេរៀនទី១៨
១៨ ៖ សូលីត ២៣៤

$ក% &ង
េមេរ នទី១ ៖ ចំនន
ួ សនិ ន

ម នទ ចនន ន ន

1/. ច គ
9 , 16 , 36 , − 64 , − 100
121 , − 144 , 625 , 3
8 , 3
−8
− 3 27 , − 3 64 , 3
125 , 3
216 , 3
1000
2/. ច គ
9 49 81 169 196 400
, , − , , , −
16 9 4 49 25 225
1 8 64 512 216
3 , 3 − , 3 , 3 , 3 −
8 27 125 343 1000
3/. ច ម

( )
3
163 , − 3 362 , 643 , 3
−8

−273 ( −27 )
3 3 2
, 15 , 3
82 , 3
642 , 3

4/. ច ម
y2 , x4 , x2 y 4 , − y6
16 100
, , 3
8x3 , − 64m3
x2 n4
5/. ច ម កន ម
(2 x) 2 , 3
( −5 y ) 3 , (4 − a) 2 , 3
( x + 3)3 , 16b2 + 24b + 9
9 x 2 − 30 x + 25 , 4m 2 − 20mn + 25n 2 , 49 x 2 − 112 xy + 64 y 2
6/. ច ប ! ញមយចននព
30 10
18 , − 48 , 75 , , , 3
40
49 121
3m
3
54 , − 3 128 , 3
192 , 3 , 3
16a 5
8n3
7/. ច ប ! ញមយចននព
36a 2b3 , 27a 4b3 , 72 x5 y 2 , − 112a 3b 4 , 80m 4 n3
64 x 2 y 3 , 3
16m3n3 , 3
−54 x 4b3 , − 3 128a 5 y 3 , 3
24 p 3q 5
8/. ច ប ! មយចននក%& ង x , y , m , n នង p (ចនន)ជ+,ន -
3
23 3m
5 6 , 2m m , , 2 3 5 , 2x 3 4 ,
y3 2n

កំែណេ យ ៃហ ហុន
ិ , ៃហ ចរ និងយ៉ត ពនក 1
េមេរ នទី១ ៖ ចំនន
ួ សនិ ន

9/. ច គ
3 2 −4 2 +5 2 −3 2 , 5 2 −3 3 −6 2 +5 3
3 15 − 4 3 − 3 15 + 6 3 , 4 3 − 2 17 + 3 17 − 3 3 − 2 3
2 3 2 − 83 3 + 3 2 + 33 3 , 83 2 − 33 3 − 53 2 + 2 3 3
10/. ច គ
2 3 1 1 3 2
27 − 48 , 288 − 72 , 75 − 27
3 4 4 6 5 3
1
5 3 128 − 3 3 250 , 3 3 81 − 3 192 , 4 3 54 − 3 3 128
2
11/. ច គ
2 8 − 3 98 − 2 200 , − 3 50 − 32 + 5 200
3 175 − 2 28 + 3 63 − 112 , 108 − 2 27 − 40 − 5 160
1
2 3 16 + 3 3 54 − 2 3 128 , 3 3 81 + 3 128 − 3 3 192 + 4 3 54
2
4 3 54 − 6 3 81 − 4 3 16 + 3 3 24 , − 2 40 − 3 3 135 + 5 3 320 + 8 3 5
3

12/. ច គ
( ) (
−2 2 12 − 18 − 5 3 32 − 27 , 3 3 3 40 − 3 135 + 4) ( ) ( 3
320 − 3 40 )
2 27 4 50 4 18 23 1 2 3 135 3 3 40
− 3 48 + − , 81 − 3 24 + −
3 5 3 3 2 3 2
13/. ច គ ក ន ម.ង ម a , b , x , y នង z (ចនន)ជ+,ន -
A = −3 32 x + 6 8 x , B = 2 125 x 2 z + 8 x 80 z
C = 7 a b3 + b 4a 2b − 4b , D = 8b 49b − 7 9b3 + a 4a + a 3
E = 3 xy x 2 y − 2 x 4 y 3 , F = −3a a 3b5 − 2b a 5b3 + 5 a 3b3
G = 8a 3 54a + 6 3 16a 4 , H = 3 3 x3 y − 6 x 3 xy 4 + 2 3 x 4 y 4

14/. ច គ /0ម1 ខ0នក ន ម A ច 34 a = 5 , b = 3 -


A = ab − ab3 − 9a 3b3 − a 3b -

15/. ច គ /0ម1 ខ0នក ន ម


A = 4a + a a 2b + b2 a + b 9b ច 34 a = 3 , b = 2 -

16/. ច គ
( 2 3 )(3 2 ) , ( 4 6 )( −2 5 ) , (3 5 )(5 3 )
( 6 2 )( −12 3 ) , (3 8 )( −3 48 ) , ( −3 75 )( −2 48 )
( 2 3 )  − 12 2 
3 3
, (3 2 )(5 15 ) , ( 6 8 )( −3 2 )
3 3 3 3

កំែណេ យ ៃហ ហុន
ិ , ៃហ ចរ និងយ៉ត ពនក 2
េមេរ នទី១ ៖ ចំនន
ួ សនិ ន

17/. ច គ

(
3 5 2 18 − 3 48 ( ) ) , 12 3 ( 2 48 − 3 32 )
, −3 3 3 6 −3 2

2 ( 2 18 − 3 48 ) , − 4 3 ( 2 6 − 2 5 ) , 2 5 ( 3 3 − 5 2 )
3 3 3 3 3 3 3

2
3 3 ( 3 8 − 2 18 )
3 3
, − 3 5 ( 4 20 − 2 45 )
3 3 3 3

18/. ច គ
( 2 3 − 8)(9 + 2 5 ) , (3 5 − 2 10 )( 50 − 2 80 )
( 50 − 75 )( 32 − 48 ) , ( 125 − 75 )( 80 − 48 )
(3 18 + 3 27 )( 2 8 − 2 12 )
3 3 3 3
, ( 80 − 2 27 )( −3 20 − 3 12 )
3 3 3 3

19/. គ56 a = 3 5 − 2 10 , b = 5 7 + 2 10 , c = 3 18 − 3 27 នង d = 3 3 6 + 3 8 -
ចគ cd , c 2 − b 2 , c 2 + 2cd -
1
−3ab , a 2 + b 2 , a 2 − 2b 2 , b 2 − 2ab ,
2
20/. ច គ
33 − 48 2 84 5 25
, , 2 75 ÷ 15 , , ÷ 5
11 27 − 12 75
12 3 6 5 3 100  3 72  3 6 3 7 4 3 21
3
, ,  3  2 , ÷
2 25 3 5 2 8 33 6 2 3 3

21/. ច ម ក ន ម.ង ម
b2 49 x 2 − 56 x + 16
A= , B=
b 2 − 14b + 49 36 x 2
a 2 + 16ab + 64b 2 25b 2 + 10ab + a 2
C= , D=
a 2 + 10ab + 25b 2 16b 2 + 24ab + 9a 2
22/. ច ប7/ ព8គ9បង
3
3 3 3 6 8 9 4 5 5 9
, , , , , , 3
10 33 48 27 18 72 10
23 3 3
18 7m 11 p 3
, , 3 , 3 , 3
3
30 3
20 36n 49q 4 y2
23/. ច ប7/ ព8គ9បង
36 − 6 3+ 5 8 2 3
, , ,
8 3 20 2 75 − 3 50 2 80 − 45
9− 3 3 53 4 + 3 3 23 6 23 2
, , ,
2 3 32 8 3 13 2 3 27 − 3 9 3
16 − 3 12

កំែណេ យ ៃហ ហុន
ិ , ៃហ ចរ និងយ៉ត ពនក 3
េមេរ នទី១ ៖ ចំនន
ួ សនិ ន

24/. ច ប7/ ព8គ9បង9: ; ថ (ចនន)ជ+,ន


3 3a 3 x 8a 27 3 m 15
, , , , ,
x 8ab 5 2 x3 y
3
25ab 3
7 mn 2
2a − b
3
7 2x 11 7 3 7x
, ,
2 x + 3y 3
2m − 3 12n 3
x2 + 2 3 y
25/. គ56 / 9កងមយ,ន), / a , b , c 9: c ( ប9)ង0ន

;&ប /ន& -ចគ ជ=ងមយ0ន / 9កង ន4 ប> គ:ង


? @
ក. a = 3 , b = 4 ខ. a = 5 , b = 12 គ. a = 6 , b = 8
ឃ. c = 26 , b = 10 ង. b = 30 , c = 50 -
26/. គ56 m = 3 8 + 5 នង n = 3 8 − 5 -
mn + m 2 m2 − n2 n 2 − 2mn
ចគ , , -
m m+n n2
27/. ច បCប ធ បចនន 2 នង 3 3 -
28/. ច ម ក ន ម A = 22 − 288 -
29/. ច គ r 0ន & Eង ប,
> F,ន
GH 628000 cm3 គ56 π = 3.14 -

30/. ក. ច បGIញ@ 217 + 217 = 218 -


ខ. ច ក / x 9: 2 x ⋅ 2 x+3 = 85 -

❈❈❈❈❈❈❈❈❈❈❈❈❈❈❈❈❈❈❈❈❈❈❈❈
0ថJទ K មL MK M , ,ង ប9O ( KP ព ប -
,ន Q4@%កច ច R មយ Fងម/ Dream125
Dream125 S MK K
(មយEនក&ងទនT UV ង W .ង ជង
> ផ ង Yង

= 7ទ ខ/Z[9ក) - ;%ក ង Q49: ជ4ម/ 4 គខ
\ & ] ន4^ង,
ព 4គ\ ខ]& ចញពភ% ពញ :ម
> ` aផb4ក%&ង5 ឈប ,ក
ច d%ថe ប0ព (/9ខe នង :ម
> ` aជប ង WLf
)ទg TយបSន
& ន OS&ន9 នL1 ម 0ថJ9 ក
h 9: ម/ZភកZខ]&7ន
ផVច ផVម
> ធHជ
> ន fក គi ;%ក គi ( ? កគ& : ពកQ/ -
Uង9: 9: ប& 6ខ]&,ន ទប
> ជយ56ខ]&,ន ប /ច/ច -
កំែណេ យ ៃហ ហុន
ិ , ៃហ ចរ និងយ៉ត ពនក 4
េមេរ នទី១ ៖ ចំនន
ួ សនិ ន

ច មយ
1/. គ
9 = 32 = 3 16 = 4 2 = 4
36 = 6 2 = 6 − 64 = − 82 = −8
− 100 = − 102 = −10 121 = 112 = 11
− 144 = − 122 = −12 625 = 252 = 25

8 = 3 23 = 2 −8 = ( −2 ) = −2
3 3 3
3

− 3 27 = − 3 33 = −3 − 3 64 = − 3 43 = −4
3
125 = 3 53 = 5 3
216 = 3 63 = 6
3
1000 = 3 103 = 10
2/. គ
9 32 3 49 72 7
= = = 2 =
16 42 4 9 3 3
81 92 9 169 132 13
− =− 2 =− = =
4 2 2 49 72 7
196 142 14 400 202 20 4
= = − =− =− =−
25 52 5 225 15 2
15 3
1 3 13 1 8 23 2
3 = = 3 − =− 3 =−
3
8 23 2 27 3 3
64 3 43 4 512 3 83 8
3 = 3 = 3 = 3 =
125 5 5 343 7 7
216 63 6 3
3 − = −3 3 = − = −
1000 10 10 5
3/. ម

(4 ) ( )
3 2
163 = 2
= 43 = 64 − 3 362 = − 3 62 = − 3 6 4 = −6 3 6

(8 ) ( )
3
2 3
643 = = 83 = 512 3
−8 = −8
3
−273 = −27 3
15 = 1
3
82 = 3
(2 ) 3 2
= 22 = 4 3
642 = 3
(4 )3 2
= 42 = 16

( −27 ) (3 )
2
= 3 27 2 = = 32 = 9
2 3
3 3

4/. ម
y2 = y x4 = x2 = x2

( xy ) (y )
2 2
x2 y 4 = 2
= xy 2 = y 2 x , − y 6 = − 3
= − y3

កំែណេ យ ៃហ ហុន
ិ , ៃហ ចរ និងយ៉ត ពនក 5
េមេរ នទី១ ៖ ចំនន
ួ សនិ ន

16 42 4 4 100 102 10 10
= = = = = =
x2 (n ) 2 2
2
x x x n4 n2 n2

8 x3 = ( 2x) = 2x
3 3
3

− 64m3 = − ( 8m ) m = − 8m m = −8 m m
2

5/. ម កន ម
(2 x) 2 = 2 x = 2 x 3
(−5 y )3 = ( −5 y ) = −5 y
(4 − a ) 2 = 4 − a 3
( x + 3)3 = x + 3

16b 2 + 24b + 9 = ( 4b + 3) = 4b + 3
2

9 x 2 − 30 x + 25 = (3x − 5) = 3x − 5
2

4m 2 − 20mn + 25n 2 = ( 2 m − 5n ) = 2 m − 5n
2

49 x 2 − 112 xy + 64 y 2 = (7x − 8 y) = 7x − 8y
2

6/. ប ! ញមយចននព
18 = 9 ⋅ 2 = 32 ⋅ 2 = 3 2 − 48 = − 16 ⋅ 3 = − 4 2 ⋅ 3 = −4 3
30 30 30
75 = 25 ⋅ 3 = 52 ⋅ 3 = 5 3 = 2
=
49 7 7
10 10 10
= 2
= 3
40 = 3 8 ⋅ 5 = 3 23 ⋅ 5 = 2 3 5
121 11 11
3
54 = 3 27 ⋅ 2 = 3 33 ⋅ 2 = 3 3 2 − 3 128 = − 3 64 ⋅ 2 = − 3 43 ⋅ 2 = −4 3 2
3
3m 3m 3m
3
192 = 3 64 ⋅ 3 = 3 43 ⋅ 3 = 4 3 3 3 = = , ( n ≠ 0)
( 2n )
3 3
8n3 2n
3
16a 5 = 3 8a 3 ⋅ 2a 2 = 2a 3 2a 2
7/. ប ! ញមយចននព

( 6ab ) ( 3a b )
2
36a 2b3 = b =6 a b b 27 a 4b3 = 3b = 3a 2b 3b
2 2

(6x y ) ( 4ab )
2 2
72 x5 y 2 = 2
2 x = 6 x2 y 2 x − 112a 3b 4 = − 2
7 a = −4ab 2 7 a

( 4m n ) ( 8 xy )
2
80m4 n3 = 5n = 4 m 2 n 5n 64 x 2 y 3 = y =8 x y y
2 2

16m3 n3 = ( 2mn ) 2 = 2mn 3 2 −54 x 4b3 = − 3 ( 3 xb ) 2 x = −3 xb 3 2 x


3 3 3 3
3

− 3 128a 5 y 3 = − 3 ( 4ay ) 2a 2 = −4ay 3 2a 2 24 p 3q 5 = ( 2 pq ) 3q 2 = 2 pq 3 3q 2


3 3 3
3

8/. ប ! មយចននក%&ង 9: x , y , m , n នង p (ចនន)ជ+,ន

5 6 = 52 ⋅ 6 = 25 ⋅ 6 = 150 2m m = ( 2m ) m = 4m 2 ⋅ m = 4m3
2

23 23 23 3
= = 2 5 = 3 23 ⋅ 5 = 3 40
y3 (y )
3 2 6
y

កំែណេ យ ៃហ ហុន
ិ , ៃហ ចរ និងយ៉ត ពនក 6
េមេរ នទី១ ៖ ចំនន
ួ សនិ ន

3
3m 3m 3m
2x 3 4 = ( 2 x ) 4 = 3 32 x3 = =
3
3 3

( 2n )
3 3
2n 8n3

9/. គ
3 2 − 4 2 + 5 2 − 3 2 = 2 ( 3 − 4 + 5 − 3) = 2
5 2 − 3 3 − 6 2 + 5 3 = 2 ( 5 − 6 ) − 3 ( 3 − 5) = 2 3 − 2
3 15 − 4 3 − 3 15 + 6 3 = 2 3
4 3 − 2 17 + 3 17 − 3 3 − 2 3 = 17 − 3
2 3 2 − 83 3 + 3 2 + 33 3 = 33 2 − 53 3
83 2 − 33 3 − 53 2 + 2 3 3 = 33 2 − 3 3
10/. គ
2
3
27 −
3
4
2
( ) ( )
3
48 = 3 3 − 4 3 = 2 3 − 3 3 = − 3
3 4
1
4
288 −
1
6
1
( ) ( )
1
72 = 12 2 − 6 2 = 3 2 − 2 = 2 2
4 6
3
5
75 −
2
3
3
( ) ( )
2
27 = 5 3 − 3 3 = 3 3 − 2 3 = 3
5 3
5 128 − 3 250 = 5 ( 4 2 ) − 3 ( 5 2 ) = 20 2 − 15 2 = 5
3 3 3 3 3 3 3
2

192 = 3 ( 3 3 ) − ( 4 3 ) = 9 3 − 2 3 = 7 3
1 1
3 81 −
3 3 3 3 3 3 3

2 2
4 54 − 3 128 = 4 ( 3 2 ) − 3 ( 4 2 ) = 12 2 − 12 2 = 0
3 3 3 3 3 3

11/. គ
(
2 8 − 3 98 − 2 200 = 2 2 2 − 3 7 2 − 2 10 2 ) ( ) ( )
= 4 2 − 21 2 − 20 2 = −37 2
(
−3 50 − 32 + 5 200 = −3 5 2 − 4 2 + 5 10 2 ) ( )
= −15 2 − 4 2 + 50 2 = 31 2
(
3 175 − 2 28 + 3 63 − 112 = 3 5 7 − 2 2 7 + 3 3 7 − 4 7 ) ( ) ( )
= 15 7 − 4 7 + 9 7 − 4 7 = 16 7
108 − 2 27 − 40 − 5 160 = 6 3 − 2 3 3 − 2 10 − 5 4 10 ( ) ( )
= 6 3 − 6 3 − 2 10 − 20 10 = −22 10
( ) (
2 3 16 + 3 3 54 − 2 3 128 = 2 2 3 2 + 3 3 3 2 − 2 4 3 2 ) ( )
= 43 2 + 93 2 − 83 2 = 53 2
1 1
( ) (
3 3 81 + 3 128 − 3 3 192 + 4 3 54 = 3 3 3 3 + 4 3 2 − 3 4 3 3 + 4 3 3 2
2 2
) ( ) ( )
= 9 3 3 + 2 3 2 − 12 3 3 + 12 3 2 = 14 3 2 − 3 3 3

កំែណេ យ ៃហ ហុន
ិ , ៃហ ចរ និងយ៉ត ពនក 7
េមេរ នទី១ ៖ ចំនន
ួ សនិ ន

( ) ( ) (
4 3 54 − 6 3 81 − 4 3 16 + 3 3 24 = 4 3 3 2 − 6 3 3 3 − 4 2 3 2 + 3 2 3 3 ) ( )
= 12 3 2 − 18 3 3 − 8 3 2 + 6 3 3 = 4 3 2 − 12 3 3
( ) ( ) (
−2 3 40 − 3 3 135 + 5 3 320 + 8 3 5 = −2 2 3 5 − 3 3 3 5 + 5 4 3 5 + 8 3 5 )
= −4 3 5 − 9 3 5 + 20 3 5 + 8 3 5 = 15 3 5
12/. គ
( ) ( ) (( )
−2 2 12 − 18 − 5 3 32 − 27 = −2 2 2 3 − 3 2 − 5 3 4 2 − 3 3 ) (( ) )
(
= −2 4 3 − 3 2 − 5 12 2 − 3 3 ) ( )
= −8 3 + 6 2 − 60 2 + 15 3 = 7 3 − 54 2

( ) (
3 3 3 40 − 3 135 + 4 3
) (( )
320 − 3 40 = 3 3 2 3 5 − 3 3 5 + 4 4 3 5 − 2 3 5 ) ( )
= 9 3 5 + 8 3 5 = 17 3 5
( ) ( ) ( )
4 50 4 18 2 3 3
( )
2 27 4 5 2 4 3 2
− 3 48 + − = −3 4 3 + −
3 5 3 3 5 3
= 2 3 − 12 3 + 4 2 − 4 2 = −10 3
2 33 5 3 2 3 5 ( ) ( )
23
3
81 −
13
2
24 +
2 3 135 3 3 40 2 3
3

2
1 3
= 3 3 − 2 3 +
3 2 3
( ) (

2
)
= 2 3 3 − 3 3 + 2 3 5 − 33 5 = 3 3 − 3 5
13/. គ ក ន ម.ង ម a , b , x , y នង z (ចនន)ជ+,ន
(
A = −3 32 x + 6 8 x = −3 4 2 x + 6 2 2 x ) ( )
= −12 2 x + 12 2 x = 0

(
B = 2 125 x 2 z + 8 x 80 z = 2 5 x 5 z + 8 x 4 5 z) ( )
= 10 x 5 z + 32 x 5 z = 42 x 5 z
C = 7 a b3 + b 4a 2b − 4b = 7 ab b + b 2a b − 2 b ( )
= 9ab b − 2 b = b ( 9ab − 2 )

(
D = 8b 49b − 7 9b3 + a 4a + a 3 = 8b 7 b − 7 3b b + a 2 a + a a ) ( ) ( )
= 56b b − 21b b + 2a a + a a = 35b b + 3a a

(
E = 3 xy x 2 y − 2 x 4 y 3 = 3 xy x y − 2 x 2 y y) ( )
= 3x 2 y y − 2 x 2 y y = x 2 y y

(
F = −3a a 3b5 − 2b a 5b3 + 5 a 3b3 = −3a ab 2 ab − 2b a 2b ab + 5ab ab ) ( )
(
= ab −3a 2b 2 − 2a 2b 2 + 5ab )
= ab ( 5ab − 5a b ) = 5ab ab (1 − ab )
2 2

(
G = 8a 3 54a + 6 3 16a 4 = 8a 3 3 2a + 6 2a 3 2a ) ( )
= 24a 3 2a + 12a 3 2a = 36a 3 2a

កំែណេ យ ៃហ ហុន
ិ , ៃហ ចរ និងយ៉ត ពនក 8
េមេរ នទី១ ៖ ចំនន
ួ សនិ ន

H = 3 3 x 4 y − 6 x 3 xy 4 + 2 3 x 4 y 4 = 3x 3 xy − 6 xy 3 xy + 2 xy 3 xy
= 3x 3 xy − 4 xy 3 xy = x 3 xy ( 3 − 4 y )

14/. គ /0ម1 ខ0នក ន ម A


យង
> ,ន
A = ab − ab3 − 9a3b3 − a 3b = ab − b ab − 3ab ab − a ab
A = ab (1 − b − 3ab − a )

ច 34 a = 5 , b = 3 គ7ន
A = 5 ⋅ 3 (1 − 3 − 3 ⋅ 5 ⋅ 3)
A = 15 (1 − 3 − 45 − 5 )
A = −52 15
:ច ន4 A = −52 15 /i)7នក /-
15/. គ /0ម1 ខ0នក ន ម A
យង
> ,ន
A = 4a + a a 2b + b2 a + b 9b
A = 2 a + a 2 b + b a + 3b b
(
A = a ( 2 + b ) + b a 2 + 3b )
ច 34 a = 3 , b = 2 គ7ន
(
A = 3 ( 2 + 2 ) + 2 32 + 3 ⋅ 2 )
A = 4 3 + 15 2
:ច ន4 A = 4 3 + 15 2 /i)7នក /-
16/. គ
( 2 3 )(3 2 ) = 2 ⋅ 3 3 ⋅ 2 = 6 6
( 4 6 )( −2 5 ) = −4 ⋅ 2 6 ⋅ 5 = −8 30
(3 5 )(5 3 ) = 3 ⋅ 5 5 ⋅ 3 = 15 15
( 6 2 )( −12 3 ) = −6 ⋅12 2 ⋅ 3 = −72 6
(3 8 )( −3 48 ) = −3 ⋅ 3 8 ⋅ 8 ⋅ 6 = −9 ⋅ 8 6 = −72 6
( −3 75 )( −2 48 ) = ( −3)( −2) 25 ⋅ 3 ⋅16 ⋅ 3 = 6 ⋅ 5 ⋅ 4 ⋅ 3 = 360
( 2 3 )  − 12 2  = 2  − 12  3 ⋅ 2 = − 6
3 3 3 3

(3 2 )(5 15 ) = 3 ⋅ 5 2 ⋅15 = 15 30
3 3 3 3

( 6 8 )( −3 2 ) = −18 2 ⋅ 2 = −36 2
3 3 3 3 3

កំែណេ យ ៃហ ហុន
ិ , ៃហ ចរ និងយ៉ត ពនក 9
េមេរ នទី១ ៖ ចំនន
ួ សនិ ន

17/. គ
( )
3 5 2 18 − 3 48 = 3 5 2 ⋅ 3 2 − 3 ⋅ 4 3 = 18 10 − 36 15 ( )
( )
−3 3 3 6 − 3 2 = −9 18 + 9 6 = −9 ⋅ 3 2 + 9 6 = −27 2 + 9 6
1
2
3 (2
1
48 − 3 32 = 
2

) 1
3  2 42 ⋅ 3 − 
 2

3  3 42 ⋅ 2 = 4 32 − 6 6

( ) ( )
= 12 − 6 6
3
2
(
2 2 18 − 3 48 =
3
2
) ( )
2 2 ⋅ 3 2 − 3 ⋅ 4 3 = 18 − 18 6

−4 3 ( 2 6 − 2 5 ) = ( −4 3 )( 2 6 ) − ( −4 3 )( 2 5 ) = −8
3 3 3 3 3 3 3 3
18 + 8 3 15

2 5 ( 3 3 − 5 2 ) = 6 15 − 10 10
3 3 3 3 3

3 3 ( 3 8 − 2 18 ) = 9 3 ⋅ 2 − 6 3 ⋅ 2 = 18 3 − 18 2
3 3 3 3 3 3 3 3 3

−3 5 ( 4 20 − 2 45 ) = −12 100 + 6 225


3 3 3 3 3

18/. គ
(2 )( ) (
3 − 8 9 + 2 5 = 2 3 9 + 2 3 2 5 − 8 (9) − 8 2 5 ) ( )( ) ( )
= 18 3 + 4 15 − 72 − 16 5

(3 5 − 2 10 )( )
50 − 2 80 = 3 52 ⋅10 − 6 202 − 2 102 ⋅ 5 + 4 202 ⋅ 2

= 15 10 − 120 − 20 5 + 80 2
( 50 − 75 ) ()( )( )
32 − 48 = 5 2 − 5 3 4 2 − 4 3

= 20 ( 2 − 3 ) = 20 ( 5 − 2 6 ) = 100 − 40 6
2

( 125 − 75 )( 80 − 48 ) = (5 5 − 5 3 )( 4 5 − 4 3 )
= 20 ( 5 − 3 ) = 20 ( 8 − 2 15 ) = 160 − 40 15
2

(3 18 + 3 27 )( 2 8 − 2 12 ) = (3 18 + 3 ⋅ 3)( 2 ⋅ 2 − 2 12 )
3 3 3 3 3 3

= 3 ( 18 + 3) ⋅ 2 ( 2 − 12 ) 3 3

(
= 6 2 3 18 − 3 32 ⋅ 2 ⋅ 22 ⋅ 3 + 6 − 3 3 12 )
= 12 3 18 − 6 ⋅ 6 + 36 − 18 3 12
= 12 3 18 − 18 3 12
( 3
)( ) (
80 − 2 3 27 −3 3 20 − 3 3 12 = 2 3 10 − 2 ⋅ 3 −3 3 20 − 3 3 12 )( )
= −6 ( 10 − 3)( 20 + 12 )
3 3 3

= −6 ( 200 + 120 − 3 20 − 3 12 )
3 3 3 3

= −6 3 23 ⋅ 25 − 6 3 23 ⋅15 + 18 3 20 + 18 3 12
= −12 3 25 − 12 3 15 + 18 3 20 + 18 3 12

កំែណេ យ ៃហ ហុន
ិ , ៃហ ចរ និងយ៉ត ពនក 10
េមេរ នទី១ ៖ ចំនន
ួ សនិ ន

19/. គ
1
−3ab , a 2 + b 2 , a 2 − 2b 2 , b 2 − 2ab , cd , c 2 − b 2 , c 2 + 2cd
2
យង
> ,ន
a = 3 5 − 2 10 , b = 5 7 + 2 10
c = 3 18 − 3 27 = 3 18 − 3
នង d = 3 3 6 + 3 8 = 3 3 6 + 2
គ7ន

( )( )
−3ab = −3 3 5 − 2 10 5 7 + 2 10 = −3 15 35 + 6 50 − 10 70 − 2 10  ( )
2

 
(
= −3 15 35 + 30 2 − 10 70 − 40 )
= 30 70 + 120 − 45 35 − 90 2

( ) + (5 7 + 2 10 )
2 2
a 2 + b 2 = 3 5 − 2 10

= ( 3 5 ) − 2 ( 3 5 )( 2 10 ) + ( 2 10 ) + ( 5 7 )
2 2 2

+2 ( 5 7 )( 2 10 ) + ( 2 10 )
2

= 45 − 12 50 + 40 + 175 + 20 70 + 40
= 300 − 60 2 + 20 70

( ) − 2 (5 7 + 2 10 )
2 2
a 2 − 2b 2 = 3 5 − 2 10

= ( 3 5 ) − 2 ( 3 5 )( 2 10 ) + ( 2 10 )
2 2

−2  ( 5 7 ) + 2 ( 5 7 )( 2 10 ) + ( 2 10 ) 
2 2

 
= 45 − 12 50 + 40 − 2 (175 + 20 70 + 40 )

= 85 − 60 2 − 430 − 40 70
= −345 − 60 2 − 40 70

( ) − 2 (15 35 + 30 2 − 10 70 − 40)
2
b 2 − 2ab = 5 7 + 2 10

= ( 5 7 ) + 2 ( 5 7 )( 2 10 ) + ( 2 10 )
2 2

−2 (15 35 + 30 2 − 10 70 − 40 )
= 175 + 20 70 + 40 − 30 35 − 60 2 + 20 70 + 80
= 295 + 40 70 − 30 35 − 60 2
1
2
1
2
( )( 1
) (
cd = 3 18 − 3 3 3 6 + 2 = 3 3 32 ⋅ 2 ⋅ 2 ⋅ 3 + 2 3 18 − 9 3 6 − 6
2
)
1
(
= 9 3 4 + 2 3 18 − 9 3 6 − 6
2
)
93 4 3 93 6
= + 18 − −3
2 2

កំែណេ យ ៃហ ហុន
ិ , ៃហ ចរ និងយ៉ត ពនក 11
េមេរ នទី១ ៖ ចំនន
ួ សនិ ន

( ) ( ) = ( 18 ) − 6 18 + 9
2 2 2
c 2 − b2 = 3
18 − 3 − 5 7 + 2 10 3 3

−  ( 5 7 ) + 2 ( 5 7 )( 2 10 ) + ( 2 10 ) 
2 2

 

(3 ⋅ 2) ( )
2
= 3 2
− 6 3 18 + 9 − 175 + 20 70 + 40

= 3 3 12 − 6 3 18 + 9 − 215 − 20 70
= 3 3 12 − 6 3 18 − 20 70 − 206
(
c 2 + 2cd = 3 3 12 − 6 3 18 + 9 + 2 9 3 4 + 2 3 18 − 9 3 6 − 6 )
= 3 3 12 − 6 3 18 + 9 + 18 3 4 + 4 3 18 − 18 3 6 − 12
= 3 3 12 − 2 3 18 + 18 3 4 − 18 3 6 − 3
20/. គ

ក ន មមនjច ម 7ន ទ / ទ -
33
11
− 48 4 3 4
=− =−
27 3 3 3
2 75 75
2 75 ÷ 15 = =2 =2 5
15 15
2 84 84
= −2 = −2 7
− 12 12
5 25 25 1 5 15 15
÷ 5 =5 =5 = =
75 5 ⋅ 75 15 15 3
12 3 6 6
3
= 12 3 = 12 3 3
2 2
5 3 100 1 3 100 3 20
= =
25 3 5 5 5 5
 3 72  3  23 9  3 3
18
 3  2 =   2 =
2 8  2⋅2  2
6 3 7 4 3 21 6 3 7 2 3 3 12 3 7 ⋅ 3 3 1 3 36
÷ = ⋅ = = =
3 3 6 2 3 3 3 3 6 4 3 21 12 6 ⋅ 21 6 6
21/. ម ក ន ម.ង ម
b2 b b
A= = =
b 2 − 14b + 49 (b − 7 )
2 b−7

( 7 x − 4)
2
49 x 2 − 56 x + 16 7x − 4
B= = =
36 x 2 6x 6x

( a + 8b )
2
a 2 + 16ab + 64b 2 a + 8b
C= = =
a 2 + 10ab + 25b 2 ( a + 5b ) a + 5b
2

25b 2 + 10ab + a 2  5b + a  5b + a
2

D= =   =
16b + 24ab + 9a
2 2
 4b + 3a  4b + 3a

កំែណេ យ ៃហ ហុន
ិ , ៃហ ចរ និងយ៉ត ពនក 12
េមេរ នទី១ ៖ ចំនន
ួ សនិ ន

22/. ប7/ ព8គ9បង


3 3 10 3 10
= =
( )
2
10 10 10

3 3 3 3 ( 33 ) 3 32 ⋅11 3 11
= = =
( )
2
33 33 33 11

6 6 3 3 3
= = =
( )
2
48 4 3 2 3 2

8 8 8 3 8 3
= = =
( )
2
27 3 3 3 3 9

9 4 18 6 2
= = =3 2
18 3 2 2

5 5 5 5 5 5 2
= = =
5 10 ( )
72 6 2 12 12

93
=
3
9 3 100
=
3 (
900 )
3
10 3
103 10
23 3 3 1 2 2 3 100 3 100
3
= 2 3 = 2 3 = = =
30 30 10 3 10 10 5
3 3
18 3 18 3 9 900
3
= = =
20 20 10 10

7m 3
= 3
7m
=
3
7m ( 3
6n 2 ) =
3
42mn 2
( )( )
3
36n 36n 3
36n 3
6n 2 6n

11 p 3 11 p
= =
3 11 p
( 3
)=
7q 2 3
77 pq 2

( 7q )
3
49q 3 7 2 q 3 7 2 q 3 2 7q

3
=
3
3
=
3
3 ( 3 2y ) =
6y
3

( )
3
4 y2 (2 y) (2 y) 2y
2 2
3 3 3 2y

23/. ប7/ ព8គ9បង


36 − 6 36 − 6
= =
36 − 6 ( ) 2
=
36 2 − 12 18 2 − 3
=
8 2 2 4 4 2

3+ 5
=
3+ 5
=
( 3+ 5 ) 5
=
15 + 5
3 20 6 5 6⋅5 30

កំែណេ យ ៃហ ហុន
ិ , ៃហ ចរ និងយ៉ត ពនក 13
េមេរ នទី១ ៖ ចំនន
ួ សនិ ន

8
=
8
=

8 2 3 +3 2 

( )
2 75 − 3 50 10 3 − 15 2 5  2 3 − 3 2 2 3 + 3 2 
  ( )( )
8 2 3 +3 2  8 2 3 +3 2 
=   = − 
5  12 − 18  5  6 
8 3 + 12 2
=−
15
2 3
=
2 3
=
2 3 2 3 5
=
( )
2 80 − 45 8 5 − 3 5 5 5 25
2 15
=
25
9− 3 3 9− 3 3
= =
)( 2 ) = 9 2 − 6
9− 3 3 ( 3
3 3

2 3 32 4 3 22 ( 2)
4 3 22 8 3

5 4 + 3 ( 5 4 + 3 )( 169 ) 5 676 + 507


3 3 3
3 3 3 3
= =
3
8 13 8 13 ( 169 ) 104 3 3

2 6 3
2 6 2 6 ( 36 + 6 9 + 81 )
3 2 6 ( 36 + 6 9 + 3 3 ) 3 3 3 3 3 3

= = =
2 27 − 9 6 − 9 ( 6 − 9 )( 36 + 6 9 + 81 )
3 3 3
216 − 9 3 3 3

2 6 (12 + 2 9 + 3 ) 24 6 + 4 3 ⋅ 2 + 2 183 3 3
3 3 3 3
= =
69 69
24 3 6 + 12 3 2 + 2 3 18
=
69
23 2
=
23 2
=
(
2 4 + 2 3 6 + 3 36 )
3
16 − 3 12 3
2 ( 3
8−36 ) ( 2 − 6 )( 4 + 2 6 + 36 )
3 3 3

2 ( 4 + 2 6 + 36 ) 3 3

= = 4 + 2 6 + 36 3 3

8−6
24/. ប7/ ព8គ9បង9: ; ថ (ចនន)ជ+,ន
3 3 x
=
x x
3a 3a 3a 2ab 3 2ab
= = =
8ab 2 2ab 4ab 4b
3 x 3 x 3 1 3 2y 3 2y
= 3
= ⋅ = ⋅ =
5 2 x3 y 5 2 x y 5 2 x2 y 5x 2 y 10 xy

8a
=
8a ( 3
5a 2 b 2 ) =
8a 3 5a 2 b 2 8 3 5a 2 b 2
=
3
25ab ( 3
25ab )( 3
5a 2 b 2 ) 5ab 5b

27 3 m 27 3 m 27 1 27 3 m 2 n 2 27 3 m 2 n 2
= = ⋅ = =
7 3 m2n 2 m 2 n 7 3 mn 7 3 mn 3 m 2 n 2 7mn
( )
កំែណេ យ ៃហ ហុន
ិ , ៃហ ចរ និងយ៉ត ពនក 14
េមេរ នទី១ ៖ ចំនន
ួ សនិ ន

15
=
15 ( 2a + b ) =
15 ( 2a + b )
2a − b ( 2a − b )( 2a + b ) 2a − b

7 2x 7 x− 2x ( 2
14 x 2 − 7 6 xy 3y )
= =
2 x + 3y (
2 x + 3y 2 x − 3y 4x − 3 y )( )
11  3 ( 2m ) + 3 ( 2m )(12n ) + 3 (12n ) 
3 2 2
3
11
=  
2m − 3 12n
( )
2m − 3 12n  3 ( 2m ) + 3 ( 2m )(12n ) + 3 (12n ) 
3 2 2
3

 

=
3
11 ( 3
4m 2 + 3 24mn + 3 144n 2 )
2m − 12n
3
44m 2 + 3 264mn + 3 1584n 2
=
2m − 12n

7 3 7x
=
7 3 7x ( 3
x4 − 2 3 x2 y + 4 3 y 2 )
3
x2 + 2 3 y ( 3
x2 + 2 3 y )( 3
x4 − 2 3 x2 y + 4 3 y 2 )
7 3 7 x 5 − 14 3 7 x3 y + 28 3 7 xy 2
=
x2 + 8 y
7 x 3 7 x 2 − 14 x 3 7 y + 28 3 7 xy 2
=
x2 + 8 y
25/. គ ជ=ងមយ0ន / 9កង ន4 ប> គ:ង
? @
[ម ទ? បទព
V [គT c2 = a2 + b2 4,

ក. a = 3 , b = 4
គ7ន
c 2 = 32 + 4 2 = 25
c=5
:ច ន4 c = 5 /i)7នក /-
ខ. a = 5 , b = 12
គ7ន
c 2 = 52 + 12 2 = 169
c = 13

កំែណេ យ ៃហ ហុន
ិ , ៃហ ចរ និងយ៉ត ពនក 15
េមេរ នទី១ ៖ ចំនន
ួ សនិ ន

:ច ន4 c = 13 /i)7នក /-
គ. a = 6 , b = 8
គ7ន
c 2 = 62 + 82 = 100
c = 10
:ច ន4 c = 10 /i)7នក /-
ឃ. c = 26 , b = 10
គ7ន
a 2 = 262 − 102 = 576
a = 24
:ច ន4 a = 24 /i)7នក /-
ង. b = 30 , c = 50
គ7ន
a 2 = 502 − 302 = 1600
a = 40
:ច ន4 a = 40 /i)7នក /-
mn + m 2 m2 − n2 n 2 − 2mn
26/. គ , ,
m m+n n2
យង
> ,ន m = 3 8 + 5 នង n = 3 8 − 5 គ7ន
mn + m 2 m ( n + m )
= = m + n = 3 8 + 5 + 3 8 − 5 = 12 2
m m
m 2 − n 2 ( m − n )( m + n )
= = m−n = 3 8 + 5 −3 8 + 5 = 2 5
m+n m+n
n 2 − 2mn n ( n − 2m ) 2m 3 8+ 5 
= = 1− = 1 − 2  
3 8− 5 
2 2
n n n

( )
2
2 3 8+ 5
= 1−
(3 8 ) − ( 5 )
2 2

2 ( 72 + 6 40 + 5 )
= 1−
72 − 5

= 1−
(
2 77 + 6 4 ⋅10 )
67
67 − 154 − 24 10
=
67
−87 − 24 10
=
67

កំែណេ យ ៃហ ហុន
ិ , ៃហ ចរ និងយ៉ត ពនក 16
េមេរ នទី១ ៖ ចំនន
ួ សនិ ន

27/. បCប ធ បចនន 2 នង 3 3


យង
> ,ន

( )=
3
2= 3 2 6
23 = 6 8

នង 3 3 = ( 3) =
2
3 6
32 = 6 9

kយ 9 > 8 56 6 9 > 6 8
:ច ន4 3 3 > 2 /i)7ន បCប ធ ប ច -
28/. ម ក ន ម A = 22 − 288
គ7ន
A = 22 − 288 = 22 − 12 2
A = 18 − 12 2 + 4

(3 2 ) − 2 ⋅ 3
2
A= 2 ⋅ 2 + 22

(3 2 − 2)
2
A=

A= 3 2 −2 = 3 2 −2

:ច ន4 ក ន ម A = 3 2 − 2 /i)7ន ម -
29/. គ r

[ម បមនZ, 0ន & Eង

V = Ah = π r 2 h
kយ V = 628000 cm3 នង h = 2r គ7ន
V = π r 2 ( 2r )
V 628 000 cm3
r3 = =
2π 2 ⋅ 3.14
r = 100 000 cm3
3

r = 3 105 cm3 = 10 3 100 cm

:ច ន4 r = 10 3 100 cm /i)7នគ -
30/. ក. បGIញ@ 217 + 217 = 218
យង
> ,ន
217 + 217 = 217 (1 + 1) = 217 ⋅ 2 = 218 ព/

:ច ន4 /i)7នបGIញ 217 + 217 = 218 -

កំែណេ យ ៃហ ហុន
ិ , ៃហ ចរ និងយ៉ត ពនក 17
េមេរ នទី១ ៖ ចំនន
ួ សនិ ន

ខ. ក / x
យង
> ,ន 2 x ⋅ 2 x+3 = 85 គ7ន ,

( )
5
2 x + x + 3 = 23
22 x + 3 = 215
2 x + 3 = 15
15 − 3
x= =6
2
:ច ន4 x = 6 /i)7នក /-

...េពលែដល
...េពលែដល នេ ះ ក់េ ះ គឺ នម៉ត
ូ ម
ូ យ
ួ េបើក ងមុខខ!"ំ យឺត

$ងេល%&នម៉ត
ូ រូ បស់ខ!"ំ , ខ!ក
"ំ ប
៏ ន+យេល%&ន ពី ៦០គ.ម
៦០គ.ម ក/ង
" ១ម៉. មក

េ1ែត បែហល$ ៥០គ.ម


៥០គ.ម ក/ង
" ១ម៉. វ5ញ ។ 8ប់ែតេពលេ ះ

9រម:ណខ
៍ $
!"ំ ប់ដត
ិ >មេរឿងែដលនឹងេកើត នេ1ៃថCែសDក(េ18

F) 8ប់ែតHនកុងៃតន័រ មកពីេ Jយ េហើយេពលKេបើកែជងខ!"ំ

Mន បែហល$$ង Nក់កOPលរថយនQ 8ប់ែត ទុងរបស់KMន

Sក់$មួយ កTUកម
់ ត
៉ូ រូ បស់ខ!"ំ េ Nះេល%&នរបស់Hន Kេលឿន$ង

េល%&នម៉ត
ូ រូ បស់ខ!"ំ េហើយKក៏រញ
ុ ខ!ឲ
"ំ WដួលែតមXង ។ េពលេ ះ េបើខ!"ំ

បែន+មេល%&នបនQច
ិ បែហល$ផុត ែតមិន9ចឯO , េបើខប
!"ំ ែន+ម

េល%&ន េ ះខ!ច
"ំ \ស់$បុកម៉ត
ូ ម
ូ យ
ួ ងមុខមិន ន ។ េពលែដល

Hនរុញខ!"ំ និងម៉ត
ូ ឲ
ូ Wដួល គឺដល
ួ េ] ង8^ំ េហើយម៉ត
ូ រូ បស់ខM
!"ំ ន

យកខ_ន
` រង$មួយផ_វ
a $តិ េហើយKក៏ បឹងបTbប់ខន
_` K ែតេ>យ8

ែតនិចលcព Kក៏ តdវបងeច


ំ ត
ិ Q េMកមក ងេឆgងវ5ញ េហើយអូសខ!"ំ

ដល់កOPលផ_វ
a $តិ េទើបឈប់ េ>យយកខ_ន
` K ទខ!"ំ , សំOងOស់

ែដលេពលេ
ែដលេពលេ ះHនMនេ]ផុត កុអ
ំ ី កង់Hនកិនខ!េ
"ំ ]$េចកេចៀន

Mត់េ]េហើយ ។ អរគុណ Oស់ដល់មត


៉ូ ូ និងមួកសុវត+c
ិ ពរបស់ខ!"ំ

េ Nះ ខ!េ
"ំ ឃើញ ម៉ត
ូ រូ បស់ខM
!"ំ នរង$មួយផ_វ
a $តិ េJស Kច

គួសមែដរ េហើយែបកមួយដុេ
ំ ទៀត ឯមួកវ5ញរួស$មួយផ_វ
a $តិែប៉ក

កំែណេ យ ៃហ ហុន
ិ , ៃហ ចរ និងយ៉ត ពនក 18
េមេរ នទី២ ៖ ស ត

ម នទ

1/. គ n

ក. 30% ន n នង 48
ខ. 25% ន 28 នង n
គ. 17 n% ន 85

ឃ. n% ន 150 នង 20
ង. n នង 30% ន 400
2/. មយ ន 20 " ធ$ &% 'ន 17 " ក(គ យ ន "
ធ$ &%
3/. )&មយ*+ប- ក- ថ+ 25 000 /យ012& ក- 20 000
ច2 ក(គ យ ន4 ក-ប56
ប567 8 ថ+
4/. 9: ន A ន ; < 2 400 ក- /យ9: ន B ន ; <
3000 ក- ច2 ក(គ
ក(គ យ ន ; <9: ន A ធ បនង9: ន B
5/. &=> ចង-ទញ
; កង-មយ ថ+ 350 000 012& ន8@ - ន<"'ន 60% ន ថ+
កង- ច /យ @ - Aខ$8 'ក-ប7B ន ទ ទបទញ
; កង- 8'ន ?
6/. 7 ខ ន<"'ន 'ក- 60$ &% 80% ន ម+ B 7 នថ ប
2 B 7ន
8 ថ+ប7B ន ?
7/. DE " ន89: នទទ ; <'ន 4 200 ក- ទ2 ខ ន8 កន ងDEម
" ន
7
20% ច2 កច"
កច"នន ; < នDEF
" -

; មយ'ន ក-ប567 8 ម+ 16% Gយ ក-ក7 ង


8/. ទ"នញ E ម+ 4 200
ក ម+មន
7 ប567 8 ម+
9/. មង " 'នខ6 'ក-ពធ @ 1 :ន នI J4 'ក- 12%
@ យ
- ក 'ក- 600 000 K ក 7 ច" ញ'ន 180 000 /យ 'ក-
A - 400 000 @ យក

- ក K បនចង4 LM គកE7ងI J 14% កE7ង
មយDE" Aព "
= ច-DE" មង " ច" ញ NO ?

កំែណេ យ ៃហ ហុន
ិ , ៃហ ចរ និងយ៉ត ពនក 19
េមេរ នទី២ ៖ ស ត

10/. ADE " ន8 គផ -)Q ប


2 ក R LM ; <ច"
<ច"នន 8% ន ; < 'ន ប1ង
ប- STប មធMម ; កUទ7 យ
; ភ2ម; មWបន ;កU AមQ&ទX
; Yយ
ច2 កច"នន ; < 'នទទ
'នទទ )Q ប
2 ក R ប4 ប1ង STប
មធMម ; កUទ7 យ
; ភ2ម; ប- ម 70% កE7ងច" =ម បកZ
បកZជន
ជន
80 000 ក-

ច មយ
1/. គ n

ក. 30% ន n នង 48
Jមប"\ប-, គ'ន ^
30%n = 48
0.3n = 48
48
n= = 160
0.3
2ច ន8 n = 160 &% 'នក" -
ខ. 25% ន 28 នង n
Jមប"\ប- , គ'ន ^
25% ⋅ 28 = n
n = 0.25 ⋅ 28 = 7
2ច ន8 n = 7 &% 'នក" -
គ. 17 n% ន 85

Jមប"\ប- , គ'ន ^
n% ⋅ 85 = 17
17 1
n% = = = 0.2 = 20%
85 5
n = 20
2ច ន8 n = 20 &% 'នក" -
ឃ. n% ន 150 នង 20
Jមប"\ប- , គ'ន ^
n% ⋅150 = 20
20 2
n% = =
150 15
2 40
n = × 100 =
15 3

កំែណេ យ ៃហ ហុន
ិ , ៃហ ចរ និងយ៉ត ពនក 20
េមេរ នទី២ ៖ ស ត

2ច ន8 n = &% 'នក" -
40
3
ង. n នង 30% ន 400
Jមប"\ប- , គ'ន ^
n = 30% ⋅ 400
30
n= ⋅ 400 = 120
100
2ច ន8 n = 120 &% 'នក" -
2/. ក(គ យ ន " ធ$ &%
Jមប"
Jមប"\ប-, ន 20 " /យ ធ$ &% 'ន 17 " ទ
គ'ន(គ យ ន " ធ$ &% គ_ ^
17 17 ⋅ 5 85
= = = 85%
20 20 ⋅ 5 100
2ច ន8(គ យ ន " ធ$ &% គ_ 85%
3/. ក(គ យ ន4 ក-ប56
ប567 8 ថ+
Jមប"\ប-, ថ+)& 25 000 /យ012& ក- 20 000 , គ'ន ^
(គ យ ន4 ក-ប567 8 ថ+គ_ ^
25 000 − 20 000 5 000 1
= = = 0.2 = 20%
25000 25000 5
2ច ន8)& 8 ក-ប567 8 ម+ 20%
4/. ក(គ
ក(គ យ ន ; <9: ន A ធ បនង9: ន B
Jមប"\ប-, 9: ន A ន ; < 2 400 ក- /យ9: ន B ន

; < 3000 ក- ,
8(គ យ ន ; <9: ន A ធ បនង9: ន B គ_ ^
3000 − 2 400 600 1
= = = 20%
3000 3000 5
2ច ន89: ន A ន ; < ច
; ង9: ន B ច"នន 20%
5/. ក 'ក- @ - Aខ$8
Jមប"\ប-, &=> ចង-ទញ
; កង-មយ ថ+ 350 000 012& ន8@ - ន<"'ន
យ,,
60% ន ថ+កង- ច /យ ននYយ` @ - Aខ$8 40% ប នaម ទ
គ'ន , 40% ន 'ក- ថ+កង- ប
7 គ_ ^
40% ⋅ 350 000 = 0.4 ⋅ 350 000 = 140 000

2ច ន8 'ក- @ - Aខ$8គ_ 140 000

កំែណេ យ ៃហ ហុន
ិ , ៃហ ចរ និងយ៉ត ពនក 21
េមេរ នទី២ ៖ ស ត

6/. គ ថ+ B 7 នថ
នថ ប
2 8
Jមប"\ប-, 7 ខ ន<"'ន 'ក- 60$ &% 80% ន ម+ B 7 នថ ប
2
ប x ម+ ប - B 7 នថ ប
2 8, គ'ន ^
60 = 80% x
60 = 0.8 x
60
x= = 75$
0.8
2ច ន8 ម+ ន B 7 នថ ប
2 8គ_ 75$ &% 'នគ
7/. កច"
កច"នន ; < នDEF
" -
Jមប"\ប-, DE " ន89: នទទ ; <'ន 4 200 ក- គ_ កន ងDEម
" ន
7
20%

ប x ច"នន ; <DEF
" - ន9: ន 8, គ'ន ^
4 200 − x
= 20%
x
4 200 − x = 0.2 x
4 200 = 0.2 x + x
1.2 x = 4 200
4 200
x= = 3500
1.2
2ច ន8DEF
" -9: ន 8 ន ; < 3500 ក-
8/. ក ម+មន
7 ប567 8 ម+
Jមប"\ប-, ទ"នញ
; មយ ក-ប567 8 ម+ 16% Gយ ក-ក7 ង
E ម+ 4 200
ប x ម+មន
7 ប567 8, គ'ន ^
x − 4 200
= 16%
x
x 4 200
− = 0.16
x x
4 200
1− = 0.16
x
4 200
1 − 0.16 =
x
0.84 x = 4 200
4 200
x= = 5 000
0.84
2ច ន8ទ"នញ
; 8 ក-មន
7 ប567 8 ម+គ_ 5000

កំែណេ យ ៃហ ហុន
ិ , ៃហ ចរ និងយ៉ត ពនក 22
េមេរ នទី២ ៖ ស ត

9/. A ព "
= ច-DE" មង " ច" ញ NO ?
Jមប"\ប-,
'ក- មង " ខព ងb
6 ធ @ 1 :ន I J4 'ក- កE7ងbDE "
មងយក 'ក- 600 000 K ក 7 'នច" ញ 180 000
'ក- 400 000 ទ @ យ
- ក Kចង4 កE7ងI J4 'ក- 14% កE7ងb
ងbDE "
គ'ន ^
4 'ក- មង " &% ងធ @ &ញ
; គ_ ^
12% ⋅1000 000 = 120 000

'ក- មង " )ច ក'នព 'ក- ម 1 គ_ 180 000 នង


;
14% ⋅ 400 000 = 56 000

8 'ក- មង " ក'ន ប


7 គ_ ^ 180 000 + 56 000 = 236 000
Gយ 'ក- មង " ក'ន ព4 'ក- ប -ធ @
2ច ន8 A ព "
= ច-DE " មង "គ_ ច" ញ
10/. ច2 កច"នន ; < 'នទទ )Q ប
2 ក R
Jមប"\ប-, បកZជន ប1ង ន 80 000 ក-
បកZជន ប1ង ប- ន 70% បB7 => 8
បកZជន នងទទ )Q ប
2 ក R គ_ 8% ន ; <
ប1ង ប-
គ'ន ^
បកZជន ប1ង STប ប-គ_ ^ 70% ⋅ 80 000 = 56 000 ក-
បកZជន នងទទ 'ន)Q ប
2 ក R គ_ ^ 8% ⋅ 56 000 = 4 480 ក-
2ច ន8ច"នន បកZជន នងទទ 'ន)Q ប
2 ក R គ_ 4 480 ក-

... ងមុខ គួរសមែដរ េបើ មខំគត


ិ េបើ នមួកេទ មុខខំ បែហល

!ន"ច់េ"ចមិន ន ។ ទឹក ក'ចមួយេកះ ែដល!*យរបស់ +វ-ន


កូនសិស/ខំជន
ូ ក1ងឲ3សបុណ5ចូល67ំ 8ន តេ:ចិ
9 េ;<ម
ើ ង+*ំ

ៃនផ?វ
@ Aំងេកះ , 3 បរបស់ខំ មួយ េឡបថប ត'វEFក់េGេលើផវ
?@


ំ ពីមត
៉ូ ូ េទើបនឹងដួលេម?ះ៉ ។ ខំេលើកម៉ត
ូ ូ រួចរុ
ចរុញមកចិេ;<ម
ើ ផ?វ
@

កំែណេ យ ៃហ ហុន
ិ , ៃហ ចរ និងយ៉ត ពនក 23
េមេរ នទី៣ ៖ កេន មពីជគណិត

ម នទ ក ន មពជគ

1/. ច ព កន ម ង ម
A = ( x + 4)( x 2 − 4 x + 1) , B = ( x + 3)( x 2 + 4 x − 3)
C = (a − b)(a 2 − 2ab − 3b 2 ) , D = ( x + 2)( x 2 − 5 x − 1)
E = ( y + 5)( y 2 − 3 y − 8) , F = (3n + 4)(n + 5)
G = ( x + 3)( x − 1) 2
, H = ( y − 1)( y + 2) 2
2/. ច ក a ម
ក. ( x + a)( x − 8) = x 2 − 12 x + 32
ខ. (2 y − 6)( y + a) = 2 y 2 + 8 y − 42
3/. ច "ក#ក ន ម$ផ&គ' (នក)*
2 x 4 + 8 x , 7b3 + 21b , 8ax 2 − 12a 2 x3 , 10 x3 y 2 − 15 xy 3
6 x 2 − 9 y 2 , 15 x − 20 y 2 , 4 x3 − 2 x 2 + 14 x , 3a 4 + 9a 2 − 15
2 x 3 + 3 x 2 + 4 x , 9 xy − 3 x 2 + 4 xy 2 , 8abc 2 − 4b 2 c + 12a 2bc
6 x 2 yz + 2 xy 2 z − 4 xyz , 12 x 4 y 3t 2 − 4 x 3 yt 2 + 8 x 2t − 16 xy
4/. ច "ក#ក ន ម ង ម$ផ&គ'
ម$ផ&គ' (នក)*
a ( x + 1) + b( x + 1) , z ( y − 3) + 2( y − 3) , ab − 3a + 9b − 27
ab + 7 a + 4b + 28 , xy + 2 x − 7 y − 14 , ap − 2 pk + ya − 2 yk
am − mb − an + nb , 12 xy + 15 x + 4 y + 5 , 2ab − 8a + 3b − 12
3ab + 12a − b − 4 , xy − 8 x − 3 y + 24
5/. ច "ក#ក ន ម ង ម$ផ&គ'
ម$ផ&គ' (នក)*
A = at + bt + ct + 2a + 2b + 2c , B = ax + 2ay + 3az − 4 x − 8 y − 12 z
C = ax + ay − az − bx − by + bz , D = y 2 − cy − ay + ac − by + bc
6/. ច "ក#ក ន ម ង ម$ផ&គ'
ម$ផ&គ' (នក)*
A = ( x + y)2 − z 2 , B = ( x + y )2 − ( z + t )2
C = ( x 2 − 2 xy + y 2 ) − t 2 , D = ( a + b) 2 − ( a − b) 2
7/. ច ក (ម(ន a ម ផ+,ង-+ .
# ម/ព ង ម0
ក. 4 x 2 − 3ay 4 = ( 2 x + 9 y 2 )( 2 x − 9 y 2 )
ខ. 16 x 2 + 5ay 4 = ( 4 x + 5 y 2 )( 4 x − 5 y 2 )

កំែណេ យ ៃហ ហុន
ិ , ៃហ ចរ និងយ៉ត ពនក 24
េមេរ នទី៣ ៖ កេន មពីជគណិត

8/. ច "ក#ក ន ម ង ម$ផ&គ'


ម$ផ&គ' (នក)
(នក)*
x 2 − 16 , y 2 − 121 , y 2 − 1 , 4 z 2 − 49 , 3a 2 − 12 , 25b 2 − 64
36 x 4 − y 2 , 84 x 2 − 21 , 3 x 2 − 75 , 4m2 − 144
8 x 2 − 160 x + 800 , 5 x 2 y 2 − 500 , 3t 2 z 4 − 147 , 2 xy 2 − 32 x
3a 2b 4 − 192a 3 , 5ab 2 − 20a + 30b 2 − 120
4cx 2 − 4c − 12 x 2 + 12 , 2 xy 2 − 2 x + 4 y 2 − 4
9/. ច "ក#ក ន ម ង ម$ផ&គ'
ម$ផ&គ' (នក)*
9( x − 1) 2 − 4(2 x + 3)2 , 4( y + 2) 2 − y 2 , 3( x + 1) − x 2 − 2 x − 1
(2 x − 5) 2 − 4 x 2 + 25 , 16( x + 3)2 − ( x − 1) 2 , x3 − x , t 4 − 1
4 x 2 − 16 x , ( x + 1)2 − ( x 2 − 1) + 2 x + 2 , x5 − x , x( y − 1) 2 − 4 x
10/. ច ក1 # (ម(ន a ម P = 2 x3 + 3x 2 − 8x + 3 .2នង
3 ( x − 1)(2 x + ax − 3)
2


4 "ក# P $ផ&គ' (នក)* 5 កទ1 0
11/. ច "ក#ក ន ម ង ម$ផ&គ'
ម$ផ&គ' (នក)* )ម6ធប1 ពញ នងបន:យ 4 <
)ម6ធគ' =ខ>ង 0
x2 + 5x + 4 , x2 + 5x + 6 , t 2 + 8t + 15 , x 2 − 10 x + 9
t 2 − 11t + 28 , x2 + 7 x − 8 , x 2 + x − 6 , x 2 + 11x − 12
b 2 + 6b − 7 , x 2 + 3x − 4 , y 2 − y − 12 , y 2 − 2 y − 35
n 2 − 4n − 12 , a 2 − 3a − 18 , x 2 − 6 x − 7 , 5t 2 + 12t + 7
2 x 2 + 13 x − 7 , 2 x2 + 5x − 3
12/. ច "ក#ក ន ម ង ម$ផ&គ'
ម$ផ&គ' (នក)*
2a 2 + 24a + 70 , 3x 2 + 21x + 36 , 5a 2 − 15a − 90
2 x 2 − 4 x − 160 , 4bc 2 + 12bc − 40b , 6 xy 4 + 18 xy 2 − 168 x
13/. ច គ ក ន ម.ន?ន ង ម
x+2 2x − 5 y−4
3x − , + 3x − 1 , y +
x 3x 3y + 4
5 5 2n + 3
a −3− , b+5+ , 3n +
a +1 b−5 4n + 5
2x − x + 3
2
14/. ច ក1 # a, b, c = & 0
c
= ax + b +
x −1 x −1
15/. ច គ ក ន ម.ន?ន ង ម
6x −1 2x 10 y − 1 5y 11 x2
− , − , −
81 − x 2 x + 9 100 − y 2 y + 10 121 − x 2 x + 11
t 2 4 x + 16 −4 4z + 2 4
− , + , +
2(t + 3) 3(t + 3) ( x + 4)( x − 4) x + 4 16 − z 2
z−4

កំែណេ យ ៃហ ហុន
ិ , ៃហ ចរ និងយ៉ត ពនក 25
េមេរ នទី៣ ៖ កេន មពីជគណិត

16/. ច . ម@&ក ន ម.ន?ន ង ម


1 3 2 3 1 1 4 8 4 b
− + − +5 −9 +
x y x y x y 2
, , , x , x , a a
4 5 6 5 6 2
− − − 12
y x 5y y y a
3x 7 2b 3 1
+ − 2
x − 49 x + 7 ,
2
b−4 b , a +1 ,
2
1 7 5 3 1 1
− + 2 a− c+
x − 7 2 x + 14 5b − 20 4b − 16b 1 c +1
a+ 1+
a 5−c
2x − 8x
3
17/. គ ក ន ម.ន?ន A = 3 ចគ (ម(ន A ច1 AB x = 2013 0
2 x − 8x2 + 8x

ច មយ
1/. ព កន ម ង ម
A = ( x + 4)( x 2 − 4 x + 1) = x 3 − 4 x 2 + x + 4 x 2 − 16 x + 4
A = x3 − 15 x + 4
B = ( x + 3)( x 2 + 4 x − 3) = x3 + 4 x 2 − 3 x + 3 x 2 + 12 x − 9
B = x3 + 7 x 2 + 9 x − 9
C = (a − b)(a 2 − 2ab − 3b 2 ) = a 3 − 2a 2b − 3ab 2 − ba 2 + 2ab 2 + 3b3
C = a 3 − 3a 2b − ab 2 + 3b3
D = ( x + 2)( x 2 − 5 x − 1) = x3 − 5 x 2 − x + 2 x 2 − 10 x − 2
D = x3 − 3 x 2 − 11x − 2
E = ( y + 5)( y 2 − 3 y − 8) = y 3 − 3 y 2 − 8 y + 5 y 2 − 15 y − 40
E = y 3 + 2 y 2 − 23 y − 40
F = (3n + 4)(n + 5) = 3n 2 + 15n + 4n + 20
F = 3n 2 + 19n + 20
(
G = ( x + 3)( x − 1) 2 = ( x + 3) x 2 − 2 x + 1 )
G = x − 2 x + x + 3x − 6 x + 3 = x + x − 5 x + 3
3 2 2 3 2

(
H = ( y − 1)( y + 2)2 = ( y − 1) y 2 + 4 y + 4 )
H = y3 + 4 y 2 + 4 y − y2 − 4 y − 4 = y3 + 3 y2 − 4
2/. ក a ម
ក. ( x + a)( x − 8) = x 2 − 12 x + 32
គCន
x 2 − 8 x + ax − 8a = x 2 − 12 x + 32
x 2 − ( 8 − a ) x − 8a = x 2 − 12 x + 32
8 − a = 12
 ⇒ a = −4
−8a = 32

កំែណេ យ ៃហ ហុន
ិ , ៃហ ចរ និងយ៉ត ពនក 26
េមេរ នទី៣ ៖ កេន មពីជគណិត

ច នB a = −4 6D Cនក1 #0
ខ. (2 y − 6)( y + a) = 2 y 2 + 8 y − 42
គCន
2 y 2 + 2ay − 6 y − 6a = 2 y 2 + 8 y − 42
2 y 2 + 2 ( a − 3 ) y − 6a = 2 y 2 + 8 y − 42
a − 3 = 4
 ⇒a=7
6a = 42
ច នB a = 7 6D Cនក1 #0
3/. "ក#ក ន ម$ផ&គ' (នក)*
2 x 4 + 8 x = 2 x x3 + 4)(
7b 3 + 21b = 7b ( b + 3) 2

8ax 2 − 12a 2 x3 = 4ax 2 ( 2 − 3ax )


10 x 3 y 2 − 15 xy 3 = 5 xy 2 2 x 2 − 3 y ( )
( )
6 x2 − 9 y 2 = 3 2 x2 − 3 y 2
15 x − 20 y = 5 ( 3 x − 4 y )
2 2

4 x − 2 x + 14 x = 2 x ( 2 x − x + 7 )
3 2 2

3a + 9a − 15 = 3 ( a + 3a − 5 )
4 2 4 2

2 x + 3x + 4 x = x ( 2 x + 3x + 4 )
3 2 2

9 xy − 3 x + 4 xy = x ( 9 y − 3 x + 4 y )
2 2 2

8abc − 4b c + 12a bc = 4bc ( 2ac − b + 3a )


2 2 2 2

6 x 2 yz + 2 xy 2 z − 4 xyz = 2 xyz ( 3 x + y − 2 )
(
12 x 4 y 3t 2 − 4 x 3 yt 2 + 8 x 2t − 16 xy= 4 x 3 x 3 y 3t 2 − x 2 yt 2 + 2 xt − 4 y )
4/. "ក#ក ន ម$ផ&គ' (នក)*
a ( x + 1) + b( x + 1) = ( x + 1)( a + b )
z ( y − 3) + 2( y − 3) = ( y − 3)( z + 2 )
ab − 3a + 9b − 27 = ( ab − 3a ) + ( 9b − 27 ) = a ( b − 3) + 9 ( b − 3) = ( b − 3)( a + 9 )
ab + 7 a + 4b + 28 = a ( b + 7 ) + 4 ( b + 7 ) = ( b + 7 )( a + 4 )
xy + 2 x − 7 y − 14 = x ( y + 2 ) − 7 ( y + 2 ) = ( y + 2 )( x − 7 )
ap − 2 pk + ya − 2 yk = p ( a − 2k ) + y ( a − 2k ) = ( a − 2k )( p + y )
am − mb − an + nb = m ( a − b ) − n ( a − b ) = ( a − b )( m − n )
12 xy + 15 x + 4 y + 5 = 3 x ( 4 y + 5 ) + ( 4 y + 5 ) = ( 4 y + 5 )( 3 x + 1)
2ab − 8a + 3b − 12 = 2a ( b − 4 ) + 3 ( b − 4 ) = ( b − 4 )( 2a + 3 )
3ab + 12a − b − 4 = 3a ( b + 4 ) − ( b + 4 ) = ( b + 4 )( 3a − 1)
xy − 8 x − 3 y + 24 = x ( y − 8 ) − 3 ( y − 8 ) = ( y − 8 )( x − 3)

កំែណេ យ ៃហ ហុន
ិ , ៃហ ចរ និងយ៉ត ពនក 27
េមេរ នទី៣ ៖ កេន មពីជគណិត

5/. "ក#ក ន ម$ផ&គ' (នក)*


A = at + bt + ct + 2a + 2b + 2c = t ( a + b + c ) + 2 ( a + b + c ) = ( a + b + c )( t + 2 )
B = ax + 2ay + 3az − 4 x − 8 y − 12 z = a ( x + 2 y + 3 z ) − 4 ( x + 2 y + 3 z )
= ( x + 2 y + 3 z )( a − 4 )
C = ax + ay − az − bx − by + bz = a ( x + y − z ) − b ( x + y − z ) = ( x + y − z )( a − b )
(
D = y 2 − cy − ay + ac − by + bc = y 2 − by − ay + ( ac − cy + bc ) )
= y ( y − b − a) + c ( a − y + b)
= y ( y − b − a ) − c ( y − b − a ) = ( y − b − a )( y − c )

6/. "ក#ក ន ម$ផ&គ' (នក)*


A = ( x + y ) 2 − z 2 = ( ( x + y ) − z ) ( ( x + y ) + z ) = ( x + y − z )( x + y + z )
B = ( x + y ) 2 − ( z + t )2 = ( ( x + y ) − ( z + t ) ) ( ( x + y ) + ( z + t ) )
= ( x + y − z − t )( x + y + z + t )
C = ( x 2 − 2 xy + y 2 ) − t 2 = ( x − y ) − t 2 = ( ( x − y ) − t ) ( ( x − y ) + t )
2

= ( x − y − t )( x − y + t )
D = ( a + b) 2 − ( a − b) 2 = ( ( a + b ) − ( a − b ) ) ( ( a + b ) + ( a − b ) )
= ( a + b − a + b )( a + b + a − b ) = 4ab

7/. ក (ម(ន a ម ផ+,ង-+ .


# ម/ព ង ម
ក. 4 x 2 − 3ay 4 = ( 2 x + 9 y 2 )( 2 x − 9 y 2 )
គCន
(
4 x 2 − 3ay 4 = 2 x + 9 y 2 2 x − 9 y 2 )( )
4 x 2 − 3ay 4 = ( 2 x ) − 9 y 2 ( )
2 2

4 x 2 − 3ay 4 = 4 x 2 − 81 y 4
⇒ 3a = 81
81
a = = 27
3
ច នB a = 27 6D Cនក1 #0
ខ. 16 x 2 + 5ay 4 = ( 4 x + 5 y 2 )( 4 x − 5 y 2 )
គCន
(
16 x 2 + 5ay 4 = 4 x + 5 y 2 4 x − 5 y 2 )( )
16 x 2 + 5ay 5 = ( 4 x ) − 5 y 2 ( )
2 2

16 x 2 + 5ay 4 = 16 x 2 − 25 y 4
⇒ 5a = −25
25
a=− = −5
5

កំែណេ យ ៃហ ហុន
ិ , ៃហ ចរ និងយ៉ត ពនក 28
េមេរ នទី៣ ៖ កេន មពីជគណិត

ច នB a = −5 6D Cនក1 #0
8/. "ក#ក ន ម$ផ&គ' (នក)
(នក)*
x 2 − 16 = x 2 − 42 = ( x − 4 )( x + 4 )
y 2 − 121 = y 2 − 112 = ( y − 11)( y + 11)
y 2 − 1 = y 2 − 12 = ( y − 1)( y + 1)
4 z 2 − 49 = ( 2 z ) − 7 2 = ( 2 z − 7 )( 2 z + 7 )
2

( )
3a 2 − 12 = 3 a 2 − 22 = 3 ( a − 2 )( a + 2 )

25b 2 − 64 = ( 5b ) − 82 = ( 5b − 8 )( 5b + 8 )
2

( ) − y = ( 6 x − 7 )( 6 x + 7 )
2
36 x 4 − y 2 = 6 x 2 2 2 2

84 x − 21 = 21( 4 x − 1) = 21( 2 x − 1)( 2 x + 1)


2 2

3 x − 75 = 3 ( x − 5 ) = 3 ( x − 5 )( x + 5 )
2 2 2

4m − 144 = 4 ( m − 6 ) = 4 ( m − 6 )( m + 6 )
2 2 2

8 x − 160 x + 800 = 8 ( x − 20 x + 100 ) = 8 ( x − 2 ⋅ x ⋅10 + 10 )


2 2 2 2

= 8 ( x − 10 ) = 8 ( x − 10 )( x − 10 )
2

(
5 x 2 y 2 − 500 = 5 ( xy ) − 102 = 5 ( xy − 10 )( xy + 10 )
2
)
((
3t 2 z 4 − 147 = 3 tz 2 )
2
)
− 7 2 = 3 tz 2 − 7 tz 2 + 7 ( )( )
(
2 xy 2 − 32 x = 2 x y 2 − 42 = 2 x ( y − 4 )( y + 4 ) )
3a 2b 4 − 192a 3 = 3a 2 b 4 − 64a ( )
5ab 2 − 20a + 30b 2 − 120 = 5a ( b 2
) ( ) (
− 4 + 30 b 2 − 4 = b 2 − 4 ( 5a + 30 ) )
= 5 ( a + 6 )( b − 2 )( b + 2 )
(
4cx 2 − 4c − 12 x 2 + 12 = 4c x 2 − 1 − 12 x 2 − 1 = x 2 − 1 ( 4c − 12 ) ) ( ) ( )
= 4 ( x − 1)( x + 1)( c − 3)
(
2 xy 2 − 2 x + 4 y 2 − 4 = 2 x y 2 − 1 + 4 y 2 − 1 = y 2 − 1 ( 2 x + 4 ) ) ( ) ( )
= 2 ( y − 1)( y + 1)( x + 2 )

9/. "ក#ក ន ម$ផ&គ' (នក)*


9( x − 1) 2 − 4(2 x + 3)2 = ( 3 ( x − 1) ) − ( 2 ( 2 x + 3) )
2 2

= ( 3 ( x − 1) − 2 ( 2 x + 3) ) ( 3 ( x − 1) + 2 ( 2 x + 3) )
= ( 3 x − 3 − 4 x − 6 )( 3 x − 3 + 4 x + 6 ) = ( − x − 9 )( 7 x + 3)
4( y + 2) 2 − y 2 = ( 2 ( y + 2 ) ) − y 2 = ( 2 ( y + 2 ) − y ) ( 2 ( y + 2 ) + y )
2

= ( 2 y + 4 − y )( 2 y + 4 + y ) = ( y + 4 )( 3 y + 4 )
3( x + 1) − x 2 − 2 x − 1 = 3 ( x + 1) − ( x + 1) = ( x + 1) ( 3 − ( x + 1) ) = ( x + 1)( − x + 2 )
2

(2 x − 5) 2 − 4 x 2 + 25 = 4 x 2 − 20 x + 25 − 4 x 2 + 25 = −20 x + 50 = 10 ( −2 x + 5)

កំែណេ យ ៃហ ហុន
ិ , ៃហ ចរ និងយ៉ត ពនក 29
េមេរ នទី៣ ៖ កេន មពីជគណិត

16( x + 3) 2 − ( x − 1)2 = ( 4 ( x + 3) ) − ( x − 1) = ( 4 ( x + 3) − ( x − 1) ) ( 4 ( x + 3) + ( x − 1) )
2 2

= ( 4 x + 12 − x + 1)( 4 x + 12 + x − 1) = ( 3 x + 13)( 5 x + 11)


( )
x3 − x = x x 2 − 1 = x ( x − 1)( x + 1)

t4 −1 = (t )
2 2
( )( )
− 12 = t 2 − 1 t 2 + 1 = ( t − 1)( t + 1) t 2 + 1 ( )
4 x − 16 x = 4 x ( x − 4 )
2

( x + 1) 2 − ( x 2 − 1) + 2 x + 2 = ( x + 1) − ( x − 1)( x + 1) + 2 ( x + 1)
2

= ( x + 1)( x + 1 − x + 1 + 2 ) = 4 ( x + 1)
( )
x5 − x = x x 4 − 1 = x ( x − 1)( x + 1) x 2 + 1 ( )
( )
x( y − 1) 2 − 4 x = x ( y − 1) − 4 = x ( y − 1 − 2 )( y − 1 + 2 ) = x ( y − 3)( y + 1)
2

10/. ក1 # (ម(ន a
យងEន P = 2 x3 + 3x 2 − 8x + 3 នង ( x − 1)(2 x 2 + ax − 3)
"យ P = ( x − 1) ( 2 x 2 + ax − 3) គCន
2 x 3 + 3 x 2 − 8 x + 3 = 2 x3 + ax 2 − 3 x − 2 x 2 − ax + 3
2 x 3 + 3x 2 − 8 x + 3 = 2 x3 + ( a − 2 ) x 2 − ( 3 + a ) x + 3
a − 2 = 3
 ⇒a=5
3 + a = 8
ច នB a = 5 $ច1នន
4 = & D6Cនក1 #0

4 "ក# P $ផ&គ' (នក)* 5 កទ1
ច1 AB a = 5 គCន
( ) (
P = ( x − 1) 2 x 2 + 5 x − 3 = ( x − 1) 2 x 2 + 6 x − x − 3 )
P = ( x − 1) ( 2 x ( x + 3) − ( x + 3) )
P = ( x − 1)( x + 3)( 2 x − 1)

11/. "ក#ក ន ម$ផ&គ' (នក)* )ម6ធប1 ពញ នងបន:យ 4 <


)ម6ធគ' =ខ>ង
( )
x 2 + 5 x + 4 = x 2 + 4 x + ( x + 4 ) = x ( x + 4 ) + ( x + 4 ) = ( x + 4 )( x + 1)
x + 5 x + 6 = ( x + 2 x ) + ( 3 x + 6 ) = x ( x + 2 ) + 3 ( x + 2 ) = ( x + 2 )( x + 3)
2 2

t + 8t + 15 = ( t + 3t ) + ( 5t + 15 ) = t ( t + 3) + 5 ( t + 3) = ( t + 3)( t + 5 )
2 2

x − 10 x + 9 = ( x − 10 x + 25 ) + 9 − 25 = ( x − 5 ) − 4 = ( x − 5 − 4 )( x − 5 + 4 )
2 2 2 2

= ( x − 9 )( x − 1)
( )
t 2 − 11t + 28 = t 2 − 4t − ( 7t − 28 ) = t ( t − 4 ) − 7 ( t − 4 ) = ( t − 4 )( t − 7 )
x2 + 7 x − 8 = ( x + 8 x ) − ( x + 8 ) = x ( x + 8 ) − ( x + 8 ) = ( x + 8 )( x − 1)
2

កំែណេ យ ៃហ ហុន
ិ , ៃហ ចរ និងយ៉ត ពនក 30
េមេរ នទី៣ ៖ កេន មពីជគណិត

( )
x 2 + x − 6 = x 2 + 3 x − ( 2 x + 6 ) = x ( x + 3) − 2 ( x + 3) = ( x + 3)( x − 2 )

( )
x 2 + 11x − 12 = x 2 + 12 x − ( x + 12 ) = x ( x + 12 ) − ( x + 12 ) = ( x + 12 )( x − 1)

( )
b 2 + 6b − 7 = b 2 + 7b − ( b + 7 ) = b ( b + 7 ) − ( b + 7 ) = ( b + 7 )( b − 1)
x2 + 3x − 4 = ( x 2
+ 4 x ) − ( x + 4 ) = x ( x + 4 ) − ( x + 4 ) = ( x + 4 )( x − 1)
y2 − y − 12 = ( y 2
− 4 y ) + ( 3 y − 12 ) = y ( y − 4 ) + 3 ( y − 4 ) = ( y − 4 )( y + 3)

y 2 − 2 y − 35 = ( y − 1) − 6 2 = ( y − 1 − 6 ) ( y − 1 + 6 ) = ( y − 7 )( y + 5 )
2

n 2 − 4n − 12 = ( n − 2 ) − 42 = ( n − 2 − 4 )( n − 2 + 4 ) = ( n − 6 )( n + 2 )
2

( )
a 2 − 3a − 18 = a 2 − 6a + ( 3a − 18 ) = a ( a − 6 ) + 3 ( a − 6 ) = ( a − 6 )( a + 3)

x 2 − 6 x − 7 = ( x − 3) − 42 = ( x − 3 − 4 )( x − 3 + 4 ) = ( x − 7 )( x + 1)
2

( )
5t 2 + 12t + 7 = 5t 2 + 5t + ( 7t + 7 ) = 5t ( t + 1) + 7 ( t + 1) = ( t + 1)( 5t + 7 )

( )
2 x 2 + 13 x − 7 = 2 x 2 + 14 x − ( x + 7 ) = 2 x ( x + 7 ) − ( x + 7 ) = ( x + 7 )( 2 x − 1)

( )
2 x 2 + 5 x − 3 = 2 x 2 + 6 x − ( x + 3) = 2 x ( x + 3) − ( x + 3) = ( x + 3)( 2 x − 1)

12/. "ក#ក ន ម$ផ&គ' (នក)*


( ) (
2a 2 + 24a + 70 = 2 a 2 + 12a + 35 = 2 ( a + 6 ) − 1 = 2 ( a + 6 − 1)( a + 6 + 1)
2
)
= 2 ( a + 5 )( a + 7 )

( ) ((
3 x 2 + 21x + 36 = 3 x 2 + 7 x + 12 = 3 x 2 + 4 x + ( 3 x + 12 ) ) )
= 3 ( x ( x + 4 ) + 3 ( x + 4 ) ) = 3 ( x + 4 )( x + 3 )

(
5a 2 − 15a − 90 = 5 a 2 − 3a − 18 = 5 a 2 − 6a + ( 3a − 18 ) ) (( ) )
= 5 ( a ( a − 6 ) + 3 ( a − 6 ) ) = 5 ( a − 6 )( a + 3)

(
2 x 2 − 4 x − 160 = 2 x 2 − 2 x − 80 = 2 ( x − 1) − 9 2 ) ( 2
)
= 2 ( x − 1 − 9 )( x − 1 + 9 ) = 2 ( x − 10 ) ( x + 8 )

(
4bc 2 + 12bc − 40b = 4b c 2 + 3c − 10 = 4b c 2 + 5c − ( 2c + 10 ) ) (( ) )
= 4b ( c + 5 )( c − 2 )

(
6 xy 4 + 18 xy 2 − 168 x = 6 x y 4 + 3 y 2 − 28 = 6 x ) ( ) (( y 4
+ 7 y 2 − 4 y 2 + 28 ))
= 6 x ( y ( y + 7 ) − 4 ( y + 7 ) ) = 6 x ( y + 7 )( y
2 2 2 2 2
−4 )
= 6 x ( y + 7 ) ( y − 2 )( y + 2 )
2

13/. គ ក ន ម.ន?ន ង ម
x + 2 3x − ( x + 2 ) 3x 2 − x − 2
2

3x − = =
x x x
2x − 5 2 x − 5 + ( 3 x − 1) 3 x 2 x − 5 + 9 x 2 − 3 x 9 x 2 − x − 5
+ 3x − 1 = = =
3x 3x 3x 3x
y − 4 y (3 y + 4) + y − 4 3 y2 + 4 y + y − 4 3 y 2 + 5 y − 4
y+ = = =
3y + 4 3y + 4 3y + 4 3y + 4

កំែណេ យ ៃហ ហុន
ិ , ៃហ ចរ និងយ៉ត ពនក 31
េមេរ នទី៣ ៖ កេន មពីជគណិត

5 ( a − 3)( a + 1) − 5 a 2 − 2a − 8
a −3− = =
a +1 a +1 a +1
5 ( b + 5)( b − 5 ) + 5 b − 25 + 5 b 2 − 20
2
b+5+ = = =
b−5 b−5 b−5 b −5
2n + 3 3n ( 4n + 5 ) + 2n + 3 12n + 15n + 2n + 3 12n 2 + 17 n + 3
2
3n + = = =
4n + 5 4n + 5 4n + 5 4n + 5
14/. ក1 # a, b, c
2 x2 − x + 3
យងEន គCន
c
= ax + b +
x −1 x −1
2 x 2 − x + 3 ( ax + b )( x − 1) + c
=
x −1 x −1
2 x − x + 3 ax − ax + bx − b + c
2 2
=
x −1 x −1
2 x 2 − x + 3 ax + ( − a + b ) x + ( −b + c )
2

=
x −1 x −1
a = 2 a = 2
 
− a + b = −1 ⇔ b = −1 + a = −1 + 2 = 1
 −b + c = 3 c = 3 + b = 3 + 1 = 4
 
ច នB a = 2 , b = 1, c = 4 6D Cនក1 #0
15/. គ ក ន ម.ន?ន ង ម
6x −1 2x 6x −1 2x 6x −1 − 2x (9 − x )
− = − =
81 − x 2
x + 9 ( 9 − x )( 9 + x ) x + 9 ( 9 − x )( 9 + x )
6 x − 1 − 18 x + 2 x 2 2 x 2 − 12 x − 1
= =
( 9 − x )( 9 + x ) ( 9 − x )( 9 + x )
10 y − 1 5y 10 y − 1 5y 10 y − 1 − 5 y (10 − y )
− = − =
100 − y 2 y + 10 (10 − y )(10 + y ) y + 10 (10 − y )(10 + y )
10 y − 1 − 50 y + 5 y 2 5 y 2 − 40 y − 1
= =
(10 − y )(10 + y ) (10 − y )(10 + y )
11 x2 11 x2 11 − x 2 (11 − x )
− = − =
121 − x 2 x + 11 (11 − x )(11 + x ) x + 11 (11 − x )(11 + x )
11 − 11x 2 + x3 x 3 − 11x 2 + 11
= =
(11 − x )(11 + x ) (11 − x )(11 + x )
t 2 3t − 4
− =
2(t + 3) 3(t + 3) 6 ( t + 3)
4 x + 16 −4 4 x + 16 − 4 ( x − 4 ) 4 x + 16 − 4 x + 16 32
+ = = =
( x + 4)( x − 4) x + 4 ( x + 4)( x − 4) ( x + 4)( x − 4) ( x + 4)( x − 4)
4z + 2 4 4z + 2 4 4z + 2 − 4(4 + z )
+ = − =
16 − z 2
z − 4 ( 4 − z )( 4 + z ) 4 − z ( 4 − z )( 4 + z )
4 z + 2 − 16 − 4 z −14
= =
( 4 − z )( 4 + z ) ( 4 − z )( 4 + z )

កំែណេ យ ៃហ ហុន
ិ , ៃហ ចរ និងយ៉ត ពនក 32
េមេរ នទី៣ ៖ កេន មពីជគណិត

16/. . ម@&ក ន ម.ន


ម.ន?ន ង ម
1 3 y − 3x

x y xy y − 3x y y − 3x
= = ⋅ = , ( y ≠ 0)
4 4 xy 4 4x
y y
2 3 2 y + 3x
+
x y xy 2 y + 3x  x  2 y + 3x
= = ⋅ −  = − , ( x ≠ 0)
5 5 xy  5 5y
− −
x x
1 1 y−x

x y xy y − x  5y  5( x − y)
= = ⋅ −  = , ( y ≠ 0)
6 6 xy  6  6 x
− −
5y 5y
4 4 + 5x
+5
x 4 + 5x y y ( 4 + 5x )
= x = ⋅ =
5 5 x 5 5x
y y
8 8 − 9x
−9
x 8 − 9 x y y (8 − 9 x )
= x = ⋅ =
6 6 x 6 6x
y y
4 b 4 + ba
+
a 2
a = a 2 = 4 + ba ⋅ a = 4 + ba , ( a ≠ 0)
2
− 12
2 − 12a a 2
2 − 12 a 2 a ( 1 − 6 a )
a a
3x 7 3x + 7 ( x − 7 ) 3 x + 7 x − 49
+
x 2 − 49 x + 7 = ( x − 7 )( x + 7 ) = ( x − 7 )( x + 7 )
1 7 1 7 2( x + 7) − 7 ( x − 7)
− −
x − 7 2 x + 14 x − 7 2 ( x + 7 ) 2 ( x − 7 )( x + 7 )
10 x − 49 2 ( x − 7 )( x + 7 ) 2 (10 x − 49 )
= ⋅ = , ( x ≠ ±7 )
( x − 7 )( x + 7 ) −5 x + 63 −5 x + 63

2b 3 2b3 − 3 ( b − 4 ) 2b3 − 3b + 12
− b2 (b − 4) b2 ( b − 4 )
b − 4 b2 = =
5 3 1 3 4b + 3
+ 2 +
5b − 20 4b − 16b b − 4 4b ( b − 4 ) 4b ( b − 4 )

= 2 ⋅ =
(
2b3 − 3b + 12 4b ( b − 4 ) 4 2b − 3b + 12
3
) , ( b ≠ 0, b ≠ 4 )
b (b − 4) 4b + 3 b ( 4b + 3)
1 1 1 1
a +1 = a +1 = a +1 = a +1 1 a2 +1 a2 +1
= ⋅ 3 = 3
a−
1
a− 2
1
a− 2
a a a2 +1 − a a +1 a (
a ( a + 1) )
a+
1 a +1 a +1 a2 + 1
a a

កំែណេ យ ៃហ ហុន
ិ , ៃហ ចរ និងយ៉ត ពនក 33
េមេរ នទី៣ ៖ កេន មពីជគណិត

2 2 2 2 12 12
= = = = =
1 1 5 − c 6c + 5 − c 5c + 5 5 ( c + 1)
c+ c+ c+
c +1 5 − c + c +1 6 6
1+
5−c 5−c
17/. គ (ម(ន A ច1 AB x = 2013
2 x3 − 8 x
យងEន, ក ន ម.ន?ន A = គCន
2 x3 − 8 x 2 + 8 x

A=
2 x3 − 8 x
=
(
2 x x2 − 4 )
(
2 x3 − 8 x 2 + 8 x 2 x x 2 − 4 x + 4 )
A=
( x − 2 )( x + 2 ) = x + 2
( x − 2) x−2
2

2013 + 2 2015
ច1 AB x = 2013 គCន A= =
2013 − 2 2011
ច នB A = 6D Cនក1 #0
2015
2011

... ងរហ័
ងរហ័ស រួចរត់េ េលើក ប ង
ំ នឹក តិចខូច េឡបថប ខ!េ
"ំ

រួចក៏េមើលម៉ត
ូ ូ េតើ&នខូចអីខះ) ? ែតេពលយប់ េមើលមិនច/ស់

0ស់េទ េហើយកំពង
ុ ែតឈឺ ផង ។ ខ!ក
"ំ ប
៏ េ78ះ9 េឡើង េហើយ:ក់ទក

;ក<ច និង ប រួចបន=ដេ


ំ ណើរេ េទៀត ។ េពលេ ដល់មខ
ុ ធBCរ

េអសុល
ី :
ី ផDរសំេEងេ ង ;Fប់ែតទឹក;ក<ច ៣, ៤ កំបង
៉ុ JKក់

េចញពីេកះ;ក:ស េ េលើផវ
)M េហើយJKយOញ់ទក
ឹ េទើបខ!ដ
"ំ ង

&នទឹក;ក<ចខ)ះJKយ ង
ំ ពីដល
ួ មកេម)ះ៉ េ ះបី9េPកQង
" េកះក៏េ:យ

េហើយេពលេBះ9Oន;Rបទឹ
េពលេBះ9Oន;Rបទឹកមក េហើយេធSឲ
ើ Uេកះេសើម ក៏JយOត
K

JKក់េ ដី ខ!ក
"ំ ប
៏ ន=េ មុខ Oនបន=ច
ិ ដល់មខ
ុ EVសFំង ផ)វ
M េ ; W
ំ រX

9ក៏បន=Jក់
K Oញ់ YZស េពញ ។ ខ!ក
"ំ ស
៏ េ;មចចិតេ
= សុទ
ំ ញ
ិ ថង់មយ

ពីអក
Q អ៊ត
ុ សក់&ក់
X េPWងេកើតផ)វ
M ែដលេពលេBះCត់កព
ំ ង
ុ ែត

េរៀបចំបទ
ិ រ\ Cត់&នចិ
&នចិតល
= ] េរ^ថង់មយ
ួ ឲUខ!"ំ េហើយអត់យកលុយេទ។

ខ!ក
"ំ អ
៏ រគុណCត់ េហើយយកមកេរ^ទក
ឹ ;ក<ច`ែដលេPល]:ក់កង
Q"

ថង់ Oន;ប&ណR ១២ កំបង


៉ុ ។ រួចក៏បន=េ ដល់ផះc េ ។ េពលខ!"ំ

េ ដល់ផះc កូន ខ!"ំ សំែដង dរមeណស


៍ ប/យរ^
ប/យរ^កEយ ។ េពលែដលេគ

ដឹង ខ!&
"ំ នេ;Cះ ក់
X េគក៏;Oប់ខ!"ំ គឺមកពី Oឯងមិន;បយ័តQ ៕

កំែណេ យ ៃហ ហុន
ិ , ៃហ ចរ និងយ៉ត ពនក 34
េមេរ នទី៤ ៖ សមី រដេឺ កទី១ នមួយអ ត

ម នទ ម កទ នមយ

1/. ច យ ម ង ម
ក. 4( x − 5) = 2 x − 14 ខ. x + 7 = −2( x + 8)
គ. 3(2 y − 1) = −8(6 + y ) ឃ. −( z + 12) = 5(2 z − 1)
ង. n − (8 + 4n) = 2(3n − 4) ច. (2m + 3) 2 − (2m + 5) = 4m(m + 4) + 4
ឆ. 5 p − (8 − p) = 2 [ −4 − (3 + 5 p) − 13] ជ. − [ k − (4k + 2) ] = 2 + (2k + 7)
ឈ. 2 − 4 {−2 x + 3[ −1 + 5(1 − x) ] − 1} = 0 ញ. 3 ( x − 2 ) − 2 x 2 = 2 ( x 3 − 3 − x 2 )
. 6(7 x + 8) − 7(6 x + 4) = 5(6 + 7 x) − 35( x − 1)
(. 8(3 − 2 p) − 16(1 − p ) − 3( p − 4) = 20 − 3 p
2/. ច យ ម ង ម
2 y +1 5 − y 3y + 2
ក. ខ.
5x 9 7 x 3
+ = + + = 1−
3 8 6 16 3 4 6
2(3 z − 8) 5( z − 3) 3( z − 8) 17 3t − 5 t + 12
គ. − = + ឃ. + =1
9 3 4 36 8 12
x+4 x−4 3x − 1 1− n
ង. ច. 1 + − − n =
2n 1
− = 2+
3 5 15 3 2 3
5k − 3 3( x − 1.6) 8.4 x − 4.9
ឆ. 3k + = 37 ជ. 2.4 x + =
2 5 2
ឈ.  x −  +  x + 0.6  −  x − 0.3  = 5.8
3 2 2 7
4 5 3   12 
3 4x − 3 5 x − 12 2x −1
ញ. x + − = 1− . 8( x − 2) + 3 = 7 x + −3
2 4 6 2
(2 y + 1) 2 3( y + 2) 2 y ( y − 2) ( z − 2)2 z ( z − 3) (8 z + 7)( z − 1) 4
(. − = ). + = +
4 6 2 3 5 15 5
3/. ច យ ម ង ម
ក. x( x + 3) = 0 ខ. ( x + 5)( x − 1) = 0
គ. ( x − 1) 2 − 4 = 0 ឃ. 4( x − 1) 2 − (2 x − 3)2 = 0
ង. x 2 − 6 x = 0 ច. 2 x 2 − 2 = 0
4/. ច យ ម ង ម
ក. x 2 − 2 x + 3 = 0 ខ. x 2 − 12 x + 32 = 0

កំែណេ យ ៃហ ហុន
ិ , ៃហ ចរ និងយ៉ត ពនក 35
េមេរ នទី៤ ៖ សមី រដេឺ កទី១ នមួយអ ត

គ. y 2 − 14 = 5 y ឃ. x 2 − x = 3x + 12
5/. * មយ បក
, ប-. យទក
/ ពក1ពង23ផ A 5ក1ពង23ផ B 3 6 នច 7 យ
57km 8 ព6 * បក
, 62ច*
1 9 ចមយ គ ;: ឈប2 ក 2 1 / 3h
<យ
, 5 62កព
1 ង23ផ B ;: ច1-យ ព6 2 10 / 3h 8 គ ង
/ =ច ន.
ទក
/ ន 6>?ន 2km / h 8 ច គ*@ 6>?ន * AB62 8
6/. CទD E ច កមយ គ?ង ធG, 1 *, ចញពH
គ?ង Iន J ន ន. K ភ
L N1 ពញ OP
5ទ ម ខ . មQប យ 6>?ន 288km / h 8 20 @ទ យមកយន. R
មយ គ?ង ទ Sន R ចញពក3នTង3 មយ 5 ន2ទ ម ខ . មQប3
កN9ង 6>?ន 480km / h 8 យ
, ន. R ន R 2 យU ព6បV@
9 W ន ទប
, Xម
Yន2CទD E ច ក ?

7/. គយក ; [ 280 ក\6 53ចក 62 L ]មយ=Nក2 8 យ ព63ចក


;[ 2 គ ង^ ឃញ
, = L ] N ក2_ទទ6Sន ; [ 4 ក\6 8
ច ក1* ច
2 ន
1 ន L ]កN9ង =Nក2 @ 8
8/. ក
N ជL កង2 N ក2 ចញពទ ក`ង A 5ទ ក`ង B 3 6 នច 7 យ 50km 8 មយ
Vង យមក
យមក ក
N ជL មV N ក2 ចញពទ ក`ង A 3 <យ
, 5 62ទ ក`ង B
ម9ន ក
N ជL កង2មយ V ង 8 ច គ*@ 6>?នaនIនY1ងព ប, គ ង
/ =
6>?នមV ,ន
W ង
/ 2.5 ងaន 6>?នកង2 8
9/. គយក 69 យ b9ងH 9 40% 3 6 ន ) 20 ℓ 5cយKមយ
69 យ b9ងH 9 70% 8 , គ ;: cយ 69 យ b9ង 70% បV@
9 W ន6

, >Sន6\យមយ3 6 នH
នH 9 50% ?

10/. ក
N 9 ; Sនយក
. Sក2មយច1នន 5ចង 8 មយ3ផNកaន Sក2 ន d 2
ទទ6Sន X Sក2 9% នង
L មយ3ផNក ទ 3 6 6, ព3ផNកម9ន
8 000 000 e ទទ6Sន X Sក2 10% 8 កN9ងមយONd
1 ទ
2 ទ6Sន
Sក2Yង
1 2ចន
1 ន 4 600 000 e 8 d
, 2 ន Sក2 ម
, កN9ង X Sក2

កំែណេ យ ៃហ ហុន
ិ , ៃហ ចរ និងយ៉ត ពនក 36
េមេរ នទី៤ ៖ សមី រដេឺ កទី១ នមួយអ ត

នមយ_បV@
9 Wន 6?
11/. =Nក2 នមយ ន L ] ប` 30 @ក2 កN9ង @ L ] 22 @ក2 6ងS62Y 2
<យ
, L ] 17 @ក2 6ងS62 S ន L ] 03 @ក2មន
L 6ងS62 S
នង
L S62Y 2 8 F B
17 − x
1 *91 F Xងfb ក
N 6ងS62Y 2 22 − x x

1 *91 B Xងfb ក
N 6ងS62 S 3

1 *91 F ∩ B Xងfb ក
N 6ងS62Y 2 នង
L S62 S 8
ច កច1នន L ]3 6 6ងS62Y ផ
2 ង នង
L S62 S ផង 8
12/. កN9ង បគ62 ម យ 5fbកgក Sន ប ព/ . 5Xម61 ប2 ច
ង ម
aថ7ទ1 គSន បគ62 2iក2ក-^ 6aន ម យ
aថ7ទ 2 គSន បគ62 2iក2ក-^ 6aន ម យ3 6 62ពaថ7ទ1
aថ7ទ 3 គSន បគ62 2iក2ក-^ 6aន ម យ3 6 62ពaថ7ទ 2
aថ7ទ 4 គSន បគ62 2iក2ក-^ 6aន ម យ3 6 62ពaថ7ទ 3
<យ
, j 62 ម យ 2 8 ច
, ន
1 នម យ ប
9 នច1ននបV@
9 Wន?
13/. ទ ពBa មយ6ក2
មយ6ក2បkl9 aមT 30% aន aមT3 66ក2ពម9ន 8 យគL Y1ង
ពនDHក ប3នmម 10% aនaថT6ក2 ,ព 9 ខ ;: បង2aថT ប
9 2 Sក2
184800 e 8 ច ក aមT3 6Sន
6Sន6ក2
Sន6ក2ពម9ន 8

ច មយ
1/. យ ម ង ម
ក. 4( x − 5) = 2 x − 14
គSន,
4( x − 5) = 2 x − 14
4 x − 20 = 2 x − 14
4 x − 2 x = −14 + 20
6
x= =3
2
ច ន ច មTយ
, ប 2 ម គ x=3 8
ខ. x + 7 = −2( x + 8)
គSន,

កំែណេ យ ៃហ ហុន
ិ , ៃហ ចរ និងយ៉ត ពនក 37
េមេរ នទី៤ ៖ សមី រដេឺ កទី១ នមួយអ ត

x + 7 = −2( x + 8)
x + 7 = −2 x − 16
3 x = −23
23
x=−
3
ច ន ច មTយ
, ប 2 ម គ x=− 8
23
3
គ. 3(2 y − 1) = −8(6 + y )
គSន,
3(2 y − 1) = −8(6 + y )
6 y − 3 = −48 − 8 y
14 y = −45
45
y=−
14
ច ន ច មTយ
, ប 2 ម គ y=− 8
45
14
ឃ. −( z + 12) = 5(2 z − 1)
គSន,
−( z + 12) = 5(2 z − 1)
− z − 12 = 10 z − 5
− 7 = 11z
7
z=−
11
ច ន ច មTយ
, ប 2 ម គ z=− 8
7
11
ង. n − (8 + 4n) = 2(3n − 4)
គSន,
n − (8 + 4n) = 2(3n − 4)
n − 8 − 4 n = 6n − 8
− 3n = 6n
n=0
ច ន ច មTយ
, ប 2 ម គ n=0 8
ច. (2m + 3) 2 − (2m + 5) = 4m(m + 4) + 4
គSន,
(2m + 3)2 − (2m + 5) = 4m(m + 4) + 4
4m 2 + 12m + 9 − 2m − 5 = 4m 2 + 16m + 4
10m + 4 = 16m + 4
m=0
ច ន ច មTយ
, ប 2 ម គ m=0 8

កំែណេ យ ៃហ ហុន
ិ , ៃហ ចរ និងយ៉ត ពនក 38
េមេរ នទី៤ ៖ សមី រដេឺ កទី១ នមួយអ ត

ឆ. 5 p − (8 − p) = 2 [ −4 − (3 + 5 p) − 13]
គSន,
5 p − (8 − p ) = 2 [ −4 − (3 + 5 p ) − 13]
5 p − 8 + p = 2 ( −4 − 3 − 5 p − 13)
6 p − 8 = −10 p − 40
16 p = −32
32
p=− = −2
16
ច ន ច មTយ
, ប 2 ម គ p = −2 8
ជ. − [ k − (4k + 2) ] = 2 + (2k + 7)
គSន,
− [ k − (4k + 2) ] = 2 + (2k + 7)
− k + 4 k + 2 = 2 + 2k + 7
k =7
ច ន ច មTយ
, ប 2 ម គ k =7 8
ឈ. 2 − 4 {−2 x + 3[ −1 + 5(1 − x) ] − 1} = 0
គSន,
2 − 4 {−2 x + 3 [ −1 + 5(1 − x) ] − 1} = 0
1 − 2 ( −2 x − 3 + 15 − 15 x − 1) = 0
1 − 2 (11 − 17 x ) = 0
1 − 22 + 34 x = 0
34 x = 21
21
x=
34
ច ន ច មTយ
, ប 2 ម គ x= 8
21
34
ញ. 3 ( x − 2 ) − 2 x 2 = 2 ( x 3 − 3 − x 2 )
គSន,
(
3 ( x − 2) − 2x 2 = 2 x 3 − 3 − x 2 )
3x − 2 3 − 2 x 2 = 2 x 3 − 6 − 2 x 2
−2 3+6= x 3

x=
−2 3 + 6
=
(
−2 3 + 6 ) 3

( )
2
3 3

−6 + 6 3
x= = −2 + 2 3
3

កំែណេ យ ៃហ ហុន
ិ , ៃហ ចរ និងយ៉ត ពនក 39
េមេរ នទី៤ ៖ សមី រដេឺ កទី១ នមួយអ ត

ច ន ច មTយ
, ប 2 ម គ x = −2 + 2 3 8
. 6(7 x + 8) − 7(6 x + 4) = 5(6 + 7 x) − 35( x − 1)
គSន,
6(7 x + 8) − 7(6 x + 4) = 5(6 + 7 x) − 35( x − 1)
42 x + 48 − 42 x − 28 = 30 + 35 x − 35 x + 35
20 = 65 មន
L ពL
ចន ម នច
dWនច មTយ
, 8
(. 8(3 − 2 p) − 16(1 − p ) − 3( p − 4) = 20 − 3 p
គSន,
8(3 − 2 p ) − 16(1 − p ) − 3( p − 4) = 20 − 3 p
24 − 16 p − 16 + 16 p − 3 p + 12 = 20 − 3 p
− 3 p = −3 p
0 = 0 ផBoងAB 2 គប2 aមT p

ចន ម នច
នច មTយ
, ចន
, Qប2មន
L 28
2/. យ ម ង ម
ក.
5x 9 7 x 3
+ = +
3 8 6 16
គSន,
5x 9 7 x 3
+ = +
3 8 6 16
5x 7 x 3 9
− = −
3 6 16 8
10 x − 7 x 3 − 18
=
6 16
3x 15
=−
3 8
15
x=−
8
ច ន ច មTយ , ប 2 ម គ x=− 8 8
15

2 y +1 5 − y 3y + 2
ខ. + = 1−
3 4 6
គSន,
2 y +1 5 − y 3y + 2
+ = 1−
3 4 6
4 ( 2 y + 1) + 3 ( 5 − y ) = 12 − 2 ( 3 y + 2 )
8 y + 4 + 15 − 3 y = 12 − 6 y − 4
11 y = −11
y = −1

កំែណេ យ ៃហ ហុន
ិ , ៃហ ចរ និងយ៉ត ពនក 40
េមេរ នទី៤ ៖ សមី រដេឺ កទី១ នមួយអ ត

ច ន ច មTយ
, ប 2 ម គ y = −1 8
2(3 z − 8) 5( z − 3) 3( z − 8) 17
គ. − = +
9 3 4 36
គSន,
2(3 z − 8) 5( z − 3) 3( z − 8) 17
− = +
9 3 4 36
8 ( 3 z − 8 ) − 60 ( z − 3) = 27 ( z − 8 ) + 17
24 z − 64 − 60 z + 180 = 27 z − 216 + 17
− 63 z = −315
315
z= =5
63
ច ន ច មTយ
, ប 2 ម គ z=5 8
3t − 5 t + 12
ឃ. + =1
8 12
គSន,
3t − 5 t + 12
+ =1
8 12
3 ( 3t − 5 ) + 2 ( t + 12 ) = 24
9t − 15 + 2t + 24 = 24
11t = 15
15
t=
11
ច ន ច មTយ
, ប 2 ម គ t = 11 8
15

x+4 x−4 3x − 1
ង. − = 2+
3 5 15
គSន,
គSន,
x+4 x−4 3x − 1
− = 2+
3 5 15
5 x + 20 − 3 x + 12 = 30 + 3 x − 1
3= x
ច ន ច មTយ
, ប 2 ម គ x=3 8
1− n
ច. 1 +
2n 1
− −n =
3 2 3
គSន,
2n 1 1− n
1+ − −n =
3 2 3
1 1 − n 2n
= n+ −
2 3 3
1 1
= n+ −n
2 3

កំែណេ យ ៃហ ហុន
ិ , ៃហ ចរ និងយ៉ត ពនក 41
េមេរ នទី៤ ៖ សមី រដេឺ កទី១ នមួយអ ត

= មន
L Hច
1 1
2 3
ចន ម នច
dWនច មTយ
, 8
5k − 3
ឆ. 3k + = 37
2
គSន,
5k − 3
3k + = 37
2
6k + 5k − 3 = 74
11k = 77
77
k= =7
11
ច ន ច មTយ
, ប 2 ម គ k =7 8
3( x − 1.6) 8.4 x − 4.9
ជ. 2.4 x + =
5 2
គSន,
3( x − 1.6) 8.4 x − 4.9
2.4 x + =
5 2
24 x + 6 ( x − 1.6 ) = 5 ( 8.4 x − 4.9 )
24 x + 6 x − 9.6 = 42 x − 24.5
− 12 x = −14.9
14.9 149
x= =
12 120
ច ន ច មTយ
, ប 2 ម គ x = 120 8
149

ឈ.  x −  +  x + 0.6  −  x − 0.3  = 5.8


3 2 2 7
4 5 3   12 
គSន,
3 2 2  7 
 4 x − 5  +  3 x + 0.6  −  12 x − 0.3  = 5.8
     
3 2 2 7
x − + x + 0.6 − x + 0.3 = 5.8
4 5 3 12
9 x + 8x − 7 x 2
= 5.8 − 0.3 − 0.6 +
12 5
10 x
= 5.3
12
x = 5.3 ⋅1.2 = 6.36
ច ន ច មTយ
, ប 2 ម គ x = 6.36 8
4x − 3 5 x − 12
ញ. x + −
3
= 1−
2 4 6
គSន,
គSន,

កំែណេ យ ៃហ ហុន
ិ , ៃហ ចរ និងយ៉ត ពនក 42
េមេរ នទី៤ ៖ សមី រដេឺ កទី១ នមួយអ ត

3 4x − 3 5 x − 12
x+ − = 1−
2 4 6
12 x + 18 − 12 x + 9 = 12 − 10 x + 24
27 = 36 − 10 x
10 x = 9
9
x=
10
ច ន ច មTយ
, ប 2 ម គ x= 8
9
10
2x −1
. 8( x − 2) + 3 = 7 x + −3
2
គSន,
2x −1
8( x − 2) + 3 = 7 x + −3
2
16 x − 32 + 6 = 14 x + 2 x − 1 − 6
0 x = 19 មន
L Hច
ចន ម នច
dWនច មTយ
, 8
(2 y + 1) 2 3( y + 2) 2 y ( y − 2)
(. − =
4 6 2
គSន,
(2 y + 1)2 3( y + 2) 2 y ( y − 2)
− =
4 6 2
( ) ( ) (
3 4 y2 + 4 y +1 − 6 y2 + 4 y + 4 = 6 y2 − 2 y )
12 y 2 + 12 y + 3 − 6 y 2 − 24 y − 24 = 6 y 2 − 12 y
0 y = 21 មន
L Hច
ចន ម នច
dWនច មTយ
, 8
( z − 2)2 z ( z − 3)2 (8 z + 7)( z − 1) 4
). + = +
3 5 15 5
គSន,
( z − 2) 2 z ( z − 3) (8 z + 7)( z − 1) 4
+ = +
3 5 15 5
( )
5 z 2 − 4 z + 4 + 3 z ( z − 3) = 8 z 2 − z − 7 + 12
5 z 2 − 20 z + 20 + 3 z 2 − 9 z = 8 z 2 − z + 5
− 28 z = −15
15
z=
28
ច ន ច មTយ
, ប 2 ម គ x= 8
15
28

កំែណេ យ ៃហ ហុន
ិ , ៃហ ចរ និងយ៉ត ពនក 43
េមេរ នទី៤ ៖ សមី រដេឺ កទី១ នមួយអ ត

3/. យ ម ង ម
ក. x( x + 3) = 0
គSន,
x = 0 x = 0
 x + 3 = 0 ⇒  x = −3
 
ច ន ច មTយ
, ប 2 ម គ x = 0 នង
L x = −3 8
ខ. ( x + 5)( x − 1) = 0
គSន,
 x + 5 = 0  x = −5
 x −1 = 0 ⇒  x = 1
 
ច ន ច មTយ
, ប 2 ម គ x = −5 នង
L x =1 8
គ. ( x − 1) 2 − 4 = 0
គSន,
( x − 1) − 22 = 0
2

( x − 1 − 2 )( x − 1 + 2 ) = 0
( x − 3)( x + 1) = 0
x +1 = 0  x = −1
x − 3 = 0 ⇒ x = 3
 
ច ន ច មTយ
, ប 2 ម គ x = −1 នង
L x=3 8
ឃ. 4( x − 1) 2 − (2 x − 3)2 = 0
គSន,
( 2( x − 1) ) − (2 x − 3)2 = 0
2

( 2 ( x − 1) − ( 2 x − 3) ) ( 2 ( x − 1) + ( 2 x − 3) ) = 0
( 2 x − 2 − 2 x + 3)( 2 x − 2 + 2 x − 3) = 0
4x − 5 = 0
5
x=
4
ច ន ច មTយ
, ប 2 ម គ x= 8
5
4
ង. x 2 − 6 x = 0
គSន,
x ( x − 6) = 0
x = 0 x = 0

x − 6 = 0 x = 6
 

កំែណេ យ ៃហ ហុន
ិ , ៃហ ចរ និងយ៉ត ពនក 44
េមេរ នទី៤ ៖ សមី រដេឺ កទី១ នមួយអ ត

ច ន ច មTយ
, ប 2 ម គ x = 0 នង
L x=6 8
ច. 2 x 2 − 2 = 0
គSន,
(
2 x2 −1 = 0 )
( x − 1)( x + 1) = 0
 x + 1 = 0  x = −1
 x −1 = 0 ⇒  x = 1
 
ច ន ច មTយ
, ប 2 ម គ x = −1 នង
L x =1 8
4/. យ ម ង ម
ក. x 2 − 2 x + 3 = 0
គSន,
គSន,

(x 2
)
− 2x +1 + 2 = 0

( x − 1) + 2 = 0 មន
L Hច i ( x − 1) + 2 > 0 គប2 x
2 2

ចន ម dWនច មTយ
, 8
ខ. x 2 − 12 x + 32 = 0
គSន,
(x 2
)
− 12 x + 36 − 4 = 0

( x − 6) − 4 = 0
2

( x − 6 − 2 )( x − 6 + 2 ) = 0
( x − 8)( x − 4 ) = 0
x − 4 = 0 x = 4
x − 8 = 0 ⇒ x = 8
 
ច ន ច មTយ
, ប 2 ម គ x = 4 នង
L x =8 8
គ. y 2 − 14 = 5 y
គSន,
y 2 − 5 y − 14 = 0
y 2 − 7 y + 2 y − 14 = 0
y ( y − 7) + 2( y − 7) = 0
( y − 7 )( y + 2 ) = 0
 y + 2 = 0  y = −2
y −7 = 0 ⇒ y = 7
 
ច ន ច មTយ
, ប 2 ម គ x = −2 នង
L x=7 8

កំែណេ យ ៃហ ហុន
ិ , ៃហ ចរ និងយ៉ត ពនក 45
េមេរ នទី៤ ៖ សមី រដេឺ កទី១ នមួយអ ត

ឃ. x 2 − x = 3x + 12
គSន,
x 2 − 4 x − 12 = 0
( x − 2 ) − 16 = 0
2

( x − 2 − 4 )( x − 2 + 4 ) = 0
( x − 6 )( x + 2 ) = 0
 x + 2 = 0  x = −2
x − 6 = 0 ⇒ x = 6
 
ច ន ច មTយ
, ប 2 ម គ x = −2 នង
L x=6 8
5/. គ*@ 6>?ន * AB62
Xង v1 K 6>?នAB62 ប 2 *
v2 K 6>?ន ប 2ទក
/ <
Xម បមន., d = ( v1 + v2 ) t
យ d = 57 km , v2 = 2 km / h , t = 10 / 3 h − 1 / 3 h = 3 h
គSន
57 = ( v1 + 2 ) ⋅ 3
57
v1 + 2 =
3
v1 = 19 − 2
v1 = 17 km / h
ច ន 6>?នAB62 ប 2 * គ 17 km / h ;: Sនក1* 2 8
6/. យ
, ន. R ន R 2 យU ព6បV@ , XមYន2CទD E គច ក ?
9 W ន ទប
Xង t1 នង
L t2 K យU ព63 6CទD E ច ក នង
L យន. R ជបdN ងdN
យpប2គL ព ព6 R qង
, ចdN 8
Xម បមន., គSន
d1 = v1t1
d 2 = v2t2
3 យ d1 = d 2 (ច 7 យជបគL ពក3នTង ចញ 1 *, គ ,d
W N)
<យ
, Xមប1Qប2 , t1 − t2 = 20 mn = 3 h , v1 = 288 km / h , v2 = 480 km / h
1

គSន
288t1 = 480t2 3t1 = 5t2 (1)
 
 1 ⇔ 1
t1 − t2 = 3 t1 − t2 = ( 2)
 3

កំែណេ យ ៃហ ហុន
ិ , ៃហ ចរ និងយ៉ត ពនក 46
េមេរ នទី៤ ៖ សមី រដេឺ កទី១ នមួយអ ត

Xម ( 2 ) គSន t1 = + t2 ជន
1
1 កN9ង ម (1) ,
3
1 
3  + t2  = 5t2
3 
1 + 3t2 = 5t2
2t2 = 1
1
t2 = h
2
ចន ម
, >XមYន2CទD E ច ក យន. R :; R កN9ង យU ព6 30 @ទ 8
7/. ក1* ច
2 ន
1 ន L ]កN9ង =Nក2 @
ប, x Kច1នន L ]កN9ង=Nក2 ន @
Xមប1Qប2, គSន
280
=4
x
280
x=
4
x = 70
ច ន កN9ង=Nក2 ន @ ន L ]ច1នន 70 @ក2 8
8/. គ*@ 6>?ន ប 2កង2 នង
L មV
Xង v1 K 6>?ន ប 2កង2
v2 K 6>?ន ប 2មV
Xមប1Qប2, គSន
d1 = v1t1

d 2 = v2t2 យ d1 = d 2 = 50 km , t2 = t1 − 2 , t1 នង
L t2 គL K h
v = 2.5v
 2 1

យង
, Sន
v1t1 = v2t2 v1t1 = v2 ( t1 − 2 ) (1)
 ⇔
v2 = 2.5v1 v2 = 2.5v1 ( 2)
យក ( 2 ) ជ កN9ង (1) គSន
v1t1 = 2.5v1 ( t1 − 2 )
t1 = 2.5t1 − 5
1.5t1 = 5
5 10
t1 == h
1.5 3
@ t2 = t1 − 2 = − 2 = h
10 4
3 3
គSន ,

កំែណេ យ ៃហ ហុន
ិ , ៃហ ចរ និងយ៉ត ពនក 47
េមេរ នទី៤ ៖ សមី រដេឺ កទី១ នមួយអ ត

 d1 50 km
v1 = t = 10 = 15 km / h
 1 h
 3

v = d 2 = 50 km = 37.5 km / h
 2 t2 4
 h
3
ច ន 6>?ន ប 2

N ជL កង2គ 15 km / h នង N ជL មV គ 37.5 km / h 8
L 6>?ន ប 2 ក
9/. គ*@ ) 69 យ b9ង 70% 3 6 ;: cយ
Xង V K ) 69 យ b9ង 70% 3 6 ;: cយ
Xមប1Qប2, គSន
t 9 9 ទD ប 2H 9 កN9ង 69 យ b9ង 40% គ 20 ⋅ 40%
t 9 9 ទD ប 2H 9 កN9ង 69 យ b9ង 70% គ V ⋅ 70%
t 9 9 ទD ប 2H 9 កN9ង 69 យ b9ង 50% គ ( 20 + V ) ⋅ 50%
Xម d6 *uព v; , t 9 9 ទD ប
9 ម9នcយ នង
L យcយdN មន
L
S ប
2 ង2 ទ , គSន
20 ⋅ 40% + V ⋅ 70% = ( 20 + V ) ⋅ 50%
20 ⋅ 0.4 + 0.7V = ( 20 + V ) ⋅ 0.5
8 + 0.7V = 10 + 0.5V
0.2V = 2
V = 10 ℓ
ចន ) 69 យ b9ងH 9 70% 3 6 ;: យកមកcយគ 10 ℓ 8
10/. កច1នន Sក2 ម
, ប 2d កN

2 9ង
N X Sក2នមយ_
Xង x K Sក2 ម
, មយ3ផNក3 6d ច
2 ង Sន Sក2 9 %
@ x + 8000 000 K Sក2 ម
, មយ3ផNក ទ 3 6d S
2 ន Sក2 10 %
គSន
Sក2 ប 2d ព
2 3ផNកទ គ 9 % ⋅ x
Sក2 ប 2d ព
2 3ផNកទw គ 10 % ⋅ ( x + 8000 000 )
3 Xមប1Qប2, Sក2 ប
9 ប 2d ក
2 9ង
N ON 1 គ 4 600 000 6, គSន
9 % ⋅ x + 10 % ⋅ ( x + 8 000 000 ) = 4 600 000
0.09 x + 0.1( x + 8 000 000 ) = 4 600 000
0.09 x + 0.1x + 800 000 = 4 600 000
0.19 x = 4 600 000 − 800 000
0.19 x = 3800 000

កំែណេ យ ៃហ ហុន
ិ , ៃហ ចរ និងយ៉ត ពនក 48
េមេរ នទី៤ ៖ សមី រដេឺ កទី១ នមួយអ ត

6
3800 000
x= = 20 000 000
0.19
@ x + 8 000 000 = 20 000 000 + 8 000 000 = 28000 000 6
ចន ក
N 9; . ន
, កN9ង X
Sក2 ម Sក2 9 % ច1នន 20 000 000 6
Sក2 ម
, កN9ង X Sក2 10 % ច1នន 28 000 000 68
11/. កច1នន L ]3 6 6ងS62Y ផ L S62 S ផង ( ក x )
2 ង នង
កN9ង=Nក2 ន @ ន L ] 30 @ក2 3 6 ន

N 6ងS62Y 32 ម9ខ 22 − x

N 6ងS62 S 3 ម9ខ 17 − x

N 6ងY1ង w ម9ខ x

N មន
L 6ងY1ងw ម9ខ 3 @ក2
គSន
30 = ( 22 − x ) + (17 − x ) + x + 3
30 = 22 − x + 17 − x + x + 3
x = 42 − 30 = 12
ចន L ]3 6 6ងS62Y ផ
2 ង នង
L S62 S ផង នច1នន 12 @ក2
12/. កច1
កច1នន ម យ ប
9
Xង x Kច1នន ម យ ប
9
Xមប1Qប2, គSន
3ចកaថ7ទ 2ចន
1 ន
x
2
1 x x
3ចកaថ7ទw 2ចន
1 ន  =
22 4
1 x x
3ចកaថ7ទx 2ចន
1 ន  =
24 8
1 x x
3ចកaថ7ទ 2ចន
1 ន  =
2  8  16
j 62 ម យ 2 ទ
@ ម យ ប
9 កN9ង 3ចក គ
x x x x
+ + + +2
2 4 8 16
3 ម យ ប
9 គ x , គSន
x x x x
+ + + +2= x
2 4 8 16
8 x + 4 x + 2 x + x + 32 = 16 x

កំែណេ យ ៃហ ហុន
ិ , ៃហ ចរ និងយ៉ត ពនក 49
េមេរ នទី៤ ៖ សមី រដេឺ កទី១ នមួយអ ត

15 x + 32 = 16 x
x = 32
ចន ម យ ប
9 គ នច1នន 32 ម យ 8
13/. ក aមT3 6Sន
6Sន6ក2
Sន6ក2ពម9ន
Xង x K aមT3 6Sន6ក2ពម9ន
Xមប1Qប2, គSន
70% x + 10% ( 70% x ) = 184800
0.7 x + 0.1( 0.7 x ) = 184800
0.7 x + 0.07 x = 184800
0.77 x = 184800
184800
x= = 240 000
0.77
ច ន aថT6ក2ពម9ន ប 2ទ ពBa @ គ 240 000 68

េយើងខំកស
៏ ម
ូ អភ័យេ សផងែដរ ចំេ ះ ល់កហ
ំ ស

ឆងែដលេកើត"នេ$យអេចត% េ&ក'ងេសៀវេ*
ងេសៀវេ*
មួយក,-លេនះ េហើយក៏សម
ូ រង់.េំ $យរ/ក យ នូវមតិ
រ1ះគន់ 34ប
, % និង63យចម7លព
់ ស
ី :
ំ ក់មត
ិ អ
; ក
' សិក<
,
ពី6គប់មជ,>$?ន
, ។ ទូរសពB ០១២ ៣៤៧ ៦២៤
ឬ អុីែម៉ល ៖ pahen_hay@yahoo.com

កំែណេ យ ៃហ ហុន
ិ , ៃហ ចរ និងយ៉ត ពនក 50
េមេរ នទី៥ ៖ វសមី រដេឺ កទី១ នមួយអ ត

ម នទ ម កទ នមយ

1/. ច យ ម ង ម!
ក. x + 5 > 3 , 3 − x < 7 − 2x

ខ. 5 x ≥ 10 , −6 x < 12 , −5 x > −20

គ. 2 x ≤ −
1 x 1 9
, > , −3 x ≥
3 7 4 2
ឃ. 5 x − 1 > 4 , −8 x + 9 ≥ −7 , x −1 < x + 2

ង. 3x − 5 < x + 7 , −2 x + 11 > 5 x + 31
 1
ច. −4 x + 9 ≤ 8 x − 3 , −2 x − 1 ≥ 2  x + 
 2
x +1 2x − 3
ឆ.
2 5
x + 3 > x − 27 , <
3 3 2 4
1 + 2 x 1 − 3x x + 1 1 3 + 2x
ជ. x − ≥ , − ≤
4 3 5 2 2
2/. ច យ បព*ន+ ម ង ម!
 2x − 3 ≤ 9  3x − 4 ≤ 8
ក.  , 
 2 x − 3 ≥ −5 3 x − 4 > −7
2 x + 5 > 5 x − 4 2 x − 3 > 5 x − 1
ខ.  , 
 x − 7 < 2x − 3  x + 4 ≥ 3x − 2
 3x 2 4 x − 3
 4 − 3 ≤ 2  2x + 1 < 0

គ.  ,  x
 2 x − 1 > 3x − 4 − 2 + 2 < 0
 4
3/. គ ក
, ន ឈច
. ន
, ន 40 ម +ជ/, ញច 1 មយ2ងច /
. ព*ទជ/ 34កង 5 ច
. ,
6 7ច ន 8ផ: ក;ធ,បផ
, / ប=6
/ > ន m 2 ប. គ ង
? @ គ ក
, ន ឈA
. B ព
CD 2 m ?
4/. ច បFGញ@ ជHងនមយI8ន 3 ន ប4 ងខBJងកនB ប 8ន
36 5
5/. KកLកMN O8នមយច, ? 5 ធ,Jង Q
O8នមយច,នន4ថមនង .ន ? −3 5 ច កច,នន 6 5
> ង
6/. បចR/បSនD ន Tព/ក ន7យ/ 32 UD , Vយ កនC
. កនC L ន7យ/ 12 UD , 5 .W
កD/ងច 6B ប=6
/ > ន UD , Xម/ខ ទ ទប7យ/
.ប7យ/Tព/ក ចJងប ង ប=4/ នN ចន
.
Jងព ង8ន7យ/កន ?

កំែណេ យ ៃហ ហុន
ិ , ៃហ ចរ និងយ៉ត ពនក 51
េមេរ នទី៥ ៖ វសមី រដេឺ កទី១ នមួយអ ត

7/. / ខ ន ក YកLព /, 5 /ទ
, 1 នក YកL 500 Z ច,នន 20 នBក
?
, 2
/ទ នក YកL 1000 Z ច,នន 30 នBក
? 5 8ថ[មយ / ខV 25 នBក
?
ពក YកL\,
\ង , ព /, 6 5 ប. / ខចងLYន YកL]=ង ច 17 500 Z
Vយ
. ]=ង ចន
. 20 000 Z . / ខ7ច ន ក YកL 500 Zប=6
/ > ន នBក
? ?
8/. កD/ង ប^ង O. 8 ម/ខ J_ 4 Oម/ខ J_នមយI នពន:/ 100 5 គក,3 L
ន ទ: A ច, K `M4 O នមធaមbគ]=ង ច .ន
> ង
? 90 Vយ
.
ទទOYនន ទ: B c នមធaមbគ]=ង ច .ន ? 80 5
> ង O ប^ង
Yន 7 ម/ខ J_ យទទOYនពន:/ 84 93 78 87 89 70 នង 81 Vយ
. W
OLមយម/ខ J_ ទ 4 O c ប^ងបនN 5
ក. . O7ចទទOYនន ទ: A Qទ?

ខ. . O7ចទទOYនន ទ: B Q ទ ? ច ក,3 ព
L ន:/8នម/ខ J_ច/ង
យ ម
. Sda គទទOន ទ: B 5

9/. នមយ2ងច / 34កង 5 ប MN យ ន Fe L O. f ង8នទទង


? ប4 ង
15 m 5 យ ង
? @ប 8ន ន4 ងប,ផ/ .ន
> ង
? 94 m , ច ក
ប 8ន ន 6 ប. Fe L ជHងJច,ននគ L 5
10/. , ប/ 4 O c OកLda ភij នព ប ភទ
ភទ A នង B 4 O ប ភទ A ន
ច,នន O. ប ភទ B ច,នន 5 នBក
? 5 ប. គOកL , ប/ ]=ង ចន
. Yន
37 នBក កច,
? 5ច កច,នន , ប/ 8ន ប ភទនមយI4 O7ចOកLYន ចន
.
ប,ផ/ 5
11/. ផOបកព ច,ននគ L CDធ,Jង 72 ច ក,3 ច
L ន
, នគ L \,ងព
4 O ន 8មB ចJង
ចJង គ 5
12/. Oកkខ3lកD/ង ជO ថយនNក3
, L ច ង ម!
ម!

D ជOJ ក
D បក
. ប នង ចញ បង យខBនmង
ឈDOជOកD
OជOកD/ងមយ8ថ[គ J YកL 80 000 Z ពម\,ង YកLប4នnម8ថB 400 Z
កD/ងច [ យ ផB 1km

D ជO c កកL YកL 400 000 Z បLជO 1 8ថ[ 5
. ក
D ជO
ជO c បក
. ប ច [ យផB ប=6
/ >ន ម
. Sក/d
, aឈDOជO O.
ព YកLកកL ?

កំែណេ យ ៃហ ហុន
ិ , ៃហ ចរ និងយ៉ត ពនក 52
េមេរ នទី៥ ៖ វសមី រដេឺ កទី១ នមួយអ ត

13/. គda ∆ABC 4 O ន ជHង AB = 12 cm , AC = 3 cm 5 . ជHង BC ន


ប4 ងប=6
/ > ន ? ប. ង
? @ ប4 ង ប LoJពV/គ/38ន 3 ( ប4 ង BC គ J
ច,ននគ )L 5
14/. ច កច,ននគ 4L Oព ង8ន 8មB ប Lo]=ង ច .ន
> ង
? 8 5
15/. បMចងL Lក daYន2ងច / 34កងមយ 4 O នទទង
? ប4 ង
3cm នង ន8ផ: ក; WកD/ងច 6B ព 15cm 2 X 24 cm 2 5 ប
. M c
ជ. . បMN យ ប4 ងប=6
/ > នខB ? ( ប4 ងប MN យJច,ននគ )L 5

ច មយ
1/. យ ម ង ម!
ក. ច, K x + 5 > 3
x > 3−5
x > −2
ច, K 3 − x < 7 − 2 x
3 − x < 7 − 2x
2x − x < 7 − 3
x<4
ខ. ច, K 5 x ≥ 10
10
x≥
5
x≥2
ច, K −6 x < 12
12
−x <
6
x > −2
ច, K −5 x > −20
−5 x > −20
20
−x>−
5
x<4

គ. ច, K 2 x ≤ −
1
3
1
x≤−
3⋅ 2
1
x≤−
6

កំែណេ យ ៃហ ហុន
ិ , ៃហ ចរ និងយ៉ត ពនក 53
េមេរ នទី៥ ៖ វសមី រដេឺ កទី១ នមួយអ ត

ច, K
x 1
>
7 4
7
x>
4
ច, K −3x ≥
9
2
9
−3 x ≥
2
9
−x ≥
3⋅ 2
3
x≤−
2
ឃ. ច, K 5 x − 1 > 4
5x > 4 + 1
5
x>
5
x >1
ច, K −8 x + 9 ≥ −7
−8 x + 9 ≥ −7
− 8 x ≥ −7 − 9
16
−x≥−
8
x≤2
ច, K x − 1 < x + 2

x − x < 1+ 2
0x < 3 ព គបL x
ម . JនចR គបL 8មB x 5
នច មBយ
ង. ច, K 3x − 5 < x + 7
3x − x < 7 + 5
2 x < 12
x<6
ច, K −2 x + 11 > 5 x + 31
−2 x − 5 x > 31 − 11
− 7 x > 20
20
x<−
7
ច. ច, K −4 x + 9 ≤ 8 x − 3
−4 x − 8 x ≤ −3 − 9
− 12 x ≤ −12
x ≥1

កំែណេ យ ៃហ ហុន
ិ , ៃហ ចរ និងយ៉ត ពនក 54
េមេរ នទី៥ ៖ វសមី រដេឺ កទី១ នមួយអ ត

ច, K −2 x − 1 ≥ 2  x + 
1
 2
− 2x −1 ≥ 2x + 1
−2 x − 2 x ≥ 1 + 1
− 4x ≥ 2
2
x≤−
4
1
x≤−
2
ឆ. ច, K
2 5
x + 3 > x − 27
3 3
2 x + 9 > 5 x − 81
2 x − 5 x > −81 − 9
− 3 x > −90
x < 30
x +1 2x − 3
ច, K <
2 4
2x + 2 < 2x − 3
2 x − 2 x < −3 − 2
0 x < −5 មន7ច

ម C>នច មBយ
. ទ5
1 + 2 x 1 − 3x
ជ. ច, K x − ≥
4 3
12 x − 3 − 6 x ≥ 4 − 12 x
6 x + 12 x ≥ 4 + 3
18 x ≥ 7
7
x≥
18
x +1 1 3 + 2x
ច, K − ≤
5 2 2
2 x + 2 − 5 ≤ 15 + 10 x
2 x − 10 x ≤ 15 + 3
− 8 x ≤ 18
9
x≥−
4
2/. យ បព*ន+ ម ង ម!
 2x − 3 ≤ 9
ក. ច, K 
 2 x − 3 ≥ −5
− 5 ≤ 2x − 3 ≤ 9
−5 + 3 ≤ 2 x ≤ 9 + 3
− 2 ≤ 2 x ≤ 12

កំែណេ យ ៃហ ហុន
ិ , ៃហ ចរ និងយ៉ត ពនក 55
េមេរ នទី៥ ៖ វសមី រដេឺ កទី១ នមួយអ ត

2 12
− ≤x≤
2 2
−1 ≤ x ≤ 6
ច ន បព*ន+ ម នច មBយ
. −1 ≤ x ≤ 6 5
 3x − 4 ≤ 8
ច, K 
3 x − 4 > −7
− 7 < 3x − 4 ≤ 8
−7 + 4 < 3 x ≤ 8 + 4
− 3 < 3 x ≤ 12
3 12
− <x≤
3 3
−1 < x ≤ 4
ច ន បព*ន+ ម នច មBយ
. −1 < x ≤ 4 5
2x + 5 > 5x − 4 (1)
ខ. ច, K 
 x − 7 < 2 x − 3 ( 2)
ម (1) : 2 x + 5 > 5 x − 4
2 x − 5 x > −4 − 5
− 3 x > −9
x<3
ម ( 2) : x − 7 < 2 x − 3
x − 2 x < −3 + 7
−x<4
x > −4
ច មBយ
. ប L បព*ន+ ម . \,ងព គ −4 < x < 3 5
J ប ពe8នច មBយ

 2 x − 3 > 5x − 1 (1)
ច, K 
 x + 4 ≥ 3 x − 2 ( 2)
ម (1) : 2 x − 3 > 5 x − 1
2 x − 5 x > −1 + 3
− 3x > 2
2
x<−
3
ម ( 2 ) : x + 4 ≥ 3x − 2
x − 3 x ≥ −2 − 4
− 2 x ≥ −6
x≤3

កំែណេ យ ៃហ ហុន
ិ , ៃហ ចរ និងយ៉ត ពនក 56
េមេរ នទី៥ ៖ វសមី រដេឺ កទី១ នមួយអ ត

ច មBយ
. ប L បព*ន+ ម J ប ពe8នច មBយ
. \,ងព គ x < − 5
2
3

 3x 2 4 x − 3
 4 − 3 ≤ 2 (1)
គ. ច, K 
2 x − 1 > 3 x − 4 ( 2)
 4
3x 2 4 x − 3
ម (1) : − ≤
4 3 2
8
3x − ≤ 8 x − 6
3
8
− 5x ≤ − 6
3
10
−x≤−
15
2
x≥
3
3x − 4
ម ( 2) : 2x −1 >
4
8x − 4 > 3x − 4
8 x − 3 x > −4 + 4
5x > 0
x>0

ច មBយ
. ប L បព*ន+ ម J ប ពe8នច មBយ
. \,ងព គ x ≥ 5
2
3

 2x +1 < 0
ច, K  x
− 2 + 2 < 0
 2 x < −1  1
 x<− (1)
 x ⇔ 2
− 2 < −2 x > 4 ( 2)

ម (1) :

ម ( 2) :

កំែណេ យ ៃហ ហុន
ិ , ៃហ ចរ និងយ៉ត ពនក 57
េមេរ នទី៥ ៖ វសមី រដេឺ កទី១ នមួយអ ត

ច មBយ
. ប L បព*ន+ ម J ប ពe8នច មBយ
. \,ងព គC>នច មBយ
.
K C>នច,3ច ប ពe 5

3/. គ36ច,
គ36ច, 8ផ: ក;ធ,បផ
, / ប Lច,
rង x នង y J (ប Ms យ នងទទង
? )8នច, 6
rមប,2បL, គYន !
ប 8នច, 6 គ ! 2 ( x + y ) = 2 ⋅ 40 ⇒ x + y = 40 (1)
Vយ
. 8ផ: ក; ប Lច, គ ! A = xy
យង
. ង
? @ ! ( x + y ) = x 2 + 2 xy + y 2
2

( x − y)
2
= x 2 − 2 xy + y 2

គYន ! 4xy = ( x + y ) − ( x − y )
2 2

6,da A = xy =
1
4
(( x + y ) − ( x − y ) )
2 2

ព (1) 6,da យង
. Yន !
A = xy =
1
4
(
40 2 − ( x − y )
2
)
1
(
= 1600 − ( x − y )
4
2
)
1
= 400 − ( x − y ) ≤ 400
2

4
K ( x − y ) ≥ 0 គបL x នង y
2

នន*យ@ ក;8ផ: 6 ប .ន
> ង
? 400 m OM ( x − y ) = 0
2 2

6,da x = y =
40
= 20 m
2
ច ន ក;8ផ: ប ប Lច, គ 400 m2 5

ច មន ធ ម,
x + y = 40
⇒ ( x + y ) = 1600
2

x 2 + 2 xy + y 2 = 1600
1
(
xy = 1600 − x 2 + y 2
2
( ))
1
(
A = xy = 800 − x 2 + y 2 ≤ 800
2
)

កំែណេ យ ៃហ ហុន
ិ , ៃហ ចរ និងយ៉ត ពនក 58
េមេរ នទី៥ ៖ វសមី រដេឺ កទី១ នមួយអ ត

ផក ប នង 800 m 2

ធ ប!ប ន"គ$ នក%& '" ប A = 800 m 2 (&" ) x= y=0

) ប x = y = 0 មន ក ទ

4/. បFGញ@ ជHងនមយI8ន 3 ខBJងកនB ប 8ន 36


rង a , b , c J Fe L ជHង\,ង8ន 3មយ
យង
. នង
? បFGញ@ !
a+b+c
a<
2
a+b+c
b<
2
a+b+c
c<
2
rម មbពកD/ង 3( Fe L ជHងមយ ខBJងផងបក8ន Fe L ជHងព
ផ`ង ទ កD/ង 34 មយ), គYន !
 a < b + c  a + a < b + c + a  2a < b + c + a
  
b < c + a ⇒ b + b < c + a + b ⇒ 2b < c + a + b
c < a + b c + c < a + b + c 2c < a + b + c
  
 a+b+c
a < 2

 a+b+c
⇒ b <
 2
 a+b+c
c <
 2
ច ន ច, Mទ c Yន យប _ កL ច2OL 5
5/. កច,នន 6
rង x Jច,នន 6
rមប,2បL, គYន !
x
+ 5 ≥ −3
2
x
≥ −3 − 5
2
x
≥ −8
2
x ≥ −16
ច ន ច,នន4 O ផ:jងt: ច
L , MទគJច,នន\,ង;យM4 Oធ,Jង
Q ? −16 5
. > នង

កំែណេ យ ៃហ ហុន
ិ , ៃហ ចរ និងយ៉ត ពនក 59
េមេរ នទី៥ ៖ វសមី រដេឺ កទី១ នមួយអ ត

6/. កច
កច 6B UD , Xម/ខ ទ
rង x Jច,ននច 6B UD , Xម/ខ ទ ( x ≥ 0)
rមប,2បL, គYន !
2 (12 + x ) < 32 + x < 3 (12 + x )
24 + 2 x < 32 + x < 36 + 3 x
24 + 2 x < 32 + x

32 + x < 36 + 3 x
 x < 32 − 24

32 − 36 < 2 x
x < 8

 −2 < x
x < 8
4 x ≥ 0 6,da  ⇒0≤ x≤7
x ≥ 0
ច ន 0 ≤ x ≤ 7 (ច 6B ព 0 OL 7 UD), Xម/ខ ទ ទប
. 7យ/Tព/ក ច
Jងf
Jងf ង 4 ចន
. Jង u ង7យ/កន 5
7/. កច,នន ក YកL 500 Z
rង x Jច,នន ក YកL 500 Z
6 25 − x Jច,នន ក YកL 1000 Z
rមប,2បL, គYន !
17 500 ≤ 500 x + 1000 ( 25 − x ) ≤ 20 000
175 ≤ 5 x + 10 ( 25 − x ) ≤ 200
35 ≤ x + 50 − 2 x ≤ 40
35 − 50 ≤ − x ≤ 40 − 50
− 15 ≤ − x ≤ −10
10 ≤ x ≤ 15
ចន ក YកL 500 Z c ន]=ង ច 10 នBក
? នង]=ង ចន
.
15 នBក
? 5
8/. ក. O7ចទទOYនន ទ: A Qទ?

O c ប^ង vម/ខ4 O wម/ខ Yនពន:/ ប


/ !
84 + 93 + 78 + 87 + 89 + 70 + 81 = 582 ពន:/

យម/ខ J_នមយI នពន:/]=ង ចន


. 100
6 ពន:/ ប ប L OកD/ង ប^ង\,ង v ម/ខ J_ គ !
582 + 100 = 682 ពន:/

កំែណេ យ ៃហ ហុន
ិ , ៃហ ចរ និងយ៉ត ពនក 60
េមេរ នទី៥ ៖ វសមី រដេឺ កទី១ នមួយអ ត

6 , មធaមbគ ប ប L Oគ!
682
= 85.25 < 90
8
ចន Oមន7ចទទOYនន ទ: A Yន ទ 5

ខ. O7ចទទOYនន ទ: B Qទ?

យ មធaមbគ
មធaមbគ ប ប L O គ ! 85.25 6 O7ចទទO
Yនន ទ: B 5

ក,3 ព
L ន:/8នម/ខ J_ច/ង យ ម
. Sda គទទOន ទ: B


. SទទOYនន ទ: B មធaមbគ ប L O c 4 ]=ង ច 80
ប. x Jពន:/8នម/ខ J_ច/ង យប L O ( x ≤ 100 ) , គYន !
582 + x
80 ≤
8
640 ≤ 582 + x
x ≥ 640 − 582
x ≥ 58
ចន ម
. SYនទទOន ទ: B O c យកពន:/ O.មខ
/ J_ច/ង យ

? 58 ពន:/ 5
daYន]=ង ច ម
9/. ក ប 8ន ន 6 ( Fe L ជHងJច,ននគ L )
rង x Jទទង
? ប L ន ( x > 0, x Jច,ននគ )L
6 3x + 15 Jប Ms យប L ន
យប ប L ន4 ងប,ផ/ .ន ? 94 m , គYន !
> ង
2 ( x + ( 3 x + 15 ) ) ≤ 94
x + 3 x + 15 ≤ 47
4 x ≤ 47 − 15
32
x≤
4
x≤8
6 , គYន !
3 x ≤ 24
3 x + 15 ≤ 24 + 15
3 x + 15 ≤ 39
ចន ប 8ន នគ !
ទទង
? .ន ? 8m , ប M
> ង s យ .ន
> ង
? 39 m 5

កំែណេ យ ៃហ ហុន
ិ , ៃហ ចរ និងយ៉ត ពនក 61
េមេរ នទី៥ ៖ វសមី រដេឺ កទី១ នមួយអ ត

10/. កច,នន , ប/ 8ន ប ភទនមយI4 O7ចOកLYន ចន


. ប,ផ/
rង x Jច,នន , ប/ ប ភទ A
6 x − 5 Jច,នន , ប/ ប ភទ B
rមប,2បL, គYន !
x + ( x − 5 ) ≤ 37
x + x − 5 ≤ 37
2 x ≤ 37 + 5
42
x≤
2
x ≤ 21
6 x − 5 ≤ 16
ចន , ប/ ប ភទ A 7ចOកLYន ចន
. ប,ផ/ 21 នBក
?
, ប/ ប ភទ B 7ចOកLYន ចន
. ប,ផ/ 16 នBក
? 5
11/. ក,3 ច
L ន
, នគ L \,ងព 4 O ន 8មB ចJង
ចJង គ
rង 2 x − 1 Jច,ននគ L ទ ( x Jច,ននគ )L
6 2 x + 1 Jច,ននគ L ទu
rមប,2បL, គYន !
( 2 x − 1) + ( 2 x + 1) > 72
2 x − 1 + 2 x + 1 > 72
4 x > 72
72
x>
4
x > 18
⇒ x ≥ 19
យក x = 19 6 ច,ននគ L ចប,ផ/ \,ងព គ 2 ⋅19 − 1 = 37 នង 39
ច ន ច,ននគ
នគ L \,ងព 4 O ន 8មB ចJង
ចJង គ គ 37 នង 39 5

ប បមយ ទ

rង x Jច,នន ទ ( x យក4 ច,នន )


6 x + 2 Jច,នន ទu
rមប,2បL, គYន !
x + ( x + 2 ) > 72
x + x + 2 > 72
2 x > 70
x > 35

កំែណេ យ ៃហ ហុន
ិ , ៃហ ចរ និងយ៉ត ពនក 62
េមេរ នទី៥ ៖ វសមី រដេឺ កទី១ នមួយអ ត

យ x Jច,នន 6 x ចប,ផ/ គ x = 37
ច ន ច,នន ទ នងទu គ 37 នង 39 c Yនក,
Yនក,3 L 5
កច [ យផB ធe. , 3.
12/. កច ម
. Sក/d
, aឈDOជO O. ព YកLកកL
rង x Jច [ យផB ធe. , 3. ប L ក
D ជO
rមប,2បL, គYន ! 8ថBចM
, យកD/ង ជO ថយនN បL ធe. , 3. ច [ យ
x គ ! 80 000 + 400x

4 យ គYនកកL YកL 400 000 Z ម/ន


6 ម
. Sក/d
, aឈDOជO O. YកLកកL គ c !
80 000 + 400 x ≤ 400 000
400 x ≤ 400 000 − 80 000
320 000
x≤
400
x ≤ 800 km
ចន D ជOមន c ធe. , 3. daV ព 800 km ទ 5

13/. ក ប4 ង BC ( BC JពV/គ/38ន 3 )

rម មbពកD/ង 3, គYន !
AB − AC < BC < AB + AC
12 − 3 < BC < 12 + 3
9 < BC < 15
យ BC Jច,ននគ L Vយ
. JពV/គ/38ន 3 6 BC = 12 cm 5
ច ន ប4 ង BC = 12 cm c Yនក,3 L 5
14/. កច,ននគ L 6
rង x Jច,ននគ L 6
rមប,2បL, គYន !
2x ≥ 8
x≥4
ច ន ច,ននគ L 6 គJបMs ច,ននគ \
L ង
, ;យM4
;យM4 Oធ,Jង Q
.ន ? 4 5 គ
> ង ! {4,5, 6, 7,⋯} 5

កំែណេ យ ៃហ ហុន
ិ , ៃហ ចរ និងយ៉ត ពនក 63
េមេរ នទី៥ ៖ វសមី រដេឺ កទី១ នមួយអ ត

15/. ក ប4 ងប Ms យ
rង x J ប4 ងប Ms យ ( x Jច,ននគ )L
rមប,2បL, គYន !
15 < 3 x < 24
15 24
<x<
3 3
5< x<8
យ x Jច,ននគ L 6,da x = 6 Q x = 7
ច ន ប4 ងប Ms យ8ន ក 6 គ 6 cm Q 7 cm 5

េ ង ៧ និង ១៥ ទី ច ៃថ ពហស ត ទី ១០ ែខ ឧស !"ំ ២០១២ , កូន បុស


របស់ខ,-ំ មកពីងត
ូ ទឹកវ1ញ េគ4ត់ ក់របស់េគ 5កុ6
ំ ន់7លហូប9យ រង់;9
ំ សិន ។
ភរ1>ខ,ក
-ំ 9
៏ ន 9ប់េគ5 9មិន6ន់មកេទ ៃថេនះ ។ េគក៏ 9ប់ ក់េគ5 ខ,ន
-ំ ក
ឹ 9Aស់
េហើយភរ1>ខ,ក
-ំ ទ
៏ រូ សពCមកខ,េ-ំ ដើម ី ឲFកូន បុស 9ននិ>យGមួ
យGមួយ ខ,-ំ ។
ហុ.ឺ ..គិតៗេL ខ,ព
-ំ ត
ិ Gមិន9នផNល់ ពកក់េOP គប់ Qន់ឲFកូនខ,េ-ំ ទ កRង
- មួយ!"េំ នះ
មិនដឹង57រមSណរ៍ បស់េគ>ងAេL ។
សូម ៃី ថឈប់ស ក ក៏មន
ិ WXនឈប់ , ខំបេ ងៀនេកSង ZមផCះេ[\ជ^នីភេRំ ពញ
េដើម 9
ី ន 9ក់ខះ_ េLប`ងaបGមួ
់Gមួយ 9ក់ែខ េ[ឯ សុក ២៨ មុន
ឺ េទៀត ស ប់ជវី ពកRង
-
មួយែខៗ ។ មិនែតប៉េុ Aeះ េ[ភRេំ ពញ តfវេរៀន មួយៃថ ៧ េ ង េហើយ ឥឡiវ តfវចុះេធkើ
កមSសក
ិ l េទៀត ។ េពលទំេនរពីmរសិកl និងmរបេ ងៀន តfវេមើលឯកnរេផoងៗ េដើម ី
ឲF9នចំេណះmន់ែតេ ចើន េហើយ តfវេឆ_qត េរៀបេរៀងេសៀវេrគណិ
េរៀបេរៀងេសៀវេrគណិតវ1ទs ស ប់សស
ិ l
នុសស
ិ o nវ GវដូចGេសៀវេrេនះGេដើម ។ េតើ9នអីខះ_ ពីmរេរៀបេរៀង ? គឺ9នថវ1m
បនvច
ិ បនvច
w Qន់ៗនឹងផoំ;យ xយេលើmរទិញ ឬ ពីន ឬកូពី ឯកnរ ... ។ េហើយmរ
េរៀបេរៀងេនះ គឺGmរេធkឲ
ើ F ន ពសម រi ែបបស ប់អក
R សិកl ។ េតើmរខំ បឹងរបស់ខ,-ំ
6ំងអស់េនះ xនឹងទទួល9នអkខ
ី ះ_ េLន៎ ? ពីសA
ំ ក់មត
ិ អ
v ក
R សិកl ។
េតើ\ល់ឯកnរែដលខ,9
-ំ នេរៀបេរៀង 6ំងប៉ុ ន
{ ទទួលmរQំ ទពីសA
ំ ក់
អRកសិកl ក មិតA ?
ៃហ 9ហុន
ិ (ទូរសពC ០១២ ៣៤៧ ៦២៤)

កំែណេ យ ៃហ ហុន
ិ , ៃហ ចរ និងយ៉ត ពនក 64
េមេរ នទី៦ ៖ បំែណងែចកេ បកង់

ម នទ ប ង ចក បកង

1/. កង ក នមយ គ ង នក ក ទ!"ន #$ក%&'ច(ង )ម *


3 2 1 6 1 7 9 7 10 10 10 3 5 7 2 5 1 6 4 8 1
2 7 1 10 3 5 5 3 7 6 2 10 6 4 3 4 5 5 7
ក. បម',ផ.ទនន/យ ន012 ងប34ញព បកង បកង កន
7 នងថយ
បងកង ធ ប នង បកង ធ ប កន
7 នងថយ :
ខ. <7 ក ន0$ន #ប=>?ន>ក ?
គ. <7 #ប=>?ន>ក &,$នក 10 ? យ
7 #ប=>?ន>ក ទ <
<A 1ប=>?ន Bគ យ &,$នក C=ង ចន
7 5?
ឃ. <7 #ប=>?ន>ក <A 1ប=>?នBគ យ &,$នក C=ង<ច 9 ?
ង. ង )ប "ន បកង ធ បថយ1Bគ យ :
2/. គ ង < )ម*
E ចននក'ន2ម គFG នមយ% កង<បនមយ ន2 ង(ង )ម*

ក. ប ងH<
7 2 ង &,ប34ញព បកង បកង កន
7 នងថយ :
7 ង<បន ន0$នប=>?ន គFG ? <$
ខ. <ក 7 នប=>?ន គFG &,$នក'ន
7 5 >ក ? នងប=>?ន គFG ទ < &,$នក'នC=ង<ច 7>ក ?
C=ង ចន
គ. ង )ប 7 :
"ន បកង កន
3/. គ ងចននក'
ងចននក'ន &,GIបកង J3Kម
ម2ម
2ម គFG នមយ% ទទ, ន
,ទ!ផ,&'ច(ង )ម *

ក. 2ងទនន/យ(ង ,7 ន01 )ប "ន បកង បកង ធ ប :


ខ. បក Gយ L ,7 )បMងព <ងច ច"ន &,$នNប

កំែណេ យ ៃហ ហុន
ិ , ៃហ ចរ និងយ៉ត ពនក 65
េមេរ នទី៦ ៖ បំែណងែចកេ បកង់

x = 3:

4/. (ង )ម ន0 1 <
E "នNយកម?ក កង O មយ *
25 22 30 45 28 51 30 32 34 33
18 30 15 20 24 17 24 41 38 27
16 25 28 31 41 28 26 15 25 19
32 36 21 18 34 41 53 25 42 41
ក. ទនន/យ(ង ,712 ង &,$ន 8 ក យ
7 កនមយ%$ន
ប ងច >I0 7ន
? ង
P 5 &'ច1 15-
15-20 20-
20-25 …:
ខ. ង ' . )ម :
គ. ប ពញកង2 ង បកង កន
7 បកង ធ ប កន
7 1Bគ យ ចគ' )បQ :
ឃ. បក Gយ ,7 )ប <ងច ច &,$នNប 25 :

5/. កង) បក< , 45>ទ កSមកTក ទទ,ជ/យជន0


0កង ថ R 45>ទ 0
,ច',ទ ន,ទ!
ន,ទ!ផ,&'ច2 ង(ង )ម *

ក. <7 កSមកTក ន0 0ច',ទ នប=>?ន ,កង យV ព, 45>ទ


45>ទ ?
ខ. <7 កSមកTក ន0 0ច',ទ នប=>?ន ,កង យV ព, )ម
15>ទ&ប'ង ? LកនI0 $=ងចង )យ <7 គ
15>ទ 0ច',ទ នប=>?ន ,

7 <A 1ប=>?នBគ យ ?
គ. ច' គ' )ប"នព ) បកង កន
7 :
6/. (ង )ម ន0 12 ងចនន&ង"ន) ផWយX ជYកម?2មទ' ទ #នZ1<
&, ប ព<
P . [\បព
[\បព $=ង 2 , ( យV ព,ផWយគ<1>ទ) :

ក. <ក
7 ងR_ច ន0 គផWយX ជYកម? នប=>?ន&ង ? យV ព,
ប=>?ន>ទ ?
ខ. <7 គផWយX ជYកម? នប=>?ន&ង កង យV ព, 20>ទ )យ
20>ទចង )យ?

កំែណេ យ ៃហ ហុន
ិ , ៃហ ចរ និងយ៉ត ពនក 66
េមេរ នទី៦ ៖ បំែណងែចកេ បកង់

30>ទ&ប'ង ?
កង យV ព, <ច1ង 30>ទ
គ. ច' គ' )ប"នព ) 7 នងថយ :
បកង កន
7/. 2 ង(ង )ម 1ចនន ប1ជនGIប នង ប [2មNយ )យពចប
J3Kម *

ក. ច' ង ' . )ម"នទនន/យ ប1ជនGIប នង ប :


ខ. ច' ប ងH<
7 2 ង បកង បកង កន ប1ជនG
7 "ន ប1ជនGIប នង ប :
គ. ច' ក < កចនន ប1ជនGIប នង ប :
ឃ. <7 ប1ជន &,$នNយ<ច1ង 35`
35`GIបប=>?ន>ក ? នង ប
ប=>?ន>ក ?

ច មយ
1/. ក. បម',ផ.ទនន/យ 12 ងប34ញព បកង បកង កន
7 នងថយ
7 នងថយ
បងកង ធ ប នង បកង ធ ប កន

ក បកង បកង បកង បកង បកង


បកង
ក ទ កន ថយ ធប ធ ប កន ធ បថយ
1 5 5 40 12.5 12.5 100
2 4 9 35 10 22.5 87.5
3 5 14 31 12.5 35 77.5
4 3 17 26 7.5 42.5 65
5 6 23 23 15 57.5 57.5
6 4 27 17 10 67.5 42.5
7 6 33 13 15 82.5 32.5
8 1 34 7 2.5 85 17.5
17.5
9 1 35 6 2.5 87.5 15
10 5 40 5 12.5 100 12.5

40 100

កំែណេ យ ៃហ ហុន
ិ , ៃហ ចរ និងយ៉ត ពនក 67
េមេរ នទី៦ ៖ បំែណងែចកេ បកង់

ខ. ក ន0$ន # បចនន ab>ក


ab>ក :
គ. # &,$នក 10 $នចនន c >ក
# &,$នក 7 5 $នចនន de >ក <A 1 57.5% :
C=ង ចន
ឃ. # &,$នក C=ង<ច 9 $នចនន >ក <A 1 15 % :
ង. ង )ប "ន បកង ធ បថយ1Bគ យ

2/. ក. ប ងH<
7 2 ង &,ប34ញព បកង បកង កន
7 នងថយ
ចននក
នកន x ចនន គ f បកង
បកង កន បកងថយ
0 144 144 685
1 195 339 541
2 130 469 346
3 80 549 216
4 58 607 136
5 45 652 78
6 24 676 33
7 6 682 9
8 3 685 3

685

ខ. កង<បន ន0$ន ប1ជនចនន 685 គFG :


$ន 652 គFG &,$នក'នC=ង ចន
7 5 >ក :
ង$ន
នង$ន 9 គFG ទ < &,$នក'នC=ង<ច 7>ក :
គ. ង )ប "ន បកង កន
7

កំែណេ យ ៃហ ហុន
ិ , ៃហ ចរ និងយ៉ត ពនក 68
េមេរ នទី៦ ៖ បំែណងែចកេ បកង់

3/. ក. 2ងទនន/យ ន01 )ប "ន បកង បកង ធ ប


មននង
P ង )ប បកង នង បកង ធ ប យង
7 ង2 ង បកង
នង បកង ធ ប1មន ន :

ចននក
នកន ប x ចនន គ f បកង ធ ប
1 78 2.96
2 1270 48.27
3 680 25.85
4 320 12.16
5 150 5.70
6 133 5.06

2631 100

)ប "ន បកង *

កំែណេ យ ៃហ ហុន
ិ , ៃហ ចរ និងយ៉ត ពនក 69
េមេរ នទី៦ ៖ បំែណងែចកេ បកង់

)ប "ន បកង ធ ប *

ខ. បក Gយ L ,7 )បMងព <ងច ច"ន


&,$នNប x = 3 $នន/យ * $ន 680 គFG <A 1
25.85 % $នក'នGIបចនន 3 >ក :
fនCយ * គFG &,$នក'នGIប 3 >ក គ680
g680 គFG <A 1
25.85 % :
4/. ក. ទនន/យ12 ង &,$ន 8 ក យ
7 កនមយ%$នច >I0 5

!"ក#យ (%") បកង


15-
15-20 7
20-
20-25 5
25-
25-30 9
30-
30-35 9
35-
35-40 2
40-
40-45 5
45-
45-50 1
50-
50-55 2

40

កំែណេ យ ៃហ ហុន
ិ , ៃហ ចរ និងយ៉ត ពនក 70
េមេរ នទី៦ ៖ បំែណងែចកេ បកង់

ខ. ង ' . )ម

គ. ប ពញកង2 ង បកង កន
7 បកង ធ ប កន
7 1Bគ យ
!" ក#យ (%" ) បកង បកង កន បកង ធ ប កន
15-
15-20 7 7 17.5
20-
20-25 5 12 30
25-
25-30 9 21 52.5
30-
30-35 9 30 75
35-
35-40 2 32 80
40-
40-45 5 37 92.5
45-
45-50 1 38 95
50-
50-55 2 40 100

40

គ' )ប បកង កន
7

កំែណេ យ ៃហ ហុន
ិ , ៃហ ចរ និងយ៉ត ពនក 71
េមេរ នទី៦ ៖ បំែណងែចកេ បកង់

គ' )ប បកង ធ ប កន
7 1Bគ យ

ឃ. បក Gយ ,7 )ប <ងច ច &,$នNប 25

<ងច ច &,$នNប 25 $នន/យ $នកម?ក &,$ន


Nយ<ច1ង 25 `ចនន 12 >ក <A 1 30 % :
5/. &ម
7 h3យ ,
F ឆIយ
7 Mងប យង
7 ង2 ង បកង បកង កន
7 នង
បកង កន
7 ធ ប1Bគ យ 1មន ន *
យ' ព)(*ទ
ព)(*ទ+) ចនន,) បកង កន បកង កន ធ ប
0-5 4 4 4.82
5-10 5 9 10.84
10-
10-15 9 18 21.69
15-
15-20 12 30 36.14
20-
20-25 15 45 54.22
25-
25-30 14 59 71.08
71.08
30-
30-35 10 69 83.13
35-
35-40 8 77 92.77
40-
40-45 6 83 100

83

ក. កSមកTក ន0 0ច',ទ ន 83 45>ទ :


,កង យV ព, 45>ទ
ខ. កSមកTក ន0 0ច',ទ ន 18 ,កង យV ព, )ម 15>ទ
15>ទ
&ប'ង :
LកនI0 $=ងចង )យ(ប>
)យ(ប>jបព 15 >ទ&ប'ង) គ 0ច',ទ ន
83 − 18 = 65 ,

7 <A 1 100 % − 21.69 % = 78.31% :

កំែណេ យ ៃហ ហុន
ិ , ៃហ ចរ និងយ៉ត ពនក 72
េមេរ នទី៦ ៖ បំែណងែចកេ បកង់

គ. គ' )ប"នព ) បកង កន


7

6/. &ម
7 h3យ ,
F ឆIយ
7 Mងប យង
7 ង2 ង បកង
បកង បកង កន
7 នង
បកងថយ1មន ន :

យ' ព)ផ.យ
ព)ផ.យ(*ទ
(*ទ+) បកង បកង កន បកងថយ

/ម 5 5 5 50
5-10 1 6 45
10-
10-15 1 7 44
15-
15-20 3 10 43
20-
20-25 2 12 40
25-
25-30 6 18 38
30-
30-35 8 26 32
35-
35-40 12 38 24
40-
40-45 12 50 12

50

ក. កងR_ច ន0 គផWយX ជYកម? ន 50 &ង : យV ព, 45 >ទ :


ខ. គផWយX ជYកម? ន 38 &ង កង យV ព, 20>ទ
20>ទចង )យ :
គផWយX ជYកម? ន 18 &ង កង យV ព, <ច1ង 30>ទ
30>ទ&ប'ង :

កំែណេ យ ៃហ ហុន
ិ , ៃហ ចរ និងយ៉ត ពនក 73
េមេរ នទី៦ ៖ បំែណងែចកេ បកង់

គ. គ' )ប"នព ) 7 នងថយ


បកង កន

7/. ក. ង ' . )ម"នទនន/យ ប1ជនGIប

កំែណេ យ ៃហ ហុន
ិ , ៃហ ចរ និងយ៉ត ពនក 74
េមេរ នទី៦ ៖ បំែណងែចកេ បកង់

ង ' . )ម"នទនន/យ ប1ជន ប

ឬេយង ចេធម ៉ ងេទ ត (េ យ រែតតៃមេលខធំៗេពក)

មបំ ប់ , េគ ន ង៖

#យ (%") ប1ជន ប កម3 ប1ជន


ប1ជន ប កម3
0-5 1 0.2 12 2.4
5-15 104 10.4 3010 301
15-
15-35 65 3.25 3645 182.25
35-
35-45 45 4.5 1700 170
45-
45-55 70 7 1175 117.5
55-
55-70 135 9 1800 120
70-
70-85 195 13 2400 160
85-
85-100 210 14 1170 78
េយើង ន , អុីស&
ស('& )ម(ប*ជន,-ប់(ចំនន
ួ (ប*ជន,-ប់គ*
ឺ ៃផ2(ក3)
(ក3) ៖

កំែណេ យ ៃហ ហុន
ិ , ៃហ ចរ និងយ៉ត ពនក 75
េមេរ នទី៦ ៖ បំែណងែចកេ បកង់

អុីស&
ស'(& )ម(ប*ជនរបួស(ចំនួន(ប*ជនរបួសគឺ*ៃផ2(ក3) ៖

ខ. ប ងH<
7 2 ង បកង បកង កន
7 "ន ប1ជនGIប នង ប
បកង បកង កន
#យ(%" )
ប ប ប ប
0-5 1 12 1 12
5-15 104 3010 105 3022
15-
15-35 65 3645 170 6667
35-
35-45 45 1700 215 8367
45-
45-55 70 1175 285 9542
55-
55-70 135 1800 420 11342
70-
70-85 195 2400 615 13742
85-
85-100 210 1170 825 14912

825 14912

គ. ក < កចនន ប1ជនGIប នង ប


2ម2 ង(ង ,7, យង
7 ឃញ
7 * ប1ជនGIប$នចនន 825 >ក :
នង ប1ជន ប $នចនន 14912 >ក :
ឃ. ប1ជន &,$នNយ<ច1ង 35`
35` *GIប 170 >ក នង ប
6667 >ក :

េ567ំ ២០០៣ ខ<(=ំ ត>វ ន*ប់*សិស@ពូែកកBង


= ,C D7ក់ទ៩
ី *មួយមិត&
ខ<=ំ ៤ Hក់េទៀត គឺ ៖ ែអ៊ល សមLត&ិ , ,ំង រទMិ , េN ,យ(Cង ,រន)
,រន) និង យិន រសOី
េពលេP(បឡងD7ក់ក(មង(ទូRង
ំ (សុក) ខ<=ំ ន*ប់ចS
ំ ត់Dក់
7 េលខ ៣ ែដល
)លេHះ គឺេយើង*ប់ ន ៤Hក់ V7ក់ ែអ៊ល សមLត&ិ W7ក់ ែដលេធYឲ
ើ [េគខូចចិត& ។
េពល(បឡងទូRង
ំ េខត& , អYែី ដលខ<គ
=ំ ត
ិ Dពិ ក េរៀន េហើយមិនខំេរៀន ]ក៏េចញ
(បឡង ។ ខ<=ំ និងមិត& ២Hក់េទៀត V-ក់*,`ពរ គឺ *ប់ែត េN ,យ W7ក់ ែដលទី
បំផត
ុ េគក៏Vក់
- េទៀត េ5ទូRង
ំ (បេទស ។

កំែណេ យ ៃហ ហុន
ិ , ៃហ ចរ និងយ៉ត ពនក 76
េមេរ នទី៧ ៖ មធ មសិតិ

ម នទ មធ ម

1/. គ ធ ក ចនន មក ម ក !យទទ #នទនន$យ


%&ច'ង ម)

ច& កមធ ម ម%*ន នងម+& ,នចនន ក- ងមយ ក .


2/. មន- មយ ក/ម0ក 1ង2 #3 %ម1ប56- 78+ គ 9 kg !យទទ
#ន ទ:ផ %&ច'ង ម)

គ3<មធ ម ម%*ន នងម+& .


3/. ច& គ3<មធ ម ម%*ន នងម+& មទនន$យ'ង ម)

4/. ច& គ3<មធ ម ម%*ន នងម+& មទនន$យ'ង ម)

ក.

ខ.

កំែណេ យ ៃហ ហុន
ិ , ៃហ ចរ និងយ៉ត ពនក 77
េមេរ នទី៧ ៖ មធ មសិតិ

គ.
5/. ទនន$យ'ង
'ង ម9ចននក&ន ប ប-គ? ក,ន @ងច កមយ )

ក. ច& ប ងA @ង បកង បកង ធ ប កន9Bគ យ .


ខ. ច& គ3< ម%*ន .
6/. @ង'ង ម ន7 9បC3ងCចក -3DBព មCខនមយF .

ក. ច& គ3<ម+& .
ខ. ច& គ3< -3DBពមធ ម .
7/. Gឆ8 ទ1 - ខ បIង 8ម-ខJ9Kទទ #នមធ មBគពនL- 89 . G
ឆ8 ទ2 ន7 - ខ JM បង
N ប ងAនពនL-ប+<
- Oនទ ទបP<Q មធ មBគពនL-
ប គ ន
O ង
N 90 ?
8/. ក- ង ព 0ធប3- S 7ក&ន ប+ង #7 Tយ បជគមV យW ព មយ #S!X
គទទ #ន ទ:ផ ,នបC មប មY កមZ ក&ន ប+ង #7%&ច @ង
'ង ម)

ក. ច& ប ងA @ងC% ប[Dញព បកង បកង កន នងថយ .


ខ. ច& កចននក&ន ប+ង #7C% 8នកមZ ^+ង ច 20 mm នង 35 mm .
គ. ច& កម+& ម%*ន នងកមZ មធ ម,នក&ន ប+ង #7 . ច& ប_ប ធ ប ម%*ន
នងមធ ម .

កំែណេ យ ៃហ ហុន
ិ , ៃហ ចរ និងយ៉ត ពនក 78
េមេរ នទី៧ ៖ មធ មសិតិ

9/. @ង'ង ម9ប 83ទក


N T7 ` មa C% b ក cV ក- ងមយ,ថd .

ក. ច& កចននb កC% cទក


N T7 `ក- ងមយ,ថd .
ខ. ក- ងមយ,ថdគ 9មធ មb ក8ក cទក
N T7 ` មaV ប+<
- Oន ម?
គ. ច& គ3<ម+& .
ឃ. ច& គ3< ម%*ន ចបក 0យ ម ប.
10/. យពf កមZ ,នJទ* $យមយ គ#ន ទ:ផ %&ច @ង
'ង ម)

ក. ច& ប ងA @ង បកង បកង កន .


ខ. ច& ង ប,នព!- 3 បកង កន .
គ. ច& គ3< ម%*ន ចបក 0យ ម ប.

ច មយ
1/. កមធ ម ម%*ន នងម+& ,នចនន ក- ងមយ ក
%ម1[យ Y ក ម%*ន យង JM បទនន$យ ម Tប កន9ម-ន ន
គ#ន
គ#ន ,)
19 , 19 , 20 , 20 , 20 , 20 , 20 , 20 , 21 , 21 , 21 , 22 , 22 , 22 , 22 , 23 , 23
23 , 23 , 23 , 23 , 24 , 24 , 24 , 26 , 27 , 27
គ#ន )
មធ ម,នចនន ក- ងមយ កគh )
2 ⋅19 + 6 ⋅ 20 + 3 ⋅ 21 + 4 ⋅ 22 + 6 ⋅ 23 + 3 ⋅ 24 + 26 + 2 ⋅ 27
x=
27
= 22 <ក
599
x=
27

កំែណេ យ ៃហ ហុន
ិ , ៃហ ចរ និងយ៉ត ពនក 79
េមេរ នទី៧ ៖ មធ មសិតិ

27 + 1
ម%*ន,នចនន គh G ង ទ ) = 14
2
គ#ន ) ម%*ន me = 22 <ក .
ម+& គh mo = 23 នង 20 .(8ន បកងខZ 9ង គ)
2/. គ3<មធ ម ម%*ន នងម+&
%ម1[យ Y ក ម%*ន
ម%*ន យង JM បទនន$យ ម Tប កន9ម-ន ន
គ#ន ,)
1 , 2 , 2 , 3 , 3 , 4 , 4 , 4 , 4 , 4 , 4 , 5 , 5 , 5 , 6 , 6 , 6 , 7 , 7 , 8 , 8 , 8 , 10 , 10
គ#ន )
មធ ម គh )
1 + 2 ⋅ 2 + 3 ⋅ 2 + 4 ⋅ 6 + 5 ⋅ 3 + 6 ⋅ 3 + 7 ⋅ 2 + 8 ⋅ 3 + 10 ⋅ 2
x=
24
126
x= = 5.25 kg
24
24 + 1
ម%*ន G ង ទ ) = 12.5 8នន$យ f Gច <k7 ទ 12 នង
2
5+5
13 គ#ន ) ម%*ន me = = 5 kg .
2
ម+& គh mo = 4 kg .(8ន បកងខZ 9ង គ)
3/. គ3<មធ ម ម%*ន នងម+&
មប@ប , គ#ន )
មធ ម គh )
16 ⋅1 + 18 ⋅ 4 + 19 ⋅ 9 + 20 ⋅ 3 + 21 ⋅ 2 + 30 ⋅1
x=
1+ 4 + 9 + 3 + 2 +1
391
x= = 19.55
20
20 + 1
ម%*ន G ង ទ ) = 10.5 8នន$យ f Gច <k7 ទ 10 នង
2
19 + 19
11 គ#ន ) ម%*ន
ម%*ន me = = 19 .
2
ម+& គh mo = 19 .(8ន បកងខZ 9ង គ)
4/. គ3<មធ ម ម%*ន នងម+&
ក. យង8ន ) 2 , 4 , 4 , 6 , 6 , 6 , 6 , 6 , 6 , 8 , 8 , 10 , គ#ន )
មធ ម គh )
2 + 4 ⋅ 2 + 6 ⋅ 6 + 8 ⋅ 2 + 10
x=
12
72
x= =6
12

កំែណេ យ ៃហ ហុន
ិ , ៃហ ចរ និងយ៉ត ពនក 80
េមេរ នទី៧ ៖ មធ មសិតិ

12 + 1
ម%*ន G ង ទ ) = 6.5 8នន$យ f Gច <k7 ទ 6 នង
2
6+6
7 គ#ន ) ម%*ន me = =6 .
2
ម+& គh mo = 6 .(8ន បកងខZ 9ង គ)
គ)
ខ. យង8ន ) 2 , 4 , 4 , 4 , 6 , 8 , 10 , 10 , 10 , 10 , 34 , 34 , គ#ន )
មធ ម គh )
2 + 4 ⋅ 3 + 6 + 8 + 10 ⋅ 4 + 34 ⋅ 2
x=
1+ 3 +1+1+ 4 + 2
136
x= = 11.33
12
12 + 1
ម%*ន G ង ទ ) = 6.5 8នន$យ f Gច <k7 ទ 6 នង
2
8 + 10
7 គ#ន ) ម%*ន me = =9 .
2
ម+& គh mo = 10 .(8ន បកងខZ 9ង គ)
គ. យង8ន ) 5 , 5 , 10 , 10 , 10 , 10 , 15 , 15 , 25 , 25 , 30 , 30 , 30 , 30
35 , 35 , គ#ន )
មធ ម គh )
5 ⋅ 2 + 10 ⋅ 4 + 15 ⋅ 2 + 25 ⋅ 2 + 30 ⋅ 4 + 35 ⋅ 2
x=
2+4+2+2+4+2
320
x= = 20
16
16 + 1
ម%*ន G ង ទ ) = 8.5 8នន$យ f Gច <k7 ទ 8 នង
2
15 + 25
9 គ#ន ) ម%*ន me = = 20 .
2
ម+& គh mo = 10 នង 30 .(8ន បកងខZ 9ង គ)
5/. ក. ប ងA @ង បកង បកង ធ ប កន9Bគ យ

ចននកន x បកង f បកង កន បកង ធ ប កន គយ


0 11 11 11.47
1 23 34 44.74
2 9 43 56.58
3 18 61 80.26
4 6 67 88.16
5 4 71 93.42
6 5 76 100

76

កំែណេ យ ៃហ ហុន
ិ , ៃហ ចរ និងយ៉ត ពនក 81
េមេរ នទី៧ ៖ មធ មសិតិ

ខ. គ3< ម%*ន
76 + 1
ម @ង , ម%*ន G ង ទ = 38.5 គច
h <k7 បកង កនទ 38
2
នងទ 39 .
2+2
គ#ន ម%*ន me = =2 .
2
េយើង ចនិ យមួយែបបេទៀត គឺ ៖ េមដ&ន'ត(វនឹងេ'បកង់េធៀបេកើន-
.គរយ 50% ។ 0ម01ង , េគ ន ៖ me = 2 ។

6/. ក. គ3<ម+&
មប@ប , គ#ន ) ម+& គh mo = 14 oC នង 30 oC (8ន បកងខZ 9ង គ)
ខ. គ3< -3DBពមធ ម
មប@ប , គ#ន ) មធ ម គh )
14 ⋅ 2 + 15 + 18 + 22 + 23 + 25 + 29 + 30 ⋅ 2 + 32 + 35
x=
12
287
x= = 23.91 oC
12
7/. កពនL-C% JM ប ងAន
មប@ប, គ#ន ពនL- - Gឆ8 ទl គh 8 × 89 = 712 ពនL-

ង x 9ពនL-C% JM ប ងAន , គ#ន )
712 + x
= 90
8
712 + x = 90 ⋅ 8
x = 720 − 712
x = 8 ពនL-

%&ច ន7ពនL-C% JM ប ងAន គh 8 ពនL- .


8/. ក. ប ងA @ងC%
@ងC% ប[Dញព បកង បកង កន នងថយ

កម (mm) ច.កន. f បកង កន បកងថយ ផ ក x x⋅ f

0-5 3 3 239 2.5 7.5


5-10 15 18 236 7.5 112.5
10-
10-15 72 90 221 12.5 900
15-
15-20 15 105 149 17.5 262.5
20-
20-25 91 196 134 22.5 2047.5
25-
25-30 35 231 43 27.5 962.5
30-
30-35 8 239 8 32.5 260

239 4552.5

កំែណេ យ ៃហ ហុន
ិ , ៃហ ចរ និងយ៉ត ពនក 82
េមេរ នទី៧ ៖ មធ មសិតិ

ខ. កចននក&ន ប+ង #7C% 8នកមZ ^+ង ច 20 mm នង 35 mm


ម @ង'ង , គ#ន )
ក&ន ប+ង #+7C% 8នកមZ ^+ង ច 20 mm 8ន 134 %ម .
នង ក&ន ប+ង #+7C% 8នកមZ ^+ង ច 35 mm គ`
h Oន.
គ. កម+& ម%*ន នងកមZ មធ ម,នក&ន ប+ង #7
ម @ង , គ#ន )
ក 20 − 25 9ម+& (8ន បកងខZ 9ង គ) .
239 + 1
ម%*ន8នទ ង = 120
2
%& ច7 ក 20 − 25 9 ម%*ន
មធ ម ) x =
4552.5
= 19.05 mm
239
ប_ប ធ ប ម%*ន នងមធ ម
Tយ me Gក- ង ក 20 − 25 !យមធ ម x = 19.05 mm Gក- ង ក
15 − 20 ( ក ផ ង`) 8នន$យ មធ ម &ច9ង ម%*ន .
9/. %ម1[យ Y ឆkយ 3 យងប ងA @ង%&ច'ង ម)

!" ទ$ក. ( g ) ចនន កង


% f ផ &ន ក x
f ր f ց x⋅ f
40-
40-45 8 8 315 42.5 340
45-
45-50 11 19 307 47.5 522.5
50-
50-55 31 50 296 52.5 1627.5
55-
55-60 61 111
111 265 57.5 3507.5
60-
60-65 54 165 204 62.5 3375
65-
65-70 58 223 150 67.5 3915
70-
70-75 43 266 92 72.5 3117.5
75-
75-80 25 291 49 77.5 1937.5
80-
80-85 17 308 24 82.5 1402.5
85-
85-90 7 315 7 87.5 612.5

315 20357.5

ក. កចននb កC% cទក


N T7 `ក- ងមយ,ថd
b កC% cទក `ក
N T7 `ក- ងl
ងl ,ថd 8នចនន 315 <ក .
ខ. កមធ
កមធ មb ក
ម @ង , គ#ន )

កំែណេ យ ៃហ ហុន
ិ , ៃហ ចរ និងយ៉ត ពនក 83
េមេរ នទី៧ ៖ មធ មសិតិ

20357.5
x= = 64.63 g
315
%&ច ន79មធ មក- ងមយ,ថdb ក8ក cទក
N T7 `V 64.63 g .

គ. គ3<ម+&
ម @ង គ#ន )
ក 55 − 60 9ម+& .
ឃ. គ3< ម%*ន
315 + 1
ម%*ន8នទ ង ង = 158
2
គ#ន ក 60 − 65 9 ម%*ន .
បក 0យ ម ប
ម%*ន គ9
h ប ព fងCខ ង ច,នព!- 3 បកង កន នង
បកង
បកងថយ .

ម ប យង ឃញ Cខ ច,នព!- 3 បកង កន នងថយ `


ងmប - Gច <k7 ក 60 − 65 .
n យងmចន^យម*+ង ទ ) V$ក ឈ ងV T ន 158 យង
ធប<L បនង
N V$ក %ក <7fនង
N ប ព9មយCខ
ព9មយCខ ច,នព!- 3
បកង កន នងព!- 3 បកងថយ ពម` ចទpកច q Cកង

កំែណេ យ ៃហ ហុន
ិ , ៃហ ចរ និងយ៉ត ពនក 84
េមេរ នទី៧ ៖ មធ មសិតិ

V$ក %ក យងនង
N #ន G V$ក mប - ច <k7 60 − 65 .
10/. ក. ប ងA @ង បកង បកង កន

កម ( cm ) ចនន ' f បកង កន f ր


145-
145-150 31 31
150-
150-155 95 126
155-
155-160 131 257
160-
160-165 272 529
165-
165-170 120 649
170-
170-175 77 726
175-
175-180 48 774

774

ខ. ង ប,នព!- 3 បកង កន

គ. គ3< ម%*ន
774 + 1
ម%*ន8នទ ង = 387.5
2
Gច <k7 ទ 387 នង 388 គ#ន )
ម%*ន me G ង ក 160 − 165 .

កំែណេ យ ៃហ ហុន
ិ , ៃហ ចរ និងយ៉ត ពនក 85
េមេរ នទី៧ ៖ មធ មសិតិ

បក 0យ ម ប
មV T ន 387.5 យង ងប<L %កម
%កមយ
យ,f ព!- 3
បកង កន ងមយច3-ចC% គr ម%*ន ចទpក
ច q Cកង V$ក mប - គ ឃញ f JM នង
N mប - G
ច <k7ព 160 − 165 .

េ234ំ ២០០៦
២០០៦ ខ9':ំ ត(វ ន-ប់-សិស<ពូែកក>ង
: ?@ A4ក់ទ១២
ី១២ -មួយមិតD
តD
េ យ?រែតវទ&ល័យតូច េគត'ម(វឲHសិស<ពូែកែដល'ត(វេI'បឡងA4ក់'សុកពី
?@ខ9:ំ Kនែត ២Lក់បេ៉ុ MNះ ក>ង
: មួយមុខវ-P ។ មិតភ
D កDខ
ិ R
9:ំ ង
ំ ' ំ ែដល-អតីត
សិស<ពូែកគណិតវទ&A4ក់?@ 0ំងេ2A4ក់ទ៩
ី េពលដល់ទ១២
ី១២ គឺេ2Kន ៣
Lក់ ែដលេរៀន-មួយខ9:ំ ។ ពីរLក់េទៀត
េទៀត គឺ េV ?យ និង ?ំង រទW'ិ ត(វពHXរYរសិកZ
េ យ?រសុខ.ព ។ េពលេនះ ៣Lក់ េគយក ២ Lក់ ។ ខ9ក
:ំ '៏ ត(វដកខXន េដើម\ីទក

ឲYសឲHមិតD ។ ខ9ក
:ំ '៏ ត(វ នេ'ជើសេរ^សឲH'បឡងរូបវទ&វញ ។ ខ9:ំ នខំ'បឹងេរៀនរូប
វទ&េនះ ង`ស់ៃដ ែត'គឹះពីAក់
4 ១០ និង ១១ េ2តិច ។ ទូRង
ំ 'សុក េពល'បឡង
រួចខ9:ំ នចំMត់Aក់
4 េលខ ១ ទនaម
ឹ នឹង មិតK
D ក់
4 េទៀតេbcះ លី . ែដល'បឡង
រូបវទ&-មួយខ9:ំ និងមិតព
D រី Lក់េទៀត ែដល'បឡងគណិតវទ& នdeក់ ត់េI
េហើយ ។ េពលេLះគឺ-ប់ នពីLក់ គឺខ9:ំ និង អ៊ក
ុ េណង មិតខ
D ែ9:ំ ដលេគ'បឡងេលើ
មុខវ-Pអក<រសិ
អក<រសិស\៍ែខhរ ( នេលខ៣)
នេលខ៣) ។ ខ9`
:ំ ំ ៃថl'បឡងេពលេLះគឺ ១០ មីL ២០០៦
ៃថlែដលមិតភ
D កDខ
ិ េ9:ំ ផ<ងៗេទៀត ែដលមិន'ត(វ'បឡង រួមRំងភរ ខ9េ:ំ ពលេនះ , េគ ន
Lំoេរៀបចំ
4 ពធិ ជ
ី ប់េលៀងមួយេ2ផaះ អ៊ក
ុ េណង ។ េពលែដល'បឡងចប់ ('បឡង១
('បឡង១
'ពឹក) ពួកខ9:ំ នLំo4 េIYន់ពធ
ិ ជ
ី ប់េលៀង ែដលេគេធp
ែដលេគេធpម
ើ ប
qr ឆtន
ិ េ'តៀម-េ'សច ។
ហូប uរ រួចស'Kកេលងេ2ទី
ស'Kកេលងេ2ទីេLះ រហូតដល់រេសៀលបនDច
ិ ខ9ក
:ំ ជ
៏ ះ
ិ កង់របស់ខ9:ំ
េI?@ែដលេ2ចKvយ ១គ.ម ពីកែនងជប់េលៀង េដើម\ី0ម នលទWផល ។
េពលដឹងAdeក់ សម\តDិ មិតភ
D កDខ
ិ ក
9:ំ ខ
៏ ច
ូ ចិត-
D ថhី ។ ខ9:ំ និងមិតេD ផ<ងេទៀត ក៏ជយ
ួ លួង
េ@ម
េ@ម ។ េ2ៃថlទី ២៨ មីL -ៃថlែដលខ9:ំ និង អ៊ក
ុ េណង មិតខ
D '9:ំ ត(វេI'បឡងេ2
វទ&ល័យ-សុម
ី 0ែកវ ែដលKនចKvយ ៤០ គ.ម ពីផះ
a របស់ខ9:ំ ។ ខ9ក
:ំ យ
៏ កម៉ត
ូ ូ
របស់ខ9:ំ ជិះឌុប េណង េI'បឡង0ំងពី'ពឹក េហើយក៏ នសន&-មួយក{| ណិត
`ន់ ឲHេ2រង់`ជ
ំ ប
ួ ខ9:ំ េ2េKង ៥ @vច េពលខ9ម
:ំ កពី'បឡងវញ ។ េពល'បឡង
ឲេ@កេអើយ លំuត់Rង
ំ ពីរវ{|?សរុប ១២ លំuត់ ។ Kនលំuត់មយ
ួ េLះ
ខ9េ:ំ ទើបែតេធpេើ ហើយ បីៃថlមន
ុ េហើយលំuត់េផ<ងៗេទៀត ខ9េ:ំ ធpើ នែ'កលែដរ ។ េពល
@vយ ខ9':ំ ត(វេចញពី'បឡង េKង ជិត' េំ Iេហើយ ។ ខ9ក
:ំ ខ
៏ កMត់ េ'~ះទ'Kំខ9:ំ
មកដល់?@
?@ េKង ០៦ ត់េIេហើយ ។ ែសtកេទើប Lង' ប់ខA
9:ំ ម<ិលមិញ`ំខ9:ំ
រហូតដល់េKង ៥ និង ៣០ Lទី ។ ខ9ក
:ំ ស
៏ េំ RសLង ។ ប៉L
ុ នៃថl
c េ'Yយមក លទWផល
ក៏េចញ គឺ អ៊ក
ុ េណង មិតខ
D 9:ំ ន'បឡង-ប់ ចំែណកឯខ9d
:ំ ក់
e ត់ ។ ខ9:ំ នេបើកេមើល
លំuត់កង
>: ឯក?ររបស់ខ9:ំ េទើបខ9ដឹ
:ំដង
ឹ A លំuត់ចន
ំ ន
ួ ១០ ក>ង
: ចំេMមលំuត់Rង

កំែណេ យ ៃហ ហុន
ិ , ៃហ ចរ និងយ៉ត ពនក 86
េមេរ នទី៨ ៖ ប ប

ម នទ ប ប

1/. គ បយក ក មយ ន ក PHNOM PENH យច ន


ច កប ប ប ន
ក. ក P ខ. ក H
គ. ក O # M ឃ. ក P # N
2/. %& ខន(
' យ) “ខ+&ក
, ព
, &ងនក
/ កច,នន %%មយ 01
' ក2& ងច 345ព 1
21 ” 7ន
7ន(
' យ) “ខ+&ក
, ក
8 ព
, &ងនក
/ កច,ននគមយ 01
' ក2&ងច 345ព 1
21 ” 19 ប ប នច,នន %& ខ នង
' 7ក,ព&ងនក
/ ក %:9;2
#ទ?
3/. ច ក ប ប ន ;ប=&ក>ក ចញ ខ1 ខ4
4/. ក2&ង@A
' B Cគប ពDញមយ 19 ប ប
គប 9 ផFឆ1 %:9នង
/ ផFមន
' ឆ1 # ទ ?

5/. H ងI1 9ម ធK យ ធL9 , M9 Nង ក


2 ក2&ង កOង នង
' ក
2 % PQយ កOង
ទទ ន ទន
' ន
2 យ
R ចSង Hម

ក កង កង ក យ កង ប

ថយន 182 52 234

ម 254 338 592

2 ធL9 , M9
ក. ច ក ប ប ន ក យមT1
2 ធL9 , M9
ខ. ច ក ប ប ន ក យ ថយនV
គ. ប9 គ WX 9 ញ ភ+Z@ 1 2003ក
2003ក ម
9 [ច មក2&ងពធ
' ផ ពLផ\យ M'ជXកម:
ក2&ង 35 ភ+Z@ 7603កQ
7603កQ ក
2 % Pក2&ង កOង ^យ
9 ភ+Z@ 4403កQ
4403កQ ក
2
% PQយ កOង 19 គ 1@ 1Zមច,M1 ថយនVប3
&T : នក ន4ង ?

កំែណេ យ ៃហ ហុន
ិ , ៃហ ចរ និងយ៉ត ពនក 87
េមេរ នទី៨ ៖ ប ប

6/. H CFប%Fង ក%, =ង;, ទ បក_ជន 33ក A , B , C យទទ ន


ទ`ផ ចSង
ចSង Hម

ទ មន ទ ន ប ប
A 1420 74 182 1676
B 846 26 122 994
C 570 41 60 671
ប 2836 141 364 3341

ក. ច ក ប ប ន%, =ង មន
' ;, ទ បក_ជន A
ខ. ច ក ប ប ន%, =ង
=ងម
មន' ;, ទ បក_ជន B # C
គ. ច ក ប ប នH ;:ន (ប ច, 5 បក_ជន A
ឃ. ច ក ប ប នH ;:ន (ប ច, 5 បក_ជន B # C
ង. ច បZប ធ ប ប ប ន%, =ង ;, ទ បក_ជន A នង
' ប បន
%, =ង ;, ទ បក_ជន B # C
7/. Sង Hម ន5Q ព1 / H
'V Mabច នH 55Hក
Hក 4 ង
HHHH , HTHH , THHH , TTHH
TTHH , HHHT , HTHT , THHT , TTHT ,
HHTH , HTTH , THTH , TTTH , HHTT , HTTT , THTT , TTTT
ក. ច ក ប ប 5 ន ក Hព
ខ. ច ក ប ប 5 ន ក T ព ម&ន គ
គ. ច ក ប ប 5 ន ក H មយម&ន គ
ឃ. ច ក ប ប 5 ន ក H d,ងបន
ង. ច ក ប ប 5 ន ក T មយ(Tង1ច
'
8/. ក2&ងថងមយ eន 1ឃ4ពMf% នង
' g: ច កប ប ប នឃ4
ពMf% យ ង
/ )ប ប ប នឃ4ពMf g: %:9នង
2
/ 5
9/. កOម^h&នមយ នក1 i 9 ចន
, ន ថj @1Veន ប%ប&គk 'ក ប l, 2,

ប9 គ ធL%
9 em%នaQមយប&គk 'កមយ យច ន
ក. ច ក ប ប ប&គk 'ក 35 នឈប(Tង ចន
9 ប ថj

កំែណេ យ ៃហ ហុន
ិ , ៃហ ចរ និងយ៉ត ពនក 88
េមេរ នទី៨ ៖ ប ប

ខ. ច ក ប ប ប&គk 'ក 35 នឈប(Tង1ច


' មយ ថj
គ. ច ក ប ប ប&គk 'ក 35 នឈបច 345ព 4 o 6 ថj
ឃ. ច ក ប ប ប&គk 'ក 35 9 Qង 6 ថj
នឈប ចន
10/. ថងមយeនឃ4 6 ក2&ង 35eនឃ4
ពMf g: 4 នង
' ពMf% 2 ម
9 [
pយ%'ក\ឃ4ពMf g:បង យ
ក ABCD នង
' ឃ4ពMf% 2 បង
យ ក X នង
' Y គ បឃ4មVង
ព ចញពថង
ក. ច ក ប ប ប នឃ4ពMf%d,ងព
ងព
ខ. ច ក ប ប បឃ4ពMf g:មយ(Tង1ច
'
11/. គ ក%, M ម ន 6 iង យ ក A , B , C , D , E នង
' F ^យ
9
' ក %ង យក 3 %, M
គនង ម
9 [ កq %'% ប=ងយកពន
' &F ប%
9 '%
e2ក ន ន 1 4 %, M
ក. ច ក ប ប %'% 35 1@ នព %, M គ1
ខ. ច ក ប ប %'% 35 នព %,M (Tង1ច
'
12/. គ 5HកមVងប ច កប ប 5 នម&ខ H d,ងប
13/. ថងមយeនឃ4 12 ;ប eន 1ឃ4ពMf% នង
' ពMf g: គ បឃ4មVង
មយ ចញ ពថង
ក. ច កច,នន នឃ4ពMf g: ប9 ប ប នH ប នឃ4ពMf g: %:9នង
1
/ 3
ខ. ច ក ប ប ប នឃ4ពMf%
14/. ក2&ងH (
' ' 3 23ក
Rយមយeនប& % 33ក នង គ ជ%
9 %
9 23ក
យ ធLH
9 ប l21 យច ន ច កប ប គ ជ%
9 %
9 ន
3 d,ងព 3ក
15/. ផF)%មយខMr យប ផ2កបTន
& ;2 iង
យ ក A1 , A2 , B គបងL' )%q @ '
ច ប9 ពDញគប 9 ផF)% 35ព ង ច ក
ប ប គគប ន ក A d,ងព 9ក

កំែណេ យ ៃហ ហុន
ិ , ៃហ ចរ និងយ៉ត ពនក 89
េមេរ នទី៨ ៖ ប ប

16/. ថងមយeនឃ4ពMf ក^ម នង


' ពMf ខ @ ក2&ង 35ឃ4ពMf ក^មeនច,នន
15 ;ប គ បយកឃ4មយពថង យច ន យ ង
/ )ប ប
ប នឃ4មយពMf ក^ម %:9នង
3
/ 5
ច គM3
ក. ច,ននឃ4ពMf ខ @ Pក2&ងថង 35
ខ. ច,ននឃ4ពMf ក^ម 1@ កច ប នsមក2&ងថង ម
9 [q ប ប ន
ឃ4មយeនពMf ក^ម %:9នង
2
/ 3

ច មយ
1/. ក2&ង ក PHNOM PENH eន ក d,ង % 9
P eន 2 , H eន 2 , N eន 2 , O eន 1 , M eន 1 , E eន 1
គ នក Mbចeន 9 ក M
កប ប ប ន
ក. ក P
យ ក P eន 2 គ ន , ក Mbច %ប eន 2 ក M
ច ន5 ប ប នក M ន5គu P =
2
9
ខ. ក H
យ ក H eន 2 គ ន , ក Mbច %ប eន 2 ក M
ច ន5 ប ប នក M ន5គu P =
2
9
គ. ក O # M (# គu បក)
យ ក O eន 1 , M eន 1 គ ន
1 1 2
P= + =
9 9 9
ច ន5 ប ប នក M ន5គu P =
2
9
ឃ. ក P # N
យ ក P eន 2 , N eន 2 គ ន
2 2 4
P= + =
9 9 9
ច ន5 ប ប នក M ន5គu P =
4
9
2/. 19 ប ប នច,នន %& ខ នង / ក %:9;2 # ទ ?
' 7ក,ព&ងនក
ព1 21 eនច,ននd,ង % 21 (ក Mbច 21)
21)ក2&ង 35

កំែណេ យ ៃហ ហុន
ិ , ៃហ ចរ និងយ៉ត ពនក 90
េមេរ នទី៨ ៖ ប ប

ខ %%eនច,នន 11 (ក M %ប 11)
11)
ខគeនច,នន 10 (ក M %ប 10)
10)
គ ន , ប ប នច,នន %& ខក,ព&ង កគu P =
11
21
ប ប នច,នន 7ក,ព&ង កគu P ' =
10
21
យ P នង
' P ' មន
' %:9;2
ច ន5 ប ប នច,នន %& ខ នង
' 7ក,ព&ងនក
/ កម
កមន' %:9;2 ទ
3/. ក ប ប ន ;ប=&ក>ក ចញ ខ1 ខ4
;ប=&ក>កeន ខ 6 គu 1 , 2 , 3 , 4 , 5 , 6 (ក Mbចeន 6)
;ប=
;ប=&ក>ក ចញ ខ1 ខ 4 គu 1 , 2 , 3 , 4 (ក Mbចeន 4)
ច ន5 ប ប ន ;ប=&ក>ក ចញ ខ1 ខ 4 គu
4 2
P= =
6 3
4/. 19 ប ប គប 9 ផFឆ1 %:9នង
/ ផFមន
' ឆ1 # ទ ?
ប9 a នង
' b Qប xy យ នង
' ទទង
/ ន
ច1& HM កង 35 ង;2 , គ ន

ផF ក> ផ2កឆ1 គu A1 = ab
1
2
1a 1a
ផF ក> ផ2កមន
' ឆ1 គu A2 = 2  2  b + 2  2  b = 2 ab
1
   
ផF ក>ច1& HM កង គu A = ab
1
ab
35 ប ប គប 9 ផ2កឆ1 គu P = A = ab = 2
A1 2 1

1
ab
នង ប ប គប ផ2
ក ឆ1 គ
A2 2 1
' 9 u P ' =
A
=
ab
=
2
យ P = P'
ច ន5 ប ប គប 9 ផFឆ1 នង
' ផFមន
' ឆ1
ឆ1 គu %:9;2
2 ធL9 , M9
5/. ក. ក ប ប ន ក យមT1
iមប,zប ,

2 & eនច,នន 234 + 592 = 826 3ក(ក Mbច)
1@ គ ងI1% ប

2 ធL9 , M9 យមT1eនច,នន 592 3ក (ក M %ប)

កំែណេ យ ៃហ ហុន
ិ , ៃហ ចរ និងយ៉ត ពនក 91
េមេរ នទី៨ ៖ ប ប

2 ធL9 , M9
35 ប ប ន ក យមT1គu P =
592
= 0.72
826
2 ធL9 , M9
ខ. ក ប ប ន ក យ ថយនV
យ យ P ' ប, ពញ ន P 35 P + P ' = 1 គ ន
2 ធL9 , M9
ប បន ក យ ថយនVគu P ' = 1 − P = 1 − 0.72 = 0.28
គ. កច,ននច,M1 ថយនV គ 1@ 1Zមទ&
1Zមទ&ក
iមប,zប, គ ន
ប ប ភ+Z@មកពក2&ង កOងជ5' ថយនVគu P1 =
182 91
=
182 + 254 218
ប ប ភ+Z@មកពQយ កOងជ5' ថយនVគu P2 =
52 2
=
52 + 338 15
eននRយ) ,
ប9 គ WX 9 ញ ភ+Z@ 760 3កព
3កពក2&ង កOង គ 1@ 1Zមច,M1
91
× 760 = 317
218
^យ
9 ប9 គ WX 9 ញ ភ+Z@ 440 3កពQយ កOង គ 1@ 1Zមច,M1 15 ⋅ 440 = 59
2

ច ន5 គ 1@ 1Zមច,M1(Tង1ច
' 317 + 59 = 376 ក ន4ង
6/. ក. ក ប ប ន%, =ង មន
' ;, ទ បក_ជន A
ច,ននមន&% & គu 3341 3ក(ក Mbច)
CFប%F&ង% ប
ច,ននមន&% មន
' ;, ទ បក_ជន A គu 74 3ក(ក M %ប)
គ ន , ប ប ន%, =ង ' ;, ទ បក_ជន A គu P = 3341 = 0.022
មន
74

ខ. ក ប ប ន%, =ង ' ;, ទ បក_ជន B # C


មន
ច,ននមន&% មន
' ;, ទ បក_ជន B គu 26 3ក(ក M %ប)
ច,ននមន&% មន
' ;, ទ បក_ជន C គu 41 3ក(ក M %ប)
គ ន , ប ប ន%, =ង ' ;, ទ បក_ជន B # C គu
មន
26 41
P= + = 0.020
3341 3341
គ. ក ប ប នH ;:ន (ប ច, 5 បក_ជន A
មន&% ;:ន (ប ច, 5 បក_ជន A គu 182 3ក (ក M %ប)
គ ន , ប ប ន%, =ង ;:ន (ប ច, 5 បក_ជន A គu
182
P= = 0.054
3341
ឃ. ក ប ប នH ;:ន (ប ច, 5 បក_ជន B # C
មន&% ;:ន (ប ច, 5 បក_ជន B គu 122 3ក(ក M %ប)

កំែណេ យ ៃហ ហុន
ិ , ៃហ ចរ និងយ៉ត ពនក 92
េមេរ នទី៨ ៖ ប ប

មន&% ;:ន (ប ច, 5 បក_ជន C គu 60 3ក(ក M %ប)


គ ន , ប ប ន%, =ង ;:ន (ប ច, 5 បក_ជន B # C គu
122 60
P= + = 0.054
3341 3341
ង. បZប ធ ប ប ប ន%, =ង ;, ទ បក_ជន A នង
' ប បន
%, =ង ;, ទ បក_ជន B # C
iមizង, គ ន
ប ប ន%, =ង ;, ទ បក_ជន A គu PA =
1420
3341
ប ប ន%, =ង ;, ទ បក_ជន B # C គu PB =
846 570 1416
+ =
3341 3341 3341
យង
9 ឃញ
9 ) PA > PB
ច ន5 ប ប ន%, =ង ;, ទ បក_ជន A ចន
9 Qង ប ប ន%, =ង
;, ទ បក_ជន B # C
7/. គ 5Hក 4 ង ន ទ`ផ
HHHH , HTHH , THHH , TTHH
TTHH , HHHT , HTHT , THHT , TTHT ,
HHTH , HTTH , THTH , TTTH , HHTT , HTTT , THTT , TTTT
TTTT
គ ន, ក Mbចeនច,នន 16 ក M
ក. ក ប ប 5 ន ក Hព ង
ក M %ប eន ក H ព គu TTHH , HTHT , THHT , HTTH ,
THTH , HHTT 35ក M %បeនច,នន 6 ក M
គ ន, ប ប 5 ន កH ព ង គu P = =
6 3
16 8
ខ. ក ប ប 5 ន ក T ព ម&ន គ
ក M %ប eន ក T ព ម&ន គ គu TTHH , TTHT , TTTT , TTTH
35ក M %បeនច,នន 4 ក M
គ ន, ប ប 5 ន ក T ព ម&ន គ គu P =
4 1
=
16 4
គ. ក ប ប 5 ន ក H មយម&ន គ
ក M %ប eន ក H មយម&ន គ គu HHHH , HHHT , HHTH ,
HHTT , HTHH , HTHT , HTTH , HTTT 35ក M %បeនច,នន 8 ក M
គ ន, ប ប 5 ន ក H មយម&ន គ គu P = =
8 1
16 2

កំែណេ យ ៃហ ហុន
ិ , ៃហ ចរ និងយ៉ត ពនក 93
េមេរ នទី៨ ៖ ប ប

ឃ. ក ប ប 5 ន ក H d,ងបន
ក M %ប eន ក H d,ងបន គu HHHH
35ក M %បeនច,នន 1 ក M
គ ន, ប ប 5 ន ក H d,ងបន គu P =
1
16
ង. ក ប ប 5 ន ក T មយ(Tង1ច
'
កM មន
' ន ក T C5 គe
u ននRយ) ន ក H d,ង 4 eន
ប ប P=
1
16
ប ប 5 ន ក ' គQ
T មយ(Tង1ច u ប, ពញ ន P
គ ន , P '+ P = 1 ⇒ P ' = 1 − P = 1 −
1 15
=
16 16
ច ន5 ប ប 5 ន ក ' គu P =
T មយ(Tង1ច
15
16
8/. ក ប ប ប នឃ4ពMf%
យង
9 ង
/ )ប ប ប នឃ4ពMf g: គu P =
2
5
35 P ' ប ប ប នឃ4ពMf%
គ ន , P + P ' = 1 ⇒ P ' = 1− P
2 3
P ' = 1− =
5 5
ច ន5 ប ប ប នឃ4ពMf% គu P ' =
3
5
9/. ក Mbចគu 6 + 1 + 15 + 21 + 24 + 18 + 16 + 10 + 7 = 109
ក. ក ប ប ប&គk ' ក 35 នឈប(Tង ចន
9 ប ថj
ច,ននប&គk 'ក ឈប(Tង ចន
9 ប ថjeនច,នន 6 + 1 + 15 + 12 = 34
គ ន ប ប ប&គk 'ក ឈប(Tង ចន
9 ប ថjគu P = 109 = 0.31
34

ខ. ប ប ប&គk 'ក 35 នឈប(Tង1ច ' មយ ថj


ច,ននប&គk 'ក ' មយ ថj eនច,នន 109 − 6 = 103
ឈប(Tង1ច
គ ន ប ប ' មយ ថj គu P = 109 = 0.94
ប&គk 'ក ឈប(Tង1ច
103

គ. ក ប ប ប&គk ' ក 35 នឈបច 345ព 4 ថj o 6 ថj


ច,ននប&គk 'ក ឈបច 345ព 4 ថj o 6 ថj eនច,នន 24 + 18 + 16 = 58
គ ន ប ប ប&គk 'ក ឈបច 345ព 4 ថj o6
o6 ថjគu P =
58
= 0.53
109

កំែណេ យ ៃហ ហុន
ិ , ៃហ ចរ និងយ៉ត ពនក 94
េមេរ នទី៨ ៖ ប ប

ឃ. ក ប ប ប&គk ' ក 35 នឈប ចន


9 Qង 6 ថj
ច,ននប&គk 'ក 9 Qង 6 ថj eនច,នន 10 + 7 = 17
ឈប ចន
គ ន ប ប ប&គk 'ក ឈប ចន
9 Qង 6 ថj គu P = 109 = 0.16
17

10/. ក. ក ប ប ប នឃ4ពMf%d,ងព

បSង 9Q ទ`ផ bច គ នក Mbចeនច,នន 5 ⋅ 6 = 30


ក M %ប uXY នង
ប នឃ4ពMf%d,ងព គXY ' YX eន 2 ក M
ច ន5 ប ប ប នឃ4ពMf%d,ងព គu P = =
2 1
30 15
ខ. ក ប ប បឃ4ពMf g:មយ(Tង1ច
'
ប ប មយ(
បឃ4ពMf g:មយ(Tង1ច
' Qប, ពញ ន ប ប ប ន
ឃ4ពMf%d,ងព , គ ន
1 14
P ' = 1− P = 1− =
15 15
ច ន5 ប ប ' គu P ' =
បឃ4ពMf g:មយ(Tង1ច
14
15
11/. ក. ក ប ប %'% 35 1@ នព %,M គ1
%'% 35 1@ នព %, M គ1 eននRយ) 1@ព នង
' ខ&%មយ%, M
iម ; H Maផ គ&M, គ ន
ប ប %'% 35 1@ នព %,M គ1 គu
4 3 2 4 2 3 2 4 3 3
P= ⋅ ⋅ + ⋅ ⋅ + ⋅ ⋅ =
6 5 4 6 5 4 6 5 4 5
ខ. ក ប ប %'% 35 នព %, M (Tង1ច
'
%'% 35 1@ នព %, M (Tង1ច
' eននRយ) 1@ព # 1@d,
1@d,ងប%,M
គ ន, ប ប %'% 35 នព %,M (Tង1ច
' គu
3 4 3 2 4
P= + ⋅ ⋅ =
5 6 5 4 5

កំែណេ យ ៃហ ហុន
ិ , ៃហ ចរ និងយ៉ត ពនក 95
េមេរ នទី៨ ៖ ប ប

12/. ក ប ប 5 នម&ខ H d,ងប


ក Mbច នH 5Hកប ងគu
{HHH , HHT , HTH , HTT , THH , THT , TTH , TTT } eន 8 កM

ក M %ប 5 នម&ខ H d,ងប គu { HHH } eន 1 ក M


គ ន, ប ប 5 នម&ខ H d,ងប គu
1
P=
8
13/. ក. កច,នន នឃ4ពMf g:
យង
9 ង
/ ) ក2&ងថងeន 1ឃ4ពMf % នង
' g: % ប
& 12 ;ប ^យ
9 ប ប
នH ប នឃ4ពMf g: %:9នង
1
/ 3
iង b Qច,ននឃ4ពMf g:
គ នប ប ប នឃ4ពMf g:គu P = 1 P=
b 1
12 3
គ ន
b 1
= ⇒b=4
12 3
ច ន5ក2&ងថង 35eនឃ4ពMf g:ច,នន 4 ;ប
ខ. ក ប ប ប នឃ4ពMf%
ប ប ប នឃ4ពMf% នង
' ប ប ប នឃ4ពMf g:
Q ប បប, ពញ;2 , គ ន
ប ប ប នឃ4ពMf% គu P ' = 1 − P = 1 − =
1 2
3 3
ច ន5 ប ប ប នឃ4ពMf% គu P ' =
2
3
14/. ក ប ប គ ជ%
9 %
9 ន3 d,ងព 3ក
គ ជ% % ន3 d,ងព 3ក eននRយ) ទ{Q3 eន ប ប
2
9 9 5
នង
' ទ|Q3 eន ប ប 4
1

iម ; H Maផ គ&M គ ន
ប ប គ ជ% % ន3 d,ងព 3ក គu
2 1 1
9 9 P= ⋅ =
5 4 10
15/. ក ប ប គគប ន ក A d,ងព 9ក
ក Mbចeន 3
ក M %ប 9កទ{ ន A គu 2
ក M %ប 9កទ| ន A គu 2
គ ន, ប ប គគប ន ក A d,ងព 9ក គu
2 2 4
P= ⋅ =
3 3 9

កំែណេ យ ៃហ ហុន
ិ , ៃហ ចរ និងយ៉ត ពនក 96
េមេរ នទី៨ ៖ ប ប

16/. ក. កច,
កច,ននឃ4ពMf ខ @ Pក2&ងថង 35
iង b Qច,ននឃ4ពMf ខ @ Pក2&ងថង
35 b + 15 Qច,ននឃ4% ប
& Pក2&ងថង (ក Mbច)
ប ប ប នឃ4ពMf ក^ម គu P =
15
b + 15
1 iមប,zប ប ប ប នឃ4ពMf ក^ម គu P =
3
5
គ ន
15 3
=
b + 15 5
b + 15 = 25
b = 25 − 15 = 10
ច ន5ច,ននឃ4ពMf ខ @ក2&ងថងគu 10 ;ប
ខ. កច,
កច,ននឃ4ពMf ក^ម 1@ កច ប នsមក2& ងថង
iង r Qច,ននឃ4ពMf ក^ម 1@ ថម , 35 គ ន
ច,ននឃ4ពMf ក^មក2&ងថងគu 15 + r
^យ
9 ច,ននឃ4% ប
& ក2&ងថងគu 25 + r
15 + r
35 ប ប ប នឃ4ពMf ក^មគu P =
25 + r
1iមប,zប , ប ប ប នឃ4ពMf ក^មគu
2
P=
3
គ ន,
15 + r 2
=
25 + r 3
3 (15 + r ) = 2 ( 25 + r )
45 + 3r = 50 + 2r
r = 50 − 45 = 5
ច ន5ច,
ន5ច,ននឃ4ពMf ក^ម 1@ កច ប នsមក2&ងថង គu 5 ;ប

...១២ េ ះ គឺ"នេ#ក$ង
% ឯក'ររបស់ខ,%ំ -.ន់ែតលំ0ត់1ង
ំ េ ះខ,េ%ំ ធ3ម
ើ ន
ិ អស់ ។
ែតមិនអីេទ េ-:ះពី
េ-:ះពី-សុកេ; ខ,េ%ំ លខ១
លខ១ ដល់ខ=
,%ំ ក់
> ពួកេគែដលេលខេ-@មខ,=
%ំ ក់
> អស់
មិតខ
A ែ,%ំ ដល-តBវេ;-បឡងទូ1ង
ំ -បេទស នបេងFន
ើ @រសិកG្ាេទ3ដង ។ ទីបផ
ំ ត
ុ េគក៏
=>ក់ នសេ-មចL'Mពរ ។
ឲស">ញ់េអើយ េនះគឺL@ររPលក
ឹ អនុសRវរSយរ៍ បស់ពក
ួ េយើង ។ េពលែដល
គិតដល់េពលេ ះមUងៗ
ងៗ ខ,"
%ំ នWរមXណរ៍ ក
S Yយ េហើយេគងលក់-សួលេទៀតផង ។

កំែណេ យ ៃហ ហុន
ិ , ៃហ ចរ និងយ៉ត ពនក 97
េមេរ នទី៩ ៖ ច យរ ងពីចណ
ំ ុ ច

ម នទ ច យ ងពច ច

1/. ច គ ច យ ងព ច ច នក ន ច ង ម
ក. ( 9, 2 ) នង
" ( 6,8) ខ. ( 5, −3) នង
" ( −2, 4 )
គ. ( 2,5 ) នង
" ( 0, 0 ) ឃ. ( −6, −3) នង
" ( 2,1)
ង. (8, 7 ) នង
" ( 8, −6 ) ច. ( 3,12 ) នង
" ( 7,12 )
ឆ. (100, −203) នង
" ( 97, −200 ) ជ. ( −3,8 ) នង
" ( −4, 7 ) '
2/. ចង(ងព )ន ង*+ផ
, "+
. ង/ងម
, យ
0 នក ន ង12 ( 3,1) នង
" ( −2,5 ) '
ចគ ង/ង, ច
0 3ង, ង/ង, ន4 '
3/. ច គ ប 6ង ង*+, ទ7ង)នច+ PQRS P ( −2,3) , Q ( 5,5) , R ( 6, −6 )

នង
" S ( −3, −3) '
4/. ច ប89ញ; + KLM K ( 2,8 ) , L (10,11) នង
" M ( 5, 0 ) < +
3ម=+ '
5/. ច គ ប " +)ន + ABC A ( 3, 7 ) , B ( 5, 2 ) នង
" C ( −7,3) '
ញ; ABC < +
ច ប89ញ; កង ច
0 គ )ផ> ក? ប3, '
6/. ច ក+)ម@ y ម
A BCDច យ ងព ច ច នក ន ( 4, 2 ) នង
" ( 4, y )
3EA នង
F 5 GកH '
7/. ច កច ច C ( x,1) ក2ង ង,ទ1 ម
A BCD + នកព
A (1,1) , B ( 4, 7 ) នង
" C ( x,1) < + 3ម=+ AB = AC '

8/. កព )ន + ម0យ នក ន ( −3, 2 ) , ( 9, 2 ) នង


" ( 3,10 ) '
ក. ច គ ប 6ង ជIង(ងប)ន + '
ខ. ច =ប,
=ប, ប ភទ)ន + ន4 '
9/. កព )នច+ ម0យ នក ន ( −1, −2 ) , ( 5, −2 ) , ( −1, 4 ) នង
" ( 5, 4 ) '
ក. ច គ ប 6ង ជIង នង
" ប 6ង ង*+, ទ7ង)នច+ '
ខ. +A គ3ន2" K ន ច មLចច M4ច+ ន4 ?

កំែណេ យ ៃហ ហុន
ិ , ៃហ ចរ និងយ៉ត ពនក 98
េមេរ នទី៩ ៖ ច យរ ងពីចណ
ំ ុ ច

10/. ក នកព ប3,ច+ ម0យ ន ( 2, −1) , ( 6, 4 ) , ( 2,9 ) នង


" (−2 , 4) '
ក. ច គ ប 6ង ជIង នង
" ប 6ង ង*+, ទ7ង)នច+ '
ខ. ច =ប, ប ភទ)នច+ ន4 '
11/. ច+ ម0យ នកព ( −2, −3) , ( −3,1) , ( 5, −1) នង
" ( 4,3) '
+ច
A + ន4 នOង< / ?
12/. ច គ ក ន)នច
ន)នច ចកPL )ន AB ច
0 Qច ចក2ង+ មIយ
+ មHម
មHមក
Hមក នម0យR ច ង ម
ក. A ( 5,8 ) នង
" B ( 5,12 ) ខ. A ( −5, 2 ) នង
" B ( −5,14 ) '
13/. បA បមនLក ន)នច ចកPL ង*+, ' ច គ ក នច ច
កPL )ន ង*+, AB ក2ងក នម0យR ច ង ម
ក. A ( −3, −5 ) នង
" B ( 7, −5 ) ខ. A ( −8, −6 ) នង
" B ( 4, 2 )
គ. A ( 3t + 5, −7 ) នង
" B ( t + 7, 7 ) , t ≠ 1 ឃ. A ( a, b ) នង
" B ( a, c ) '
14/. ច គ ក ន)នច ចកPL M )ន ង*+ , ងព ច ច
A(−3, − 1) នង
" B(5, 7) ' ច គ ប 6ង AM នង
" BM '
15/. គCD M (1, −2 ) <ច ចកPL )ន ង*+, AB A ( −2, 2 ) '

ចគ ក ន)នច ច B '
16/. ង/ងម
, យ
0 " F ( −2,6 ) ' ង*+ផ
នផ.+ , +
". ង/ង, ន4 នច ចចងម0យ +ង,
V ( 3,8) ' គ ក ន)នច ចT <ច ចចង S ង ទ +)ន
ង*+ផ ". TV ច
, + 0 3ង, ង/ង, ន4 '
17/. ក ន)នកព ប T មម0យ 3EA (1,1) , ( 2, −3) , ( −3, −1) នង
" ( −2, −5 ) '
ចគ ក ន)នច ចកPL )ន ង*+, ទ7ង(ងព ' +A គUច
ន=នV
3ន2" K ន=នVWង ច មLច ?
18/. ច+ ម0យ នកព E ( 5,3) , F ( 5, −5 ) , G ( −3, −3) នង
" H ( −1,5 ) ' ច
គ ក ន)នច ចកPL A , B , C នង
" D )ន ជIង EF , FG
GH នង
" EH ' +ច
A + ABCD នOង< / ?
19/. ច+ កងម0យក +, យច ច A ( 3, 2 ) , B ( 9, 2 ) , C ( 9, 6 ) នង
" D ( 3, 6 ) '
ច XយបYZ ក,;ច+ ក +, យច ចកPL )ន ជIង(ង
ប0ន ប3, ច+ កង ង A <ច+ 3EA '

កំែណេ យ ៃហ ហុន
ិ , ៃហ ចរ និងយ៉ត ពនក 99
េមេរ នទី៩ ៖ ច យរ ងពីចណ
ំ ុ ច

20/. ច+ ម0យ នកព P ( −3, −2 ) , Q ( −1,5 ) , R ( 3,1) នង


" S ( 5, −4 ) ' បA
A , B , C នង
" D <ច ចកPL ង12)ន ជIង PQ , QR , RS នង
"
SP ' ច ប89ញ; ABCD < ប T ម'
21/. គ ន ង*+, P1 P2 P1 ( x1 , y1 ) នង
" P2 ( x2 , y2 ) ' ច គ ក ន
)នច ច A នង
" B <ច ច ចក ង*+, P1 P2 <ប ផ2ក 3EAR12 '
x1 + x2 y1 + y2 
22/. គ នច ច P1 ( x1 , y1 ) , Pm  ,  នង
" P2 ( x2 , y2 ) '
 2 2 
ច ប89ញ;
ញ; P1 Pm + Pm P2 = P1 P2 '

23/. គCDច ច A (1,0 ) នង


" B ( 3, 2 ) ច
0
3ង, ង/ង, ន ង*+ផ ". AB '
, +
ក. ច កក ន)នផ.
ន)នផ.+
" ង/ង,
Hង យច ច I '
ខ. M <ច ចម0យ ន

ក ន  , y  ច ក +,
5

2 
y ម
A BCD M [+
" \ A ង/ង, ច
0 បY
បYZក,; +A M នទHងបW E ន ?

ច មយ
1/. គ ច យ ងព ច ច នក ន ច ង ម
Hម បមនLច យ ងព ច ច A ( x1 , y1 ) នង
" B ( x2 , y2 ) គ]
AB = ( x2 − x1 ) + ( y2 − y1 ) គ=ន
2 2

ក. ( 9, 2 ) នង
" ( 6,8) , 4ច យ ងព ច ច 4គ]
( 6 − 9) + (8 − 2) = 9 + 36 = 45 = 3 5 GកH ប 6ង '
2 2

ខ. ( 5, −3) នង
" ( −2, 4 ) , 4ច យ ងព ច ច 4គ]

( −2 − 5 ) + ( 4 − ( −3) ) = 49 + 49 = 7 2 GកH ប 6ង '


2 2

គ. ( 2,5 ) នង
" ( 0, 0 ) , 4ច យ ងព ច ច 4គ]
( 0 − 2 ) + ( 0 − 5) = 4 + 25 = 29 GកH ប 6ង '
2 2

ឃ. ( −6, −3) នង
" ( 2,1) , 4ច យ ងព ច ច 4គ]

( 2 − ( −6 ) ) + (1 − ( −3) ) = 64 + 16 = 80 = 4 5 GកH ប 6ង '


2 2

កំែណេ យ ៃហ ហុន
ិ , ៃហ ចរ និងយ៉ត ពនក 100
េមេរ នទី៩ ៖ ច យរ ងពីចណ
ំ ុ ច

ង. (8, 7 ) នង
" ( 8, −6 ) , 4ច យ ងព ច ច 4គ]
(8 − 8) + ( −6 − 7 ) = 0 + 132 = 13 GកH ប 6ង '
2 2

ច. ( 3,12 ) នង
" ( 7,12 ) , 4ច យ ងព ច ច 4គ]
( 7 − 3) + (12 − 12 ) = 42 + 0 = 4 GកH ប 6ង '
2 2

ឆ. (100, −203) នង
" ( 97, −200 ) , 4ច យ ងព ច ច 4គ]
( 97 − 100 ) + ( −200 − ( −203) ) = 9 + 9 = 3 2 GកH ប 6ង '
2 2

ជ. ( −3,8 ) នង
" ( −4, 7 ) , 4ច យ ងព ច ច 4គ]

( −4 − ( −3 ) ) + ( 7 − 8 ) = 1 + 1 = 2 GកH ប 6ង '
2 2

2/. គ ង/ង,
HមបOប, , ចង(ងព )ន ង*+ផ
, +
". ង/ង, នក ន ( 3,1) នង
" ( −2,5 )
គ=ន
ង*+ផ
, +
". )ន ង/ងគ
, ] d= ( −2 − 3) + ( 5 − 1) = 25 + 16 = 41 GកH ប 6ង
2 2

4, )ន ង/ង, 4គ] GកH ប 6ង '


d 41
r= =
2 2
3ង, ង/ង,

3/. គ ប 6ង ង*+, ទ7ង)ន PRQS

កំែណេ យ ៃហ ហុន
ិ , ៃហ ចរ និងយ៉ត ពនក 101
េមេរ នទី៩ ៖ ច យរ ងពីចណ
ំ ុ ច

យង
A ន P ( −2,3) , R ( 6, −6 ) , Q ( 5,5 ) នង
" S ( −3, −3)
គ=ន , ង*+, ទ7ង)នច+ PRQS គ]

( 6 − ( −2 ) ) + ( −6 − 3 ) = 64 + 81 = 145 GកH ប 6ង
2
PR =
2

នង
" QS = ( −3 − 5 ) + ( −3 − 5 ) = 8 + 8 = 8 2 GកH ប 6ង
2 2 2 2

4/. ប89ញ; + KLM < + 3ម=+


+ KLM ន K ( 2,8 ) , L (10,11) នង
" M ( 5, 0 ) , គ=ន
KL = (10 − 2 ) + (11 − 8) = 64 + 9 = 73 GកH ប 6ង
2 2

LM = ( 5 − 10 ) + ( 0 − 11) = 25 + 121 = 146 GកH ប 6ង


2 2

MK = ( 2 − 5) + (8 − 0 ) = 9 + 64 = 73 GកH ប 6ង
2 2

យ KL = MK
ច ន4 + KLM < + 3ម=+ កព K '

5/. គ ប " +)ន + ABC

+ ABC ន A ( 3, 7 ) , B ( 5, 2 ) នង
" C ( −7,3) , គ=ន
AB = ( 5 − 3) + ( 2 − 7 ) = 4 + 25 = 29 GកH ប 6ង
2 2

BC = ( −7 − 5 ) + ( 3 − 2 ) = 144 + 1 = 145 GកH ប 6ង


2 2

( 3 − ( −7 ) ) + ( 7 − 3) = 100 + 16 = 116 = 2 29 GកH ប 6ង


2 2
CA =

គ=ន , ប " +)ន + ABC គ]

P = AB + BC + CA = 29 + 145 + 2 29 = 3 29 + 145 GកH ប 6ង '

ប89ញ;
ញ; ABC < + កង
Hម3 យ ង A , យង
A ន AB = 29 , BC = 145 , CA = 116

គ=ន
AB 2 = 29
BC 2 = 145
CA2 = 116

⇒ AB 2 + CA2 = BC 2
ច ន4 ABC < + កង +ង,កព A '

គ )ផ> ក? ប3,
យ ABC < + កង +ង,កព A , គ=ន

ក?)ផ>)ន + ABC គ]
1 1
A= ⋅ AB ⋅ AC = ⋅ 29 ⋅ 2 29 = 29
2 2

កំែណេ យ ៃហ ហុន
ិ , ៃហ ចរ និងយ៉ត ពនក 102
េមេរ នទី៩ ៖ ច យរ ងពីចណ
ំ ុ ច

ច ន4 ABC ន)ផ> ក? A = 29 GកH)ផ> '


6/. ក+)ម@ y
HមបOប, ,ច
,ច យ ងព ច ច ( 4, 2 ) នង
" ( 4, y ) 3EA នង
F 5 GកH
គ=ន
( 4 − 4) + ( y − 2) =5
2 2

( y − 2) =5
2

y−2 =5
 y − 2 = −5  y = −3
y −2 = 5 ⇒ y = 7
 
ច ន4 y = −3 _ y = 7 +76=នក +, '
7/. កច ច C ( x,1) ក2ង ង,ទ1
យង
A ន A (1,1) , B ( 4, 7 ) នង
" C ( x,1) `យ
A C \ក2ង ង,ទ1 4x >0
គ=ន AB = ( 4 − 1) + ( 7 − 1) = 9 + 36 = 45 = 3 5 GកH ប 6ង
2 2

នង
" AC = ( x − 1) + (1 − 1) = ( x − 1) = x − 1 GកH ប 6ង
2 2 2

+ យ AB = AC , គ=ន
x −1 = 3 5
 x −1 = 3 5 x = 1+ 3 5
 ⇒
 x − 1 = −3 5  x = 1 − 3 5
យ x = 1 − 3 5 < 0 យកមន
" ក+
A
ច ន4 C (1 + 3 5,1) +76=នក +, '
8/. ក. គ ប 6ង ជIង(ងប)ន +
យង
A នកព (ងប)ន + គ] ( −3, 2 ) , ( 9, 2 ) នង
" ( 3,10 ) គ=ន
ប 6ង ជIង(ងប)ន + ន4គ] ន

( 9 − ( −3 ) ) + ( 2 − 2 ) = 12 2 + 0 = 12 GកH ប 6ង
2 2

( 3 − 9 ) + (10 − 2 ) = 36 + 64 = 10 GកH ប 6ង
2 2

( 3 − ( −3) ) + (10 − 2 ) = 36 + 64 = 10 GកH ប 6ង


2 2

ខ. =ប, ប ភទ)ន + ន4
ក2ង + ង A, ន ជIងព ន 8/3, 10 GកH 3EA12 '
ច ន4 + ង A គ<
] + 3ម=+ '

កំែណេ យ ៃហ ហុន
ិ , ៃហ ចរ និងយ៉ត ពនក 103
េមេរ នទី៩ ៖ ច យរ ងពីចណ
ំ ុ ច

9/. ក. គ ប 6ង ជIង នង
" ប 6ង ង*+, ទ7ង)នច+

យង
A ន ,កព
,កព )នច+ គ] ( −1, −2 ) , ( 5, −2 ) , ( 5, 4 ) នង
" ( −1, 4 ) គ=ន
ប 6ង ជIង ប3, គ]
ជIង ( −1, −2 ) នង
" ( 5, −2 ) ( 5 − ( −1) ) + ( −2 − ( −2 ) ) = 6 GកH ប 6ង
2 2

ជIង ( 5, −2 ) នង
" ( 5, 4 ) ( 5 − 5 ) + ( 4 − ( −2 ) ) = 6 GកH ប 6ង
2 2

ជIង ( 5, 4 ) នង
" ( −1, 4 ) ( −1 − 5 ) + ( 4 − 4 ) = 6 GកH ប 6ង
2 2

ជIង ( −1, 4 ) នង
" ( −1, −2 ) ( −1 − ( −1) ) + ( 4 − ( −2 ) ) = 6 GកH ប 6ង
2 2

ប 6ង ង*+, ទ7ង ប3, គ]


ង*+, ( 5, −2 ) នង
" ( −1, 4 ) ( −1 − 5 ) + ( 4 − ( −2 ) ) = 6 2 GកH ប 6ង
2 2

ង*+, ( −1, −2 ) នង
" ( 5, 4 ) ( 5 − ( −1) ) + ( 4 − ( −2 ) ) = 6 2 GកH ប 6ង
2 2

ខ. +A គ3ន2" K ន ច មLចច M4ច+ ន4 ?


ច+ ង A ន ជIង(ងប0ន 3EA12 នង
" ង*+, ទ7ង(ងព 3EA12
4 គUច
គUច3ន2 ន=ន;
3ន2" K ន=ន; , ច+ ង A< W'
10/. ក. គ ប 6ង ជIង នង
" ប 6ង ង*+, ទ7ង)នច+

កំែណេ យ ៃហ ហុន
ិ , ៃហ ចរ និងយ៉ត ពនក 104
េមេរ នទី៩ ៖ ច យរ ងពីចណ
ំ ុ ច

យង
A ន, កព ប3,ច+ 4 គ] ( 2, −1) , ( 6, 4 ) , ( 2,9 ) នង
" (−2 , 4)
គ=ន
ប 6ង ជIង ប3, គ]
ជIង ( 2, −1) នង
" ( 6, 4 ) ( 6 − 2 ) + ( 4 − ( −1) ) = 41 GកH ប 6ង
2 2

ជIង ( 6, 4 ) នង
" ( 2,9 ) ( 2 − 6) + (9 − 4) = 41 GកH ប 6ង
2 2

ជIង ( 2,9 ) នង
" ( −2, 4 ) ( −2 − 2 ) + ( 4 − 9 ) = 41 GកH ប 6ង
2 2

ជIង ( −2, 4 ) នង
" ( 2, −1) ( 2 − ( −2 ) ) + ( −1 − 4 ) = 41 GកH ប 6ង
2 2

ប 6ង ង*+, ទ7ង ប3, គ]


ង*+, ( 2, −1) នង
" ( 2,9 ) ( 2 − 2 ) + ( 9 − ( −1) ) = 10 GកH ប 6ង
2 2

ង*+, ( 6, 4 ) នង
" ( −2, 4 ) ( −2 − 6 ) + ( 4 − 4 ) = 8 GកH ប 6ង
2 2

ខ. =ប, ប ភទ)នច+ ន4
ច+ ង A ន 8/3, ជIង(ងa
(ងa 3EA12 + ង*+, ទ7ង(ងb
(ងb
មន
" 3EA12 ទ '
ច ន4ច+
ន4ច+ ង A < ច+ 3EA '
11/. +ច
A + ន4 នOង< / ?
យង
A ន, កព ប3,ច+
ច+ 4 ( −2, −3) , ( −3,1) , ( 4,3) នង
" ( 5, −1) '

គ=ន
ប 6ង ជIង ប3, គ]
ជIង ( −2, −3) នង
" ( −3,1) ( −3 − ( −2 ) ) + (1 − ( −3) ) = 17 GកH ប 6ង
2 2

ជIង ( −3,1) នង
" ( 4,3) ( 4 − ( −3) ) + ( 3 − 1) = 53 GកH ប 6ង
2 2

កំែណេ យ ៃហ ហុន
ិ , ៃហ ចរ និងយ៉ត ពនក 105
េមេរ នទី៩ ៖ ច យរ ងពីចណ
ំ ុ ច

ជIង ( 4,3) នង
" ( 5, −1) ( 5 − 4 ) + ( −1 − 3 ) = 17 GកH ប 6ង
2 2

ជIង ( 5, −1) នង
" ( −2, −3) ( −2 − 5 ) + ( −3 − ( −11) ) = 53 GកH ប 6ង
2 2

ប 6ង ង*+, ទ7ង ប3, គ]


ង*+, ទ7ង ( −2, −3) នង
" ( 4,3) ( 4 − ( −2 ) ) + ( 3 − ( −3 ) ) = 72 GកH ប 6ង
2 2

ង*+, ទ7ង ( −3,1) នង


" ( 5, −1) ( 5 − ( −3) ) + ( −1 − 1) GកH ប 6ង
2 2
= 68

យច+ ន4 ន 8/3, ជIងឈម 3EA12ព R នង


" ប 6ង ង*+, ទ7ង
(ងព មន
" 3EA12 '
ច ន4 ច+ ង A<ប T ម'
12/. គ ក ន)នច ចកPL )ន AB ច
0 Qច ចក2ង+ មIយ
+ ម
ក. A ( 5,8 ) នង
" B ( 5,12 )
យ A នង
" B នUប,33 5 A Hង M ( 5, y ) <ច
ច12 , យង ច
កPd )ន ង*+, AB គ=ន
AM = BM

( 5 − 5) + ( y − 8) = ( 5 − 5) + ( y − 12 )
2 2 2 2

( y − 8 ) = ( y − 12 )
2 2

( y − 8 ) − ( y − 12 ) = 0
2 2

( ( y − 8) − ( y − 12 ) ) ( ( y − 8) + ( y − 12 ) ) = 0
4 ( 2 y − 20 ) = 0
2 y = 20
y = 10
ច ន4ច ចកPd )ន AB គ] M ( 5,10 ) '
Qច ចក2ង+ មIយ + ម

កំែណេ យ ៃហ ហុន
ិ , ៃហ ចរ និងយ៉ត ពនក 106
េមេរ នទី៩ ៖ ច យរ ងពីចណ
ំ ុ ច

ខ. A ( −5, 2 ) នង
" B ( −5,14 )
យ A នង
" B នUប,33 −5 A Hង M ( −5, y ) <ច
ច12 , យង ច
កPd )ន ង*+, AB គ=ន
AM = BM

( −5 − ( −5) ) + ( y − 2 ) ( −5 − ( −5) ) + ( y − 14 )
2 2
=
2 2

( y − 2 ) = ( y − 14 )
2 2

( y − 2 ) − ( y − 14 ) = 0
2 2

( ( y − 2 ) − ( y − 14 ) ) ( ( y − 2 ) + ( y − 14 ) ) = 0
12 ( 2 y − 16 ) = 0
2 y = 16
y =8
ច ន4ច ចកPd )ន AB គ] M ( −5,8) '
Qច ចក2ង+ មIយ + ម

13/. បA បមនLក ន)នច ចកPL )ន ង*+, , គ ក នច ច


កPL )ន ង*+, AB ក2ងក នម0យR ច ង ម
Hម បមនL, ច ចកPd )ន AB A ( x1 , y1 ) នង
" B ( x2 , y2 ) គ]

កំែណេ យ ៃហ ហុន
ិ , ៃហ ចរ និងយ៉ត ពនក 107
េមេរ នទី៩ ៖ ច យរ ងពីចណ
ំ ុ ច

x1 + x2 y + y2
M ( x, y ) x= នង
" y= 1
2 2
ក. A ( −3, −5 ) នង
" B ( 7, −5 )
គ=ន
x1 + x2 −3 + 7
x= = =2
2 2
y +y −5 + ( −5 )
y= 1 2 = = −5
2 2
ច ន4ច ច M ( 2, −5) +76=នក +, '
ខ. A ( −8, −6 ) នង
" B ( 4, 2 )
គ=ន
គ=ន
x1 + x2 −8 + 4
x= = = −2
2 2
y + y 2 −6 + 2
y= 1 = = −2
2 2
ច ន4ច ច M ( −2, −2 ) +76=នក +, '
គ. A ( 3t + 5, −7 ) នង
" B ( t + 7, 7 ) , t ≠ 1
គ=ន
x1 + x2 3t + 5 + t + 7 4t + 12
x= = = = 2t + 6 , t ≠ 1
2 2 2
y + y2 −7 + 7
y= 1 = =0
2 2
ច ន4ច ច M ( 2t + 6, 0 ) , t ≠ 1 +76=នក +, '
ឃ. A ( a, b ) នង
" B ( a, c )
គ=ន
x1 + x2 a + a
x= = =a
2 2
y + y2 b + c
y= 1 =
2 2
b+c
ច ន4ច ច M  a,  +76=នក +, '
 2 
14/. គ ក ន)នច ចកPL M

យង
A ន M <ច ចកPd )ន ង*+, ងព ច ច A(−3, − 1) នង
"
B (5, 7) គ=ន
x1 + x2 −3 + 5
x= = =1
2 2
y + y 2 −1 + 7
y= 1 = =3
2 2

កំែណេ យ ៃហ ហុន
ិ , ៃហ ចរ និងយ៉ត ពនក 108
េមេរ នទី៩ ៖ ច យរ ងពីចណ
ំ ុ ច

ច ន4ច ច M (1,3) +76=នក


=នក +, '
គ ប 6ង AM នង
" BM
យង
A ន A(−3, − 1) នង
" B(5, 7) គ=ន
( 5 − ( −3) ) + ( 7 − ( −1) )
2 2
AB =

AB = 64 + 64
AB = 8 2 GកH
GកH ប 6ង ប 6ង
ច ន4 AM = BM = = 4 2 GកH ប 6ង +76=នក +, '
AB
2
15/. គ ក ន)នច ច B
យង
A ន , M (1, −2 ) <ច ចកPL )ន ង*+, AB A ( −2, 2 )

គ=ន
 x1 + x2
 x = 2

 y = y1 + y2
 2
 −2 + x2
1 = 2

 −2 = 2 + y 2
 2
 x2 = 4

 y 2 = −6
ច ន4 ក នច ច B គ] ( 4, −6 ) +76=នក +, '
16/. គ ក ន)នច ចT
យ TV < ង*+ផ
, +
". )ន ង/ង,
ង, នផ.+
" F F ( −2,6 ) នង
" V ( 3,8)
ននeយ; F <ច ចកPd )ន ង*+, TV , គ=ន
 x1 + x2
 x = 2

 y = y1 + y2
 2
 3 + x2
 − 2 =
2

 6 = 8 + y2
 2
 x2 = −7

 y2 = 4
ច ន4 T ( −7, 4 ) +76=នគ '

កំែណេ យ ៃហ ហុន
ិ , ៃហ ចរ និងយ៉ត ពនក 109
េមេរ នទី៩ ៖ ច យរ ងពីចណ
ំ ុ ច

3ង, ប

17/. គ ក ន)នច ចកPL )ន ង*+, ទ7ង(ងព


យង
A ន, កព )ន ប T មគ
មគ] (1,1) , ( 2, −3) , ( −2, −5 ) នង
" ( −3, −1)

គ=ន
ច ចកPd )ន ង*+, ទ7ង (1,1) , ( −2, −5) គ]
 1 + ( −2 ) 1 + ( −5 )   1 
 ,  =  − , −2 
 2 2   2 
ច ចកPd )ន ង*+, ទ7ង ( 2, −3) , ( −3, −1) គ]
 2 + ( −3) −3 + ( −1)   1 
 ,  =  − , −2 
 2 2   2 
ច ន4ច ចកPd )ន ង*+, ទ7ង ប3, ប T ម ង Aគ<
]
ច ច  − , −2  ម
0 12 '
1
 2 

កំែណេ យ ៃហ ហុន
ិ , ៃហ ចរ និងយ៉ត ពនក 110
េមេរ នទី៩ ៖ ច យរ ងពីចណ
ំ ុ ច

ន=នV
+A គUច3ន2" K ន=នVWង ច មLច ?
យ យង
A 3 ង*+ ឃញ
A ;ច ចកPd )ន ង*+, ទ7ង(ងព <ច ច
+ម0យ , 4 យង
A Uច3ន2" K ន=ន; ង*+, ទfង(ងព )ន ប T ម
ម0យ នច ចកPd ម
0 12 _ UចនV
" យ; +,12 +ង,ច ច
កPd '
18/. គ ក ន)នច ច A , B , C នង
" D

យង
A ន , កព )នច+
)នច+ គ] E ( 5,3) , F ( 5, −5 ) , G ( −3, −3) នង
" H ( −1,5 )
`យ
A ច ច A , B , C នង
" D )ន ជIង EF , FG , GH នង
" EH , គ=ន
 5 + 5 3 + ( −5 ) 
A ,  = A ( 5, −1)
 2 2 
 5 + ( −3) −5 + ( −3) 
B ,  = B (1, −4 )
 2 2 
 −3 + ( −1) −3 + 5 
C ,  = C ( −2,1)
 2 2 
 5 + ( −1) 3 + 5 
D ,  = D ( 2, 4 )
 2 2 

ច ន4 A ( 5, −1) , B (1, −4 ) , C ( −2,1) , D ( 2, 4 ) +76=នក +, '


+ច
A + ABCD នOង< / ?
ច+ ABCD < ប T ម M4
A <ច ចកPd )ន EF
B <ច ចកPd )ន FG
C <ច ចកPd )ន GH
D <ច ចកPd )ន HE

កំែណេ យ ៃហ ហុន
ិ , ៃហ ចរ និងយ៉ត ពនក 111
េមេរ នទី៩ ៖ ច យរ ងពីចណ
ំ ុ ច

CD , AB <=+មធDម ប3, + EFG ⇒ AB / / EG (1)


BC <=+មធDម ប3, + FGH ⇒ BC / / FH ( 2)
CD <=+មធDម ប3, + GHE ⇒ CD / / GE ( 3)
DA <=+មធDម ប3, + HEF ⇒ DA / / HF ( 4)
Hម (1) នង
" ( 3) CD AB / / CD ( 5)
( 2 ) នង
" ( 4) CD BC / / DA ( 6)
Hម ( 5 ) នង
" ( 6 ) គ=ន ABCD < ប T ម'

យង ច កក នច ចកPd )ន ង*+, ទ7ង(ងព AC នង


" BD ច
0 (ញ

ព 3ន2" K នក2ង h+, ខ ij កk=ន '


19/. XយបYZ ក,;ច+ ក +, យច ចកPL )ន ជIង(ង
ប0ន ប3, ច+ កង ង A <ច+ 3EA

យង
A ន, ច+ កងម0យ ន A ( 3, 2 ) , B ( 9, 2 ) , C ( 9, 6 ) នង
" D ( 3, 6 )
<កព , គ=ន ក នច ចកPd )ន ជIង AB , BC , CD នង
" DA គ]
 3+9 2+ 2 
 ,  = ( 6, 2 )
 2 2 
9+9 2+6
 ,  = ( 9, 4 )
 2 2 
9+3 6+6
 ,  = ( 6, 6 )
 2 2 
 3+3 2+ 6 
 ,  = ( 3, 4 ) ង12 '
 2 2 
`យ
A គ=ន ច+ នកព ( 6, 2 ) , ( 9, 4 ) , ( 6, 6 ) , ( 3, 4 ) < ប
T ម ' ( មA h+,ទ im) (1)
A 3ព ន4 o ទ + , ច+ ង A ន 8/3,

កំែណេ យ ៃហ ហុន
ិ , ៃហ ចរ និងយ៉ត ពនក 112
េមេរ នទី៩ ៖ ច យរ ងពីចណ
ំ ុ ច

ជIង ( 6, 2 ) , ( 9, 4 ) គ] (9 − 6) + ( 4 − 2) = 9 + 4 = 13 GកH ប 6ង
2 2

នង
" ជIង ( 9, 4 ) , ( 6, 6 ) គ] (6 − 9) + ( 6 − 4) = 9 + 4 = 13 GកH ប 6ង
2 2

CDច+ ង A ន ជIង<ប,ព ន 8/3, 3EA12 ( 2)


Hម (1) នង
" ( 2)
CD ច+ ង A ន4<ច+ 3EA +76=ន XយបYZក, '
20/. ប89ញ; ABCD < ប T ម

យង
A ន ច+ នកព P ( −3, −2 ) , Q ( −1,5 ) , R ( 3,1) នង
" S ( 5, −4 )
`យ
A A , B , C នង
" D កPL ង12)ន ជIង PQ , QR , RS នង
" SP
គ=ន
 −3 + ( −1) −2 + 5   3
A ,  = A  −2, 
 2 2   2
 −1 + 3 5 + 1 
B ,  = B (1,3)
 2 2 
 3 + 5 1 + ( −4 )   3
C ,  = C  4, − 
 2 2   2
 −3 + 5 −2 + ( −4 ) 
D ,  = D (1, −3)
 2 2 
ច+ ABCD ន ង*+, ទ7ង AC នង
" BD ច ចកPd )ន
 3  3
 −2 + 4 2 +  − 2  
AC គ]  ,    = 1, 0
( )
 2 2 
 
 

កំែណេ យ ៃហ ហុន
ិ , ៃហ ចរ និងយ៉ត ពនក 113
េមេរ នទី៩ ៖ ច យរ ងពីចណ
ំ ុ ច

 1 + 1 3 + ( −3) 
នង
" BD គ]  2 ,  = (1, 0 )
 2 

យង A ; ង*+, ទ7ង AC នង
A ឃញ " BD នច ចកPd ម
0 12 '
ច ន4 ច+ ABCD < ប T ម +76=នក +, '
21/. គ ក ន )នច ច A នង
" B
យង
A ន ង*+, P1 P2 P1 ( x1 , y1 ) នង
" P2 ( x2 , y2 ) `យ
A A នង
" B<
ច ច ចក ង*+, P1 P2 <ប ផ2ក 3EAR12 , គ=ន
4 P1 A = AB = BP2
ននeយ; A កPd P1 B នង
" B កPd AP2

Hង A ( xA , y A ) នង
" B ( xB , y B ) 4 គ=ន
  x1 + xB y1 + yB 
( x A , y A ) =  2 , 2 
  

( x , y ) =  x A + x2 , y A + y2 
 B B  2 2 

2 xA = x1 + xB (1)

2 y A = y1 + yB ( 2)

2 xB = x A + x2 ( 3)
2 y = y + y
 B A 2 ( 4)
 2 x1 + x2
 x =
2 x A − xB = x1 A
Hម (1) នង
" ( 3) គ=ន  ⇒
3
2 xB − x A = x2  x = x1 + 2 x2
 B 3
 2 y + y2
 yA = 1
 A
2 y − y = y 
Hម ( 2 ) នង
" ( 4 ) គ=ន 
B 1
⇒
3
 2 y B − y A = y2  y = y1 + 2 y2
 B 3

កំែណេ យ ៃហ ហុន
ិ , ៃហ ចរ និងយ៉ត ពនក 114
េមេរ នទី៩ ៖ ច យរ ងពីចណ
ំ ុ ច

2 x1 + x2 2 y1 + y2   x + 2 x2 y1 + 2 y2 
ច ន4 A  ,  នង
" B 1 ,  +76=នក +, '
 3 2   3 3 
22/. ប89ញ;
ញ; P1 Pm + Pm P2 = P1 P2
x1 + x2 y1 + y2 
យង
A ន ច ច P1 ( x1 , y1 ) , Pm  ,  នង
" P2 ( x2 , y2 )
 2 2 
គ=ន
P1 P2 = ( x2 − x1 ) + ( y2 − y1 )
2 2

x +x  y +y
2 2

P1 Pm =  1 2 − x1  +  1 2 − y1 
 2   2 
 x + x − 2 x1   y1 + y2 − 2 y1 
2 2

=  1 2  + 
 2   2 

( x2 − x1 ) (y − y )
2 2

= + 2 1
4 4
( x2 − x1 ) + ( y2 − y1 )
2 2

=
2
P1 P2
=
2
x +x   y +y 
2 2

នង
" P2 Pm =  x2 − 1 2 2  +  y2 − 1 2 2 
   
 2x − x − x   2 y − y − y 
2 2

=  2 1 2  + 2 1 2 
 2   2 

( x2 − x1 ) (y − y )
2 2

= + 2 1
4 4
( x2 − x1 ) + ( y2 − y1 )
2 2

=
2
P1 P2
=
2
គ=ន P1 Pm + Pm P2 = 1 2 + 1 2 = P1 P2
PP PP
2 2
ច ន4 P1 Pm + Pm P2 = P1 P2 +76=ន XយបYZក, '
23/. ក. កក ន)នផ.+
" ង/ង, Hង យច ច I
យង
A ន ង/ង, ន ង*+ផ
, +
". AB A (1,0 ) នង
" B ( 3, 2 )
4ផ.+
" )ន ង/ង, គ] I <ច ចកPd AB , 4ក ន)ន I គ]
1+ 3 0+2
x= =2, y= =1
2 2
ច ន4 I ( 2,1) +76=នក +, '

កំែណេ យ ៃហ ហុន
ិ , ៃហ ចរ និងយ៉ត ពនក 115
េមេរ នទី៩ ៖ ច យរ ងពីចណ
ំ ុ ច

ខ. ក y

យង ន M  , y  <ច ច\ A ង/ង, , គ=ន


5
A
2 
IM = IA ( ង/ង),
2
5 
 − 2  + ( y − 1) = (1 − 2 ) + ( 0 − 1)
2 2 2

2 
1
+ ( y − 1) = 1 + 1
2

4
1
+ ( y − 1) = 2
2

4
1
( y − 1) = 2 −
2

4
7
y = 1±
2

ច ន4 y = 1 − _ y = 1+ +76=នក +, '
7 7
2 2
+A M នទHងបW E ន ?

យ M ន+)ម@ព ផpង12 គ] y = 1 − នង
7 7
2
" y = 1+ 2
5 7 5 7
ច ន4 M នព ទHងគ] M  ,1 −  នង
" M  ,1 +  '
2 2  2 2 

េ ំ ២០០៦ ប"ន
# $សីខ()ំ ៃហ ចរ ន$ត*វ,ប់,សិស.ពូ
ស.ពូែកគណិតវទ2
3ក់ទី ៩ ក5ង
) 67 េហើយ$ត*វ$បឡងទូ:ង
ំ $សុក ។ ែតេពលេ=ះខ(ម
)ំ ន
ិ @នលទABព
CចជួយបFGត់បេ$ងៀនប"ន
# ៗ ន =ងក៏ នKLក់េ 3ក់$សុក=$Mេ=ះេN ។
េ ំ ២០០៩ ប"ន
# $សី ៃហ ចរ ន$ត*វ,ប់,សិស.ពូែកអក.រសិលP៍ែខRរ
3ក់ទ១២
ី១២ ក5ង
) 67 េហើយក៏ ន$បឡង,ប់ទ:
ូ ង
ំ $សុកេទៀត ។ ែតទីបផ
ំ ត
ុ េ
ែតមិនCចឆងទូ:ង
ំ េខតVរច
ួ ដែដល ។ េ យXរេធZស$បែហស របស់=ង គឺ ន
េឆើយខុសមួយសំណរ\ ១៨ ពិន^) េ យXរភ័ន$^ ចឡំ ។
េ ំ ២០០៨ ប"ន
# $សីខ()ំ ៃហ កល2ណ ន$ត*វ,ប់,សិស.ពូែកគណិតវទ2
3ក់ទ៩
ី ក5ង
) 67 េហើយ$ត*aបន$បឡង,ប់ទ:
ូ ង
ំ $សុកេទៀត ។ ែតេ ែតមិនCច
ឆងទូ:ង
ំ េខតVរច
ួ ដែដល ។
េ ំ ២០១១ ប"ន
# $សី ៃហ កល2ណ ន$ត*វ,ប់,សិស.ពូែកអក.រសិលP៍ែខRរ
3ក់ទ១២
ី១២ ក5ង
) 67 ែត$ត*វKLក់េពល$បឡងទូ:ង
ំ $សុក ។
េពលេនះខ(អ
)ំ ស់បងប"ន
# ែដល$ត*វ$បឡងេទៀតេហើយ កbស
ី ងcឹម,បនV គឺ@នែត
កRយ
e ៗ របស់ខ()ំ ដូច, ៖ សុខ វួចេផង (3ក់ទី ៨ ំ ២០១១/២០១២) , ... ។

កំែណេ យ ៃហ ហុន
ិ , ៃហ ចរ និងយ៉ត ពនក 116
េមេរ នទី១០
១០ ៖ សមី រប ត់

ម នទ ម ប

1/. ច ងប មមយច ច នង ន មគ !"បទ #


ក. a = 2 , ( −4,1) ខ. a = −
2
, (1,5)
3
2/. ច ក មគ !"បទ 'នប មព ច ច#
ក. ( 3,1) , ( 5,4 ) ខ. ( 3,0 ) , ( −3, −6 )
គ. ( −2, −1) , ( −4, −4 ) ឃ. (1, 4) , (4, − 2) *

3/. ច ក ម 'នប មព ច ច#
ក. ( −6,1) , ( 6, −2 ) ខ. ( 3, −4 ) , ( −9, 2 )
គ. ( 2,0 ) , (1, −8) ឃ. ( −4.2, −4 ) , ( −2.2,6 )
ង. ( 2.5, −1) , ( 0.5,1) ច. ( 0,5) , ( 3, −2.5) *

4/. កច !មន
+ 'ន ផ-ក. /0កម1 នង 2កម1 "នប34 56ចនន
ក ក ថយច8 9 :ប ; ម<! 0.8 % ក-ងមយ=- * បក
+ ក ន
ចនន 80% >=- 2000 * ច ក ម ?កទង ;នង
@ Aគ យ'ន
ក ក "នថយច8 ; ម=- * !ប+ ម ន8 ម
+ B"CនD1ន ក
ចននក ក >=- 2008 *
5/. យ ង
@ <ព !E8F-ក'ន" ប34 56ចនន<-កជក"នថយច8Hបព=-
1992 មក * ប<
+ ក
- ជក នចនន 38 % >=- 1992 0យ
+ ចនន ន8"ន
@ 15.5% > =- 2007
ថយច8មក! ម 2007 * ច ក ម ?កទងនង
@
ចនន<-កជក នងចនន=- *
6/. ម នង ងប ន មគ !"បទ m 0យ
+ ម
ច ច P ច កក< : ន'នច ច!ប ពI 2ងប នង<JកK *
ក. m = 1 , P ( 0,2 ) ខ. m = −1 , P (1,0 ) គ. m = −2 , P ( 3, −1)
ឃ. m = , P ( −1, −4 ) ង. m = 0 , P ( 2,1) ច. m = −
3 3
, P ( 0,3)
5 2

កំែណេ យ ៃហ ហុន
ិ , ៃហ ចរ និងយ៉ត ពនក 117
េមេរ នទី១០
១០ ៖ សមី រប ត់

7/. ក-ងច 3មប បLង ! ម ម


+ យ3! បE- ?
ក. 2 x + 3 y = −11 ខ. 4 x + 8 y − 1 = 0 គ. y = − x + 1
2
3
8/. ច ក មគ !"បទ 'នប ង ម Lង ! ម ច!"បF គ
+ ម
3ខN8 នប ! បE- ?
ក. y = x + 4 , x − y + 5 = 0 ខ. y = 2 x − 3 , x + 2 y + 1 = 0
គ. 3x − y + 4 = 0 , y − 2 = 3( x + 1) ឃ. 2 y = 5 x + 6 , 5 x + 2 y − 1 = 0 *

9/. គ
+ ម 3ខN8 នប កងE- ?
ក. y = 3x + 1 , y = − x
1
3
ខ. y = 2 x + 5 , x − 2 y + 6 = 0
គ. y = 5 x − 4 , x + 5 y − 1 = 0
10/. ច ក ម ប @ #
! បនង
ក. y = 3x − 4 0យ
+ មច ច ( 5,1)
ខ. 3x − 2 y + 5 = 0 0យ
+ មច ច ( −2, 4 )

គ. 2 x + 2 y + 9 = 0 0យ មច ច  − ,0 
5
+
 3 
ឃ. x − 5 y + 6 = 0 0យ
+ មច ច ( 0,0 ) *
11/. ច ក ម ប @ #
កងនង
ក. y = x + 4 0យ មច ច ( 5,0 )
1
2
+
ខ. x − y + 5 = 0 0យ
+ មច ច ( 0,0 )
គ. 8 x + 3 y + 1 = 0 0យ
+ មច ច ( −1,4 )

ឃ. y = − x + 6 0យ មច ច  −4, − 
2
+
 3
12/. ក. ច ក ម 'នប d1 មច ច A ( −3,5) ន
មគ !"បទ
!"បទ +ន
1 ង
@ −2 ច ងប ន8 *
ខ. គ ន ម ប d 2 : y = ( m − 1) x + 2 * ច ក 'មN m ម
+ B56
ប d2 ! ប d1 * + ន 'មN m 56 d2 កងនង
@ d1 O ទ ?
ច ម 'នប
'នប ! QP នង
@ 'មN m នមយR ច ងប ?ង
8*
គ. B 9ច ច នក< : ន ( 6,0 ) * ច ម 'នប

កំែណេ យ ៃហ ហុន
ិ , ៃហ ចរ និងយ៉ត ពនក 118
េមេរ នទី១០
១០ ៖ សមី រប ត់

ម A នង B ចបSTកច @ <JកK y′y *


ចជបនង
13/. ក-ង !មUយ< ម គ56បច ច A ( 0,6 ) , B ( −3,0 ) នង C ( 6,0 ) *
ក. ច ក ម 'នប AB នង AC *
ខ. ច ក ម 'នប D មច ច C 0យ
+ កងនង
@ ប AB *

គ. ច ក ម 'នប D′ មច ច B 0យ
+ ! បនង
@ ប AC *

14/. ក. ក-ង !មUយ< ម ក ម 'នប មច ច A ( 6,2 ) នង


B ( 2, −2 ) * ច ក ម 'ន ខK ម VទJ ប AB *

ខ. គ56ច ច C ( −2,6 ) * ច បWXញF CAB 9! ម" *


គ. ច ក ម 'នប C2 ម C 0យ
+ ! បនង
@ AB *
ឃ. ច បWXញFប C2 ម M ( −5,3) *
15/. ក-ង !មUយ< ម គ56ច ច A ( −6,0 ) , B ( 6,0 ) នង C ( 3,9 ) *
ក. ច ក ម 'ន!ជUង! ABC *

ខ. ច ក ម ង!
'ន ខK ម VទJ ប !ជUង! ABC *

16/. បច
+ ច ( a,6) នង ( 2, a ) Z > +ប 4x + 2 y = b * ច ក 'មN
a នង b ច ងប 8 *
17/. ម < ង[ \ងច!ក + ក ]ក !បF
+ មព ព !បង"ន
ថយច8ពមយ =- ;មយ=- * >=- 1973 \ងច!ក !បF
+ មព
ព !បង ន 47% 0យ
+ >=- 1990 ចនន ន8"នថយច8! ម
@ 38% *
ចក ម ?កទង ;នង
@ !បF
+ មព ព !បង ; ម
=- x * គក យក x = 0 ! QP នង
@ =- 1970 *

កំែណេ យ ៃហ ហុន
ិ , ៃហ ចរ និងយ៉ត ពនក 119
េមេរ នទី១០
១០ ៖ សមី រប ត់

ច មយ

1/. ងប មមយច ច នង ន មគ !"បទ #


ក. a = 2 , ( −4,1)
បង គ ងច ច ( −4,1)
ប ន មគ !"បទ a=2

ននJយF ប+ x កន
+ 1 ^ក 8 y
នង + 2 ^ក
@ កន *
គ 1 ^ក ចញពច ច ( −4,1) ម
ទ Ox ប បមកគ _ង
+ + 2
^ក មទ Oy * គ"នច ច
មយ ទ *
ច ន8ប ! QP ងគ` មច ច?ងព Lង +*
ខ. a = −
2
, (1,5)
3
បង គ ងច ច (1,5 )
ប ន មគ !"បទ
ននJយF ប+ x កន
2
a=−
3
+ 3
^ក 8 y នង
@ ថយច8 2 ^ក *
គ 3 ^ក ចញព
ចញពច ច (1,5 )
មទ Ox ប បមកគ ច8 ! ម
2 ^ក មទ Oy * គ"នច ចមយ ទ *
ច ន8ប ! QP ងគ` មច ច?ងព Lង +*
2/. មគ !"បទ 'នប មព ច ច#
ម បមន4 , មគ !"បទ 'នប មព ច ច A ( x1 , y1 )
y −y
នង B ( x2 , y2 ) គ` # a = 2 1 *
x2 − x1
ក. ( 3,1) , ( 5,4 )
y2 − y1 4 − 1 3
គ"ន # a = = =
x2 − x1 5 − 3 2

ច ន8 មគ !"បទ 'នប មព ច ច 8គ` a = *


3
2

កំែណេ យ ៃហ ហុន
ិ , ៃហ ចរ និងយ៉ត ពនក 120
េមេរ នទី១០
១០ ៖ សមី រប ត់

ខ. ( 3,0 ) , ( −3, −6 )
y2 − y1 −6 − 0 −6
គ"ន # a = = = =1
x2 − x1 −3 − 3 −6
ច ន8 មគ !"បទ 'នប មព ច ច 8គ` a = 1 *
គ. ( −2, −1) , ( −4, −4 )
y2 − y1 −4 − ( −1) −3 3
គ"ន # a = = = =
x2 − x1 −4 − ( −2 ) −2 2

ច ន8 មគ !"បទ 'នប មព ច ច 8គ` a = *


3
2
ឃ. (1, 4) , (4, − 2)
y2 − y1 −2 − 4 −6
គ"ន # a = = = = −2
x2 − x1 4 −1 3
ច ន8 មគ !"បទ 'នប មព ច ច 8គ` a = −2 *
3/. ក ម 'នប មព ច ច#
ក. ( −6,1) , ( 6, −2 )
−2 − 1
គ"ន , មគ !"បទ 'នប 8គ` # a =
1
=−
6 − ( −6 ) 4

ប 8 ន មគ !"បទ គ"ន #
1
a=−
4
ម ប គ` y = − x + b 0យ
+ 2 មច ច ( −6,1) គ"ន #
1
4
1
1= − ( −6 ) + b
4
3
1= +b
2
1
b=−
2
ច ន8 ម ! QP ក គ` y = − x − *
1 1
4 2
ខ. ( 3, −4 ) , ( −9, 2 )
ម ប ន\ង # y = ax + b
:យប Lង + មច ច ( 3, −4 ) នង ( −9, 2 ) គ"ន #
 1
a = −
−4 = 3a + b  2
 ⇒
 2 = − 9 a + b b = − 5
 2
ច ន8 ម ! QP ក គ` y = − x − *
1 5
2 2

កំែណេ យ ៃហ ហុន
ិ , ៃហ ចរ និងយ៉ត ពនក 121
េមេរ នទី១០
១០ ៖ សមី រប ត់

គ. ( 2,0 ) , (1, −8)


ង A ( 2, 0 ) , B (1, −8 ) នង M ( x, y ) M 9ច ចមយ > +ប AB
−8 − 0 −8
ប AB ន មគ !"បទ # a= = =8
1− 2 −1
y−0
ប ន មគ !"បទ # a' =
y
AM =
x−2 x−2
ប AB នង AM 9ប មយ(! បE-) 8 a = a ' គ"ន #
y
8=
x−2
y = 8 ( x − 2)
y = 8 x − 16
ច ន8 ម ! QP ក គ` y = 8 x − 16 *
ឃ. ( −4.2, −4 ) , ( −2.2,6 )
ម ប ន\ង # y = ax + b
:យប Lង + មច ច ( −4.2, −4 ) នង ( −2.2, 6 ) គ"ន #
−4 = −4.2a + b a = 5
 ⇒
6 = −2.2a + b b = 17
ច ន8 ម ! QP ក គ` y = 5 x + 17 *
ង. ( 2.5, −1) , ( 0.5,1)
ម ប ន\ង # y = ax + b
:យប Lង + មច ច ( 2.5, −1) នង ( 0.5,1) គ"ន #
−1 = 2.5a + b a = −1
 ⇒
 1 = 0.5a + b b = 1.5
ច ន8 ម ! QP ក គ` y = − x + 1.5 *
ច. ( 0,5) , ( 3, −2.5)
ម ប ន\ង # y = ax + b
:យប Lង + មច ច ( 0,5) នង ( 3, −2.5) គ"ន #
 5=b a = −2.5
 ⇒
−2.5 = 3a + b b = 5
ច ន8 ម ! QP ក គ` y = −2.5 x + 5 *
4/. ក ម ?កទង ;នង
@ Aគ យ'នក ក "នថយច8 ; ម=-
ង x 9=-
=- 2000 ! QP នង + =- 2008 ! QP នង
@ x = 0 0យ @ x =8

កំែណេ យ ៃហ ហុន
ិ , ៃហ ចរ និងយ៉ត ពនក 122
េមេរ នទី១០
១០ ៖ សមី រប ត់

ង y 9Aគ យ'នក ក
y = 80% ! QP នង
@ x=0
<! ថយច8 0.8% 9 មគ !"បទ (ថយច8គ` Sd ក)
គ"ន ម # y = −0.8% x + b b 9 'មN! QP ក

:យ x = 0 ! QP នង
@ y = 80% , គ"ន # b = 80%
ច ន8 ម ! QP ក គ` y = −0.8% x + 80% O y = −0.008 x + 0.8 *
"CនD1ន កចននក ក >=- 2008
យង
+ ន ម # y = −0.8% x + 80%
>=- 2008 ! QP នង
@ x =8 8 គ"ន #
y = −0.8% ⋅ 8 + 80%
y = −6.4% + 80%
y = 73.6%
ច ន8 >=- 2008 ក ក នចនន 73.6% *
5/. ក ម ?កទងនង
@ ចនន<-កជក នងចនន=-
ង x 9=-
=- 1992 ! QP នង + =- 2007 ! QP នង
@ x = 0 0យ @ x = 15
ង y 9Aគ យ'ន<-កជក"
y = 38% ! QP នង
@ x = 0 នង y = 15.5% ! QP នង
@ x = 15
:យ ម ន\ង # y = ax + b a នង b 9 'មN! QP ក

:យ x = 0 ! QP នង
@ y = 38% នង x = 15 ! QP នង
@ y = 15.5% , គ"ន #
38% = b a = −1.5%
 ⇒
15.5% = 15a + b b = 38%
ច ន8 ម ! QP ក គ` y = −1.5% x + 38% O y = −0.015% x + 0.38 *
6/. ម នង ងប ន មគ !"បទ m 0យ
+ ម
ច ច P ច កក< : ន'នច ច!ប ពI 2ងប នង<JកK #
ក. m = 1 , P ( 0,2 )

មប\ប,ប ន ម y = x + b 0យ
+ មច ច P ( 0, 2 )
គ"ន
គ"ន # 2 = 0 + b ⇒ b = 0
ច ន8 ម ប ! QP កគ` y = x + 2 *

កំែណេ យ ៃហ ហុន
ិ , ៃហ ចរ និងយ៉ត ពនក 123
េមេរ នទី១០
១០ ៖ សមី រប ត់

កក< : ន'នច ច!ប ពI 2ងប នង<JកK #


!ប ពI 2ងប @ <JកK x ' x # យក y = 0
នង 8 x + 2 = 0 ⇒ x = −2
!ប ពI 2ងប @ <JកK y ' y # យក x = 0
នង 8 y = 0+2 = 2
ច ន8ប <JកK x ' x ! ង ( −2, 0 ) នង y ' y ! ង ( 0, 2 ) *
ងប
eច ច ( −2, 0 ) នង ( 0, 2 ) ចATបច ច?ងព គ"នប ! QP
ង*

ខ. m = −1 , P (1,0 )

មប\ប,ប ន ម y = − x + b 0យ
+ មច ច P (1,0 )
គ"ន # 0 = −1 + b ⇒ b = 1
ច ន8 ម ប ! QP កគ` y = − x + 1 *
កក< : ន'នច ច!ប ពI 2ងប នង<JកK #
!ប ពI 2ងប @ <JកK x ' x # យក y = 0
នង 8 −x +1 = 0 ⇒ x = 1
!ប ពI 2ងប @ <JកK y ' y # យក x = 0
នង 8 y = −0 + 1 = 1
ច ន8ប <JកK x ' x ! ង (1, 0 ) នង y ' y ! ង ( 0,1) *
ងប
eច ច (1, 0 ) នង ( 0,1) ចATបច ច?ងព គ"នប ! QP
ង*

កំែណេ យ ៃហ ហុន
ិ , ៃហ ចរ និងយ៉ត ពនក 124
េមេរ នទី១០
១០ ៖ សមី រប ត់

គ. m = −2 , P ( 3, −1)

មប\ប,ប ន ម y = −2 x + b 0យ
+ មច ច P ( 3, −1)
គ"ន # −1 = −2 ⋅ 3 + b ⇒ b = 5
ច ន8 ម ប ! QP កគ` y = −2 x + 5 *
កក< : ន'នច ច!ប ពI 2ងប នង<JកK #
!ប ពI 2ងប @ <JកK x ' x # យក y = 0
នង 8 −2 x + 5 = 0 ⇒ x =
5
2
!ប ពI 2ងប @ <JកK y ' y # យក x = 0
នង 8 y = 0+5 = 5

ច ន8ប <JកK x ' x ! ង  , 0  នង y ' y ! ង ( 0,5 ) *


5
2 
ងប

eច ច  , 0  នង ( 0,5 ) ចATបច ច?ងព គ"នប ! QP


5
2 
ង*

កំែណេ យ ៃហ ហុន
ិ , ៃហ ចរ និងយ៉ត ពនក 125
េមេរ នទី១០
១០ ៖ សមី រប ត់

ឃ. m = , P ( −1, −4 )
3
5

មប\ប,ប ន ម x + b 0យ មច ច P ( −1, −4 )
3
5
y= +

គ"ន # −4 = ( −1) + b ⇒ b = −
3 17
5 5
ច ន8 ម ប ! QP កគ` y = x − *
3 17
5 5
កក< : ន'នច ច!ប ពI 2ងប នង<JកK #
!ប ពI 2ងប @ <JកK x ' x # យក y = 0
នង 8
3 17 17
x− =0⇒ x=
5 5 3
!ប ពI 2ងប @ <JកK y ' y # យក x = 0
នង 8 y = 0− = −
17 17
5 5
ច ន8ប <JកK x ' x ! ង  , 0  នង y ' y ! ង  0, −  *
17 17
 3   5 
ងប
\ង 'មN ខ#

x −1 4
3 17
y= x− −4 −1
5 5

eច ច ( −1, −4 ) នង ( 4, −1) ចATបច ច?ងព គ"នប


! QP ង *

ង. m = 0 , P ( 2,1)

មប\ប,ប ន ម y = b 0យ
+ មច ច P ( 2,1)
គ"ន # 1 = b
ច ន8 ម ប ! QP កគ` y = 1 *

កំែណេ យ ៃហ ហុន
ិ , ៃហ ចរ និងយ៉ត ពនក 126
េមេរ នទី១០
១០ ៖ សមី រប ត់

កក< : ន'នច ច!ប ពI 2ងប នង<JកK #


!ប ពI 2ងប @ <JកK x ' x គ` ( 0,1) !f8 y = 1 !គប 'មN x
នង
ប មន <JកK y ' y ទ !f8 y = 1 ≠ 0 !គប 'មN x
ងប
eច ច ( 0,1) នង (1,1) ចATបច ច?ងព គ"នប ! QP
ង *( !f8 y = 1 !គប 'មN x )

ច. m = −
3
, P ( 0,3)
2

មប\ប,ប ន ម y = − x + b 0យ មច ច P ( 0,3)
3
2
+
គ"ន # 3 = 0 + b ⇒ b = 3
ច ន8 ម ប ! QP កគ` y = − x + 3 *
3
2
កក< : ន'នច ច!ប ពI 2ងប នង<JកK #
!ប ពI 2ងប @ <JកK x ' x # យក y = 0
នង 8 − x+3= 0⇒ x = 2
3
2
!ប ពI 2ងប @ <JកK y ' y # យក x = 0
នង 8 y = 0+3= 3
ច ន8ប <JកK x ' x ! ង ( 2, 0 ) នង y ' y ! ង ( 0,3) *
ងប
eច ច ( 2, 0 ) នង ( 0,3) ចATបច ច?ងព គ"នប ! QP
ង*

កំែណេ យ ៃហ ហុន
ិ , ៃហ ចរ និងយ៉ត ពនក 127
េមេរ នទី១០
១០ ៖ សមី រប ត់

7/. ក-ងច 3មប បLង ! ម ម


+ យ3! បE- ?
យង
+ ន#
ក. ម 56 y = − x − ន មគ !"បទ #−
2 11 2
2 x + 3 y = −11
3 3 3
ខ. ម 56 y = − x + ន មគ !"បទ #−
1 1 1
4x + 8 y −1 = 0
2 8 2
គ. ម ន មគ !"បទ #−
2 2
y=− x +1
3 3
:យ ម (ក) នង (គ) ន មគ !"បទ ចE- *
ច ន8 ម 2 x + 3 y = −11 នង y = − x + 1 9 ម ងប ! បE- *
2
3
8/. ក មគ !"បទ 'នប ង ម Lង ! ម ច!"បF គ
+ ម
3ខN8 នប ! បE- ?
យង
+ ន#
ក. ប y = x+4 ន មគ !"បទ a =1

ប x− y+5 = 0 ⇔ y = x+5 ន មគ !"បទ a' =1

:យ a = a ' ច ន8ប ?ងព ន8! បE- *


ខ. ប y = 2x − 3 ន មគ !"បទ a=2

ប ន មគ !"បទ
1 1 1
x + 2 y +1 = 0 ⇔ y = − x − a' = −
2 2 2
:យ a ≠ a ' ច ន8ប ?ងព ន8មន! បE- ទ(9ប E-) *
គ. ប 3x − y + 4 = 0 ⇔ y = 3x + 4 ន មគ !"បទ a=3

ប y − 2 = 3 ( x + 1) ⇔ y = 3x + 5 ន មគ !"បទ a' = 3

:យ a = a ' ច ន8ប ?ងព ន8! បE- *

កំែណេ យ ៃហ ហុន
ិ , ៃហ ចរ និងយ៉ត ពនក 128
េមេរ នទី១០
១០ ៖ សមី រប ត់

ឃ. ប ន មគ !"បទ a =
5 5
2 y = 5x + 6 ⇔ y = x+3
2 2
ប ន មគ !"បទ a ' = −
5 1 5
5x + 2 y −1 = 0 ⇔ y = − x +
2 2 2
:យ a ≠ a ' ច ន8ប ?ងព ន8មន! បE- ទ(9ប E-) *
9/. គ
+ ម 3ខN8 នប កងE- ?
យង
+ ន#
ក. ប y = 3x + 1 ន មគ !"បទ a=3

ប ន មគ !"បទ
1 1
y=− x a' = −
3 3
គ"ន # a ⋅ a ' =  −  ⋅ 3 = −1
1
 3
ច ន8ប ?ងព ន8 កងE- *
ខ. ប y = 2x + 5 ន មគ !"បទ a=2

ប ន មគ !"បទ
1 1
x − 2y + 6 = 0 ⇔ y = x+3 a' =
2 2
គ"ន # a ⋅ a ' = 2 ⋅ = 1 ≠ −1
1
2
ច ន8ប ?ងព ន8មន កងE- ទ *
គ. ប y = 5x − 4 ន មគ !"បទ a=5

ប ន មគ !"បទ
1 1 1
x + 5 y −1 = 0 ⇔ y = − x + a' = −
5 5 5
គ"ន # a ⋅ a ' = 5 ⋅  −  = −1
1
 5
ច ន8ប ?ងព ន8 កងE- *
10/. ក ម ប @ #
! បនង
ក. y = 3x − 4 0យ
+ មច ច ( 5,1)
:យប y = 3x − 4 ន មគ !"បទ a=3 8ប ! QP
កគ` y = 3x + b 0យ
+ មច ច ( 5,1) , គ"ន #
1 = 3⋅5 + b
⇒ b = −14
ច ន8 ម ប ! QP ក គ` y = 3x − 14 *
ខ. 3x − 2 y + 5 = 0 0យ
+ មច ច ( −2, 4 )
3 5 3
:យប 3x − 2 y + 5 = 0 ⇔ y =
2
x+
2 ន មគ !"បទ a=
2
3
8ប ! QP កគ` y = 2 x + b 0យ
+ មច ច ( −2, 4 ) , គ"ន #

កំែណេ យ ៃហ ហុន
ិ , ៃហ ចរ និងយ៉ត ពនក 129
េមេរ នទី១០
១០ ៖ សមី រប ត់

3
4= ⋅ ( −2 ) + b
2
⇒b=7
ច ន8 ម ប ! QP ក គ` y = x + 7 *
3
2
គ. 2 x + 2 y + 9 = 0 0យ មច ច  − ,0 
5
+
 3 
:យប ន មគ !"បទ
2
2x + 2 y + 9 = 0 a=− = −1
2
8ប ! QP កគ` y = − x + b 0យ មច ច  − , 0  គ"ន #
5
+
 3 
 5 5
0 = − −  + b ⇒ b = −
 3 3
ច ន8 ម ប ! QP ក គ` y = − x − *
5
3
ឃ. x − 5 y + 6 = 0 0យ
+ មច ច ( 0,0 )
:យប ន មគ !"បទ
1
x − 5y + 6 = 0 a=
5
8ប ! QP កគ` y = x + b 0យ មច ច ( 0,0 ) គ"ន #
1
5
+

0 = 0+b ⇒ b = 0
ច ន8 ម ប ! QP ក គ` y = x *
1
5
11/. ក ម ប @ #
កងនង
ប ព កងE- 3 # a ⋅ a ' = −1 ⇒ a ' = − , គ"ន #
1
a
ក. y = x + 4 0យ មច ច ( 5,0 )
1
2
+

:យប ន មគ !"បទ
1 1 1
y= x+4 a= ⇒ a ' = − = −2
2 2 1
2
8ប ! QP កគ` y = −2 x + b 0យ
+ មច ច ( 5,0 ) គ"ន #
0 = −2 ( 5) + b ⇒ b = 10

ច ន8 ម ប ! QP ក គ` y = −2 x + 10 *
ខ. x − y + 5 = 0 0យ
+ មច ច ( 0,0 )
:យប ន មគ !"បទ
1 1
x− y+5 = 0 a = = 1 ⇒ a ' = − = −1
1 1
8ប ! QP កគ` y = − x + b 0យ
+ មច ច ( 0,0 ) គ"ន #

0 = 0+b ⇒ b = 0
ច ន8 ម ប ! QP ក គ` y = − x *

កំែណេ យ ៃហ ហុន
ិ , ៃហ ចរ និងយ៉ត ពនក 130
េមេរ នទី១០
១០ ៖ សមី រប ត់

គ. 8 x + 3 y + 1 = 0 0យ
+ មច ច ( −1,4 )
:យប ន មគ
មគ !"បទ
8 1 3
8x + 3 y + 1 = 0 a = − ⇒ a' = − =
3 8 8

3
8ប ! QP កគ` y = x + b 0យ មច ច ( −1,4 ) គ"ន #
3
8
+
3 35
4 = ⋅ ( −1) + b ⇒ b =
8 8
ច ន8 ម ប ! QP ក គ` y = x + *
3 35
8 8
ឃ. y = − x + 6 0យ មច ច  −4, − 
2
+
 3
:យប ន មគ !"បទ
1
y = −x + 6 a = −1 ⇒ a ' = − =1
−1
8ប ! QP កគ` y = x + b 0យ មច ច  −4, −  គ"ន #
2
+
 3
2 10
− = −4 + b ⇒ b =
3 3
ច ន8 ម ប ! QP ក គ` y = x + *
10
3
12/. ក. ក ម 'នប d1

មប\ប, ប d1 មច ច A ( −3,5) ន មគ !"ប


ទ +ន
1 ង
@ −2 គ"ន # d1 : y = −2 x + b 0យ
+ 5 = −2 ( −3) + b ⇒ b = −1
ច ន8ប d1 ន ម d1 : y = −2 x − 1 *

ច ងប ន8
\ង
\ង 'មN ខ

x 0 −1

y = −2 x − 1 −1 1

កំែណេ យ ៃហ ហុន
ិ , ៃហ ចរ និងយ៉ត ពនក 131
េមេរ នទី១០
១០ ៖ សមី រប ត់

ខ. ក 'មN m ម
+ B56ប d2 ! ប d1
:យប d1 : y = −2 x − 1 ន មគ !"បទ a1 = −2

នង ប d 2 : y = ( m − 1) x + 2 ន មគ !"បទ
!"បទ a2 = m − 1

8 ប+ d1 / / d 2 8! a1 = a2 គ"ន #
−2 = m − 1
m = −1
ច ន8ប d1 / / d 2 3 m = −1 *

+ ន 'មN m 56 d2 កងនង
@ d1 O ទ ?
! ងE- , d1 ⊥ d 2 8! a1 ⋅ a2 = −1 គ"ន #
−2 ( m − 1) = −1
1
m −1 =
2
3
m=
2
ច ន8ប 3 m= *
3
d1 ⊥ d 2
2
ម 'នប
'នប ! QP នង
@ 'មN m នមយR
ប+ m = −1 គ"ន # d 2 : y = −2 x + 2
ប+ m = គ"ន # d 2 : y = x + 2
3 1
2 2
ច ងប ?ង 8
\ង 'មN ខ#

x 0 1 x 0 −2
1
y = −2 x + 2 2 0 y= x+2 2 1
2

( ម ម+ ប >Lង +)
គ. ម 'នប ម A នង B
យង
+ ន # A ( −3,5 ) នង B ( 6, 0 )
ប AB ន\ង y = ax + b មច ច A នង B គ"ន #
5 = −3a + b

0 = 6a + b
 5
a = − 9

b = 10
 3

កំែណេ យ ៃហ ហុន
ិ , ៃហ ចរ និងយ៉ត ពនក 132
េមេរ នទី១០
១០ ៖ សមី រប ត់

ច ន8ប ន ម ! QP "នក *
5 10
AB AB : y = − x +
9 3
បSTកច ចជបនង
@ <JកK y′y
ជន x = 0 ក-ង ម Lង + , គ"ន # y = 0 + 3 = 3
10 10

ច ន8ប AB ជបនង
@ <JកK yy ' ! ង ច ច  0,  *
10
 3
13/. ក. ក ម 'នប AB នង AC
យង
+ ន , A ( 0, 6 ) , B ( −3, 0 ) នង C ( 6, 0 ) គ"ន #
y−6 0−6
AB : =
x − 0 −3 − 0
AB : y − 6 = 2 x
AB : y = 2 x + 6
y−6 0−6
AC : =
x−0 6−0
AC : y − 6 = − x
AC : y = − x + 6
ច ន8 ម ប ! QP កគ` AB : y = 2 x + 6 នង AC : y = − x + 6 *
ខ. ក ម 'នប D មច ច C 0យ
+ កងនង
@ ប AB

ប D មច ច C ( 6, 0 ) 0យ
+ កងនង
@ AB : y = 2 x + 6 8ប
ន មគ !"បទ , គ"ន #
1
D a 2a = −1 ⇒ a = −
2
D : y = − x + b នង មច ច C ( 6, 0 ) គ"ន #
1
2
1
0 = − ⋅6 + b ⇒ b = 3
2
ច ន8 ម ប ! QP កគ` D : y = − x + 3 *
1
2
គ. ក ម 'នប D′ មច ច B 0យ
+ ! បនង
@ ប AC

ប D' មច ច B ( −3, 0 ) 0យ
+ ! បនង
@ AC : y = − x + 6 8ប
D' ន មគ !"បទ a a = −1 , គ"ន #

D ' : y = − x + b នង មច ច B ( −3, 0 ) គ"ន #


0 = − ( −3) + b ⇒ b = −3

ច ន8 ម ប ! QP កគ` D : y = − x − 3 *
14/. ក. ក ម 'នប
'នប មច ច A នង B
ម 'នប មច ច A ( 6,2 ) នង B ( 2, −2 ) គ` #

កំែណេ យ ៃហ ហុន
ិ , ៃហ ចរ និងយ៉ត ពនក 133
េមេរ នទី១០
១០ ៖ សមី រប ត់

y − 2 −2 − 2
AB : =
x−6 2−6
−4
AB : y − 2 = ( x − 6)
−4
AB : y = x − 4
ច ន8 ម ប ! QP កគ` AB : y = x − 4 *
ក ម 'ន ខK ម VទJ ប AB

ប 'ន ខK ម VទJ ប AB 9ប មច ច M ក3g


AB នង កង9មយប
កង9មយប AB : y = x − 4 *
 6 + 2 2 + ( −2 ) 
:យ M  ,  = M ( 4, 0 )
 2 2 
8ប ម VទJ 8គ` # y = ax + b a ⋅1 = −1 ⇒ a = −1 គ"ន ,

y = −x + b មច ច M ( 4, 0 ) យង
+ "ន # 0 = −4 + b ⇒ b = 4
ច ន8 ម 'ន ខK ម VទJ ប AB គ` y = − x + 4 ! Q"នក
PQ"នក *
ខ. បWXញF CAB 9! ម"
យង
+ នច ច C ( −2,6 ) នង ម ម VទJ ប AB គ` y = − x + 4

ជន ក< : ន ប C ;ក-ង ម ម VទJ , គ"ន #


6 = − ( −2 ) + 4
6 = 2+4
6 = 6 ផhងi

ននJយF C Z > + ខK ម VទJ 'ន AB 56 គ"ន CA = CB


ច ន8! ABC 9! ម" កព C *

គ. ក ម 'នប C2 ម C 0យ
+ ! បនង
@ AB
ប C2 ម C ( −2,6) 0យ
+ ! បនង
@ AB : y = x − 4
8 C2 : y = x + b មច ច C ( −2,6) គ"ន # 6 = −2 + b ⇒ b = 8

ច ន8 ម ប C2 : y = x + 8 ! QP "នក *
ឃ. បWXញFប C2 ម M ( −5,3)
យង
+ ន C2 : y = x + 8 នង M ( −5,3)
ប+ x = −5 8< : ន ប C2 គ y = −5 + 8 = 3
`
ច ន8ប C2 មច ច M *

កំែណេ យ ៃហ ហុន
ិ , ៃហ ចរ និងយ៉ត ពនក 134
េមេរ នទី១០
១០ ៖ សមី រប ត់

15/. ក. ក ម 'ន!ជUង! ABC

យង
+ នច ច A ( −6,0 ) , B ( 6,0 ) នង C ( 3,9 ) គ"ន ម ប #
y−0 0−0
AB : =
x − ( −6 ) 6 − ( −6 )
AB : y = 0
y−0 9−0
BC : =
x − 6 3− 6
BC : y = −3 ( x − 6 )
BC : y = −3 x + 18
y −9 0−9
CA : =
x − 3 −6 − 3
CA : y − 9 = x − 3
CA : y = x + 6
ច ន8 ម 'ន!ជUង! ABC គ` # AB : y = 0 , BC : y = −3 x + 18 នង

CA : y = x + 6 *

ខ. ក ម 'ន ខK ម VទJ ប !ជUង! ABC

យង
+ នច ច A ( −6, 0 ) , B ( 6, 0 ) , C ( 3,9 ) នងប AB : y = 0

BC : y = −3 x + 18 , CA : y = x + 6 *

គ"ន # ច ចក3g 'ន AB គ` ( 0, 0 )

ច ចក3g 'ន BC គ`  , 
9 9
2 2
ច ចក3g 'ន CA គ`  − , 
3 9
 2 2
យង
+ "ន ម 'ន ខK ម VទJ ប #
!ជUង AB គ` # x = 0
9 19
!ជUង BC គ` # y = x + b
1
=  +b ⇒ b = 3
3 2 3 2
56 គ"ន ម ប ម VទJ y = x + 3 *
1
3
 3
!ជUង CA គ` # y = − x + b
9
= −−  + b ⇒ b = 3
2  2
56 គ"ន ម ប ម VទJ y = − x + 3 *
16/. ក 'មN a នង b
យង
+ នប 4x + 2 y = b មច
មច ច ( a,6) នង ( 2,a ) គ"ន #
4a + 12 = b 4a − b = −12 a = −2
 ⇔ ⇒
8 + 2a = b  2 a − b = −8 b = 4

កំែណេ យ ៃហ ហុន
ិ , ៃហ ចរ និងយ៉ត ពនក 135
េមេរ នទី១០
១០ ៖ សមី រប ត់

ច ន8 a = −2, b = 4 ! QP "នក jCងWយ! k *


ងប 8
ច f8 a = −2, b = 4 យង
+ eច ច ( −2, 6 ) នង ( 2, −2 ) ចATបច ច?ង
ព យង
+ នង
@ ទទ "នប ! QP ង *

17/. ក ម ?កទង ;នង


@ !បF
+ មព ព !បង ; ម=- x
គក យក x = 0 ! QP នង
@ =- 1970 8=- 1973 ! QP នង
@ x=3
នង=- 1990 ! PQនង
@ x = 20
ង y 9Aគ យ<-ក !បF
+ មព ព !បង គ"ន #
y = 47% ! QP នង
@ x=3
y = 38% ! QP នង
@ x = 20
គ"ន , ប ! QP កគ` មច ច ( 3, 0.47 ) នង ( 20, 0.38 )
យង
+ "ន #
y − 0.47 0.38 − 0.47
=
x−3 20 − 3
0.09
y=− ( x − 3) + 0.47
17
0.09 8.26
y=− x+
17 17
ច ន8 ម ! QP កគ` y = − x = 0 ! PQនង
@ =-
0.09 8.26
x+
17 17
1970 *

កំែណេ យ ៃហ ហុន
ិ , ៃហ ចរ និងយ៉ត ពនក 136
េមេរ នទី១១
១១ ៖ បព័នសមី រដេឺ កទី១ នពីរអ ត

ម នទ បពន ម កទ នព

1/. ក ន គ !ង ម#ច ម%យ ប ' បពន ម (ទ?


x− y =8 2 x + y = 3
ក.  ( 3, −5 ) ខ.  ( 2, −1)
 x + y = −2  x + 2y = 4
 y = 3x  x− y =3
គ.  ( 0,0 ) ឃ.  ( 5, 2 ) -
x + y = 3 2 x − 2 y = 6
2/. ច . /យ បពន ម !ង ម0ម ប1
x − 4 y = 1 y − 2 = 0 2 x − 3 y = 13
ក.  ខ.  គ. 
 x + y = −4  x+3= 0  x − 2y = 8
x + y =1
 y = 3x 2 x + y = 6
ឃ.  ង.  ច.  x y
x + y = 4  y = 8 − 2x  2 + 2 = 2
 x+ y =3
y = 7 − x
ឆ.  7 − 2 x ជ.  -
 y = 2  x = y +3

3/. ច . /យ បពន ម 0ម4ធ


5 ប78 ' យបក 1
 2 x + 2 y = −2 2 x + 2 y = 8  3x + 3 y = 9
ក.  ខ.  គ. 
 5x − 2 y = 9  2x − y = 5 4 x − 3 y = −16
8 x − 3 y = 17 7 x − 10 y = −1  4 x − 3 y = 15
ឃ.  ង.  ច. 
 −7 x + 6 y = 2  3 x + 2 y = −13 8 x + 2 y = −10
 2 x + 8 y = −1 5x + 3 y + 9 = 0 6 x − 5 = 2 x − 7 y
ឆ.  ជ.  ឈ. 
−10 x + 4 y = 16 3 x − 4 y + 17 = 0  2x = 5 y − 6
 4x − 3y = 9 3 x + 4 y = −1 4 x − 2 y = −19
ញ.  .  ;.  -
 −3 x + 5 y = 7  7x + 9 y = 0  −6 x − 3 y = 1.5
4/. ច . /យ បពន ម 0ម4ធ 7 <
5 ជន 1
 x − 2 y = 16  x + 2y = 6  y = −4 x + 5
ក.  ខ.  គ. 
 4x + y = 1 4 x + 3 y = 4  2 x − 3 y = 13
5 x − y = −23  −2 x + y = 2  x + 5 y = 11
ឃ.  ង.  ច. 
 3 x − y = −15  2x + 3y = 6  4x − y = 2
2 x + 3 y = 11 5 x + 3 y = 4 5 x + 7 y = −3
ឆ.  ជ.  ឈ. 
 3 x + 3 y = 18  4x − 2 y = 1  2 x + 14 y = 2
 x = 10 y x−7 = y
3 x + 6 y = 6
ញ.  .  1 ;.  1 -
 2x = 2 y  2 x = 3 y + 2  4 x − 1 = y

កំែណេ យ ៃហ ហុន
ិ , ៃហ ចរ និងយ៉ត ពនក 137
េមេរ នទី១១
១១ ៖ បព័នសមី រដេឺ កទី១ នពីរអ ត

5/. ក. ច ក =ម% m ម> បពន ម !ង ម?@នច ម%យ -


4 x + 6 y = 12 12 x − 3my = 1
 , 
 2 x + my = 5  4x + 2 y = 3
ខ. ច ក =ម% m នង
5 n ម> បពន ម !ង ម នគច ម%យAប'
មន
5 '-
 6 x + y = 30
mx + ny = 10 
 ,  1 -
 4x − 3y = 5 (n − 1) x + 3 y = n + m

6/. B ខ នង
5 Cទញ
5 4D - B ខទញ
5 4D ង
E 2កF នង
5
4D ' 3កF ' 8ក' 21 000 G - ច7 Hក C4ញទ
5 ញ
5 4D

E 3កF នង
5 4D ' 2កF ' 8ក' 19 000 G - 4D
I7ងព ប ភទ ន. ន =ម%មយ
< កF =ថ%បM
BL @ ន ?
បបម<
7/. NកOBង បបម<យ ន 8ក' 8 000 G - គAប'ចន
7 ន
< ក 8ក' ឃញ 11
ន%ក
P - ក 8ក' ន. នព ប ភទគ ប ភទ 500 G នង
5 ប ភទ
1 000 G - ច កច7នន
< ក 8ក' ប ភទនម<យQ -
8/. ព 7 ក'ទក
P 0O យ ប ខRព< បងS ចB.មក - ប? ម
' ន
5 ប ' ខR#ព ទ
M.? ប
' ងS ប7ពង'មក ' N 'កនTBយ ខR 7 m ទ - បLB នE ប? '
' ងS ប7ពង'មក
ប ' ខRព< #ព ? ប មក ' ម
@ ?@ន ' ខR /. - ប7ពង'
នង
5 ខRប7ពង' ន ប 4ង 1m - កមU ' ម 0O V4 គ5 ពក ន%ង ? '
ងWBយមក ម
P - ច ក ប 4ង ខRព< ន5ងកមU ' ម 0O -
9/. ម<យក ន%ងAងច B H កង នប
នប 5 110 m - ប គ ថម 5m បXE យ
នង
5 2m ទទង
P M.=ផT កZ កន ពមBន 150 m 2 - ច ក ប 4ង
បXE យ នង P =ន M. -
5 ទទង
10/. គយក ក 8ក' 20$ [ទញ
5 ន7 H នង
5 ន7 P -
ប គទញ
5 ន7 H 6 នង
5 ន7 P 4 M. គនង
P \ប'4ញ
5 1.5$
ប គទញ
5 ន7 H 2 នង
5 ន7 P 5 M. គនង
P \ប'4ញ
5 1$
ប គទញ
5 ន7 H 4 នង
5 ន7 P 4 គនង
P \ប'4ញ
5 8ក'បM
BL @ ន ?
11/. ផ នន
ក=នព ច7ន< < ន
@ ង
P 22 - ព ង=នច7នន
< ធ7បកនង
P ប ង=នច7នន
< ច

@ ង
P 246 - កព ច7នន
< M. -

កំែណេ យ ៃហ ហុន
ិ , ៃហ ចរ និងយ៉ត ពនក 138
េមេរ នទី១១
១១ ៖ បព័នសមី រដេឺ កទី១ នពីរអ ត

12/. ព ច7នន
< នផ បក ន
@ ង
P 490 នង នផ ធប ន
@ ង
P 7 - ច គHM
3
5
ច7នន
< I7ងព M. -
13/. ផ ធ ប=នព ច7នន
<

@ ង
P 4 នង L =នផ ក ប '] ន
@ ង 324-
5
5 P 324-
ច គHM ច7នន
< នម<យQ -
14/. H ABC ម<យ នប 5 192 m - ច គHM ប 4ង ជ^ង BC , CA

នង
5 AB យ ង
P _ ជ^ងI7ង M. ង?Oនង
P ច7នន
< 2.5 , 2 នង
5
1.5 -

15/. ផ ក=នព ច7នន


< ន P 36 នង
@ ង L =នផ ធ ប ប '] ន P 81 -
@ ង
25
5
ច គHMច7
គHMច7នន
< នម<យQ -
16/. ផ ធ ប8 I7ងព =នច B H`Oយម<យ

@ ង
P 5 នង
5 ផ បក8 ប '
3

] ន P 64 m - ច គHM ប 4ង8 នម<យQ -


@ ង
17/. បបMT ' Ax + By = 9 ច
' H
7 B ច ( 2,1) នង
5 ( −3,3) ច ក7H ' A នង
5 B-
2 2 1
x − y = 2

18/. ច . /យ បពន ម  -
1 + 5 = 3
 x y 4

19/. ក^មabBនម<យ8ន ក'ផ 5 ផ ព មBខ A នង


5 B - 0ម /4 #4=ន
ក' [0ម
[0មc
0មcOន
7 ម<យQ ន ច0Aង!ង ម - dក0គ5 #eន
គfង -
2008 2009 2010 2011
A 2.3 2.7 3.1 3.5
B 2.9 3.3 3.7 4.1
ប ក^មabBន កg ក'0ម 0ក7 Hន ច!ង ផ 5 ផ A \ច

ក' កន #ង B ( ទ ? a B h ?

កំែណេ យ ៃហ ហុន
ិ , ៃហ ចរ និងយ៉ត ពនក 139
េមេរ នទី១១
១១ ៖ បព័នសមី រដេឺ កទី១ នពីរអ ត

ច មយ
1/. ក ន គ !ង ម#ច ម%យ ប ' បពន ម (ទ?
x− y =8
ក.  ( 3, −5 )
 x + y = −2
7 <
ជន ( 3, −5) [កOBង បពន ម , គ8ន 1
 3 − ( −5 ) = 8  8=8
 ⇔ ផTjងkT '
3 + ( −5 ) = −2 −2 = −2
ច ន.ក ន ( 3, −5) #ច ម%យ ប ' បពន ម -
2 x + y = 3
ខ.  ( 2, −1)
 x + 2y = 4
7 <
ជន ( 2, −1) [កOBង បពន ម , គ8ន 1
2 ⋅ 2 + ( −1) = 3 3 = 3
 ⇔ មន
5 ផTjងkT '
2 + 2 ( −1) = 4 0 = 4
ច ន.ក ន ( 2, −1) មន
5 មន#ច ម%យ ប ' បពន ម ទ-
 y = 3x
គ.  ( 0,0 )
x + y = 3
7 <
ជន ( 0, 0 ) [កOBង បពន ម , គ8ន 1
0 = 3 ⋅ 0 0 = 0
 ⇔ មន
5 ផTjងkT '
 0 + 0 = 3 0 = 3
ច ន.ក ន ( 0, 0 ) មន
5 មន#ច ម%យ ប ' បពន ម ទ-
 x− y =3
ឃ.  ( 5, 2 )
2 x − 2 y = 6
7 <
ជន ( 5, 2 ) [កOBង បពន ម , គ8ន 1
 5−2 = 3 3 = 3
 ⇔ ផTjងkT '
2 ⋅ 5 − 2 ⋅ 2 = 6 6 = 6
ច ន.ក ន ( 5, 2 ) #ច ម%យ ប ' បពន ម -
2/. . /យ បពន ម !ង ម0ម ប1
 x − 4y =1 (1)
ក. 
 x + y = −4 ( 2)
ង'បMT ' (1) នង 7 ង 4V ង'0Aង =ម%
5 ( 2) , ប ខ=ន 1
ច7 `. ម x − 4y =1 (1)
x 1 −3

y 0 −1

កំែណេ យ ៃហ ហុន
ិ , ៃហ ចរ និងយ៉ត ពនក 140
េមេរ នទី១១
១១ ៖ បព័នសមី រដេឺ កទី១ នពីរអ ត

ច7 `. ម x + y = −4 ( 2)
x −1 −2

y −3 −2
I #ច7HBច ប ពh=នបMT ' (1) នង
5 ( 2) ម> កក នប ' I
គគ ច7 X កងព I មក កRI7ងព -

0ម ប , គ8នក ន=ន I គ 1 x = −3 , y = −1
ច ន. ( −3, −1) #គច ម%យ=ន បពន ម -
 y−2=0 (1)
ខ. 
 x + 3 = 0 ( 2)
ង'បMT ' (1) នង 7 ង 4V ង'0Aង =ម%
5 ( 2) , ប ខ=ន 1
ច7 `. ម y−2=0 (1)
x 0 1

y 2 2

ច7 `. ម x+3= 0 ( 2)
x −3 −3

y 0 1
I #ច7HBច ប ពh=នបMT ' (1) នង
5 ( 2) ម> កក នប ' I
គគ ច7 X កងព I មក កRI7ងព -

កំែណេ យ ៃហ ហុន
ិ , ៃហ ចរ និងយ៉ត ពនក 141
េមេរ នទី១១
១១ ៖ បព័នសមី រដេឺ កទី១ នពីរអ ត

0ម ប , គ8នក ន=ន I គ 1 x = −3 , y = 2
ច ន. ( −3, 2 ) #គច ម%យ=ន បពន ម -
 2 x − 3 y = 13 (1)
គ. 
 x − 2 y = 8 ( 2)
ង'បMT ' (1) នង 7បង 4V ង'0Aង =ម%
5 ( 2 ) , បង ខ=ន 1
ច7 `. ម 2 x − 3 y = 13 (1)
x 5 2

y −1 −3

ច7 `. ម x − 2y = 8 ( 2)
x 0 2

y −4 −3
I #ច7HBច ប ពh=នបMT ' (1) នង
5 ( 2) ម> កក នប ' I
គគ ច7 X កងព
កងព I មក កRI7ងព -

កំែណេ យ ៃហ ហុន
ិ , ៃហ ចរ និងយ៉ត ពនក 142
េមេរ នទី១១
១១ ៖ បព័នសមី រដេឺ កទី១ នពីរអ ត

0ម ប , គ8នក ន=ន I គ 1 x = 2 , y = −3
ច ន. ( 2, −3) #គច ម%យ=ន បពន ម -
 y = 3x (1)
ឃ. 
 x + y = 4 ( 2)
ង'បMT ' (1) នង 7 ង 4V ង'0Aង =ម%
5 ( 2) , ប ខ=ន 1
ច7 `. ម y = 3x (1)
x 0 1

y 0 3

ច7 `. ម x+ y = 4 ( 2)
x 1 2

y 3 2
I #ច7HBច ប ពh=នបMT ' (1) នង
5 ( 2) ម> កក នប ' I
គគ ច7 X កងព I មក កRI7ងព -

0ម ប , គ8នក ន=ន I គ 1 x = 1 , y = 3
ច ន. (1,3) #គច ម%យ=ន បពន ម -
2x + y = 6 (1)
ង. 
 y = 8 − 2 x ( 2)
ង'បMT ' (1) នង 7 ង 4V ង'0Aង =ម%
5 ( 2) , ប ខ=ន 1
ច7 `. ម 2x + y = 6 (1)

កំែណេ យ ៃហ ហុន
ិ , ៃហ ចរ និងយ៉ត ពនក 143
េមេរ នទី១១
១១ ៖ បព័នសមី រដេឺ កទី១ នពីរអ ត

x 3 2

y 0 2

ច7 `. ម y = 8 − 2x ( 2)
x 2 3

y 4 2

0ម ប , យង ឃញ_បMT ' (1) នង


5 ( 2) ប?O (?@នច7HBច ប ពh)
ច ន. បពន ម ?@នគច ម%យ ទ -
 x + y =1 (1)
ច.  x y
 + =2 ( 2)
2 2
ង'បMT ' (1) នង 7 ង 4V ង'0Aង =ម%
5 ( 2) , ប ខ=ន 1
ច7 `. ម x + y =1 (1)
x 0 1

y 1 0

ច7 `. ម
x y
+ =2 ( 2)
2 2

x 1 2

y 3 2

កំែណេ យ ៃហ ហុន
ិ , ៃហ ចរ និងយ៉ត ពនក 144
េមេរ នទី១១
១១ ៖ បព័នសមី រដេឺ កទី១ នពីរអ ត

0ម ប , យង ឃញ_បMT ' (1) នង


5 ( 2) ប?O (?@នច7HBច ប ពh)
ច ន. បពន ម ?@នគច ម%យ ទ -
 x+ y =3 (1)
ឆ.  7 − 2 x
y = ( 2)
 2
ង'បMT ' (1) នង 7 ង 4V ង'0Aង =ម%
5 ( 2) , ប ខ=ន 1
ច7 `. ម x+ y =3 (1)
x 2 1

y 1 2
7 − 2x
ច7 `. ម y= ( 2)
2

x 1 2
5 3
y
2 2

កំែណេ យ ៃហ ហុន
ិ , ៃហ ចរ និងយ៉ត ពនក 145
េមេរ នទី១១
១១ ៖ បព័នសមី រដេឺ កទី១ នពីរអ ត

0ម ប , យង ឃញ_បMT ' (1) នង


5 ( 2) ប?O (?@នច7HBច ប ពh)
ច ន. បពន ម ?@នគច ម%យ ទ -
y =7−x (1)
ជ. 
 x = y + 3 ( 2)
ង'បMT ' (1) នង 7 ង 4V ង'0Aង =ម%
5 ( 2) , ប ខ=ន 1
ច7 `. ម y =7−x (1)
x 3 4

y 4 3

ច7 `. ម x = y+3 ( 2)
x 3 1

y 0 −2
I #ច7HBច ប ពh=នបMT ' (1) នង
5 ( 2) ម> កក នប ' I
គគ ច7 X កងព I មក កRI7ងព -

0ម ប , គ8នក ន=ន I គ 1 x = 5 , y = 2
ច ន. ( 5, 2 ) #គច ម%យ=ន បពន ម -
3/. . /យ បពន ម 0ម4ធ
5 ប78 ' យបក 1
 2 x + 2 y = −2 (1)
ក. 
5 x − 2 y = 9 ( 2)
5 ងW=ន (1) នង
បក ងW នង 5 ( 2 ) គ8ន 1
7x = 7
x =1

កំែណេ យ ៃហ ហុន
ិ , ៃហ ចរ និងយ៉ត ពនក 146
េមេរ នទី១១
១១ ៖ បព័នសមី រដេឺ កទី១ នពីរអ ត

7 <
ជន x = 1 កOBង ម (1) យង8ន 1
2 ⋅1 + 2 y = −2
y = −2
ច ន. បពន ម នគច ម%យ (1, −2 ) -
 2x + 2 y = 8 (1)
ខ. 
2 x − y = 5 ( 2)
7 ព =ន ( 2 ) នង
គBH ងWIង P −1 គ8ន ម 1 −2 x + y = −5 ( 3)
បក ងW នង
5 ងW=ន (1) នង
5 ( 3) គ8ន 1
3y = 3
y =1
7 <
ជន y = 1 កOBង ម ( 2 ) យង8ន 1
2x −1 = 5
x=3
ច ន. បពន ម នគច ម%យ ( 3,1) -
 3x + 3 y = 9 (1)
គ. 
4 x − 3 y = −16 ( 2)
5 ងW=ន (1) នង
បក ងW នង 5 ( 2 ) គ8ន 1
7 x = −7
x = −1
7 <
ជន x = −1 កOBង ម (1) យង8ន 1
3 ( −1) + 3 y = 9
y=4
ច ន. បពន ម នគច ម%យ ( −1, 4 ) -
 8 x − 3 y = 17 (1)
ឃ. 
−7 x + 6 y = 2 ( 2)
គBH ងWI7
Iង 7 ព =ន (1) នង
P 2 គ8ន ម 1 16 x − 6 y = 34 ( 3)
5 ងW=ន ( 2 ) នង
បក ងW នង 5 ( 3) គ8ន 1
9 x = 36
x=4
7 <
ជន x = 4 កOB ង ម ( 2 ) យង8ន 1
−7 ⋅ 4 + 6 y = 2
y=5
ច ន. បពន ម នគច ម%យ ( 4,5 ) -

កំែណេ យ ៃហ ហុន
ិ , ៃហ ចរ និងយ៉ត ពនក 147
េមេរ នទី១១
១១ ៖ បព័នសមី រដេឺ កទី១ នពីរអ ត

7 x − 10 y = −1 (1)
ង. 
 3 x + 2 y = −13 ( 2)
7 ព =ន ( 2 ) នង
គBH ងWIង P 5 គ8ន ម 1 15 x + 10 y = −65 ( 3)
5 ងW=ន (1) នង
បក ងW នង 5 ( 3) គ8ន 1
22 x = −66
x = −3
7 <
ជន x = −3 កOBង ម ( 2 ) យង8ន 1
3 ⋅ ( −3) + 2 y = −13
y = −2
ច ន. បពន ម នគច ម%យ ( −3, −2 ) -
 4 x − 3 y = 15 (1)
ច. 
8 x + 2 y = −10 ( 2)
គBH ងWIង 7 ព =ន ( 2 ) នង
P − 2 គ8ន ម 1 −4 x − y = 5
1
( 3)
5 ងW=ន (1) នង
បក ងW នង 5 ( 3) គ8ន 1
−4 y = 20
y = −5
7 <
ជន y = −5 កOB ង ម ( 3) យង8ន 1
−4 x − ( −5 ) = 5
x=0
ច ន. បពន ម នគច ម%យ ( 0, −5 ) -
 2 x + 8 y = −1 (1)
ឆ. 
−10 x + 4 y = 16 ( 2)
7 ព =ន ( 2 ) នង
គBH ងWIង P −2 គ8ន ម 1 20 x − 8 y = −32 ( 3)
5 ងW=ន (1) នង
បក ងW នង 5 ( 3) គ8ន 1
22 x = −33
3
x=−
2
7 <
ជន
3
កOង ម (1) យង8ន 1
2 B
x=−

 3
2  −  + 8 y = −1
 2
1
y=
4
ច ន. បពន ម នគច ម%យ  − ,  -
3 1
 2 4

កំែណេ យ ៃហ ហុន
ិ , ៃហ ចរ និងយ៉ត ពនក 148
េមេរ នទី១១
១១ ៖ បព័នសមី រដេឺ កទី១ នពីរអ ត

 5x + 3 y + 9 = 0 (1)
ជ. 
3 x − 4 y + 17 = 0 ( 2)
7 ព =ន (1) នង
គBH ងWIង P 4 គ8ន ម 1 20 x + 12 y + 36 = 0 ( 3)
7 ព =ន ( 2 ) នង
គBH ងWIង P 3 គ8ន ម 1 9 x − 12 y + 51 = 0 ( 4)
5 ងW=ន ( 3) នង
បក ងW នង 5 ( 4 ) គ8ន 1
29 x + 87 = 0
x = −3
7 <
ជន x = −3 កOB ង ម ( 2 ) យង8ន 1
3 ( −3) − 4 y + 17 = 0
y=2
ច ន. បពន ម នគច ម%យ ( −3, 2 ) -
6 x − 5 = 2 x − 7 y  4 x + 7 y = 5 (1)
ឈ.  ⇔
 2x = 5 y − 6 2 x − 5 y = −6 ( 2)
7 ព =ន ( 2 ) នង
គBH ងWIង P −2 គ8ន ម 1 −4 x + 10 y = 12 ( 3)
5 ងW=ន (1) នង
បក ងW នង 5 ( 3) គ8ន 1
17 y = 17
y =1
7 <
ជន y = 1 កOBង ម (1) យង8ន 1
4 x + 7 ⋅1 = 5
1
x=−
2
ច ន. បពន ម នគច ម%យ  − ,1 -
1
 2 
 4x − 3 y = 9 (1)
ញ. 
−3 x + 5 y = 7 ( 2)
7 ព =ន (1) នង
គBH ងWIង P 3 គ8ន ម 1 12 x − 9 y = 27 ( 3)
7 ព =ន ( 2 ) នង
គBH ងWIង P 4 គ8ន ម 1 −12 x + 20 y = 28 ( 4)
បក ងW នង
5 ងW=ន ( 3) នង
5 ( 4 ) គ8ន 1
11 y = 55
y=5
7 <
ជន y = 5 កOBង ម (1) យង8ន 1
4x − 3⋅ 5 = 9
x=6
ច ន. បពន ម នគច ម%យ ( 6,5 ) -

កំែណេ យ ៃហ ហុន
ិ , ៃហ ចរ និងយ៉ត ពនក 149
េមេរ នទី១១
១១ ៖ បព័នសមី រដេឺ កទី១ នពីរអ ត

 3 x + 4 y = −1 (1)
. 
7 x + 9 y = 0 ( 2)
7 ព =ន (1) នង
គBH ងWIង P 7 គ8ន ម 1 21x + 28 y = −7 ( 3)
គBH ងWIងព

7 ព =ន ( 2 ) នង
P −3 គ8ន ម 1 −21x − 27 y = 0 ( 4)
5 ងW=ន ( 3) នង
បក ងW នង 5 ( 4 ) គ8ន 1 y = −7
7 <
ជន y = −7 កOBង ម ( 2 ) យង8ន 1
7 x + 9 ( −7 ) = 0
x=9
ច ន. បពន ម នគច ម%យ ( 9, −7 ) -
 4 x − 2 y = −19 (1)
;. 
−6 x − 3 y = 1.5 ( 2)
7 ព =ន (1) នង
គBH ងWIង P 2 គ8ន ម 1 2x − y = −
1 19
( 3)
2
គBH ងWIង 7 ព =ន ( 2 ) នង
P 3 គ8ន ម 1 −2 x − y =
1 1
( 4)
2
5 ងW=ន ( 3) នង
បក ងW នង 5 ( 4 ) គ8ន 1
−2 y = −9
9
y=
2
7 <
ជន
9
កOង ម ( 3) យង8ន 1
2 B
y=

9 19
2x − =−
2 2
5
x=−
2
ច ន. បពន ម នគច ម%យ  − ,  -
5 9
 2 2
4/. . /យ បពន ម 0ម4ធ 7 <
5 ជន 1
 x − 2 y = 16 (1)
ក. 
4 x + y = 1 ( 2)
0ម (1) : x − 2 y = 16 គ8ន 1 x = 2 y + 16 ( 3)
7 <
ជន ( 3) កOBង ( 2 ) យង8ន 1
4 ( 2 y + 16 ) + y = 1
9 y = −63
y = −7
7 <
ជន y = −7 កOBង ម ( 3) គ8ន 1 x = 2 ( −7 ) + 16 = 2

ច ន. បពន ម នគច ម%យ ( 2, −7 ) -

កំែណេ យ ៃហ ហុន
ិ , ៃហ ចរ និងយ៉ត ពនក 150
េមេរ នទី១១
១១ ៖ បព័នសមី រដេឺ កទី១ នពីរអ ត

x + 2 y = 6 (1)
ខ. 
4 x + 3 y = 4 ( 2)
0ម (1) : x + 2 y = 6 គ8ន 1 x = −2 y + 6 ( 3)
7 <
ជន ( 3) កOBង ( 2 ) យង8ន 1
4 ( −2 y + 6 ) + 3 y = 4
24 − 5 y = 4
y=4
7 <
ជន y = 4 កOBង ម ( 3) គ8ន 1 x = −2 ⋅ 4 + 6 = −2

ច ន. បពន ម នគច ម%យ ( −2, 4 ) -


 y = −4 x + 5 (1)
គ. 
2 x − 3 y = 13 ( 2)
7 <
ជន (1) កOBង ( 2 ) យង8ន 1
2 x − 3 ( −4 x + 5 ) = 13
14 x = 28
x=2
7 <
ជន x = 2 កOBង ម (1) គ8ន 1 y = −4 ⋅ 2 + 5 = −3

ច ន. បពន ម នគច ម%យ ( 2, −3) -


5 x − y = −23 (1)
ឃ. 
3 x − y = −15 ( 2)
0ម (1) : 5 x − y = −23 គ8ន 1 y = 5 x + 23 ( 3)
7 <
ជន ( 3) កOBង ( 2 ) យង8ន 1
3 x − ( 5 x + 23) = −15
− 2x = 8
x = −4
7 <
ជន x = −4 កOBង ម ( 3) គ8ន 1 y = 5 ( −4 ) + 23 = 3

ច ន. បពន ម នគច ម%យ ( −4,3) -


 −2 x + y = 2 (1)
ង. 
2 x + 3 y = 6 ( 2)
0ម (1) : −2 x + y = 2 គ8ន 1 y = 2 x + 2 ( 3)
7 <
ជន ( 3) កOBង ( 2 ) យង8ន 1
2x + 3( 2 x + 2) = 6
8x = 0
x=0
7 <
ជន x = 0 កOBង ម ( 3) គ8ន 1 y = 0+2 = 2

កំែណេ យ ៃហ ហុន
ិ , ៃហ ចរ និងយ៉ត ពនក 151
េមេរ នទី១១
១១ ៖ បព័នសមី រដេឺ កទី១ នពីរអ ត

ច ន. បពន ម នគច ម%យ ( 0, 2 ) -


 x + 5 y = 11 (1)
ច. 
4 x − y = 2 ( 2)
0ម ( 2 ) : 4 x − y = 2 គ8ន
គ8ន 1 y = 4 x − 2 ( 3)
7 <
ជន ( 3) កOBង (1) យង8ន 1
x + 5 ( 4 x − 2 ) = 11
21x = 21
x =1
7 <
ជន x = 1 កOBង ម ( 3) គ8ន 1 y = 4 ⋅1 − 2 = 2

ច ន. បពន ម នគច ម%យ (1, 2 ) -


2 x + 3 y = 11 (1)
ឆ. 
3 x + 3 y = 18 ( 2)
0ម (1) : 2 x + 3 y = 11 គ8ន 1 3 y = −2 x + 11 ( 3)
7 <
ជន ( 3) កOBង ( 2 ) យង8ន 1
3x + ( −2 x + 11) = 18
x=7
7 <
ជន x = 7 កOB ង ម ( 3) គ8ន 1 3 y = −2 ⋅ 7 + 11 = −3 ⇒ y = −1

ច ន. បពន ម នគច ម%យ ( 7, −1) -


5 x + 3 y = 4 (1)
ជ. 
4 x − 2 y = 1 ( 2)
0ម ( 2 ) : 4 x − 2 y = 1 គ8ន 1 y = 2 x −
1
( 3)
2
7 <
ជន ( 3) កOBង (1) យង8ន 1
 1
5x + 3 2x −  = 4
 2
3
11x = 4 +
2
1
x=
2
7 <
ជន
1
កOង ម ( 3) គ8ន 1
1 1 1
2 B
x= y = 2⋅ − =
2 2 2
ច ន. បពន ម នគច ម%យ  ,  -
1 1
2 2
5 x + 7 y = −3 (1)
ឈ. 
2 x + 14 y = 2 ( 2)
0ម ( 2 ) : 2 x + 14 y = 2 គ8ន 1 7 y = − x + 1 ( 3)

កំែណេ យ ៃហ ហុន
ិ , ៃហ ចរ និងយ៉ត ពនក 152
េមេរ នទី១១
១១ ៖ បព័នសមី រដេឺ កទី១ នពីរអ ត

7 <
ជន ( 3) កOBង (1) យង8ន 1
5 x + ( − x + 1) = −3
4 x = −4
x = −1
7 <
ជន x = −1 កOBង ម ( 3) គ8ន 1 7 y = − ( −1) + 1 = 2 ⇒ y =
2
7
ច ន. បពន ម នគច ម%យ  −1,  -
2
 7
3 x + 6 y = 6  x + 2 y = 2 (1)
ញ.  ⇔
2 x = 2 y  x = y ( 2)
7 <
ជន ( 2 ) កOBង (1) យង8ន 1
y + 2y = 2
2
y=
3
7 <
ជន
2
កOង ម ( 2 ) គ8ន 1
2
3 B
y= x= y=
3
ច ន. បពន ម នគច ម%យ  ,  -
2 2
3 3 
 x = 10 y (1)
.  1
 x = 3y + 2 ( 2)
2
7 <
ជន (1) កOBង ( 2 ) យង8ន 1
1
(10 y ) = 3 y + 2
2
5y − 3y = 2
y =1
7 <
ជន y = 1 កOBង ម (1) គ8ន 1 x = 10 ⋅1 = 10

ច ន. បពន ម នគច ម%យ (10,1) -


x − 7 = y (1)
;.  1
 x −1 = y ( 2)
4
0ម (1) : x − 7 = y គ8ន 1 y = x − 7 ( 3)
7 <
ជន ( 3) កOBង ( 2 ) យង8ន 1
1
x −1 = x − 7
4
1 
 − 1  x = −7 + 1
4 
3
− x = −6
4
x=8

កំែណេ យ ៃហ ហុន
ិ , ៃហ ចរ និងយ៉ត ពនក 153
េមេរ នទី១១
១១ ៖ បព័នសមី រដេឺ កទី១ នពីរអ ត

7 <
ជន x = 8 កOBង ម ( 3) គ8ន 1 y = 8− 7 =1

ច ន. បពន ម នគច ម%យ ( 8,1) -


5/. ក. ក =ម% m ម> បពន ម !ង ម?@នច ម%យ
4 x + 6 y = 12 (1)
ច7 `. 
2 x + my = 5 ( 2)
0ម (1) គ8ន 1 y = − x + 2
2
3
0ម ( 2 ) គ8ន 1 y = − x +
2 5
m m
បពន ម !ង ?@នគច ម%យ X1
 2 2
− 3 = − m m = 3
 ⇒ 5
2 ≠ 5 m ≠ 2
 m
ច ន. ម> បពន ម !ង ?@នច ម%យ B . 0 m=3 -
12 x − 3my = 1
ច7 `. 
4 x + 2 y = 3
បពន ម !ង ?@នគច ម%យ X1
12 −3m 1
= ≠
4 2 3
 3m
3 = − 2 m = −2

 ⇒ 2
− 3m ≠ 1 m ≠ − 9
 2 3
ច ន. ម> បពន ម !ង ?@នច ម%យ B . 0 m = −2 -

ខ. ក =ម% m នង
5 n ម> បពន ម នគច ម%យAប'មន
5 '
mx + ny = 10 (1)
ច7 `. 
4 x − 3 y = 5 ( 2)
0ម (1) គ8ន 1 y = −
m 10
x+
n n
0ម ( 2 ) គ8ន 1 y = x −
4 5
3 3
បពន ម !ង នគច ម%យ ចនAប'មន
5 ' X1
 m 4
− n = − 3 m = 8
 ⇒
10 = − 5 n = −6
 n 3
ច ន. បពន ម !ង នច ម%យ ចនAប'
នAប'មន
5 ' B. 0

កំែណេ យ ៃហ ហុន
ិ , ៃហ ចរ និងយ៉ត ពនក 154
េមេរ នទី១១
១១ ៖ បព័នសមី រដេឺ កទី១ នពីរអ ត

m = 8 នង
5 n = −6 -
6 x + y = 30
ច7 `.  1
( n − 1) x + 3 y = n + m

បពន ម !ង នគច ម%យ ចនAប'មន


5 ' X1
6 1 30
= =
n −1 1 n + m
3
6 30
=3=
n −1 n+m
n − 1 = 2 n = 3
 ⇒
n + m = 10 m = 7
ច ន. បពន ម !ង នច ម%យ ចនAប'
នAប'មន
5 ' B. 0
m = 7 នង
5 n=3 -
6/. 4 DI7ងព ប ភទ ន. ន =ម%មយ
< កF =ថ%បM
BL @ ន ?
0ង x # =ម% 4D n B ង 1 កF
ងកO
នង
5 y # =ម% 4D O 1 កF
'កB ង
0មប7Aប' , គ8ន បពន ម 1
2 x + 3 y = 21000 (1)

3 x + 2 y = 19 000 ( 2 )

គBH ងWIង
7 ព =ន (1) នង P − 2 គ8ន 1 −3x − 2 y = −31500
3 9
( 3)
បក ងWនង
P ងW=ន ម ( 2 ) នង
5 ( 3) យង8ន 1
9
2y − y = −12500
2
5
− y = −12 500
2
y = 5 000
7 ន<
ជន< y = 5000 កOBង ម (1) គ8ន 1
2 x + 3 ⋅ 5 000 = 21000
21000 − 15 000
x=
2
x = 3000
ច ន. , 4D n យកF =ថ% 3000 G
ងម<
aយ 4D 'មយ
< កF =ថ% 5000 G -

កំែណេ យ ៃហ ហុន
ិ , ៃហ ចរ និងយ៉ត ពនក 155
េមេរ នទី១១
១១ ៖ បព័នសមី រដេឺ កទី១ នពីរអ ត

7/. កច7នន
< ក 8ក' ប ភទនម<យQ
0ង x #ច7នន
< ក 8ក' ប ភទ 500 G
នង
5 y #ច7នន
< ក 8ក' ប ភទ 1000 G
0មប7Aប' , គ8ន បពន ម 1
500 x + 1000 y = 8000  x + 2 y = 16 (1)
 ⇔ 
 x + y = 11  x + y = 11 ( 2)
យក (1) − ( 2 ) យង8ន 1 y = 5
7 <
ជន y = 5 កOBង ម ( 2 ) គ8ន 1 x = 11 − 5 = 6

ច ន. , ក 8ក' ប ភទ 500 G នច7នន


< 6 ន%ក
P
នង
5 ក 8ក' ប ភទ 1000 G នច7នន
< 5 ន%ក
P -
8/. ក ប 4ង ខRព< ន5ងកមU ' ម 0O
0ង x # ប 4ង ខRព<
ន5ង y #កមU ' ម 0O
0មប7Aប', គ8ន បពន ម 1
x +1 = y + 7 (1)
  x = y + 6
x ⇔
 2 + 1 = y  x + 2 = 2 y ( 2)
ជ7ន< ម (1) [កOB ង ម ( 2 ) គ8ន 1
( y + 6) + 2 = 2 y
y =8
ជ7ន< y = 8 កOBង ម (1) យង8ន 1 x = 8 + 6 = 14

ច ន. , ខRព< M. ន ប 4ង 14 m aយ កមU ' ម 0O M.គ 8 m -


9/. ក ប 4ងប XE យ នងទទ
5ងទទង
P =ន M.
0ង x # ប 4ងប Xn យ នង
5 y # ប 4ងទទង
P =ន M. ( x > 0 , y > 0 )
0មប7Aប', គ8ន បពន ម 1
2 ( x + y ) = 110

( x + 5 )( y + 2 ) = xy + 150
 x + y = 55

 xy + 2 x + 5 y + 10 = xy + 150
 x + y = 55 (1)

2 x + 5 y = 140 ( 2)
0ម (1) គ8ន 1 y = 55 − x ( 3)

កំែណេ យ ៃហ ហុន
ិ , ៃហ ចរ និងយ៉ត ពនក 156
េមេរ នទី១១
១១ ៖ បព័នសមី រដេឺ កទី១ នពីរអ ត

7 <
ជន ម ( 3) កOBង ម ( 2 ) យង8ន 1
2 x + 5 ( 55 − x ) = 140
− 3 x = −135
x = 45
7 <
ជន x = 45 [កOBង ម ( 3) គ8ន 1 y = 55 − 45 = 10

ច ន. M. ន ប 4ងប Xn យ 45 m នង
5 ទទង
P 10 m -
10/. គHM 8ក'\ប'
\ប'
0ង x # =ម%ន7 H នង
5 y # =ម%ន7 P
0មប7Aប', គ8ន បពន ម 1
6 x + 4 y = 20 − 1.5

2 x + 5 y = 20 − 1
6 x + 4 y = 18.5 (1)

2 x + 5 y = 19 ( 2)
0ម ( 2 ) គ8ន 1 2 x = 19 − 5 y ( 3)
7 <
ជន ម ( 3) [កOBង ម (1) យង8ន 1
3 (19 − 5 y ) + 4 y = 18.5
57 − 15 y + 4 y = 18.5
11 y = 57 − 18.5
y = 3.5
7 <
ជន y = 3.5 [កOBង ម ( 3) គ8ន 1 2 x = 19 − 5 ⋅ 3.5 ⇒ x = 0.75

ច7 `. x = 0.75 នង
5 y = 3.5 M7 យង8ន 1
4x + 4 y = 4 ( x + y )
= 4 ( 3.5 + 0.75 )
= 17
= 20 − 3
ច ន. ប គទញ
5 ន7 H 4 នង
5 ន7 P 4 គនង 5 3$ -
P \ប' 8ក'4ញ
11/. កព ច7នន
< M.
0ង x #ច7នន
< ទ នង
5 y #ច7នន
< ទo ( x > y )
0មប7Aប', គ8ន បពន ម 1
 x − y = 22 (1)

2 x + 3 y = 246 ( 2)
0ម (1) គ8ន 1 y = x − 22 ( 3)
7 <
ជន ម ( 3) [កOBង ម ( 2 ) យង8ន 1

កំែណេ យ ៃហ ហុន
ិ , ៃហ ចរ និងយ៉ត ពនក 157
េមេរ នទី១១
១១ ៖ បព័នសមី រដេឺ កទី១ នពីរអ ត

2 x + 3 ( x − 22 ) = 246
5 x = 246 + 66
x = 62.4
7 <
ជន x = 62.4 [កOBង ម ( 3) គ8ន 1 y = 62.4 − 22 = 40.4

ច ន.ច7នន
< I7ងព M.គ 62.4 នង
5 40.4 4V 8នក7H ' -
12/. គHMច7នន
< I7ងព M.
0ង x នង
5 y #ច7នន
< I7ងព M.
0មប7Aប',ព ច7នន
< នផ បក ន
@ ង
P 490 នង នផ ធប ន
@ ង
3
5 P 7
គ8ន បពន ម 1
 x + y = 490 (1)

x 3
y = 7 ( 2)

0ម ( 2 ) គ8ន 1 y = x
7
( 3)
3
7 <
ជន ម ( 3) [កOB ង ម (1) យង8ន 1
7
x+ x = 490
3
x = 147

M. ( 3) : y = ⋅147 = 343
7
3
< I7ងព M.គ 147 នង
ច ន.ច7នន 5 343 4V 8នក7H p
' L ងqយ r -
13/. គHM ច7នន
< នម<យQ
0ង x នង
5 y #ច7នន
< I7ងព M.
0មប7Aប', ផ ធ ប=នព ច7នន
< ន P 4 នង
@ ង =នផ ក ប '] ន
@ ង
5
5 P
324 , គ8ន បពន ម 1
x 5
y = 4

( x − y ) 2 = 324

 5
x = y
 4
 x − y = ±18
 5
x = y (1)
កH  4
 x − y = 18
 ( 2)
7 <
ជន ម (1) [កOBង ម ( 2 ) គ8ន 1

កំែណេ យ ៃហ ហុន
ិ , ៃហ ចរ និងយ៉ត ពនក 158
េមេរ នទី១១
១១ ៖ បព័នសមី រដេឺ កទី១ នពីរអ ត

5
y − y = 18
4
y = 4 ⋅18 = 72

M. (1) : x = ⋅ 72 = 90
5
4
 5
x = y ( 3)
កH 4
 x − y = −18
 ( 4)
7 <
ជន ម ( 3) [កOBង ម ( 4 ) គ8ន 1
5
y − y = −18
4
y = 4 ⋅ ( −18 ) = −72

M. ( 3) : x = ⋅ ( −72 ) = −90
5
4
ច ន. ប < I7ងព M.គ 90 , 72 ( −90 , − 72 -
B មក ច7នន
14/. គHM ប 4ង ជ^ង BC , CA នង
5 AB
0ង a , b , c # ប 4ង ជ^ង BC , CA , AB ង?O
0មប7Aប', គ8ន បពន ម 1
a + b + c = 192

 a b c
 2.5 = 2 = 1.5

យ ប កsHt [ បពន ម គ8ន 1


a b c a+b+c
= = =
2.5 2 1.5 2.5 + 2 + 1.5
a b c 192
= = =
2.5 2 1.5 6
a b c
= = = 32
2.5 2 1.5
 a
 2.5 = 32
 a = 32 ⋅ 2.5 = 80
b 
 = 32 ⇒ b = 32 ⋅ 2 = 64
2 c = 32 ⋅1.5 = 48
 c 
1.5 = 32

ច ន. AB = 48 m , BC = 80 m , CA = 64 m 4V 8នគHM -

ច ន គ ង a = BC a ជងឈម នម A

ន បប ង កងធ "#$ %

កំែណេ យ ៃហ ហុន
ិ , ៃហ ចរ និងយ៉ត ពនក 159
េមេរ នទី១១
១១ ៖ បព័នសមី រដេឺ កទី១ នពីរអ ត

15/. គHMច7
គHMច7នន
< នម<យQ
0ង x នង
5 y #ច7នន
< I7ងព M.
0មប7Aប', គ8ន បពន ម 1
 x − y = 36  x − y = 36  x − y = 36
  
25 ⇔  x = ± 5 ⇔  x
2
 x  y
 y  = y  =±
  81  9 5 9
 x − y = 36

ក H x y
 5 = 9

ប កsHt គ8ន 1
x y x− y
= =
5 9 5−9
x y 36
= = = −9
5 9 −4
x
 5 = −9  x = −45
 ⇒
 = −9  y = −81
y
 9
 x − y = 36

ក H x y
 5 = − 9

ប កsHt គ8ន 1
x y x− y
=− =
5 9 5+9
x y 36 18
=− = =
5 9 14 7
 x 18  18 90
 5 = 7  x = 7 ⋅ 5 = 7
 ⇒
− y = 18  y = 18 ⋅ ( −9 ) = − 162
 9 7  7 7
ច ន. ប
B មកច7នន
< I7ងព M.គ −45 , − 81 ( -
90 162
,−
7 7
16/. គHM ប 4ង8 នម<យQ
0ង x នង
5 y # ប 4ង=ន8 នម<យQ
0មប7Aប' , គ8ន 1
x 3 x y
 =  =
y 5 ⇔ 3 5
 x + y = 64  x + y = 64

ប កsHt គ8ន 1

កំែណេ យ ៃហ ហុន
ិ , ៃហ ចរ និងយ៉ត ពនក 160
េមេរ នទី១១
១១ ៖ បព័នសមី រដេឺ កទី១ នពីរអ ត

x y x + y 64
= = =
3 5 3+5 8
x
 3 = 8  x = 24
 ⇒
 y = 8  y = 40
 5
ច ន.8 I7ងព =នច B H`Oយ M. ន qh ' 24 m នង
5 40 m -
17/. ក7H ' A នង
5 B
បMT ' Ax + By = 9 0
' មច7HBច ( 2,1) នង
5 ( −3,3) គ8ន បពន ម 1
2 A + B = 9 2 A + B = 9 (1)
 ⇔ 
−3 A + 3B = 9 − A + B = 3 ( 2)
យក (1) − ( 2 ) គ8ន 1 3 A = 6 ⇒ A = 2
M. ( 2 ) : −2 + B = 3 ⇒ B = 5
ច ន. A = 2 , B = 5 4V 8នក7H ' -
18/. . /យ បពន ម
2 2 1
x − y = 2
យង ន បពន ម 1 
1 + 5 = 3
 x y 4
0ង X = គ8ន បពន ម ថ@ 1
1 1
,Y =
x y
 1  1
2 X − 2Y = 2  X − Y = 4 (1)
 ⇔
3  X + 5Y = 3
 X + 5Y = ( 2)
 4  4
យក ( 2 ) − (1) គ8ន 1 6Y = ⇒ Y =
1 1
2 12
M. (1) : X = + =
1 1 1
4 12 3
1 1
=
 x 3 x = 3
គ8ន 1  ⇒
1 = 1  y = 12
 y 12
ច ន. បពន ម នគច ម%យ ( 3,12 ) -
19/. ផ 5 ផ A \ច ក' កន #ង B ( ទ ? a B h ?
0ង x #cO 7
cO 7 2008 4V នង
P x=0
0ង y #ច7នន
< ផ 5 ផ 8ន ក' (គ5 #eន គfង)

កំែណេ យ ៃហ ហុន
ិ , ៃហ ចរ និងយ៉ត ពនក 161
េមេរ នទី១១
១១ ៖ បព័នសមី រដេឺ កទី១ នពីរអ ត

គ8ន0Aង 1
x
0 1 2 3
yA
2.3 2.7 3.1 3.5

yB 2.9 3.3 3.7 4.1


0ក7 Hន=ន ក'ផ 5 ផ A គ 1 a = 0.4 ថ

0ក7 Hន=ន ក'ផ 5 ផ B គ 1 a ' = 0.4 ថ

យ a = a' ននយ_ ម បMT ' y A នង


5 yB ប?O (?@នច7HBច
ប ពh) -
aយ y A < yB គប' x = 0 ,1, 2 ,3 M7 y A < yB គប' =ម% x

ច ន.ផ 5 ផ A មន
5 \ច ក' កន#ង B 8ន ទ `.បMT '
y A / / yB aយបMT ' yB N បMT ' y A -

និ យេរឿង បឡងសិស ពូែក "នរ#$ន់សខ


ុ 'ន េ()*ក់ទ៩
ី ខ.0
/ំ ក់
1 េ()*ក់
សុក ដល់)ក់
* ទី ១០ ខ.0
/ំ ក់
1 េ()*ក់ សុកេទៀត ។ េពលដល់)ក់
* ទី ១១ និង ១២ េគក៏
;<ក=រ បឡង ។
េ(>*ំ ២០០៩ ប?ន
@ សីខA
./ំ ង
ំ ពីរ ន បឡង"នរ#$ន់សខ
ុ 'ន េនះCប់ពី
ព) ី ក់
*
សុកAំងពីរ Dក់ ។ ខ./ំ នជូនពួកេគេG បឡងទូAង
ំ េខតH េ យកIស
ី ងJឹម ែតទី
បMNប់ េ(ែត01ក់ដែដល ។

OPយខ./ំ បពនQ គR សុកែ ស ៖ េ =យពី ន'នេសៀវេUេរឿងមួយOនចំណង


េជើង) ៖ “ បពនQ គR សុកែ ស”
ស របស់អក
X និពនQ េអៀង ពីសី ខ.ក
/ំ ន
៏ ង
ឹ 'ណិតOPយខ.
យខ./ំ
ែដលYត់កC
៏ បពនQ គR សុកែ សមួយរូបែដរ ។ ឪពុកខ.O
/ំ ន ក់ែខតិចCង តួអង[
កXង
/ េរឿងេGេទៀត េហើយកូនYត់Oន ៨Dក់ ែដលខ.C
/ំ កូនទី៦ ។ េហតុអក
^ី ៏ បងប?ន
@ ខ./ំ
េ ចើនេម៉ះ ? េបើOយខ.
P យ /ំ កកូនែត ៥ ដូចកXង
/ េរឿង េតើOនខ.ដ
/ំ ច
ូ សព^ៃថbេនះ ឬអត់ ?
បងប?ន
@ របស់ឪពុកខ./ំ តRវ នd1ប់កង
X/ របបeលពិfដអស់ េ(សល់ែតYត់Oក់
*
Yត់គO
ឺ ន'រមhណឯ
៍ េ= , ឪពុកខ.Y
/ំ ត់កំ "ឪពុកkំងពី'យុ មិនAន់ ១០>*
១០>*ំ ។
មូលេហតុេនះេហើយេទើបបlmលឲoYត់យកកូនេ ចើន ។ OPយខ./ំ តRវេឆqតចិMម
Nឹ
Oន់(ទុកលក់dច់) A (យកស៊ត
ុ េធ^'
ើ vរ និងលក់) េY(យកកូនលក់ CកO1ង

េធ^ែើ ស wមក៍េធ^ជ
ើ )
ី ជRក(លក់កន
ូ និងdច់)... ។ ឪពុកខ.េ/ំ ឆqតេពលទំេនរ ំ តកួន
ផzរី នេDង តសក់ ៃឆ)វ ៃស{ ដំឡង
@ មី ដំឡង
@ C$ ឪឡឹក សែណ|ក ... េGkមរដូវ
េដើម{ី ជក់ទក
ុ ហូប េធ^ៃើ ឆប៉វូ លក់ ... ។ OPយខ./ំ តRវេដរសឹកទុក បក់ផះ
z និងលក់ ។
េពល ពឹកYត់
កYត់េ =កពីយប់េដើម{ី ប
ំ បរស ដួសទុកឲo តCក់ទក
ុ ឲoពួកខ.ហ
/ំ ប
ូ C
មួយពងAៃ ប ៃឆប៉វូ =Pមៃ ប េសៀង ជក់ៃស} ... មុនេពលេGdw ។

កំែណេ យ ៃហ ហុន
ិ , ៃហ ចរ និងយ៉ត ពនក 162
េមេរ នទី១២ ៖ ទឹសប
ី ទពី គ័រ

ម នទ ទ បទព គ

1/. ច គ x កង ប ង ន

2/. ច ! "# $ ជ&ងនម'យ) ង *ម ន +, ABC - + * .កង / ទ ?


ក. AB = 7 , BC = 9 , AC = 12

ខ. AB = 52.8 , BC = 45.5 , AC = 69.7

គ. AB = 83 , BC = 49 , AC = 67

3/. ច គ 3ផ5 ក63ន + * ABC

4/. ង#ងម
$ យ
' .789នផ:+
; O នង
; * ,ន
< ង 5cm
AB -=ង>+ធ
$ .789ន "# $ , < 8 cm
ចគ ប.@ង OM

5/. + * មងAម'យ.789នកមB $ ,<


នង 3 cm ចគ 3ផ5 ក6
3ន + * មងA ន

កំែណេ យ ៃហ ហុន
ិ , ៃហ ចរ និងយ៉ត ពនក 163
េមេរ នទី១២ ៖ ទឹសប
ី ទពី គ័រ

6/. ច ក +$ x នង
; y 3ន ប ង *មគ+
; -m

7/. ច គ កមB $3ន * 3ន ប ង



' គ 9C3ន *

8/. គបម'យ9ន "# $ ជ&ង ,ន


< ង 1cm ចគ =ង>+$ ទDង3នគប
9/. ច គ "# $ ជ&ង3ន ប ង គ+
; - cm
ក. ច គ AB

ខ. ច គ AH

គ. ច គ BH នង
; HC

10/. យនE F ព F ចញពច ច O 7ចH មទ; I ងH Ox នង


; Oy
គ7ងJយនE F Kងព បក
, ប Lយ 8MNន ,H
< Oយ
, F ចញព
ច ច O P 9Qង.+ម'យ ចគ ច9Rយច 3នយនE F នម'យ) P
ព8.78STU+ពHVន 100 km ( ម8
, ប Pទព ប 5 ប$)

កំែណេ យ ៃហ ហុន
ិ , ៃហ ចរ និងយ៉ត ពនក 164
េមេរ នទី១២ ៖ ទឹសប
ី ទពី គ័រ

ច មយ
1/. គ x

ម ប, =ន@+EនZ ទ បទព គ យង
, Vន [
10 2 = 6 2 + x 2
x 2 = 100 − 36
x 2 = 64
x = 8m ,x > 0
7ច ន x = 8 m +D@Vនក +$
2/. ច ! "# $ ជ&ងន
ងនម'យ) ង *ម ន +, ABC - + * .កង / ទ ?
ក. AB = 7 , BC = 9 , AC = 12

យង
, 9ន AB = 7 , BC = 9 , AC = 12 គVន [
AB 2 = 7 2 = 49
BC 2 = 92 = 81
AC 2 = 122 = 144
\] AB 2 + BC 2 = 49 + 81 = 130 ≠ AC 2
7ច ន + * ABC ន មន
; .មន- + * .កង ទ
ខ. AB = 52.8 , BC = 45.5 , AC = 69.7

យង
, 9ន AB = 52.8 , BC = 45.5 , AC = 69.7 គVន [
AB 2 = 52.82 = 2787.84
BC 2 = 45.52 = 2070.25
AC 2 = 69.7 2 = 4858.09
\] AB 2 + BC 2 = 2787.84 + 2070.25 = 4858.09 = AC 2

កំែណេ យ ៃហ ហុន
ិ , ៃហ ចរ និងយ៉ត ពនក 165
េមេរ នទី១២ ៖ ទឹសប
ី ទពី គ័រ

7ច ន + * ABC ន - + * .កង
គ. AB = 83 , BC = 49 , AC = 67

យង
, 9ន AB = 83 , BC = 49 , AC = 67 គVន [
AB 2 = 832 = 6889
BC 2 = 49 2 = 2401
AC 2 = 67 2 = 4489
\] AC 2 + BC 2 = 4489 + 2401 = 6890 ≠ AB 2
7ច ន + * ABC ន មន
; .មន-
.មន- + * .កង ទ
3/. គ 3ផ5 ក63ន + * ABC

ម ប, គVន [
AD 2 = 152 − 122
= 3 ⋅ 27
= 92
AD = 9 cm
Oយ
, ម^Qង ទ + [
CD 2 = 412 − 9 2
= 32 ⋅ 50
= 402
CD = 40 cm
គVន , ក63ផ53ន + * ABC គ_ [
1
A= ⋅ BC ⋅ AD
2
1
A = ⋅ (12 + 40 ) ⋅ 9
2
A = 234 cm 2
7ច ន 3ផ5 ក63ន + * ABC គ_ 234 cm 2

4/. គ ប.@ង OM
ម ទ បទព គ , គKញVន [

OM 2 = OA2 − AM 2
Lយ OA = R = 5 cm , AM = = 4 cm គVន [
AB
2
OM 2 = 52 − 42
OM 2 = 9
OM = 3 cm
7ច ន OM = 3 cm +D@Vនក +$`Qង"យ 8
a

កំែណេ យ ៃហ ហុន
ិ , ៃហ ចរ និងយ៉ត ពនក 166
េមេរ នទី១២ ៖ ទឹសប
ី ទពី គ័រ

5/. គ 3ផ5 ក63ន


ក63ន + * មងA
ង a - "# $ ជ&ង3ន + * មងA
2

ម ទ បទព គ , គVន [ h +   = a 2
a 2

2
.+ Lយ h = 3 cm គVន [
a2
3+ = a2
4
3a 2
=3
4
a2 = 4
a = 2 cm ,a > 0
ក63ផ5 ប $ + * គ_ [
1
A= ⋅ ah
2
1
A = ( 2 cm )
2
( 3 cm )
A = 3 cm2
7ច ន 3ផ5 ក6 ប $ + * មងA គ_ 3 cm 2 +D@Vនក +$
6/. ក +$ x នង
; y 3ន ប ង *មគ+
; -m
( បទ ), ម ទ បទព គ គVន [
x 2 = 62 + 82
x 2 = 102
x = 10 m
Oយ
, ម^Qង ទ + [
122 = y 2 + x 2
y 2 = 144 − 100
y 2 = 44
y = 44 = 2 11 m
7ច ន x = 10 m នង
; y = 2 11 m +D@Vនក +$
( បទ ), ម ទ បទព គ យង
, Vន [
x 2 = 16 2 + ( 50 − 38 )
2

x 2 = 256 + 144
x 2 = 400
x = 20 m
7ច ន x = 20 m +D@Vនក +$`Qងធ

កំែណេ យ ៃហ ហុន
ិ , ៃហ ចរ និងយ៉ត ពនក 167
េមេរ នទី១២ ៖ ទឹសប
ី ទពី គ័រ

7/. គ កមB $3ន *


ម ទ បទព គ , គVន [
h 2 + 12 2 = 182
h 2 = 324 − 144
h 2 = 180
h = 180 = 6 5 cm
7ច ន * 3ន ប 9នកមB $ h = 6 5 cm
គ 9C3ន *
ម បមនE9C3ន * [ V = Ah .78 A = π R 2
1
3
1
⇒ V = ⋅ π ⋅122 ⋅ 6 5
3
V = 288π 5 cm3
7ច ន 9C3ន * គ_ V = 288π 5 cm3
8/. គ =ង>+$ ទDង3នគប (គ a)

Lយគប9ន "# $ ជ&ង ,ន


< ង 1cm \] [

AB = BC = CD = DA = EF = FG = GH = HE = AE = BF = CG = DH = 1 cm
ម ទ បទព គ , គVន [
a 2 = AE 2 + AC 2
(
a 2 = AE 2 + AB 2 + BC 2 )
a 2 = 12 + 12 + 12
a2 = 3
a = 3 cm
7ច ន =ង>+$ ទDង3នគប 9ន "# $ 3 cm

កំែណេ យ ៃហ ហុន
ិ , ៃហ ចរ និងយ៉ត ពនក 168
េមេរ នទី១២ ៖ ទឹសប
ី ទពី គ័រ

9/. ក. គ AB

ម ទ បទព គ កង + * .កង ABC , គVន [


AB 2 = BC 2 − AC 2
= 20 2 − 162
= 144
AB = 12 cm
7ច ន AB = 12 cm +D@Vនគ Lយ@ធ
; មbc
ខ. គ AH

ក63ផ53ន + * ABC គ_ [ A =
1
⋅ AH ⋅ BC (1)
2
/ ម^Qង ទ + A = ⋅ AB ⋅ AC
1
( 2)
2
ផ5ម ម* (1) នង
; ( 2 ) គVន [
AH ⋅ BC = AB ⋅ AC
AB ⋅ AC
AH =
BC
12 ⋅16
AH = = 9.6 cm
20
7ច ន AH = 9.6 cm +D@Vនក +$
គ. គ BH នង
; HC
ម ទ បទព គ កង + * .កង ABH នង
; ACH គKញVន [
 BH = AB − AH
2 2 2


CH = AC − AH
2 2 2

 BH 2 = 12 2 − 9.6 2

CH = 16 − 9.6
2 2 2

 BH 2 = 51.85

CH = 163.84
2

 BH = 7.2 cm

CH = 12.8 cm
7ច ន BH = 7.2 cm នង
; CH = 12.8 cm +D@Vនក +$
10/. គ ច9Rយច 3នយនE F នម'យ) P ព8.78STU+ពHVន 100 km
ង vx នង
; v y - 8MNន ប $យនE F មទ; I Ox នង
; Oy ងH
P យd ព8 t 7ចH យនE F Kងព F O, Vនច9Rយ d x = vxt នង
;
d y = vyt

.+ មបeប$ vx = v y \] d x = d y

កំែណេ យ ៃហ ហុន
ិ , ៃហ ចរ និងយ៉ត ពនក 169
េមេរ នទី១២ ៖ ទឹសប
ី ទពី គ័រ

Oយ
, ម ទ បទព គ d x2 + d y2 = 1002
d x2 + d x2 = 1002
2d x2 = 1002
2d x = 100
100
dx = = 50 2 km
2
7ច ន d x = d y = 50 2 km +D@Vនក +$

...ពួកខំទទួល នែត"#ក់មយ
ួ រយេរៀលប៉េុ ()ះស,"ប់េ-./ េហើយ,ត1វេដើរ
២ គ.ម េទើបដល់./ ។ ឪពុកខំ8ត់ជះ
ិ កង់មយ
ួ ែដលរដ:ែចកឲ<8ត់ ។ =លមុន
8ត់ជះ
ិ កង់ក>?ស់មយ
ួ ែតេ,=យមក8ត់ក៏ នកង់រដ:ែចកឲ<េAះជិះBងថDប
ី នFច
ិ ។
ខំជន
ួ =លជិះGមួយ8ត់ ែតមិនសូវេទ េ,Iះចង់េដើរវញសបKយGងេ,Iះ"នមិតF
ភកFេិ គេដើរ,គប់8Mំ
# ងហNង
O ។ េ,=យមកេទើ
េ,=យមកេទើតឪពុកខំ8ត់ ន,បឡងGប់G
,គ1ឧេទRសបឋមសិកTជំAន់ទ១
ី េហើយ,ត1វេឡើងមកសិកTបែនVមេWភXេំ ពញ ៣ែខ ។
ដំ(ក់=លេAះមិនដឹង[8ត់លំ កប៉(
ុ េទ
) ? េ,Iះ,ត1វមក]^យ េហើយGរដូវ
េធNែើ ,សច"`រផង ។ "bយខំ,ត1វBcប់រង=រdរG"bយផង Gឪពុកផង ែតសំ(ង,តង់
[ បងៗខំធំ
ធៗ ំ ,8ន់េបើេហើយ ។ ឪពុកខំ,ត1វមក.#ក់េWេរៀនឯភXេំ ពញ មិនដឹងGហូប
ចុក,ត1វតgិតេតghតប៉(
ុ េទ
) េ,Iះ, ក់ែខតិច េហើយ, ក់បប ៉ មិនMន់េបើក ។ េពល
ំ ន
បំបន
៉ ចប់8ត់ នេបើក, ក់ េហើយទិញ នម៉ត
ូ ក
ូ >?ស់មយ
ួ (jចKស់(ស់[
8ត់kយjយតgិតេតghត កXងរយះេពល៣
ងរយះេពល៣ែខេនះគឺេដើមgីសនmំ
សនmំលយ
ុ ទិញម៉ត
ូ )
ូ ។
...បងៗរបស់ខម
ំ ន
ិ នេរៀនសូ,តឲ< នេ,ចើននឹងេគេទ គឺ នែត,តឹមមធ<មសិកT
បឋមភូម(ឌី
ិ(ឌីបម
O ) ែតប៉េុ ()ះ េ,Iះេបើេរៀនេលើសេនះ ,ត1វចំ(យេ,ចើន (ប>pក់ផង
ែដរ[ បងៗខំសទ
ុ ែq តសិសmេឆXម
ើ stប់ទទួល ន=រសរេសើរពីស(
ំ ក់េ/ក,គ1/
អXក,គw) ។ "នែតខំ និងបxន
O ,សីMង
ំ ពីររបស់ខប
ំ េ៉ុ ()ះែដល នបyzបស
់ >{ប,ត
មធ<មសិកTទុតយ
ិ ភូមិ ( ក់ឌប
ុ ) េហើយឥឡOវខំ នេធNឲ
ើ <ភរ របស់ខ=
ំ យG
t
,បពនq,គ1បេ,ងៀន,សុកែ,ស "#ក់េទៀតេ-េហើយ ។ បxន
O ,សី របស់ខក
ំ ៏ នkប់យក}
ជីពG,គ1បេ,ងៀនេនះែដរ េហើយសល់បន
xO ,សីេ~ខំ ចង់េរៀនេពទ< ែតមិន"នលទq•ព
ក៏េរៀន,តឹមែតឆDបមធ<មវញសិនេ- kំៃថ€េ,=យ េបើ"នឲ=សសឹមគិតេទៀត ។
=រែដលខំ និងបxន
O ខំេធN,ើ គ1េនះ មិនែមនG=រ8•នជេ,មើស េAះេទ គឺGនិសm័យ
Gចំណល
ƒ ចិតF ។

"bយខំ ,បពនq ,គ1,សុកែ,ស េពលេនះ,គ1,សុកែ,ស 8ត់ នចូលនិវតFន៍


នេ៍ ហើយ
អNែី ដល8ត់ទទួល នពី=រខិតខំអស់ពក
ី "tង
ំ =យ ចិតប
F េ,ងៀនសិសm ៣០ ]#គ
ំ ឺ
េម យស,មឹតមួយ និង, ក់ចល
ូ និវតFន៍ ១៨ មុន
ឺ េរៀល កXងមួយែខ ។ ស"គមន៍
បុគល
‡ ក
ិ សិកTែខDរ ˆម=ត់មយ
ួ ែខៗអស់
ែខៗអស់ ២ េ- ៣ Iន់េរៀល ៕

កំែណេ យ ៃហ ហុន
ិ , ៃហ ចរ និងយ៉ត ពនក 170
េមេរ
េមេរ នទី១៣
១៣ ៖ រងង់ និងប ត់

ម នទ ងង នងប

1/. កង បនមយ ង មន គ ន ងងផ! O នង PT "ប ប# $នង


%
ងង ង T &

ក. ច ក )ម*)នម+ a ខ. ច គ- )ម*)នម+ a នងម+ b

គ. ច ក )ម*)នម+ b ឃ. ច គ- )ម*)នម+ a នងម+ b

ង. ច ក )ម*)នម+ a , b នងម+ c ច. AOT នង BOP "ប ង


ច ក )ម*)នម+ a នងម+ b &

កំែណេ យ ៃហ ហុន
ិ , ៃហ ចរ និងយ៉ត ពនក 171
េមេរ
េមេរ នទី១៣
១៣ ៖ រងង់ និងប ត់

2/. កង បនមយ ង មន គ ន PT នង PV "ប ប# $នង


% ងងផ!
O ង T នង V & ក /0ម+123មន453 &

(ក) (ខ)

(គ) (ឃ)
3/. គ ន ងងផ! O 123 ន PA នង PB "ប ប# នង
% ងង ង8 ង A
នង B 2ច ប ង ម & ច គ- /0 + r នង /0 d & /0គ
" cm &

4/. 9ក
9កង ប ង4:+ ន ន O "ផ! )ន ងង ;យ
< PA នង PB "ប ប#
$នង
% ងង ង8 ង A នង B & /0គ " cm &
ច គ- =
ក. /0 b &
ខ. /0 c &
គ. ប )នច - OAPB &
ឃ. )ផ ក>)នច - OAPB &
5/. កង ប ង មន ន AB , BC នង AC "ប ប# $នង
% ងង ង8 ង
D , E នង F & ច គ- )ផ ក>)ន
ក>)ន - ABC & /0គ " cm &

កំែណេ យ ៃហ ហុន
ិ , ៃហ ចរ និងយ៉ត ពនក 172
េមេរ
េមេរ នទី១៣
១៣ ៖ រងង់ និងប ត់

(ប ប ទ ) (ប ប ទ )

6/. គ ន PA នង PB "ប ប# $នង


% ងង123 នផ! O នង + 3cm &
គ?@ AP = 82.4 cm នង ∠AOB = 140o & ច គ- )ផ ក> =
ក. ច - APBO &
ខ. )ផ ក>)នច មAកB0 ច AOB (យក π = )&
22
7
7/. កង ងងផ! O គ ន AT នង BT ន ប1Cង 20 cm & ប< TQO "ប
ង123 TQ = 10 cm & ច ក /0)ន + ងង &

(ប ប ទ ) (ប ប ទ )

8/. 9កង ងងផ! O 123 ន % 4.5 cm & គ ន TA នង TB "ប


+ 0D<នង
ប# នង
% ងង ង8 ង A នង B 123 TA = 9 cm &
ក. ច គ- ប )នច - OATB &
ខ. ច គ- )ផ ក>)នច - OATB &
9/. 9កង ប ង មន គ?@ ងងព 123 នផ! A នង B ;យ
< ន
/0 + 0D<8 4 cm & ច+ F ងងផ! A ន CB នង CD "ប ប#
$នង
% ងង ង8 ង B នង D & ច+ F ងងផ! B ន DA នង DC
"ប ប# $នង
% ងង
ងង ង8 ង A នង C & ច ក)ផ ក>)នច -
ABCD &

10/. TA នង TB "ប ប# $នង


% ងងផ! O 123 ន + 0D<នង
% 5 cm &
ប< ∠ATB = 60o ច គ- =
ក. ∠AOB &

កំែណេ យ ៃហ ហុន
ិ , ៃហ ចរ និងយ៉ត ពនក 173
េមេរ
េមេរ នទី១៣
១៣ ៖ រងង់ និងប ត់

ខ. )ផ ក>)នច មAកB0 ច AOB (យក π = )&


22
7

(ប ប ទ ) (ប ប ទ )

11/. ងងព នផ! ម O ន + 8 cm នង 10 cm & AB "GងH ធ)ន ងងធ+


;យ
< ប# ងង ច ង T & ច គ- AB & (1- + = គ0 OA នង OB )

(ប ប ទ ) (ប ប ទ )

12/. គ ន MA , MB នង MC "ប ប# $នង


% ងងផ! O នងផ! O' &

ច ប/JញB MA = MC &
13/. EF នង EG "ប ប# $នង
% ងងផ! H & គ ន ∠JEF = 220 &
ច គ- =
ក. ∠EGH
ខ. ∠HGF
គ. ∠FGE &

កំែណេ យ ៃហ ហុន
ិ , ៃហ ចរ និងយ៉ត ពនក 174
េមេរ
េមេរ នទី១៣
១៣ ៖ រងង់ និងប ត់

ច មយ
1/. ក. រកតៃមៃនមុំ a
េយើង ន PT ប៉ះរងង់ផត
ិ O តង់ T
េគ ន PT ⊥ OT ⇒ ∠OTP = 90o
កង តីេ ណ OTP ន ∠OTP = 90o េគ ន៖
∠TOP + ∠OPT = 90o
∠OPT = 90o − ∠TOP
េ"យ ∠OPT = a , ∠TOP = 50o (សម%តក
ិ ម%)

⇒ a = 90o − 50o = 40o


ដូចេនះ a = 40o ត*វ នកំណត់ ។
ខ. គណ.តៃមៃនមុំ a និងមុំ b
េយើង ន PT ប៉ះរងង់
រងង់ផត
ិ O តង់ T
េគ ន PT ⊥ OT ⇒ ∠OTP = 90o
កង តីេ ណ OTP ន a = ∠OTP = 90o
េហើយកង តីេ ណ OTP ន TO = TP
.ំឲ21 តីេ ណ OTP 3 តីេ ណែកងសម តកំពល
ូ T
វ7 ក ៖ ∠TOP = ∠TPO = 45o .ំឲ21 b = 45o
ដូចេនះ a = 90o និង b = 45o ត*វ នគណ. ។
គ. រកតៃមៃនមុំ b
េយើង ន PT 3ប.8ត
1 ប
់ ះ៉ រងង់ តង់ T , .ំឲ21 េគ ន៖
∠OTP = 90o
⇒ 40o + b = 90o
b = 90o − 40o = 50o
ដូចេនះ b = 50o ត*វ នគណ. ។
ឃ. គណ.តៃមៃនមុំ a និងមុំ b
េយើង ន PT 3ប.8ត
1 ប
់ ះ៉ រងង់ តង់ T , .ំឲ21 េគ ន៖
∠OTP = 90o
⇒ ∠OTS + ∠STP = 90o
a + 35o = 50o
a = 55o
េហើយេ;កង តីេ ណ OTS ន OS = OT ( ៃំ នរងង់ែតមួយ)

កំែណេ យ ៃហ ហុន
ិ , ៃហ ចរ និងយ៉ត ពនក 175
េមេរ
េមេរ នទី១៣
១៣ ៖ រងង់ និងប ត់

.ំឲ21 េគ ន តីេ ណ OTS 3 តីេ ណសម ត កំពល


ូ O , េគ ន៖

∠OTS = ∠OST = a
េយើង ន ៖ b + a + a = 180o
b = 180o − 2a
b = 180o − 2 ⋅ 55o = 70o
ដូចេនះ a = 55o និង b = 70o ត*វ នគណ. ។
ង. រកតៃមៃនមុំ a , b និងមុំ c
=មបំ>ប់, េគ ន៖
2b + 65o = 180o
180o − 65o
b= = 57.5o
2
េហើយ b + c = 90o .ំឲ21 េគ ន ៖ c = 90o − 57.5o = 32.5o
ម1?ង
@1 េទៀត , a + 65o = 90o ⇒ a = 25o
ដូចេនះរCDស
1 ម
់ ុំ a = 25o , b = 57.5o និង c = 32.5o ត*វ នគណ. ។
ច. រកតៃមៃនមុំ a និងមុំ b
=មបំ>ប់ , េគ ន៖
∠AOB = 180o − ∠AOP
= 180o − 130o = 50o
េហើយ តីេ ណ AOB 3 តីេ ណសម តកំពល
ូ O េយើង ន៖
2a + ∠AOB = 180o
180o − 50o
a= = 65o
2
េហើយ ∠AOB = ∠POT (មុទ
ំ ល់កព
ំ ល
ូ )
.ំឲ21 ∠POT = 50o
េ.ះ b = ∠OPT = 90o − 50o = 40o េ Eះ PT ប៉ះនឹងរងង់ តង់ T
ដូចេនះ a = 65o និង b = 40o ត*វ នគណ. ។
2/. រករCDស
1 ម
់ ែុំ ដលមិនGHល
1 ់
(ក) រក a , b និង c
=មរូប , េយើង ន៖
a + 22 = 90o
a = 90o − 22o = 68o
ម1?ងេទៀត
@1ងេទៀត , =មលកIណៈៃនប.8ត
1 ប
់ ះ៉ នឹងរងង់ , េគ ន៖
c = 22o និង b = 68o

កំែណេ យ ៃហ ហុន
ិ , ៃហ ចរ និងយ៉ត ពនក 176
េមេរ
េមេរ នទី១៣
១៣ ៖ រងង់ និងប ត់

ដូចេនះ a = b = 68o និង c = 22o ត*វ នកំណត់ ។


(ខ) រក a , b និង c
េ"យ PT និង PV 3ប.8ត
1 ប
់ ះ៉ នឹងរងង់ផត
ិ O តង់ T និង V េរៀងKL1 ,
េគ ន៖
∠PTO = c = 90o
∠PVO = a = 90o
ម1?ង
@1 េទៀត a + b + c + 42o = 360o (ផលបូកមុក
ំ ងៃនចតុេ ណ)

⇔ b = 180o − 42o = 138o


ដូចេនះ a = c = 90o និង b = 138o ត*វ នគណ. ។
(គ) គណ.រក a , b និង c
=មលកIណៈប.8ត
1 ប
់ ះ៉ ៃនរងង់ , េគ ន៖

b = 90o − 25o = 65o


េហើយ a = c = 90o − b = 25o
ដូចេនះ a = c = 25o និង b = 65o ត*វ នគណ. ។
(ឃ) គណ.រក a , b និង c
េ"យ PT ប៉ះរងង់ តង់ T េគ ន៖
∠PTO = 90o េ.ះ a = 90o − 21o = 69o

េហើយ b + c = a
ែតកង តីេ ណ XTO ន OT = OX ( ៃំ នរងង់ែតមួយ)
េ.ះ XTO 3 តីេ ណសម តកំពល
ូ O , េគ ន ៖ b=c
69o
.ំឲ21 b = c = =
a
= 34.5o
2 2
ដូចេនះ a = 69o និង b = c = 34.5o ត*វ នគណ. ។
3/. គណ.រCDស
1 ់ ំ r និងរCDស
1 ់ d គិត3 cm
ចំេEះរូបទី១
េ"យ PB ប៉ះរងង់ តង់ B េគ ន៖
តីេ ណ OPB 3 តីេ ណែកង តង់ B
=ម ទឹសប
Nី ទពី=គ័រ , េគ ន៖
OP 2 = OB 2 + BP 2
17 2 = r 2 + 152
r 2 = 17 2 − 152 = 2 ⋅ 32 , r>0

កំែណេ យ ៃហ ហុន
ិ , ៃហ ចរ និងយ៉ត ពនក 177
េមេរ
េមេរ នទី១៣
១៣ ៖ រងង់ និងប ត់

⇒ r = 8 cm
ដូចេនះ r = 8 cm ត*វ នគណ. ។
ចំេEះរូបទី២
េ"យ PA ប៉ះរងង់ តង់ A េគ ន៖
តីេ ណ PAO 3 តីេ ណែកង តង់ A
=ម ទឹសប
Nី ទពី=គ័រ , េគ ន៖
OP 2 = OA2 + AP 2
( 5 + d ) = 122 + 52
2

( 5 + d ) = 144 + 25 = 169 , d >0


2

5 + d = 13
⇒ d = 8 cm
ដូចេនះ d = 8 cm ត*វ នគណ. ។
4/. ក. គណ. b
=មលកIណៈៃនប.8ត
1 ប
់ ះ៉ នឹងរងង់ , េគ ន៖
PA = PB ែត PB = 9 cm

េ.ះ b = PA = 9 cm
ដូចេនះ b = 9 cm ត*វ នគណ. ។
ខ. គណ. c
=មរូប , េគ ន៖
OA = OB ( ៃំ នរងង់ែតមួយ)
ែត OA = 2 cm
េ.ះ c = OB = 2 cm
ដូចេនះ c = 2 cm ត*វ នគណ. ។
គ. រកបរ7
រកបរ7 តៃនចតុេ ណ OAPB
បរ7 តៃនចតុេ ណ OAPB គឺ ៖
P = OA + AP + PB + BO
P = 2 ( OA + AP )
P = 2 ( 2 + 9 ) = 22 cm
ដូចេនះបរ7 តៃនចតុេ ណ OAPB គឺ P = 22 cm ត*វ នកំណត់ ។
ឃ. រកៃផR
រកៃផR កSៃនចតុេ ណ OAPB
កSៃផRៃនចតុេ ណ OAPB គឺ ៖

កំែណេ យ ៃហ ហុន
ិ , ៃហ ចរ និងយ៉ត ពនក 178
េមេរ
េមេរ នទី១៣
១៣ ៖ រងង់ និងប ត់

AOAPB = A∆OAP + A∆OBP


= 2 A∆OAP
1 
= 2  ⋅ OA ⋅ AP 
2 
= 2 ⋅ 9 = 18 cm 2

ដូចេនះៃផR
េនះៃផR កSៃនចតុេ ណ OAPB គឺ A = 18 cm2 ត*វ នកំណត់ ។
5/. គណ.ៃផR កSៃន តីេ ណ ABC
=មលកIណៈៃនប.8ត
1 ប
់ ះ៉ រងង់ , េគ ន៖
CF = CE = 15 cm
BE = BD = 5 cm
AD = AF = 10 cm
.ំឲ21 BC = BE + EC = 20 cm
និង AB = AD + DB = 15 cm
េគ ន៖
1
A∆ABC = ⋅ AB ⋅ BC
2
1
= ⋅15 ⋅ 20
2
= 150 cm 2
ដូចេនះៃផR
េនះៃផR កSៃន តីេ ណ ABC គឺ A = 150 cm 2 ត*វ នកំណត់ ។
6/. ក. គណ.ៃផR កSចតុ
កSចតុេ ណ APBO
=មលកIណៈប.8ត
1 ប
់ ះ៉ រងង់ , េគ ន៖
AAPBO = A∆PAO + A∆PBO
= 2 A∆PAO
1 
= 2  ⋅ OA ⋅ PA 
2 
= 3 ⋅ 82.4 = 247.2 cm 2
ដូចេនះ ៃផR កSៃន
កSៃនចតុ
ៃនចតុេ ណ APBO គឺ A = 247.2 cm2 ត*វ នកំណត់ ។
ខ. គណ.ៃផR កSៃផR
កSៃផR កSៃនចេ មៀកTសតូច AOB
x oπ r 2
=មរូបមនU កSៃផRចេ មៀកTស ៖ A =
360o
េ"យ x = 140o , r = OA = 3 cm , π =
22
7
22 2
140o ⋅ ⋅3
.ំឲ21 េគ ន A= 7
o
= 11 cm 2
360
ដូចេនះៃផR កSៃផR កSៃនចេ មៀកTសតូច AOB គឺ A = 11cm2 ។
េនះៃផR កSៃផR

កំែណេ យ ៃហ ហុន
ិ , ៃហ ចរ និងយ៉ត ពនក 179
េមេរ
េមេរ នទី១៣
១៣ ៖ រងង់ និងប ត់

7/. រករCDស
1 ៃ់ ន រំ ងង់
េ"យ TB ប៉ះ3មួ
3មួយរងង់ តង់ B
.ំឲ21 េគ ន តីេ ណ TOB 3 តីេ ណែកង តង់កព
ំ ល
ូ B
=ម ទឹសប
Nី ទពី=គ័រ , េគ ន៖
TO 2 = TB 2 + OB 2
(TQ + QO ) = TB 2 + OB 2
2

(10 + r ) = 202 + r 2
2

100 + 20r + r 2 = 400 + r 2


20r = 300
⇒ r = 15 cm
ដូចេនះរCDស
1 ់ ៃំ នរងង់េ.ះគឺ r = 15 cm ត*វ នកំណត់ ។
8/. ក. គណ.បរ7 តៃនចតុេ ណ OATB
=មលកIណៈប.8ត
1 ប
់ ះ៉ ៃនរងង់ , េគ ន៖
OA = OB = 4.5 cm
TA = TB = 9 cm
.ំឲ21 េគ ន , បរ7 តៃនចតុេ ណ OATB គឺ ៖
P = OA + AT + TB + BO
P = 2 ( OA + AT )
P = 2 ( 4.5 + 9 )
P = 27 cm
ដូចេនះបរ7
េនះបរ7 តៃនចតុេ ណ OATB គឺ P = 27 cm ត*វ នកំណត់ ។
ខ. គណ.ៃផR កSៃនចតុេ ណ OATB
=មលកIណៈប.8ត
1 ប
់ ះ៉ ៃនរងង់ , េគ ន ៖ OA ⊥ AT
េ.ះេគ ន , ៃផR កSៃនចតុេ ណ OATB គឺ ៖
AOATB = A∆OAT + A∆OBT
= 2 A∆OAT
1 
= 2  ⋅ OA ⋅ AT 
2 
= 4.5 ⋅ 9 = 40.5 cm2
ដូចេនះៃផR
េនះៃផR កSៃនចតុេ ណ OATB គឺ 40.5 cm2 ត*វ នកំណត់ ។
9/. រកៃផR កSៃនចតុេ ណ ABCD
េយើង ន , CB និង CD ប៉ះនឹងរងង់ផត
ិ A
េហើយ DA និង DC ប៉ះនឹងរងង់ផត
ិ B
េគ ន ៖ DA ⊥ DC , CB ⊥ AB , DA ⊥ AB , CB ⊥ DC

កំែណេ យ ៃហ ហុន
ិ , ៃហ ចរ និងយ៉ត ពនក 180
េមេរ
េមេរ នទី១៣
១៣ ៖ រងង់ និងប ត់

និង AB = AD ( ៃំ នរងង់ែតមួយ)
.ំឲ21 ចតុេ ណ ABCD 3 េរែដល នរCDស
1 ់ ជុង AB = 4 cm
េគ ន , កSៃផRៃន ចតុេ ណ ABCD គឺ ៖ AABCD = 42 = 16 cm2
ដូចេនះ កSៃផRៃនចតុ
នចតុេ ណ ABCD គឺ 16 cm2 ត*វ នកំណត់ ។
10/. ក.គណ.
ក.គណ. ∠AOB
េ"យ TA និង TB 3ប.8ត
1 ប
់ ះ៉ រងង់ តង់ A និង B
េគ ន ∠ATB + ∠AOB = 180o
ែត ∠ATB = 60o ⇒ ∠AOB = 180o − 60o = 120o
ដូចេនះ ∠AOB = 120o ត*វ នកំណត់ ។
ខ. គណ.ៃផR
គណ.ៃផR កSៃនចេ មៀកTសតូច AOB
x oπ r 2
=មរូបមនU ៃផR កSចេ មៀកTស ៖ A =
360o
េ"យ x = 120o , π =
22
, r = OB = 5 cm
7
22
120o ⋅ ⋅ 52
.ំឲ21 េគ ន ៖ A= 7 550 2
= cm
360 21
ដូចេនះ ៃផR កSៃនចេ មៀកTសតូច AOB គឺ cm ត*វ នកំណត់ ។
550 2
21
11/. គណ. AB

=មបំ>ប់
>ប់, េយើង ន ៖ OA = OB = 10 cm ( ៃំ នរងង់ធ)

និង OT = 8 cm ( ៃំ នរងង់តច
ូ )
េហើយ AB ប៉ះរងង់តច
ូ តង់ T .ំឲ21 េគ ន AT ⊥ OT នន័យT
តីេ ណ OAT 3 តីេ ណែកង តង់កព
ំ ល
ូ T
=ម ទឹសប
Nី ទពី=គ័រ , េគ ន៖
OA2 = OT 2 + AT 2
102 = 82 + AT 2
AT 2 = 36 ⇒ AT = 6 cm

កំែណេ យ ៃហ ហុន
ិ , ៃហ ចរ និងយ៉ត ពនក 181
េមេរ
េមេរ នទី១៣
១៣ ៖ រងង់ និងប ត់

.ំឲ21 េយើង ន ៖ AB = AT + BT = 2 AT = 12 cm
ដូចេនះ AB = 12 cm ត*វ នគណ. ។
12/. បCXញ
1 T MA = MC
=មលកIណៈប.8ត
1 ប
់ ះ៉ រងង់ េគ ន៖
កងរងង់ផត
ិ O ន MA និង MB 3ប.8ត
1 ប
់ ះ៉ ,.ំឲ21 MA = MB (1)
កងរងង់ផត
ិ O' ន MB និង MC 3ប.8ត
1 ប
់ ះ៉ ,.ំឲ21 MB = MC ( 2)
=ម (1) និង ( 2 ) .ំឲ21 េគ ន ៖ MA = MC
ដូចេនះ MA = MC ត*វ នបCXញ
1 Z@ង
1 ខី ។
13/. ក. គណ. ∠EGH
េ"យ EG ប៉ះរងង់ផត
ិ H តង់ G .ំឲ21 េគ ន៖
HG ⊥ EG
⇒ ∠EGH = 90o
ដូចេនះ ∠EGH = 90o ត*វ នគណ.Z@ង
1 ខី ។
ខ. គណ. ∠HGF
េយើង ន EF = EG និង HF = HG
.ំឲ21 EH 3េមដ1?ទ័រៃន FG
.ំឲ21 ∠HJF = ∠HJG = 90o
.ំឲ21 ∠HGF = ∠HEG = 22o (មុំ ន ជុង ត*វKLែ1 កងេរៀងKL)
1
ដូចេនះ ∠HGF = 22o ត*វ នគណ. ។
គ. គណ. ∠FGE
េគ ន៖
∠FGE + ∠HGF = 90o
⇒ ∠FGE = 90o − 22o = 68o
ដូចេនះ ∠FGE = 68o ត*វ នគណ.Z@ង
1 ខី ។

សូមអភ័យេ!សចំេ"ះ$រយឺត វក'ង
( $រេចញផ+យេសៀវេ-កំែណគណិត
វទ0 េនះ ។ $លពីែខ តុ5 វច6$
ិ រ ធ'8 9:ំ ២០១១ ខ>(ំ នេរៀបេរៀងេសៀវេ-េនះ
ន ៦០% េBេហើយ ែតេពលេDះEFប់ែតកុព
ំ H8ទរ័ ខ>ខ
(ំ ច
ូ េហើយខ>ក
(ំ ម
៏ ន
ិ ន
យកឯកFរ!ំងKយមកទុកក'ង
( អងLចងMំ ។ េពលែដលOងេគជួសជុល
េហើយេB ឯកFរេនះក៏ ត់បង់េB ។ ខ>F
(ំ យ
R Sស់ FRយកTUង
ំ ៣ែខ ។
ខ>ន
(ំ ក
ឹ FRយមួយរយះធំ ។ េទើបែតដល់េដើមែខេមFេនះ េទើបសេEមចចិតេX ធYZ

Fរេឡើងវញ េហើយ នសេEមចOF\ពរ េ]"ក់កS^លែខ ឧស` ៕

កំែណេ យ ៃហ ហុន
ិ , ៃហ ចរ និងយ៉ត ពនក 182
េមេរ នទី១៤
១៤ ៖ លកណៈមុៃំ នរងង់

ម នទ ក ម ន ងង

1/. េគឲ រងង់ែដល នផិត O េហើយអងត់ធ AC = AB និង ∠ABC = 65o ។


ចូររករ ស់
! មុំ ∠ACB និង ∠ABO (រូប(ងេ)*
(ងេ)*ម
េ)*ម) ។

(ប ប ទ ) (ប ប ទ )
2/. ,មរូប(ងេលើ
(ងេលើេនះ
េនះ ចូររករ ស់
! មុំ ∠ACD , ∠BAE និង ∠BDC ។
3/. ក.ងរងង់ែដល នផិត O េគឲ អងត់ធ AB )សបនឹង CD ។ ចូរគណ1រ ស់
! មុំ
∠BCD , ∠ADC និង ∠ABC ។

(ប ប ទ ) (ប ប ទ )
4/. ,មរូប(ងេលើ
(ងេលើេនះ
េនះ ចូររករ ស់
! មុំ x និង y ។
5/. េគឲ រងង់ែដល នផិត O ។ អងត់ធ AC និង BD *ត់2)តង់
3 I និង

∠CAB = 40o , ∠AIB = 60o ។ ចូរគណ1រ ស់


! មុំ ៖
ក. ∠ACD ខ. ∠CAD ។

កំែណេ យ ៃហ ហុន
ិ , ៃហ ចរ និងយ៉ត ពនក 183
េមេរ នទី១៤
១៤ ៖ លកណៈមុៃំ នរងង់

(ប ប ទ ) (ប ប ទ )
6/. បីចណ
ំ ច
8 A , B , C ឋិតេ:េលើរងង់ែដល នផិត O ដូចរូប(ង
(ងេលើេនះ
េនះ ។
េគឲ ∠AOC = 120o ។ ចូរគណ1រ ស់
! មុំ ∠ABC ។
7/. ក.ងរូប(ងេ)*ម
(ងេ)*មេនះេគ
េ)*មេនះេគ ន O ;ផិតៃនរងង់ ។ អងត់ធ AD )សបនឹង BC
និង ∠ACB = 45o , ∠ACD = 25o ។ ចូរគណ1រ ស់
! មុំ ៖
ក. ∠ADC ខ. ∠OAB ។

(ប ប ទ ) (ប ប ទ )
8/. ក.ងរូប(ងេលើ
(ងេលើេនះេគ
េនះេគ នចំណច
8 A , B , C និង D េ:េលើរងង់ផត
ិ O ។ េបើេគ
ប1=យ អងត់ធ AB និង CD *ត់2)តង់
3 P េហើយ ∠CBD = 35o ,

∠CAP = 70o និង ∠DBP = 75o ។ ចូរគណ1 ៖

ក. ∠ACD ខ. ∠APC គ. ∠DAB ។


9/. ក.ងរងង់ែដល នអងត់ផត
ិ O និង AC ;អងត់ផត
ិ ។ េគប1=យអងត់ធ BA
និង CD ជួប23)តង់ T េហើយ AT = AC និង ∠ATD = 20o ។ ចូរគណ1
រ ស់
! មុំ ៖
ក. ∠ADB ខ. ∠BOC គ. ∠BDC ។

កំែណេ យ ៃហ ហុន
ិ , ៃហ ចរ និងយ៉ត ពនក 184
េមេរ នទី១៤
១៤ ៖ លកណៈមុៃំ នរងង់

(ប ប ទ ) (ប ប ទ )
10/. ក.ងរងង់ផត
ិ O ែដល នអងត់ផត
ិ QS )បសពនឹងអងត់ធ PR )តង់ M
ចំណច
8 កABលៃន PR ។ េគឲ ∠POQ = 134o ។ ចូរគណ1រ ស់
! មុំ ៖
ក. ∠PRQ ខ. ∠RPQ គ. ∠RSQ ។
11/. េគ នរងង់ែដល នផិត O និង PQ ;ប1Cត់បះ៉ រងង់)តង់ R ។ RS ;
អងត់ផត
ិ និង ∠ROQ = 48o ។ ចូរគណ1រ ស់
! មុំ ៖
ក. ∠OQR ខ. ∠RST គ. ∠TRQ ។

(ប ប ទ ) (ប ប ទ )
12/. េគ នរងង់ផត
ិ O និង ABE ;)តីេ*ណ ។ េបើរងង់*ត់ BE )តង់ C និង
AE )តង់ D ។ ចូរគណ1រ ស់
! មុំ x , y និង z ។
13/. េគ ន ABCD ;ចតុេ*ណEរFកក.ងរងង់ែដល ន AB = BC និង
∠ACE = 25o ។ េគប1=យ CD ដល់ E និង ∠ADE = 80o ។

ចូរគណ1រ ស់
! មុំ ៖ ក. ∠ABC ខ. ∠BAD ។

កំែណេ យ ៃហ ហុន
ិ , ៃហ ចរ និងយ៉ត ពនក 185
េមេរ នទី១៤
១៤ ៖ លកណៈមុៃំ នរងង់

(ប ប ទ ) (ប ប ទ )
14/. េគ ន TKXM និង SNXL ;ប1Cត់បះ៉ រងង់មយ
ួ ដូចរូ
ចរូប(ងេលើ
(ងេលើេនះ
េនះ ។ JKL
សGត
ិ េ:េលើប1Cត់ែតមួយ និង ∠TJS = 56o េហើយ ∠NJK = 40o ។ ចូរគណ1
រ ស់
! មុំ ៖ ក. ∠MLN ខ. ∠LMN គ. ∠KLN ។

ច មយ
1/. រករ ស់
! មុំ ∠ACB និង ∠ABO
ក.ង)តីេ*ណ ABC ន AB = AC (សមHតក
ិ មH)
1ំឲ )តីេ*ណ ABC ;)តីេ*ណសមIតកំពល
ូ A
វKIក ៖ ∠ABC = ∠ACB ែត ∠ABC = 65o (សមHតក
ិ មH)
េ1ះ1ំឲ េគIន ៖ ∠ACB = 65o
មLMងេទៀត , េគIន ៖ ∠BAC = 180o − 2 ⋅ 65o = 50o
50o
1ំឲ ∠BAD = ∠CAD = = 25o
2
ក.ង)តីេ*ណ ABO ន AO = BO (*ំៃនរងង់ែតមួយ)
1ំឲ )តីេ*ណ ABO ;)តីេ*ណសមIតកំពល
ូ O
វKIក ៖ ∠OAB = ∠OBA ⇒ ∠ABO = 25o
ដូចេនះ ∠ACB = 65o និង ∠ABO = 25o )តQវIនគណ1 ។
2/. រករ ស់
! មុំ ∠ACD , ∠BAE និង ∠BDC
,មរូប , េគIន ៖ ∠ABD = ∠ACD (RSត់ធរួម AD )
េTយ ∠ABD = 45o ⇒ ∠ACD = 45o
េយើង ន ៖ ∠BEC =
1
2
( )
BC + AD េហើយ AD = 2∠ABD

1ំឲ េគIន ៖ ∠BEC = BC + ∠ABD ែត ∠BEC = 130o


1
2

កំែណេ យ ៃហ ហុន
ិ , ៃហ ចរ និងយ៉ត ពនក 186
េមេរ នទី១៤
១៤ ៖ លកណៈមុៃំ នរងង់

1
⇒ 130o = BC + 45o
2
( )
BC = 2 130o − 45o = 170o

1ំឲ ∠BAE = ∠BAC = BC = 85o


1
2
េហើយេគIន ៖ ∠BDC = ∠BAC = 85o (មុR
ំ ត់
S ធរួម BC )
ដូចេនះ ∠BAE = ∠BDC = 85o និង ∠ACD = 45o )តQវIនកំណត់ ។
3/. គណ1រ ស់
! មុំ ∠BCD , ∠ADC និង ∠ABC
េយើង ន ៖ AB / /CD 1ំឲ ∠BCD = ∠DAB (មុU
ំ ស់
= កង
. )
ែត ∠DAB = 38o (សមHតក
ិ មH)
េ1ះេយើងIន ៖ ∠BCD = 38o
េហើយក.ង)តីេ*ណ OCD ន OC = OD (*ំៃនរងង់ែតមួយ)
1ំឲ )តីេ*ណ OCD ;)តីេ*ណសមIតកំពល
ូ O
វKIក
Iក ៖ ∠OCD = ∠ODC ⇒ ∠ADC = ∠ODC = 38o
េហើយ ∠ABC = ∠BCD = 38o (មុU
ំ ស់
= កង
. )
ដូចេនះ ∠BCD = ∠ADC = ∠ABC = 38o )តQវIនកំណត់ ។
4/. រករ ស់
! មុំ x និង y
,មរូប , េគIន x = y (មុRS
ំRត់
S ធែតមួយ)
េហើយ x + 34o = 60o (មុេំ )V និងមុក
ំ ង
. ៃន)តីេ*ណ)
⇒ x = 600 − 34o = 26o
ដូចេនះ x = y = 26o )តQវIនកំណត់Wង
M យ)សួល ។
5/. ក. គណ1រ ស់
! មុំ ∠ACD
េយើង ន ៖ ∠ACD = ∠ABD (មុR
ំ ត់
S ធរួម AD )
ែត ∠ABD = 180o − 40o − 60o = 80o
ដូចេនះ ∠ABD = 80o )តQវIនគណ1 ។
ខ. គណ1រ ស់
! មុំ ∠CAD
េយើង ន ៖ AB = BC ⇒ AB = BC
េ1ះ ∠ADB = ∠BAC = 40o េហើយ ∠AID = 180o − ∠AIB = 120o
ក.ង)តីេ*ណ ADI េគIន ៖ ∠DAI = 180o − 40o − 120o = 20o
1ំឲ េយើងIន ∠CAD = ∠DAI = 20o
ដូចេនះ ∠CAD = 20o )តQវIនគណ1 ។

កំែណេ យ ៃហ ហុន
ិ , ៃហ ចរ និងយ៉ត ពនក 187
េមេរ នទី១៤
១៤ ៖ លកណៈមុៃំ នរងង់

6/. គណ1រ ស់
! មុំ ∠ABC
,មបំXប់ , ∠AOC = 120o ⇒ ABC = 120o
េ1ះ AC = 360o − 120o = 240o 1ំឲ ∠ABC = AC = 120o
1
2
ដូចេនះ ∠ABC = 120o )តQវIនគណ1 ។
7/. ក. គណ1រ ស់
! មុំ ∠ADC
េយើង ន AD / / BC ⇒ ∠DAC = ∠ACB = 45o (មុU
ំ ស់
= កង
. )
ក.ង)តីេ*ណ ADC េគIន ៖ ∠ADC = 180o − 45o − 25o = 110o
ដូចេនះ ∠ADC = 110o )តQវIនគណ1 ។
ខ. គណ1រ ស់
! មុំ ∠OAB
េយើង ន ∠AOB = AB ែត ∠ACB = AB
1
2
1ំឲ េយើងIន ៖ ∠AOB = AB = 2∠ACB = 90o
ក.ង)តីេ*ណ AOB ន OA = OB (*ំៃនរងង់ែតមួយ)
1ំឲ េគIន )តីេ*ណ AOB ;)តីេ*ណសមIតកំពល
ូ O
90o
វKIក ៖ ∠OAB = ∠OBA 1ំឲ េគIន ៖ ∠OAB = ∠OBA = = 45o
2
ដូចេនះ ∠OAB = 45o )តQវIនគណ1 ។
8/. ក. គណ1 ∠ACD
េយើង ន ∠ABD + ∠DBP = 180o ⇒ ∠ABD = 180o − 75o = 105o
េTយ ABDC ;ចរុេ*ណEរFកក.ងរងង់ , េគIន ៖
∠ABD + ∠DCA = 180o
⇒ ∠ACD = 180o − 105o = 75o
ដូចេនះ ∠ACD = 75o )តQវIនគណ1 ។
ខ. គណ1 ∠APC
ក.ង)តីេ*ណ APC េគIន , ផលបូកមុក
ំ ង
. )តីេ*ណ ៖
∠APC + ∠PCA + ∠CAP = 180o
⇒ ∠APC = 180o − 75o − 70o = 35o
ដូចេនះ
ចេនះ ∠APC = 35o )តQវIនកំណត់ ។
គ. គណ1 ∠DAB
េយើង ន ៖ ∠DAB = BD =
1
2
1
2
(BC − CD )
ែត BC = 2∠CAB = 2 ⋅ 70o = 140o និង CD = 2∠CBD = 2 ⋅ 35o = 70o

កំែណេ យ ៃហ ហុន
ិ , ៃហ ចរ និងយ៉ត ពនក 188
េមេរ នទី១៤
១៤ ៖ លកណៈមុៃំ នរងង់

េគIន ៖ ∠DAB =
1
2
( )
140o − 70o = 35o

ដូចេនះ ∠DAB = 35o )តQវIនគណ1 ។


9/. ក. គណ1រ ស់
! មុំ ∠ADB
េយើង ន ៖ ∠ADB = ∠ACB (មុR
ំ ត់
S ធរួម AB )
)តីេ*ណ ADC ;)តីេ*ណEរFកក.ងកនYះរងង់ 1ំឲ ∠ADC = 90o
⇒ ∠DAT = 90o − 20o = 70o
⇒ ∠DAB = 180o − 70o = 110o
ចតុេ*ណ ABCD EរFកក.ងរងង់ , េគIន ៖
∠DAB + ∠DCB = 180o
∠DCB = 180o − 110o = 70o
⇒ ∠DCA + ∠ACB = 70o
ែតេTយ AC = AT ⇒ ∠ACT = ∠ATC = 20o
1ំឲ េគIន ៖ ∠ACB = 70o − 20o = 50o
ដូចេនះ ∠ACB = 50o )តQវIនគណ1 ។
ខ. គណ1រ ស់
! មុំ ∠BOC
ក.ង)តីេ*ណ BOC ន OB = OC (*ំៃនរងង់ែតមួយ)
េ1ះ BOC ;)តីេ*ណសមIតកំពល
ូ O
វKIក ៖ ∠OBC = ∠OCB = ∠ACB = 50o
1ំឲ ∠BOC = 180o − 2 ⋅ 50o = 80o
ដូចេនះ ∠BOC = 80o )តQវIនគណ1 ។
គ. គណ1រ ស់
! មុំ ∠BDC
េគIន ៖ ∠BDC = BC ែត BC = ∠BOC
1
2
1ំឲ េយើងIន ៖ ∠BDC = ⋅ 80o = 40o
1
2
ឬ េយើង[ចរក,មរេបៀបមួយេទៀត គឺ ៖
∠ADC = ∠ADB + ∠BDC
90o = 50o + ∠BDC
⇒ ∠BDC = 90o − 50o = 40o
ដូចេនះ ∠BDC = 40o )តQវIនគណ1 ។

កំែណេ យ ៃហ ហុន
ិ , ៃហ ចរ និងយ៉ត ពនក 189
េមេរ នទី១៤
១៤ ៖ លកណៈមុៃំ នរងង់

10/. ក. គណ1រ ស់
! មុំ ∠PRQ
េយើង ន ៖ ∠QRP = QP ែត QP = ∠QOP = 134o
1
2
េគIន ៖ ∠QRP = QP = ⋅134o = 67o
1 1
2 2
ដូចេនះ ∠QRP = 67o )តQវIនគណ1 ។
ខ. គណ1រ ស់
! មុំ ∠RPQ
ក.ង)តីេ*ណ QRP ន QM ⊥ RP និង RM = MP
1ំឲ េគIន QM ;េមដLទ័រៃន RP
1ំឲ េយើងIន QRP ;)តីេ*ណសមIតកំពល
ូ Q
វKIក ៖ ∠RPQ = ∠PRQ = 67o
ដូចេនះ ∠RPQ = 67o )តQវIនគណ1 ។
គ. គណ1រ
គណ1រ ស់
! មុំ ∠RSQ
េយើង ន ៖ ∠RSQ = ∠RPQ (មុR
ំ ត់
S ធរួម QR )
ដូចេនះ ∠RSQ = 67o )តQវIនគណ1 ។
11/. ក. គណ1រ ស់
! មុំ ∠OQR
េTយ RQ ប៉ះរងង់)តង់ R
េគIន OQR ;)តីេ*ណែកងែដល ន ∠ROQ = 48o
1ំឲ េគIន ៖ ∠OQR = 90o − 48o = 42o
ដូចេនះ ∠OQR = 42o )តQវIនគណ1 ។
ខ. គណ1រ ស់
! មុំ ∠RST
,មលក^ណៈមុំ និងរងង់ , េគIន ៖
RT ែត RT = ∠ROT
1
∠RST =
2
1ំឲ េយើងIន ៖ ∠RST = ⋅ 48o = 24o
1
2
ដូចេនះ ∠RST = 24o )តQវIនគណ1 ។
គ. គណ1រ ស់
! មុំ ∠TRQ
,មលក^ណៈមុំ និងរងង់ , េគIន ៖ ∠TRQ = RT = 24o
1
2
ដូចេនះ ∠TRQ = 24o )តQវIនគណ1 ។

កំែណេ យ ៃហ ហុន
ិ , ៃហ ចរ និងយ៉ត ពនក 190
េមេរ នទី១៤
១៤ ៖ លកណៈមុៃំ នរងង់

12/. គណ1រ ស់
! មុំ x , y និង z
ក.ង)តីេ*ណ ABE ន ∠A = 110o , ∠B = 40o
1ំឲ េយើងIន ៖ z = ∠E = 180o − 110o − 40o = 30o
ចតុេ*ណ ABCD ;ចតុេ*ណEរFកក.ងរងង់ , េគIន ៖
∠A + ∠DCB = 180o
∠DCB = 180o − 110o = 70o
ែត ∠DCB + y = 180o (មុX
ំ ប)
1ំឲ y = 180o − 70o = 110o
ក.ង)តីេ*ណ CDE ន z = 30o , y = 110o
1ំឲ េយើងIន ៖ x = 180o − (110o + 30o ) = 40o
ដូចេនះ x = 40o , y = 110o , z = 30o )តQវIនគណ1 ។
13/. ក. គណ1រ ស់
! មុំ ∠ABC
េយើង ន ABCD ;ចតុេ*ណEរFកក.ងរងង់ , េគIន ៖
∠ABC + ∠ADC = 180o
ែត ∠ADC = 180o − ∠ADE = 100o
1ំឲ ∠ABC = 180o − 100o = 80o
ដូចេនះ ∠ABC = 80o )តQវIនគណ1
Iនគណ1 ។
ខ. គណ1រ ស់
! មុំ ∠BAD
េTយ BA = BC 1ំឲ BAC ;)តីេ*ណសមIតកំពល
ូ B
180o − ∠ABC
1ំឲ េគIន ៖ ∠BAC = ∠BCA = = 50o
2
េហើយ ∠CAD = ∠ADE − ∠ACD = 80o − 25o = 55o
1ំឲ េយើងIន ៖ ∠BAD = ∠BAC + ∠CAD = 50o + 55o = 105o
ដូចេនះ ∠BAD = 105o )តQវIនគណ1 ។
14/. ក. គណ1រ ស់
! មុំ ∠MLN
េTយ MT ប៉ះរងង់)តង់ K េគIន ៖
∠MKN = ∠KJN = 40o (មុR
ំ ត់
S ធរួម KN )
ែត ∠MLN = ∠MKN (មុR
ំ ត់
S ធរួម MN )
ដូេចះ ∠MLN = 40o )តQវIនគណ1 ។

កំែណេ យ ៃហ ហុន
ិ , ៃហ ចរ និងយ៉ត ពនក 191
េមេរ នទី១៤
១៤ ៖ លកណៈមុៃំ នរងង់

ខ. គណ1រ ស់
! មុំ ∠LMN
េយើង ន ៖ ∠LKM = ∠TKJ = 56o (មុទ
ំ ល់កព
ំ ល
ូ )
េគIន ៖ ∠MNL = ∠MKL = 56o (មុR
ំ ត់
S ធរួម ML )
ក.ង)តីេ*ណ LMN េគIន ៖
∠LMN = 180o − ∠MLN − ∠MNL
= 180o − 40o − 56o = 84o
ដូេចះ ∠LMN = 84o )តQវIនគណ1 ។
គ. គណ1រ ស់
! មុំ ∠KLN
េយើង ន ៖ ∠LNK = ∠KJN = 40o (មុR
ំ ត់
S ធរួម KN )
េហើយ ∠LKN = ∠LKM + ∠MKN = 56o + 40o = 96o
ក.ង)តីេ*ណ KLM េគIន ៖ ∠KLN = 180o − 96o − 40o = 44o
ដូចេនះ ∠KLN = 44o )តQវIនគណ1 ។

អកវ
អកវ ម័ទ េ#ចើនែតវបត&
ែតវបត&ិ េ យក&
យក&េមើ
ីេមើល)យ
អកេចះ
អកេចះផង
េចះផង,-
ផង,- .ជនស12
ជនស12យ េ#3ះក&
េ#3ះក&1ក់
ី1ក់)យ
េគស6
េគស6បជុ
់ជខ8
ំខ
ុ ន
8 ។

ដក សង់េចញពី វច នុ កមែខរសេមចជួ
កមែខរសេមចជួន ត ។

កំ ន

១/. អកេន=ទ ៥ ?ក់ ស@ច


A #តី ៥ ?ទី ន#តី ៥ ។
សួរ. ៖ េបើអកេន=ទ ៥០ ?ក់ #តEវស@ច
A #តីឲG ន ៥០ េតើពក
ួ េគ
#តEវេ#បើរយះេពលប៉?
ុ ន
H ?ទី ?
២/. ១២ ែចកនឹង កនះ េតើ នលទLផលប៉?
ុ ន
H ?

៣/. េតើេគ#តEវេធO#ើ ប1ណវPធែី បបQ េបើ ២ បូក ២ ែចក


២ េសSន
ើ ង
ឹ ៣?

កំែណេ យ ៃហ ហុន
ិ , ៃហ ចរ និងយ៉ត ពនក 192
េមេរ នទី១៥ ៖ ទឹសប
ី ទ ែលស

ម នទ ទ បទ

1/. ចូរែចកអងត់ AB ែដល AB = 6 cm 5 ចំែណកេសើ ។


2/. ចូរែចកអងត់ AB ែដល AB = 11cm អងត់ស តនឹង 3 4 5 ។

3/. ចូរែចកអងត់ AB ែដល AB = 12 cm អងត់ស តនឹង 3 5 7 ។

4/. េគឲ AB = 7 cm ចូរកំណត់ចណ


ំ ច C ែដលែចកអងត់ AB !មផលេធៀប ។
3
5
5/. េគឲ AB = 9 mm ចូរកំណត់ចណ
ំ ច C និង D ែដលែចកអងត់ AB !មផល
េធៀប ។
5
2
6/. េគ ន AB / /CD / / EF / / GH !មរូប(ង)*ំ ។

ក. ចូរគណ, DF និង FH េបើ AC = 6 cm ,


CE = 12 cm , EG = 4 cm និង BD = 9 cm ។

ខ. ចូរគណ, BD , DF និង FH េបើ AC = 6 cm ,


CE = 12 cm , EG = 4 cm និង HB = 33 cm ។

គ. ចូរគណ, DF និង CG េបើ AC = 6 cm , CE = 12 cm , DB = 8 cm និង


FH = 6 cm ។

7/. េគឲ AB / / MN ចូរគណ, បែវង x ក/ង


0 រូបនីមយ
ួ ៗ(ងេ 5ម ។

កំែណេ យ ៃហ ហុន
ិ , ៃហ ចរ និងយ៉ត ពនក 193
េមេរ នទី១៥ ៖ ទឹសប
ី ទ ែលស

8/. !មរូប(ងេ
(ងេ 5មេគ
5មេគ ន BE / /CD ។ ចូរគណ, បែវង ED និង CD ។ រ67ស់
គិត cm ។

(ប ប ក ) (ប ប ក )
9/. ចូរគណ, a និង b !មរូប(ងេលើ
(ងេលើេនះ
េនះ ។ រ67ស់គត
ិ cm ។

10/. េគចង់សង់ជេណ:រមួ
ើរមួយែដល ន 13 5ំ កម;ស់ 2.40 m និង បែវង 3.40 cm ។
ក. ចូរសង់ តីេ5ណ ABC ែកង តង់ B េ<ក/ង
0 ប=ងែ់ ដល ន ត>?ន
1
20
េ>យយក AB ជុងឈរ និង BC ជុងេដក ។
ខ. េតើេគ តBវែចក AB ប៉,
ុ នែផ/
D ក េដើមEីកណ
ំ ត់ទ!
ី ង
ំ កម;សៃ់ ន5ំនម
ី យ
ួ ៗ?
េតើេគ តBវែចក BC ប៉,
ុ នែផ/
D ក េដើមEីកំ
កណ ំ ត់ទ!
ី ង
ំ មុខឈរៃន5ំនម
ី យ
ួ ៗ?
គ. ធ ធ ចក AB នង BC ច !"#ន នន$យ& ប ង 1cm
1
20
'ក()ងប*ង+ ន
, ង ប ង 20 cm -ន . +ព!

កំែណេ យ ៃហ ហុន
ិ , ៃហ ចរ និងយ៉ត ពនក 194
េមេរ នទី១៥ ៖ ទឹសប
ី ទ ែលស

11/. េគឲ កន=ះប,Iត់ពរី OX និង OY ។ េលើ OX េគេK ចំណច I និង J ែដល


OI = 2 cm និង IJ = 3 cm ។ េលើ OY េKចំណច I ' និង J ' ែដល OI ' = 3 cm

និង I ' J ' = 4.5 cm ។ េតើប,Iត់ II ' និង JJ ' សប ឬេទ ?


12/. ក/ង
0 កីM>?នមួយ េ>យចង់ដង
ឹ N េតើរOរបេ6PលេQតកម;
លេQតកម;សេ>យេNល
់េ>យេNល
េ>យ
មួយឋិតេ<កម;សប
់ ,
៉ុ ន
D ែម៉ តពីដី សុ(OនេដើរេSឈរដូចរូប(ងេ
(ងេ 5មេនះ
5មេនះ
េ>យឲ សេ លកTលរបស់U តVតេលើ សេ លៃនរOរ ។ UដឹងN U ន
កម;ស់ 1.65 m ឋិតេ<ច យ
W 6.50 m ពីេជើងបេ6Pល េហើយប=ងៃ់ នរOរឋិតេ<
ច យ
W 10 m ពីេជើ
េជើងបេ6Pល ។
ក. ចូរបក )យ)YនZពេនះ េ>យរូបធរណី ត។
ខ. េតើរOរ នកម;ស់ បែហលប៉,
ុ នែម៉
D តពីដី ?

13/. េគឲ ចតុេ5ណ ABCD មួយ និង E ចំណច បសព[រUងអងត់ ទBង\ំង


ពីរ ។ !ម E គូសប,Iត់ សបនឹង BC ែដល5ត់!ម AB តង់ F ។ !ម
E គូសប,Iត់ សបនឹង CD ែដល5ត់ AD តង់ G ។

ក. សរេសរ ទឹសប
]ី ទ!ែលសក/ង
0 តីេ5ណ AGE និង ADC រួចក/ង
0 តីេ5ណ
AFE និង ABC ។

ខ. ចូរប6^ញN FG / / BD ។

កំែណេ យ ៃហ ហុន
ិ , ៃហ ចរ និងយ៉ត ពនក 195
េមេរ នទី១៥ ៖ ទឹសប
ី ទ ែលស

ច មយ
1/. ែចកអងត់ AB ែដល AB = 6 cm 5 ចំែណកេសើ

សង់ AB = 6 cm , !មចំ
!មចំណច A សង់កន=ះប,Iត់ Ax រួចេKចំណច M , N
P , Q និង R េ<េលើកន=ះប,Iត់េនះ ែដល AM = MN = NP = PQ = QR ។

គូសZaប់ RB េហើយ!មចំណច M , N , P និង Q គូសប,Iត់ MM ', NN '


PP ' និង QQ ' សបនឹង RB េហើយ5ត់ AB តង់ M ', N ', P ' និង Q ' េរៀង

ដូចេនះេគOនចំណច M ', N ', P ' និង Q ' ែចក AB ៥ចំែណកេសើៗគ្/ា ។


2/. ែចកអងត់ AB ែដល AB = 11cm អងត់ស តនឹង 3 4 5

សង់ AB = 11cm , េហើយ!ម A សង់កន=ះប,Iត់ Ax រួចេ<េលើ Ax េK


ចំណច C , D និង E ែដល AC = 3, CD = 4 និង DE = 5 ។ គូសZaប់ចណ
ំ ច
E េS B រួចគូសប,Iត់ែដល5ត់!ម C , D សបនឹង BE េហើយ5ត់នង

AB តង់ F , G ។

េគOន ៖
AF FG GB
= =
3 4 5
ដូចេនះ F និង G ចំណចែចកអងត់ AB អងត់ស តនឹង
3 4 5 ។

កំែណេ យ ៃហ ហុន
ិ , ៃហ ចរ និងយ៉ត ពនក 196
េមេរ នទី១៥ ៖ ទឹសប
ី ទ ែលស

3/. ែចកអងត់ AB ែដល AB = 12 cm អងត់ស តនឹង 3 5 7

សង់ AB = 12 cm , េហើយ!ម A សង់កន=ះប,Iត់ Ax រួចេ<េលើ Ax េK


ចំណច C , D និង E ែដល AC = 3, CD = 5 និង DE = 7 ។ គូសZaប់ចណ
ំ ច
E េS B រួចគូសប,Iត់ែដល5ត់!ម C , D សបនឹង BE េហើយ5ត់នង

AB តង់ F , G ។

េគOន ៖
AF FG GB
= =
3 5 7
ដូចេនះ F និង G ចំណចែចកអងត់ AB អងត់ស តនឹង
3 5 7 ។

4/. កំណត់ចណ
ំ ច C ែដលែចកអងត់ AB !មផលេធៀប
3
5

សង់ AB = 7 cm , េហើយ !ម A គូសកន=ះប,Iត់ Ax រួចេ<េលើ Ax េK


ចំណច E និង F ែដល AF = 3 និង FE = 5 ។ េគគូសZaប់ចណ
ំ ច E េS B
!មចំណច F េគគូសប,Iត់ឲ សបនឹង EB េហើយ5ត់ AB តង់ C ។
េគOន ៖
AC AF 3
= =
CB FE 5
ដូចេនះ C ែចក AB !មផលេធៀប តBវOនកំណត់ ។
3
5
5/. កំណត់ចណ
ំ ច C និង D ែដលែចកអងត់ AB !មផលេធៀប
5
2

កំែណេ យ ៃហ ហុន
ិ , ៃហ ចរ និងយ៉ត ពនក 197
េមេរ នទី១៥ ៖ ទឹសប
ី ទ ែលស

សង់ AB = 9 cm , េហើយ !ម A គូសកន=ះប,Iត់ Ax រួចេ<េលើ Ax េK


ចំណច H , E និង G ែដល AH = 3, AE = 5 និង EG = 2 ។
េគគូសZaប់ចណ
ំ ច G េS B េហើយ!មចំណច E េគគូសប,Iត់ឲ សប
នឹង GB េហើយ5ត់ AB តង់ C ។
េគOន ៖ = ( នន័យN C សង់េ<ចេ,hះចំណច A និង B )
CA AE 5
=
CB EG 2
េគគូសZaប់ចណ
ំ ច H េS B េហើយ!មចំណច E េគគូសប,Iត់ឲ សប
នឹង HB េហើយ5ត់ AB តង់ D ។
េគOន ៖ = ( នន័យN D សង់េ<េលើប,hយៃន AB (ង B )
DA EA 5
=
DB EH 2
ដូចេនះ C និង D ែដលែចកអងត់ AB !មផលេធៀប តBវOនកំណត់ ។
5
2
6/. ក. គណ, DF និង FH
េយើង ន AB / /CD / / EF / / GH និង AC = 6 cm , CE = 12 cm , EG = 4 cm និង
BD = 9 cm , េគOន ទឹសប
]ី ទ!ែលស ៖
AC CE EG 6 12 4
= = ⇔ = =
BD DF FH 9 DF FH
12 ⋅ 9
⇒ DF = = 18 cm
6
4⋅9
FH = = 6 cm
6
ដូចេនះ DF = 18 cm និង FH = 6 cm តBវOនកំណត់ ។
ខ. គណ, BD , DF និង FH
េយើង ន AB / /CD / / EF / / GH និង AC = 6 cm , CE = 12 cm , EG = 4 cm និង
HB = 33 cm េ,ះ AG = 6 + 12 + 4 = 22 cm , េគOន ទឹសប
]ី ទ!ែលស ៖
AC BD AC ⋅ BH 6 ⋅ 33
= ⇒ BD = = = 9 cm
AG BH AG 22
AC BD BD ⋅ CE 9 ⋅12
= ⇒ DF = = = 18 cm
CE DF AC 6
EG FH EG ⋅ BH 4 ⋅ 33
= ⇒ FH = = = 6 cm
AG BH AG 22
ដូចេនះ BD = 9 cm , DF = 18 cm និង FH = 6 cm តBវOនកំណត់ ។
គ. គណ, DF និង CG
េយើង ន AB / /CD / / EF / / GH និង AC = 6 cm , CE = 12 cm , DB = 8 cm និង
FH = 6 cm , !ម ទឹសប
]ី ទ!ែលស េគOន ៖

កំែណេ យ ៃហ ហុន
ិ , ៃហ ចរ និងយ៉ត ពនក 198
េមេរ នទី១៥ ៖ ទឹសប
ី ទ ែលស

AC BD CE ⋅ BD 12 ⋅ 8
= ⇒ DF = = = 16 cm
CE DF AC 6
AC BD AC ⋅ DH 6 ⋅ (16 + 6 )
= ⇒ CG = = = 16.5 cm
CG DH BD 8
ដូចេនះ DF = 16 cm និង CG = 16.5 cm តBវOនកំ
វOនកំណត់ ។
7/. គណ, បែវង x ក/ង
0 រូបនីមយ
ួ ៗ(ងេ 5ម
បទ
េយើង ន MN / / BC , !ម ទឹសប
]ី ទ!ែលស េគOន ៖
AM AN 9 8
= ⇔ =
MB NC 16 x
16 ⋅ 8 128
⇒x= =
9 9
ដូចេនះ x = ឯក! បែវង ។
128
9
បទ
េយើង ន MN / / BC , !ម ទឹសប
]ី ទ!ែលស េគOន ៖
AM AN 8 x
= ⇔ =
MB NC 3 4
4 ⋅ 8 32
⇒x= =
3 3
ដូចេនះ x = ឯក! បែវង ។
32
3
បទ
េយើង ន MN / / AB , !ម ទឹសប
]ី ទ!ែលស េគOន ៖
CM CN 6 12 − 4
= ⇔ =
CA CB x 12
6 ⋅12
⇒x= =9
8
ដូចេនះ x = 9 ឯក! បែវង ។
8/. គណ, បែវង ED និង CD (រ67ស់គត
ិ cm )

េយើង ន BE / /CD , !ម ទឹសប


]ី ទ!ែលស េគOន ៖
AB AE BE 8 4 12
= = ⇔ = =
AC AD CD 8 + 2 4 + DE CD
4 ⋅10
⇒ DE = − 4 = 1 cm
8
12 ⋅10
CD = = 15 cm
8
ដូចេនះ DE = 1cm និង CD = 15 cm តBវOនកំណត់ ។

កំែណេ យ ៃហ ហុន
ិ , ៃហ ចរ និងយ៉ត ពនក 199
េមេរ នទី១៥ ៖ ទឹសប
ី ទ ែលស

9/. គណ, a និង b (រ67ស់គត


ិ cm )

!ម ទឹសប
]ី ទ!ែលស , េយើងOន ៖
a+2 6+4 8
= =
a 6 b
2 5 8
1+ = =
a 3 b
2
⇒a= = 3 cm
5
−1
3
8 24
b= = cm
5 5
3
ដូចេនះ a = 3 cm និង b = cm តBវOនកំណត់ ។
24
5
10/. ក. សង់ តីេ5ណ ABC ែកង តង់ B េ<ក/ង
0 ប=ងែ់ ដល
ដល ន ត>?ន
1
20
េ>យយក AB ជុងឈរ និង BC ជុងេដក
ត>?ន នន័យN បែវង ១ឯក!េ<ក/ង
0 ប=ងេ់ សើនង
ឹ បែវង ២០
1
20
ឯក! ៃនរ67ស់ពត
ិ ។
េ>យ រ67ស់ពិ
ពត ិ AB = 2.40 m និង BC = 3.40 m េគOន បែវងេ<េលើ
ប=ងគ
់ ឺ AB = = 0.12 m = 12 cm និង BC =
2.40 3.40
= 0.17 m = 17 cm
20 20
ដូចេនះ តីេ5ណ ABC ែដល តBវសង់គឺ តីេ5ណែដល នរ67ស់
AB = 12 cm និង BC = 17 cm ។

ខ. េតើេគ តBវែចក AB ប៉,


ុ នែផ/
D កេដើមEីកំ
កណ ំ ត់ទ!
ី ង
ំ កម;សៃ់ ន5ំនម
ី យ
ួ ៗ?
ៗ?
ជេណ:រើ ន ១៣5ំ
១៣5ំ , េ,ះេគ តBវែចក AB ១៣ ែផ/កប៉ន
ុ ៗ ។
ដូចេនះេដើមEីកណ
ំ ត់ទ!
ី ង
ំ កម;សៃ់ ន5ំនម
ី យ
ួ ៗ េគ តBវែចក
វែចក AB ១៣
ែផ/កប៉ន
ុ ៗ ។
េតើេគ តBវែចក BC ប៉,
ុ នែផ/
D កេដើមEីកណ
ំ ត់ទ!
ី ង
ំ មុខឈរៃន5ំនម
ី យ
ួ ៗ?
ៗ?
01ច2( 0 , "យជេណ:
"យជេណ:រើ ន ១៣5ំ
១៣5ំ , េ,ះេគ តBវែចក BC ១៣ ែផ/ក
ប៉ន
ុ ៗ ។
ដូចេនះេដើមEី
មEីកណ
ំ ត់ទ!
ី ង
ំ មុខឈរៃន5ំនម
ី យ
ួ ៗ េគ តBវែចក BC ១៣
ែផ/កប៉ន
ុ ៗ ។

កំែណេ យ ៃហ ហុន
ិ , ៃហ ចរ និងយ៉ត ពនក 200
េមេរ នទី១៥ ៖ ទឹសប
ី ទ ែលស

គ. ធ ចក AB នង BC

វpធែី ចក AB ៖
!ម B េយើងគូសកន=ះប,Iត់ Bx េហើយេ<េលើកន=ះប,Iត់េនះេគ កិត
១៣ អងត់បន
៉ុ ៗ េហើយេKចំណច M ចំណចចុង ។ េគZaប់ MA
េហើយ !មចំណចនីមយ
ួ ៗ\ំង ១២េទៀតែដលេ<េលើ
១២េទៀតែដលេ<េលើ Bx េយើងគូសប,I
សប,Iត់
សបនឹង MA , េ,ះេយើងOនប,Iត់ សប\ំងអស់េ,ះែចក AB ១៣
អងត់បន
៉ុ ៗ ។
វpធែី ចក BC ៖
!ម B េយើងគូសកន=ះប,Iត់ By េហើយេ<េលើកន=
កន=ះប,Iត់េនះេគ កិត
១៣ អងត់បន
៉ុ ៗ េហើយេKចំណច N ចំណចចុង ។ េគZaប់ NC
េហើយ !មចំណចនីមយ
ួ ៗ\ំង ១២េទៀតែដលេ<េលើ
១២េទៀតែដលេ<េលើ By េយើងគូសប,Iត់
សបនឹង NC , េ,ះេយើងOនប,I
ងOនប,Iត់ សប\ំងអស់េ,ះែចក BC ១៣
អងត់បន
៉ុ ៗ ។

កំែណេ យ ៃហ ហុន
ិ , ៃហ ចរ និងយ៉ត ពនក 201
េមេរ នទី១៥ ៖ ទឹសប
ី ទ ែលស

11/. េតើប,Iត់ II ' និង JJ ' សប ឬេទ ?

!មបំqប់ , OI = 2 cm , IJ = 3 cm , OI ' = 3 cm និង I ' J ' = 4.5 cm


េគOន ៖
OI 2
= (1)
IJ 3
និង
OI ' 3 2
= = ( 2)
I ' J ' 4.5 3
!ម (1) និង ( 2 ) េគOន ៖
OI OI '
=
IJ I ' J '
!ម ទឹសប
]ី ទ rសរបស់!ែលស , េគOន ៖ II '/ / JJ '
ដូចេនះ II ' និង JJ ' សប ។
12/. ក. បក )យ)YនZពេនះ េ>យរូបធរណី ត

សនត DE កម;សរ់ Oរ និង AC កម;សរ់ បស់សុ


សខ ុ ។
េគេឃើញN , សេ លចុងរOរ E និងកTលរបស់សខ
ុ តVតសុី តង់ B
េគOន DE / / AC ។
ខ. េតើរOរ នកម;ស់ បែហលប៉,
ុ នែម៉
D តពីដី ?
េ>យ DE / / AC េ,ះ!ម ទឹសប
]ី ទ!ែលស , េយើងOន ៖
AC AB AC ⋅ DB 1.65 ⋅10
= ⇒ DE = = = 4.71 m
DE DB AB 10 − 6.50
ដូចេនះរOរេ,ះ នកម;ស់ បែហល 4.71m ពីដី ។

កំែណេ យ ៃហ ហុន
ិ , ៃហ ចរ និងយ៉ត ពនក 202
េមេរ នទី១៥ ៖ ទឹសប
ី ទ ែលស

13/. ក. សរេសរ ទឹសប


]ី ទ!ែលសក/ង
0 តីេ5ណ AGE និង ADC រួចក/ង
0
តីេ5ណ AFE និង ABC

ក/ង
0 តីេ5ណ AGE និង ADC ន GE / /CD េគOន ៖
ទឹសប
]ី ទ!ែលស ៖
AG AE GE
= = (1)
AD AC DC
សេដៀង ែដរ, ក/ង
0 តីេ5ណ AFE និង ABC ន EF / / BC េគOន ៖
ទឹសប
]ី ទ!ែលស ៖
AF AE EF
= = ( 2)
AB AC BC
ខ. ប6^ញN FG / / BD
!ម (1) និង ( 2 ) (ងេលើ , េគ\ញOន ៖
AG AF
=
AD AB
!ម ទឹសប
]ី ទ rសរបស់!ែលស , េគOន FG / / BD
ដូចេនះ FG / / BD តBវOន )យបtaក់ ។

កំ ន
១. េគចង់ ប
ំ េ"#ល បួនេដើម ស*+ប់ចងអ*ងឹងឲ/ ន៦។
េតើេគ*ត2វេធ5ដ
ើ ច
ូ េម7ច ?
២. * ប
ំ ន
ួ ែចកនឹង ពីរ ន ៤ េតើេគេធ5ើ ងេម៉ច នលទ=ផល
េនះ ?
៣. គណAចំនន
ួ Bងេ*Cម ៖

( )
2+ 2÷2
ក. 2 2
2 + 2÷2
+2 −2÷2

( )
2+ 2÷2
ខ. 2 2 + 2 ÷ 2 −2÷2
2
2 2

( ) (( ) )
2+ 2 2
គ. 2
− 22
2
÷ 2 − 2 2⋅ 2 − 2 ÷ 2

កំែណេ យ ៃហ ហុន
ិ , ៃហ ចរ និងយ៉ត ពនក 203
េមេរ នទី១៦ ៖ តីេ ណដូច

ម នទ ច

1/. គូ តីេ ណ ងេ ម តីេ ណដូច ។ចូរសរេសរេ ះ និង ជុង តវ ។

2/. េតើគូ តីេ ណនីមយ


ួ ៗ ងេ មដូច ែដរ ឬេទ ? េហតុអ)ី ?

ប(ក) ប(ខ
ប(ខ)

ប(គ
ប(គ)

កំែណេ យ ៃហ ហុន
ិ , ៃហ ចរ និងយ៉ត ពនក 204
េមេរ នទី១៦ ៖ តីេ ណដូច

3/. ចូរេ បៀបេធៀប ∆ABC និង ∆A ' B ' C ' , ∆PMN និង ∆P ' M ' N ' រួចសរេសរ
ចសរេសរ
ផលេធៀបដំណច
1 (រូប ងេ ម) ។

ប (A)

ប (B)

4/. េតើគូ តីេ ណនីមយ


ួ ៗ ងេ មេនះ តីេ ណដូច ែដរ ឬេទ ? េហតុអ)ី ?

ប (A) ប (B)

ប (C)

5/. េគ4ន តីេ ណ ABC ដូច តីេ ណ GEF រូប ងេ ម ចូរសរេសរបំេពញ


ផលេធៀបដំណច
1 ។

កំែណេ យ ៃហ ហុន
ិ , ៃហ ចរ និងយ៉ត ពនក 205
េមេរ នទី១៦ ៖ តីេ ណដូច

AB BC ⋯ ? AB ⋯ ? BC
= = = =
GE ⋯ ? FG EG EF ⋯ ?
6/. ចូរ 7យប89
7យប89ក់< តីេ ណ ងេ ម តីេ ណដូច រួច=ញវ>?ក ។

ស ល់ ៖ េយើងខ#$ំ នប&រ' ()រ*+ស់មុំ ∠M និង ∠P ពីតៃមែដលឲ0 ។


េនះក៏េ45ះែត MN > PN េ6ះ74ត8វែត ∠P > ∠M ។

7/. េគឲAចតុេ ណBយ ABCD បCDយ ជុងេ ទត AD និង BC ត់ តង់ F


េEយ7Fល់ FD : FA = 8 : 5 និង FB = 2.25 m ។ ចូរគណC បែវង BC ។
8/. េគ4ន តីេ ណដូច ពីរ ែដល4នបរ>4 តេរៀង 35 dm និង 56 dm ។ ចូរ

រក បែវងកមGសម
់ យ
ួ ៃន តីេ ណធំេបើកមGសៃ់ ន តីេ ណតូចេសIន
ើ ង
ឹ 10 dm ។
9/. កKង
L តីេ ណ ABC 4ន ∠BAC = 90o េហើយ RS ⊥ BC ។ ចូរ 7យប89ក់<
∆ABC ∼ ∆SRC ។

10/. ចូរេMចំណច
N C និង D ែដលែចកអងOត់ MN ងកKង
L និង ងេ PQមផល
េធៀប ។
a
b
ក. MN = 11 dm , ខ. MN = 6 dm , ។
a 3 a 1
= =
b 5 b 3

កំែណេ យ ៃហ ហុន
ិ , ៃហ ចរ និងយ៉ត ពនក 206
េមេរ នទី១៦ ៖ តីេ ណដូច

11/. េគឲA តីេ ណ ABC TរUកកKង


L រង)ងែ់ ដល AB = α , AC = β , BC = γ ។ កនVះ
បCWត់ពះុ ∠BAC ត់ ជុង BC តង់ចណ
ំ ច
N I និងធKX BC តង់ J ។
ក. ចូរ 7យប89ក់< ∆ABI និង ∆AJC តីេ ណដូច ។
ខ. ចូរគណCផលគុណ AI ⋅ AJ ។
គ. ចូរបYZញ< ∠BAJ = ∠CBJ ។
ឃ. ចូរ 7យប89ក់< JB 2 = JA ⋅ JI ។
12/. េដើម\ី]ស់ច4^យពីចណ
ំ ច
N A េ_
ចំណច
N B ែដល4នឧបសគaមយ

មិនbចេ_ដល់ េគេ ជើសេរcសយក
ចំណN
ណច N C មួយ រួច]ស់ បែវង AC ។
េបើ D ∈ AC េCះ]ស់អងOត់ CD ,Qម
ចំណច
N D គូសបCWត់ DE / / AB រួច
]ស់អងOត់ DE ។េEយេធ)Q
ើ មរេបៀប
េនះេគbចគណCអងOត់ AB ?ន ។
េបើ CA = 1.8 km , CD = 90 m និង DE = 150 m ។ ចូរគណCច4^យ AB ។
13/. េគឲA តីេ ណ ABC TរUកកKង
L រង)ង់ ។ បCWត់មយ
ួ សបេ_នឹងបCWត់បះ៉ តង់
A ត់អងOត់ AB និង AC តង់ចណ
ំ ច
N M និង N ។
ក. ចូរ បដូច តីេ ណ ABC និង តីេ ណ AMN ។
ខ. ចូរ 7យប89ក់< AB ⋅ AM = AC ⋅ AN ។
14/. Qមចំណច
N C ៃនកនVះរង)ង់ េគគូសអងOត់ CD ែកងេ_អងOតផ
់ ត
eិ AB ។
ក. េបើ AD = 4 dm , DB = 9 dm ។ ចូរគណC CD ។
ខ. េបើ AB = 29 dm , CD = 10 dm ។ ចូរគណC AD និង DB ។
15/. 7យបំភឲ
Vឺ Aេឃើញ<កKង
L តីេ ណមួយ រYhស់កមGស់ Tសស44 តនឹង
រYhស់ ជុង គឺ aha = bhb = chc ។(ែណCំ ៖ គូសកមGសព
់ រី រួច បដូច តីេ ណ
ែកងែដល4នកមGសម
់ យ
ួ ជុងៃនមុែំ កង) ។
16/. Qមកំពល
ូ A ៃន តីេ ណសម?ត ABC េគគូសបCWត់មយ
ួ ែដល ត់ ជុង
BC តង់ D និង ត់រង)ងT
់ រUកេ P តីេ ណ តង់ចណ
ំ ច
N E ។ចូរេ បៀបេធៀប
តីេ ណ ADB និង តីេ ណ AEB រួច 7យប89ក់< AB 2 = AD ⋅ AE ។

កំែណេ យ ៃហ ហុន
ិ , ៃហ ចរ និងយ៉ត ពនក 207
េមេរ នទី១៦ ៖ តីេ ណដូច

17/. េដើមឫសkីមយ
ួ េដើម4នកមGស់ 10 m តវខAល់
តវខAល់]យ?ក់ តង់ចណ
ំ ច
N P មួយ
េហើយចុងឫសkីបះ៉ ដី តង់ចណ
ំ ច
N មួយែដល4នច4^យ 3 m ពីគល់ ។
ចូរគណCកមG
គណCកមGសេដើ
់េដើមឫសkីពគ
ី ល់េ_ចំណច
N ?ក់ P ។
18/. AB រYhស់ព?
ី តដីដល់
ដល់lឈូ
m ក េហើយែផKកផុតពីទក
ឹ 4ន បែវង AE = 3 dm
េពលេCះ4នTបមួយមកទុេំ ធ)ឲ
ើ Aឈូកេ ទត ប ន់យក7oនpព BA '
ែដល EA ' = 9 dm ។ ចូររកជេ qទឹក EB ។

19/. Qង a និង b រYhស់ ជុង=ំងពីរៃនមុែំ កងរបស់ តីេ ណែកងមួយ ។ h


រYhស់កមG
កមGសច
់ េំ Bះអុីបេ៉ូ តនុស ។ ចូរគណC h េ_QមតៃមV a និង b ។
20/. ABCD បេលឡX មដូចរូប ង7sំ ។ I ចំណច
N កtuលៃន AD ែដល
IB ⊥ AD ។

ក. ចូរគណC IB ។
ខ. ចូរគណCៃផv កwៃន បេល
ឡX ម។

21/. កKង
L រូប ង7sេំ នះ NMQ និង PNQ តីេ ណែកង ។
ក. ចូរគណCរYhស់ MQ ។
ខ. ចូរគណCរYhស់ NP ។

កំែណេ យ ៃហ ហុន
ិ , ៃហ ចរ និងយ៉ត ពនក 208
េមេរ នទី១៦ ៖ តីេ ណដូច

22/. កKង
L រូប ង7sេំ នះអងOតធ
់ KX CD និង AB ត់ តង់ I ែដល ៖

IC = 12 cm , IB = 5 cm , BD = 3 cm
ក. ចូរេ បៀបេធៀប តីេ ណ IAC និង IBD ។
ខ. ចូរគណCរYhស់ AC ។

ស ល់ ៖ ប នលំ ត់ទ២២
ី២២ េនះ នប រែដល ប នឲ រ ស់ ជុងមក
ដ៏េ ចើនែបបេនះគឺេគចង់ឲ េយើងរក*មផលេធៀប ជុង ។ ប៉ែុ ន0 តៃម2េលខែដល4ក់
មក5ែបរ6មិនសមេហតុសមផល6មួយលក:ណៈមុំ និងរង=ងេ់ >វ@ញ ។ ខBស
Cំ ម
ូ ែក
តៃម2ែដលឲ េ4យទុកែត IC = 12 cm , IB = 5 cm , BD = 3 cm ប៉េុ EFះ ។
រេធ=ែើ បបេនះគឺ Gន់ែតេធ=ឲ
ើ លំ ត់េនះ នដំេEះ Hយ តឹម តJវែតប៉េុ EFះ
23/. កKង
L រូប ង7sំ , ពីចណ
ំ ច
N P េគគូសបCWត់បះ៉ រង)ង់ តង់ A េហើយគូស
បCWត់មួ
មយ ួ េទៀត ត់រង)ង់ តង់ C
និង D ។
ក. ចូរេ បៀបេធៀប តីេ ណ PAC
និង PAD ។
ខ. ចូរ=ញឲAេឃើញ< AP 2 = PC ⋅ PD ។
24/. កKង
L រូប ងេ
ងេ មេនះ
មេនះ , េតើេ ណ=ំងពីរដូច ឬេទ ?

កំែណេ យ ៃហ ហុន
ិ , ៃហ ចរ និងយ៉ត ពនក 209
េមេរ នទី១៦ ៖ តីេ ណដូច

25/. រូប ងេ
ងេ ម េ ណពីរដូច ។
3 3

ក. បYZញ< េបើេ ណពីរដូច េCះ =   =   ែដល v និង V


v h r
V H  R
4ឌ ។
ខ. េបើេ ណតូច4ន4ឌ v = 6.28 cm3 េហើយ h = 2 cm , H = 4 cm ។
ចូរគណC4ឌ V ៃនេ ណធំ ។

ច មយ
1/. សរេសរេ ះ និង ជុង តវ
∆MNP ∼ ∆BAC 4ន ៖ ជុង MN តវនឹង ជុង BA

ជុង MP តវនឹង ជុង BC


ជុង PN តវនឹង ជុង CA ។
∆EFG ∼ ∆TRS 4ន ៖ ជុង EF តវនឹង ជុង TS
ជុង EG តវនឹង ជុង TR
ជុង GF តវនឹង ជុង SR ។
2/. េតើគូ តីេ ណនីមយ
ួ ៗ ងេ មដូច ែដរ ឬេទ ? េហតុអ)ី ?
ប(ក)

កKង
L តីេ ណ ABC 4ន ∠A = 90o , ∠B = 51o ⇒ ∠C = 90o − 51o = 39o
តីេ ណ ABC និង ADE 4នមុេំ សIើ ែតមួយប៉េុ tyះគឺ ∠BAC = ∠ADE = 90o ។
ដូចេនះគូ តីេ ណ ABC និង ADE មិនដូច េទ ។
ប(ខ)

កKង
L តីេ ណ FGH 4ន ∠F = 80o , ∠G = 70o ⇒ ∠H = 180o − 80o − 70o = 30o
តីេ ណ FGH និង TRS 4នមុេំ សIើ ែតមួយប៉េុ tyះគឺ ∠FGH = ∠SRT = 70o ។
ដូចេនះគូ តីេ ណ FGH និង TRS មិនដូច េទ ។

កំែណេ យ ៃហ ហុន
ិ , ៃហ ចរ និងយ៉ត ពនក 210
េមេរ នទី១៦ ៖ តីេ ណដូច

ប(គ)

កKង
L តីេ ណ PKL 4ន ∠P = 34o , ∠K = 82o ⇒ ∠L = 180o − 82o − 34o = 64o
តីេ ណ FGH និង TRS 4នមុព
ំ រី េសIើ គឺ ∠PLK = ∠KNM = 64o និង
∠PKL = ∠MKN = 82o (មុទ
ំ ល់កព
ំ ល
ូ )។
ដូចេនះគូ តីេ ណ FGH និង TRS គូ តីេ ណដូច េទ(ករណី
េទ(ករណីដណ
ំ ច
1 ទី១
ម.ម.) ។
3/. េ បៀបេធៀប ∆ABC និង ∆A ' B ' C ' , ∆PMN និង ∆P ' M ' N ' រួចសរេសរផលេធៀប
ដំណច
1
ប (A)

Qមរូប , កKង
L តីេ ណ ∆ABC និង ∆A ' B ' C ' 4ន ៖
∠A = ∠A ' និង ∠C = ∠C '

េCះ ∆ABC និង ∆A ' B ' C ' ដូច Qមករណីដណ


ំ ច
1 ទី១ (ម.ម.)
សរេសរផលេធៀបដំណច
1
∆ABC
AB BC AC
∼ ⇒ = =
A ' B ' B 'C ' A'C '
∆A ' B ' C '
ប (B)

Qមរូប , កKង
L តីេ ណ ∆MNP និង ∆M ' N ' P ' 4ន ៖
∠M = ∠M ' និង ∠P = ∠P '

េCះ ∆MNP និង ∆M ' N ' P ' ដូច Qមករណីដណ


ំ ច
1 ទី១ (ម.ម.)
សរេសរផលេធៀបដំណច
1
∆MNP
MN NP MP
∼ ⇒ = =
M 'N ' N 'P' M 'P'
∆M ' N ' P '

កំែណេ យ ៃហ ហុន
ិ , ៃហ ចរ និងយ៉ត ពនក 211
េមេរ នទី១៦ ៖ តីេ ណដូច

4/. េតើគូ តីេ ណនីមយ


ួ ៗ ងេ មេនះ តីេ ណដូច ែដរ ឬេទ ? េហតុអ)ី ?
ប (A)

កKង
L តីេ ណ=ំងពីរ 4នផលេធៀប ជុង
12 14 16
= =
6 7 8
ដូចេនះគូៃន តីេ ណ=ំ
ណ=ំងពីរេនះ តីេ ណដូច Qមករណីទ៣
ី (ជ.ជ.ជ)
ប (B)

កKង
L តីេ ណ=ំងពីរ 4ន ៖ មុម
ំ យ
ួ គូេសIើ ( មុទ
ំ ល់កព
ំ ល
ូ )
និង ផលេធៀប ជុង
16 10
=
32 20
ដូចេនះគូៃន តីេ ណ=ំងពីរេនះ តីេ ណដូច Qមករណីទ២
ី (ជ.ម.ជ)
ប (C)

កKង
L តីេ ណ=ំងពីរ 4ន ៖ មុម
ំ យ
ួ គូេសIើ ( មុទ
ំ ល់កព
ំ ល
ូ )
និង ផលេធៀប ជុង
27 30
=
36 40
ដូចេនះគូៃន តីេ ណ=ំងពីរេនះ តីេ ណដូច Qមករណីទ២
ី (ជ.ម.ជ)
5/. សរេសរបំេពញផលេធៀបដំ
ពញផលេធៀបដំណច
1

AB BC CA AB AC BC
= = = =
GE EF FG EG EF GF
6/. 7យប89
7យប89ក់< តីេ ណ ងេ ម តីេ ណដូច រួច=ញវ>?ក
កKង
L តីេ ណ ABC និង MNP 4នផលេធៀប ជុង ៖
AC 290
= = 2.2
NP 130.5
= 2.2 និង = 2.2 , េគ?ន
AB 348 BC 400 AC AB BC
= = = =
NM 156.6 MP 180 NP NM MP
ដូចេនះ ∆ABC ∼ ∆NMP Qមករណីដណ
ំ ច
1 ទី៣(ជ.ជ.ជ.)
៣(ជ.ជ.ជ.) ។
៣(
វ>?ក ៖ ∠ABC = ∠NMP = 45o , ∠BAC = ∠MNP = 77o ។
7/. គណC បែវង BC

កំែណេ យ ៃហ ហុន
ិ , ៃហ ចរ និងយ៉ត ពនក 212
េមេរ នទី១៦ ៖ តីេ ណដូច

េEយ AB / /CD ( ABCD ចតុេ ណBយ)


Qម ទឹសប
}ី ទQែលស , េគ?ន ៖
FD FC
=
FA FB
េEយ FD : FA = 8 : 5 និង FB = 2.25 m េគ?ន ៖
FC 8
= ⇒ FC = 3.6 m
2.25 5
េCះ BC = FC − FB = 3.6 − 2.25 = 1.35 m
ដូចេនះ BC = 1.35 m តវ?នគណC។(េយើងេធ)Q
ើ ម តីេ ណដូច ក៏?ន)
8/. រក បែវងកមGសម
់ យ
ួ ៃន តីេ ណធំេបើកមGសៃ់ ន តីេ ណតូចេសIន
ើ ង
ឹ 10 dm
េEយ តីេ ណ=ំងពីរ តីេ ណដូច េហើយ4នបរ>4 តេរៀង 35 dm

និង 56 dm , េគ?ន ៖
35 10 56 ⋅10
= ⇒h= = 16 dm
56 h 35
ដូចេនះកមGសម
់ យ
ួ ៃន តីេ ណធំគឺ 16 dm ។
9/. 7យប89ក់< ∆ABC ∼ ∆SRC
កKង
L តីេ ណ ABC និង SRC 4ន ៖
∠BAC = ∠RSC = 90o និង ∠C (មុរំ ម
ួ )។
ដូចេនះ ∆ABC ∼ ∆SRC Qមករណីដណ
ំ ច
1 ទី១ (ម.ម.) ។
10/. េMចំណច
N C និង D ែដលែចកអងOត់ MN ងកKង
L និង ងេ PQមផល
េធៀប
a
b
ក. MN = 11 dm ,
a 3
=
b 5

កំែណេ យ ៃហ ហុន
ិ , ៃហ ចរ និងយ៉ត ពនក 213
េមេរ នទី១៦ ៖ តីេ ណដូច

សង់ MN = 11 dm ។ Qមចំណច
N M និង N សង់បCWត់ d និង d ' េរៀង
ែដល d / / d ' ។
េ•េលើបCWត់ d េយើងេMចំណច
N F និង G ែដល MF = MG = 3 dm
េ•េលើបCWត់ d ' េយើងេMចំណច
N E ែដល NE = 5 dm
គូសបCWត់ EF និង EG ត់បCWត់ MN តង់ C និង D េរៀង ។
េEយ FG / / NE េគ?ន ៖ ( C េ•ចេCDះ MN )
a MC CF MF 3
= = = =
b NC CE NE 5
និង ( D េ•េ PចេCDះ MN )
a MD MG 3
= = =
b ND NE 5
ដូចេនះ C និង D ចំណច
N ែចកកKង
L និងេ PអងOត់ AB Qមផលេធៀប
3
5
តវ?នេMរួច ។
ខ. MN = 6 dm ,
a 1
=
b 3

សង់ MN = 6 dm ។ Qមចំណច
N M និង N សង់បCWត់ d និង d ' េរៀង
ែដល d / / d ' ។
េ•េលើបCWត់ d េយើងេMចំ
េMចំណច
N E ែដល ME = 1 dm
េ•េលើបCWត់ d ' េយើងេMចំណច
N F និង G ែដល NF = NG = 3 dm
គូសបCWត់ EF និង EG ត់បCWត់ MN តង់ C និង D េរៀង ។
េEយ ME / / FG េគ?ន ៖ ( C េ•ចេCDះ MN )
a MC ME 1
= = =
b NC NF 3
និង ( D េ•េ PចេCDះ MN )
a DM ME 1
= = =
b DN NG 3
ដូចេនះ C និង D ចំណច
N ែចកកKង
L និងេ PអងOត់ AB Qមផលេធៀប
Qមផលេធៀប
1
3
តវ?នេMរួច ។

កំែណេ យ ៃហ ហុន
ិ , ៃហ ចរ និងយ៉ត ពនក 214
េមេរ នទី១៦ ៖ តីេ ណដូច

11/. ក. 7យប89ក់< ∆ABI និង ∆AJC តីេ ណដូច

កKង
L ∆ABI និង ∆AJC 4ន ៖
∠BAI = ∠CAJ ( AJ កនVះបCWត់ពះុ ∠BAC )
និង ∠ABI = ∠AJC (មុ7
ំ ត់
m ធរKX ម
ួ AC )
ដូចេនះ តីេ ណ ∆ABI និង ∆AJC ដូច Qមករណីដណ
ំ ច
1 ទី១(ម.ម.)
១(ម.ម.) ។
១(
ខ. គណCផលគុណ AI ⋅ AJ
េយើង4ន តីេ ណ ∆ABI និង ∆AJC តីេ ណដូច , េគ?នផលេធៀប
AB AI
= ⇒ AI ⋅ AJ = AB ⋅ AC
AJ AC
ដូចេនះផលគុណ AI ⋅ AJ = AB ⋅ AC ។
គ. បYZញ< ∠BAJ = ∠CBJ
េយើង4ន ៖ ∠BAJ = ∠CAJ ( AJ កនVះបCWត់ពះុ ∠BAC ) (១)
និង ∠CBJ = ∠CAJ (មុ7
ំ ត់
m ធរKX ម
ួ CJ ) (២)
Qម (១) និង (២) េគ=ញ?ន ៖ ∠BAJ = ∠CBJ ។
ឃ. 7យប89ក់< JB 2 = JA ⋅ JI
កKង
L តីេ ណ ABJ និង BIJ 4ន ៖
∠BAJ = ∠CBJ (ស 4យ ងេលើ)
និង ∠J មុរំ ម

េCះេគ?ន ∆ABJ ∼ ∆BIJ Qមករណីទ១
ី (ម.ម.)
វ>?ក ៖
BJ AJ
= ⇒ BJ 2 = IJ ⋅ AJ
IJ BJ
ដូចេនះ JB 2 = JA ⋅ JI តវ?ន 7យប89ក់ ។

កំែណេ យ ៃហ ហុន
ិ , ៃហ ចរ និងយ៉ត ពនក 215
េមេរ នទី១៦ ៖ តីេ ណដូច

12/. គណCច4^យ AB
េយើង4ន AB / / DE និង CA = 1.8 km = 1800 m , CD = 90 m , DE = 150 m
Qម ទឹសប
}ី ទQែលស , េគ?ន ៖
CD DE
=
CA AB
CA ⋅ DE 1800 ⋅150
⇒ AB = = = 3000 m = 3 km
CD 90
ដូចេនះ AB = 3 km តវ?នគណC ។ (bចេធ)Q
ើ ម តីេ ណដូច ក៏?ន)
13/. ក. បដូច តីេ ណ ABC និង តីេ ណ AMN

េយើង4ន ៖ ∠AMN =
1
2
(AE + DB )
េEយ xy / / DE េគ?ន AE = AD េCះ ∠AMN =
1
2
( )
1
AD + DB = AB = ∠ACB
2
េគ?ន , កKង
L តីេ ណ ABC និង AMN 4ន ∠AMN = ∠ACB និង ∠A មុរំ ម

ដូចេនះ ∆ABC ∼ ∆ANM Qមករណីទ១
ី (ម.ម.) ។
ខ. 7យប89ក់< AB ⋅ AM = AC ⋅ AN
េយើង4ន ∆ABC ∼ ∆ANM េគ?នផលេធៀបដំណច
1 ៖
AB AC
=
AN AM
⇒ AB ⋅ AM = AN ⋅ AC
ដូចេនះ AB ⋅ AM = AC ⋅ AN តវ?ន 7យប89ក់ ។
14/. ក.គណC CD

កំែណេ យ ៃហ ហុន
ិ , ៃហ ចរ និងយ៉ត ពនក 216
េមេរ នទី១៦ ៖ តីេ ណដូច

កKង
L តីេ ណែកង ACD និង CBD 4ន ៖
∠A = ∠BCD (មុ4
ំ ន ជុង តវ ែកងេរៀង )
េគ?ន ∆ACD ∼ ∆CBD Qមករណីទ១
ី (ម.ម.)
វ>?ក ៖
CD AD
= ⇒ CD 2 = AD ⋅ BD
BD CD
េEយ AD = 4 dm , DB = 9 dm េគ?ន ៖
CD 2 = 4 ⋅ 9 = 36
CD = 6 dm
ដូចេនះ CD = 6 dm តវ?នគណC ។
ខ. គណC AD និង DB
7យដូច ងេលើែដរ េយើង4ន CD 2 = AD ⋅ BD (ស 4យ ងេលើ)
េEយ AB = 29 dm , CD = 10 dm
េយើងQង x = AD េCះ BD = 29 − x
េគ?ន ៖
10 2 = x ⋅ ( 29 − x )
100 = 29 x − x 2
x 2 − 29 x + 100 = 0
x 2 − 4 x − 25 x + 100 = 0
x ( x − 4 ) − 25 ( x − 4 ) = 0
( x − 4 )( x − 25 ) = 0
េបើ x − 4 = 0 ⇒ x = AD = 4 dm េCះ BD = 29 − 4 = 25 dm ។
េបើ x − 25 = 0 ⇒ x = AD = 25 dm េCះ BD = 29 − 25 = 4 dm ។
ដូចេនះ AD = 4 dm , BD = 25 dm ឬ AD = 25 dm , BD = 4 dm តវ?នគណC ។
15/. 7យបំភឲ
Vឺ Aេឃើញ<កKង
L តីេ ណមួយ aha = bhb = chc

កKង
L តីេ ណែកង AHB និង BJC 4ន ∠ABH = ∠JBC (មុរំ ម
ួ )
េCះ ∆AHB ∼ ∆CJB Qមករណីដណ
ំ ច
1 ទី១ (ម.ម.)

កំែណេ យ ៃហ ហុន
ិ , ៃហ ចរ និងយ៉ត ពនក 217
េមេរ នទី១៦ ៖ តីេ ណដូច

វ>?ក ៖
AH AB h c
= ⇔ a = ⇔ aha = chc (1)
CJ CB hc a
7យ សេដៀង ែដរ , េEយ បដូច តីេ ណ AHC និង BIC េគនឹង?ន
aha = bhb ( 2)
Qម (1) និង ( 2 ) េគ?ន ៖ aha = bhb = chc
ដូចេនះ aha = bhb = chc តវ?ន 7យប89ក់ ។
16/. េ បៀបេធៀប តីេ ណ ADB និង តីេ ណ AEB

េយើង4ន ∆ABC តីេ ណសម?តកំពល


ូ A េCះ AB = AC ⇒ AB = AC
កKង
L តីេ ណ ADB និង AEB 4ន ៖
∠BAE = ∠BAD (មុរំ ម
ួ )
និង ∠BDA = ∠ABE (មុ7
ំ ត់
m ធប
KX ន
៉ុ AB = AC )

ដូចេនះ ∆ADB ∼ ∆ABE Qមករណីដណ


ំ ច
1 ទី១ (ម.ម.) ។
7យប89ក់< AB 2 = AD ⋅ AE
េយើង4ន ∆ADB ∼ ∆ABE េគ?នផលេធៀបដំណច
1 ៖
AB AD
=
AE AB
⇒ AB 2 = AE ⋅ AD
ដូចេនះ AB 2 = AD ⋅ AE តវ?ន 7យប89ក់ ។
17/. គណCកមG
គណCកមGសេដើ
់េដើមឫសkីពគ
ី ល់េ_ចំណច
N ?ក់ P
Qង x កមGសេ់ ដើមឫសkីពគ
ី ល់ដល់ចណ
ំ ច
N
?ក់ ( x > 0 ) េCះច4^យពីកែនVង?ក់េ_ចុង
ឫសkីគឺ 10 − x
េEយអនុវត€ន៍ ទឹសប
}ី ទពីQគ័រ កKង
L តីេ ណ ង

កំែណេ យ ៃហ ហុន
ិ , ៃហ ចរ និងយ៉ត ពនក 218
េមេរ នទី១៦ ៖ តីេ ណដូច

េលើ េគ?ន ៖
x 2 + 32 = (10 − x )
2

x 2 + 9 = 100 − 20 x + x 2
20 x = 100 − 9
91
x= = 4.55 m
20
ដូចេនះកមG
ចេនះកមGសេ់ ដើមឫសkីពគ
ី ល់ដល់ចណ
ំ ច
N ?ក់គឺ 4.55 m តវ?នគណC ។
18/. រកជេ qទឹក EB

េ បើ ទឹសប
}ី ទពីQគ័រកKង
L តីេ ណ A ' EB េគ?ន ៖
A ' B 2 = BE 2 + A ' E 2 (១)
េEយ A ' B = AB ( បែវងlmឈូកែតមួយ)
AE = 3 dm និង EA ' = 9 dm េគ?ន ,

(១) េ_ AB 2 = BE 2 + 92
( AE + EB ) = BE 2 + 81
2

( 3 + EB ) = BE 2 + 81
2

9 + 6 BE + BE 2 = BE 2 + 81
81 − 9
BE = = 12 dm
6
ដូចេនះជេ
ចេនះជេ qទឹក សះេCះគឺ BE = 12 dm តវ?នគណC ។
19/. គណC h េ_QមតៃមV a និង b

កំែណេ យ ៃហ ហុន
ិ , ៃហ ចរ និងយ៉ត ពនក 219
េមេរ នទី១៦ ៖ តីេ ណដូច

កKង
L តីេ ណែកង CBH និង ACH 4ន ៖
∠CBH = ∠HCA (មុ4
ំ ន ជុង តវ ែកងេរៀង )
េCះ ∆CBH ∼ ∆ACH Qមករណីដណ
ំ ច
1 ទី១ (ម.ម.)
េគ?នផលេធៀបដំណច
1 ៖
CB CH BH
= =
AC AH CH
a h BH
= =
b AH h
a
⇒ h = ⋅ AH
b
េEយអនុវត€ន៍ ទឹសប
}ី ទពីQគ័រ , េគ?ន ៖
AH 2 = AC 2 − CH 2
AH = b 2 − h 2
េយើង?ន ៖
a
h= ⋅ b 2 − h2
b
( hb )
2
(
= a2 b2 − h2 )
( )
h 2 b 2 + a 2 = a 2b 2
ab
h=
a + b2
2

ដូចេនះ h = តវ?នកំណត់ ។
ab
a2 + b2
20/. ក. គណC IB

េEយ ABCD បេលឡX ម , េគ?ន ៖ AD = BC = 12 cm ( ជុងឈម)


េEយ I ចំណច
N កtuលៃន AD េCះ AI = DI =
12
= 6 cm
2
Qម ទឹសប L តីេ ណ ABI េគ?ន ៖
}ី ទពីQគ័រ កKង
AB 2 = AI 2 + IB 2
102 = 62 + IB 2
IB = 100 − 36 = 8 cm
ដូចេនះ IB = 8 cm តវ?នគណC ។

កំែណេ យ ៃហ ហុន
ិ , ៃហ ចរ និងយ៉ត ពនក 220
េមេរ នទី១៦ ៖ តីេ ណដូច

ខ. គណCៃផv កwៃន បេលឡX ម


Qមរូបមន€ កwៃផvៃន បេលឡX ម S = ah
េEយ a = AD = 12 cm និង h = IB = 8 cm េគ?ន ៖
S = (12 cm )( 8 cm ) = 96 cm 2

ដូចេនះ បេលឡX មេCះ4ន កwៃផv 96 cm 2 ។


21/. ក. គណCរYhស់ MQ

េEយអនុវត€ន៍ ទឹសប
}ី ទពីQគ័រកKង
L តីេ ណ MNQ េគ?ន ៖
MN 2 + MQ 2 = NQ 2
MQ = 52 − 32 = 4 cm
ដូចេនះរYhស់ MQ = 4 cm តវ?នគណC ។
ខ. គណCរYhស់ NP
កKង
L តីេ ណែកង MNQ និង NPQ 4ន ∠MQN = ∠NQP (សមIតក
ិ មI)
េគ?ន ∆MNQ ∼ ∆NPQ Qមករណីដណ
ំ ច
1 ទី១ (ម.ម.) ។
េគ?នផលេធៀប ៖
MN MQ 3 4
= ⇔ = ⇒ NP = 3.75 cm
NP NQ NP 5
ដូចេនះរYhស់ NP = 3.75 cm តវ?នគណC ។
22/. ក. េ បៀបេធៀប តីេ ណ IAC និង IBD

កKង
L តីេ ណ IAC និង IBD 4ន ៖

កំែណេ យ ៃហ ហុន
ិ , ៃហ ចរ និងយ៉ត ពនក 221
េមេរ នទី១៦ ៖ តីេ ណដូច

∠AIC = ∠DIB (មុទ


ំ ល់កព
ំ ល
ូ )
និង ∠ACI = ∠DBI (មុ7
ំ ត់
m ធរKX ម
ួ AD )
ដូចេនះ ∆IAC ∼ ∆IDB Qមករណីដណ
ំ ច
1 ទី១ (ម.ម.)
(ម.ម.) ។
ខ. គណCរYhស់ AC
េEយ ∆IAC ∼ ∆IDB េគ?នផលេធៀបដំណច
1 ៖
IC AC 12 AC
= ⇔ = ⇒ AC = 7.2 cm
IB DB 5 3
ដូចេនះ AC = 7.2 cm តវ?នគណC ។
23/. ក. េ បៀបេធៀប តីេ ណ PAC និង PAD

កKង
L តីេ ណ PAC និង PAD 4ន ៖
∠APC = ∠APD (មុរំ ម
ួ )
និង ∠PAC = ∠ACP (មុ7
ំ ត់
m ធរKX ម
ួ AC )
ដូចេនះ ∆PAC ∼ ∆PDA Qមករណីដណ
ំ ច
1 ទី១ (ម.ម.) ។
ខ. =ញឲAេឃើញ< AP 2 = PC ⋅ PD
េEយ ∆PAC ∼ ∆PDA េគ?នផលេធៀបដំណច
1 ៖
PA PC
= ⇒ PA2 = PC ⋅ PD
PD PA
ដូចេនះ AP 2 = PC ⋅ PD តវ?ន=ញឲAេឃើ
តវ?ន=ញឲAេឃើញ ។
24/. េតើេ ណ=ំងពីរដូច ឬេទ ?

កំែណេ យ ៃហ ហុន
ិ , ៃហ ចរ និងយ៉ត ពនក 222
េមេរ នទី១៦ ៖ តីេ ណដូច

េ ណ=ំងពីរ4ន ផលេធៀប ៖ = = និង


d 4 1 h 5 1
= =
d' 8 2 h ' 10 2
េEយផលេធៀប
d h
=
d ' h'
ដូចេនះេ ណ=ំងពីរេនះដូច ។
3 3

25/. ក. បYZញ< =   =  
v h r
V H  R

Qមប 4ប់
4ប់ , េ ណ=ំងពីរ ងេលើេនះដូច , េគ?ន ៖
(១)
r h
=
R H
មƒ„ងេទៀត , Qមរូបមន€4ឌៃនេ ណ េគ?ន ៖ v = π r 2 h និង V = π R 2 H
1 1
3 3
1 2
πr h
r 2h
េគ?នផលេធៀប = 3 (២)
v
= 2
V 1 π R2 H R H
3
Qម (១) និង (២) េគ?ន ៖
2 3
v r2 h  r   r   r 
= ⋅ =   = 
V R2 H  R   R   R 
2 3
v r2 h  h   h   h
ឬមƒ„ងេទៀត = ⋅ =   = 
V R2 H  H   H  H
3 3

ដូចេនះសរុបមកវ>ញ =   =   តវ?នបYZញ ។
v h r
V H  R
ខ. គណC4ឌ V ៃនេ ណធំ
េយើង4ន v = 6.28 cm3 េហើយ h = 2 cm , H = 4 cm
3

Qមស 4យ ងេលើ េយើង4ន , =   េគ?ន ៖


v h
V H 
3 3
H  4 cm 
 h
(
V = v   = 6.28 cm3  )  = 50.24 cm
3

 2 cm 
ដូចេនះ 4ឌៃនេ ណធំគឺ V = 50.24 cm3 តវ?នគណC ។

កំែណេ យ ៃហ ហុន
ិ , ៃហ ចរ និងយ៉ត ពនក 223
េមេរ នទី១៧ ៖ ពហុេ ណ

ម នទ ព

1/. ចូរគណ ផលបូករ ស់មក


ុំ ងៃនពហុេ ណែដល ន ជុង 7 8 9 10 និង
12 ។

2/. ចូររកចំនន
ួ ជុងៃនពហុេ ណែដល នផលបូករ ស់មក
ុំ ងេស%ន
ើ ង

1800o , 2 700o និង 4860o ។

3/. េតើ នពហុេ ណ)ែដល នផលបូករ ស់មក


ុំ ងេស%ន
ើ ង
ឹ 5130o ែដរ ឬេទ ?
4/. េ-េពលេគេធ/ើ រគណ ផលបូករ ស់មក
ុំ ងៃនពហុេ ណេ01ងមួយែដល
នរ ស់េស%ន
ើ ង
ឹ 540o េតើពហុេ ណេ01ងេ ះ ន ជុងប៉ុ ន
5 ?
5/. ចូររកតៃម6 x 7មរូបនីមយ
ួ ៗ ៃនពហុេ ណ:ងេ ម។

បក បខ

បគ

6/. ចូររកតៃម6 y 7មរូបនីមយ


ួ ៗ ៃនពហុេ ណ:ងេ ម។

បក បខ

កំែណេ យ ៃហ ហុន
ិ , ៃហ ចរ និងយ៉ត ពនក 224
េមេរ នទី១៧ ៖ ពហុេ ណ

បគ

7/. ចូរគណ ចំនន


ួ ជុងៃនពហុេ ណនិយត
័ េ<យ=>
េ<យ=>ល់រ ស់មក
ុំ ងេស%ន
ើ ង

150o , 160o និង 175o ។

8/. ផលបូករ ស់មក


ុំ ងៃនពហុេ ណនិយត
័ មួយ នរ ស់េស%ន
ើ ង
ឹ 3240o ។
ចូរគណ រ ស់មេុំ ?នីមយ
ួ ៗ។
9/. ផលបូករ ស់មក
ុំ ងរួមនឹងផលបូករ ស់មេុំ ?ៃនពហុេ ណនិយត
័ មួយេស%ន
ើ ង

900o ។ េតើពហុេ ណេ ះ ន ជុងប៉ុ ន
5 ?
10/. មុក ័ ែដល ន n ជុង នរ ស់េស%ន
ំ ងមួយៃនពហុេ ណនិយត ើ ង
ឹ 168o ។
ចូររកចំនន
ួ n ។
11/. ចូរគណ រ ស់មេុំ ?នីមួ
មយ ួ ៗ ៃនពហុេ ណនិយត
័ :ងេ ម៖
ក. ឆេ ណនិយត
័ n=6
ខ. បDEេ ណនិយត
័ n=5
គ. អដGេ ណនិយត
័ n =8
ឃ. ពហុេ ណនិយត
័ ែដល ន ជុង 12 ។
12/. ផលបូករ ស់មក
ុំ ង និងមុេំ ?េស%ន
ើ ង
ឹ 900o ។ េតើពហុេ ណេ ះ ន ជុង
ប៉ុ ន
5 ?
13/. េ<យ ជុងនីមយ
ួ ៗ ៃនឆេ ណនិយត
័ I ជុងមួយរបស់ េរែដលេគគូ
៉ែដលេគគូសេ-
:ងេ ?ឆេ ណនិយត
័ េហើយេគJKប់ពក
ី ព
ំ ល
ូ េរែដលេ-Iប់
៉ែដលេ-Iប់ៗLM ។
ចូរ =យបNKក់O រូបែដលកេកើតIពហុេ ណនិយត
័ ែដល ន ជុង 12 ។
14/. មុេំ ?ៃនអដGេ ណនិយត
័ មួយ នរ ស់ 27o តិចIងរ ស់មេុំ ?មួយៃន
ពហុេ ណនិយត
័ មួយែដល ន n ជុង ។ ចូរគណ n ។

15/. ចូរ =យបNKក់O េបើពហុេ ណមួយែដល ន n ជុង េ ះចំ


ះចំនន
ួ អងPត់
n ( n − 3)
ទQងែដលេគRចគូស0នេស%ើ ។
2
ែណ ំ ៖ េ បៀបេធៀបចំនន
ួ ជុងពហុេ ណេ នឹងចំនន
ួ អងត់ ទងែដលគូស

កំែណេ យ ៃហ ហុន
ិ , ៃហ ចរ និងយ៉ត ពនក 225
េមេរ នទី១៧ ៖ ពហុេ ណ

េចញពីកព
ំ ល
ូ មួយៃនពហុេ ណ ។
16/. ចូរគណ មុក
ំ ង និងមុេំ ?ៃនឆេ ណនិយត
័ ។
័ ែដល ន n ជុង នរ ស់ 30o េ ចើនIងរ ស់
17/. មុេំ ?ៃនពហុេ ណនិយត
មុេំ ?មួយៃនឆេ ណនិយត
័ ។ ចូរគណ n ។

ច មយ
1/. គណ ផលបូករ ស់មក
ុំ ងៃនពហុេ ណែដល ន 7 8 9 10 និង 12
7មរូបមនS , ផលបូករ ស់មក
ុំ ងៃនពហុេ ណែដល ន n ជុងគឺ ៖
( n − 2 )180o េគ0ន ៖
ករណី n = 7 េ ះផលបូករ ស់មក
ុំ ងគឺ ( 7 − 2 )180o = 900o ។
ករណី n = 8 េ ះផលបូករ ស់មក
ុំ ងគឺ (8 − 2 )180o = 1080o ។
ករណី n = 9 េ ះផលបូករ ស់មក
ុំ ងគឺ ( 9 − 2 )180o = 1260o ។
ករណី n = 10 េ ះផលបូករ ស់មក
ុំ ងគឺ (10 − 2 )180o = 1440o ។
ករណី n = 12 េ ះផលបូករ ស់មក
ុំ ងគឺ (12 − 2 )180o = 1800o ។
2/. រកចំនន
ួ ជុងៃនពហុេ ណែដល នផលបូករ ស់មក
ុំ ងេស%ន
ើ ង
ឹ 1800o
2 700o និង 4860o

ពីរប
ូ មនS , ផលបូករ ស់មក
ុំ ងៃនពហុេ ណែដល ន n ជុងគឺ ៖
S = ( n − 2 )180o េគVញ
េគVញ0ន
Vញ0ន ៖ n =
S
+2
180o
1800o
ករណី S = 1800o េគ0ន ៖ n = + 2 = 12 ជុង ។
180o
2700o
ករណី S = 2700 េគ0ន ៖ n =
o
+ 2 = 17 ជុង ។
180o
4860o
ករណី S = 4860o េគ0ន ៖ n = + 2 = 29 ជុង ។
180o
3/. េតើ នពហុេ ណ)ែដល នផលបូករ ស់មក
ុំ ងេស%ន
ើ ង
ឹ 5130o ែដរ
ឬេទ ?
ពីរប
ូ មនS ៖ n = + 2 េ<យ S = 5130o េគ0ន ៖
S
o
180
5130o
n= + 2 = 30.5 មិនែមនIចំនន
ួ គត់
180o
ដូេចះL5នពហុេ ណែដល នផលបូករ ស់មក
ុំ ងេស%ន ឹ 5130o េទ ។
ើ ង

កំែណេ យ ៃហ ហុន
ិ , ៃហ ចរ និងយ៉ត ពនក 226
េមេរ នទី១៧ ៖ ពហុេ ណ

4/. េតើពហុេ ណេ01ងេ ះ ន ជុងប៉ុ ន


5 ?
7មបំXប់ , េគេធ/ើ រគណ ផលបូករ ស់មក
ុំ ងៃនពហុេ ណេ01ងមួយ
ែដល នរ ស់េស%ន
ើ ង
ឹ 540o , េគ0ន
េគ0ន ៖
540o
n= + 2 = 5 ជុង
180o
ដូចេនះពហុេ ណេ01ងេ ះគឺ ន ៥ ជុង (បDEេ ណ) ។
5/. រកតៃម6 x 7មរូបនីមយ
ួ ៗ ៃនពហុេ ណ:ងេ ម

បក េនះIចតុេ ណ េ ះ\ តQវ នផលបូ


នផលបូករ ស់មក
ុំ ងVំងអស់េស%ន
ើ ង

( 4 − 2 )180o = 360o
ែតផលបូករ ស់មក
ុំ ងVំងអស់គឺ ៖ 3x + x + 80o + x េគ0ន ៖
5 x + 80o = 360o
360o − 80o
x= = 56o
5
ដូចេនះ x = 56o តQវ0នកំណត់ ។

បខ េនះIបDEេ ណ េ ះ\ តQវ នផលបូករ ស់មក


ុំ ងVំងអស់េស%ន
ើ ង

( 5 − 2 )180o = 540o
ែតផលបូករ ស់មក
ុំ ងVំងអស់គឺ ៖ x + 2 x + 2 x + 50o + 100o េគ0ន
េគ0ន ៖
5 x + 150o = 540o
540o − 150o
x= = 78o
5
ដូចេនះ x = 78o តQវ0នកំណត់ ។

កំែណេ យ ៃហ ហុន
ិ , ៃហ ចរ និងយ៉ត ពនក 227
េមេរ នទី១៧ ៖ ពហុេ ណ

បគ េនះIឆេ ណ េ ះ\ តQវ នផលបូករ ស់មក


ុំ ងVំងអស់េស%ន
ើ ង

( 6 − 2 )180o = 720o
ែតផលបូករ ស់មក
ុំ ងVំងអស់គឺ ៖ x + 160o + 2 x + x + 2 x + 2 x េគ0ន ៖
8 x + 160o = 720o
720o − 160o
x= = 70o
8
ដូចេនះ x = 70o តQវ0នកំណត់ ។
6/. រកតៃម6 y 7មរូបនីមយ
ួ ៗ ៃនពហុេ ណ:ងេ ម
េ<យ ផលបូករ ស់មេុំ ?Vំងអស់ៃនរូបពហុេ ណេ01ង គឺ 360o IនិចE
េគ0ន ៖

360o
បក y + 2 y + y + 2 y = 360o ⇒ y = = 60o
6

360o − 150o
បខ y + y + y + 80o + 70o = 360o ⇒ y = = 70o
3

360o
បគ 2 y + 3 y + 2 y + 2 y + 3 y + 2 y + 2 y = 260o ⇒ y = = 22.5o
16

កំែណេ យ ៃហ ហុន
ិ , ៃហ ចរ និងយ៉ត ពនក 228
េមេរ នទី១៧ ៖ ពហុេ ណ

7/. គណ ចំនន
ួ ជុងៃនពហុេ ណនិយត
័ េ<យ=>ល់រ ស់មក
ុំ ងេស%ន
ើ ង

150o , 160o និង 175o

េយើងដឹងេហើយO ផលបូករ ស់មក


ុំ ងVំងអស់ៃនពហុេ ណែដល ន n
ជុង គឺ ( n − 2 )180o េហើយពហុេ ណនិយត
័ Iពហុេ ណែដល នរ ស់

មុក
ំ ងVំងអស់េស%L
ើ ,េគ0នរ
M ស់មក
ុំ ងៃនពហុេ ណនិយត
័ គឺ៖
( n − 2 )180o
n
ករណីពហុេ ណនិយត
័ ែដល នរ ស់មក
ុំ ង 150o េគ0ន ៖
( n − 2 )180o = 150o
n
180 ( n − 2 ) = 150n
6n − 12 = 5n
n = 12
ដូចេនះពហុេ ណនិយត
័ េ ះ ន ១២ ជុង ។
ករណីពហុេ ណនិយត
័ ែដល នរ ស់មក
ុំ ង 160o េគ0ន ៖
( n − 2 )180o = 160o
n
180 ( n − 2 ) = 160n
9n − 18 = 8n
n = 18
ដូចេនះពហុេ ណនិយត
័ េ ះ ន ១៨ ជុង ។
ករណីពហុេ ណនិយត
័ ែដល នរ ស់មក
ុំ ង 175o េគ0ន ៖
( n − 2 )180o = 175o
n
180 ( n − 2 ) = 175n
36n − 72 = 35n
n = 72
ដូចេនះពហុេ ណនិយត
័ េ ះ ន ៧២ ជុង ។
8/. គណ រ ស់មេុំ ?នីមយ
ួ ៗ
7មបំXប់ , ផលបូករ ស់មក
ុំ ងៃនពហុេ ណនិយត
័ េ ះ នរ ស់េស%ន
ើ ង

3240o េគ0ន ៖
( n − 2 )180o = 3240o
3240
n= + 2 = 20
180
េ<យផលបូករ ស់មេុំ ?Vំងអស់ៃនពហុេ ណេ01ង គឺ 360o
360o
េ ះេគ0នរ ស់មុំ កីនម
ី យ
ួ ៗគឺ ៖ = 18o
20

កំែណេ យ ៃហ ហុន
ិ , ៃហ ចរ និងយ៉ត ពនក 229
េមេរ នទី១៧ ៖ ពហុេ ណ

ដូចេនះរ ស់មេុំ ?នីមយ


ួ ៗៃនពហុេ ណនិយត
័ េ ះគឺ 18o ។
9/. េតើពហុេ ណេ ះ ន ជុងប៉ុ ន
5 ?
7មបំXប់ , ផលបូករ ស់មក
ុំ ងរួមនឹងផលបូករ ស់មេុំ ?ៃនពហុេ ណ
និយត
័ មួយេស%ន
ើ ង
ឹ 900o , េគ0ន ៖
( n − 2 )180o + 360o = 900o
900o − 360o
n= +2=5
180o
ដូចេនះពហុេ ណនិយត
័ េ ះ នចំនន
ួ ៥ ជុង(បDEេ ណនិយត
័ )។
10/. រកចំនន
ួ n
7មបំXប់ , មុក
ំ ងមួយៃនពហុេ ណនិយត
័ ែដល ន n ជុង នរ ស់េស%ើ
នឹង 168o , េគ0ន ៖
( n − 2 )180o
= 168o
n
15 ( n − 2 ) = 14
n = 30
ដូចេនះ n = 30 តQវ0នកំណត់ ។
11/. គណ រ ស់មេុំ ?នីមួ
មយ ួ ៗ ៃនពហុេ ណនិយត
័ :ងេ ម៖
ផលបូករ ស់មេុំ ?Vំងអស់ៃនពហុេ ណគឺ 360o េ ះមុេំ ?នីមយ
ួ ៗៃន
360o
ពហុេ ណនិយត
័ ែដល ន n គឺ ៖
n
ក. ឆេ ណនិយត
័ n=6
360o
េគ0ន ៖ រ ស់មេុំ ?នី
?នីមយ
ួ ៗ ៃនឆ
ៃនឆេ ណនិយត
័ = 60o ។
6
ខ. បDEេ ណនិយត
័ n=5
360o
េគ0ន ៖ រ ស់មេុំ ?នីមយ
ួ ៗ ៃនបDE
ៃនបDEេ ណនិយត
័ = 72o ។
5
គ. អដGេ ណនិយត
័ n =8
360o
េគ0ន ៖ រ ស់មេុំ ?នីមយ
ួ ៗ ៃនអដG
ៃនអដGេ ណនិយត
័ = 45o ។
8
ឃ. ពហុេ ណនិយត
័ ែដល ន ជុង 12
360o
េគ0ន ៖ រ ស់មេុំ ?នីមយ
ួ ៗ ៃនពហុ
ៃនពហុេ ណនិយត
័ ១២ ជុង = 30o ។
12
12/. ដូច នឹងលំ ត់េលខ៩
លខ៩ ែដរ ។

កំែណេ យ ៃហ ហុន
ិ , ៃហ ចរ និងយ៉ត ពនក 230
េមេរ នទី១៧ ៖ ពហុេ ណ

13/. =យបNKក់O រូបែដលកេកើតIពហុេ ណនិយត


័ ែដល ន ជុង 12

7ង ABCDEF Iឆេ ណនិយត


័ េ ះ។
េគ0ន ៖ AB = BC = CD = DE = EF = FA
េគសង់ េរ៉ជវុំ ញ
b ឆេ ណនិយត
័ េ ះ ដូចរូប , េគ0ន ៖
GH = IJ = KL = MN = OP = QR (១)
េហើយេគក៏ទទួល0ន តីេ ណសម0តប៉ន
ុ ៗLMចំនន
ួ ៦ គឺ ៖

∆GAR , ∆IBH , ∆KCJ , ∆MDL , ∆OEN , ∆PFQ


េយើងនឹងប ញO
e តីេ ណសម0តVំង៦េនះសុទែf តI តីេ ណសម័ងg
កង តីេ ណសម0ត GAR កំពល
ូ A ន៖

∠GRA = 360o − ∠BAF − ∠GAB − ∠RAF


( 6 − 2 )180o
ែត ∠BAF = = 120o , ∠GAB = ∠RAF = 90o េគ0ន ៖ ∠GRA = 60o
6
តីេ ណសម0តែដល នមុម
ំ យ
ួ េស%ន
ើ ង
ឹ 60o េ ះ\I តីេ ណសម័ងg
=យ សេដៀងLMែដរ ឲ
ំ jេគ0ន តីេ ណ ∆GAR , ∆IBH , ∆KCJ , ∆MDL
∆OEN , ∆PFQ សុទែf តI តីេ ណសម័ងgប៉ន
ុ ៗLM ។
េគ0ន ៖ GR = HI = JK = LM = NO = PQ (២)
7ម (១) និង (២) េគ0ន ៖

GH = IJ = KL = MN = OP = QR = GR = HI = JK = LM = NO = PQ
ដូចេនះពហុេ ណ GHIJKLMNOPQR Iពហុេ ណនិយត
័ ែដល ន ១២
ជុង ។

កំែណេ យ ៃហ ហុន
ិ , ៃហ ចរ និងយ៉ត ពនក 231
េមេរ នទី១៧ ៖ ពហុេ ណ

14/. គណ n

7មបំXប់ , មុេំ ?ៃនអដGេ ណនិយត


័ មួយ នរ ស់ 27o តិចIងរ ស់មុំ
េ ?មួយៃនពហុេ ណនិយត
័ មួយែដល ន n ជុង , េគ0ន ៖
360o 360o
+ 27 o =
8 n
360o
72o =
n
360o
n= =5
72o
ដូចេនះពហុេ ណេ ះ នចំនន
ួ ជុង n = 5 ។
15/. =យបNKក់O េបើពហុេ ណមួយែដល ន n ជុង េ ះចំនន
ួ អងPត់
n ( n − 3)
ទQងែដលេគRចគូស0នេស%ើ
2

ពហុេ ណមួយែដល ន n ជុង េ ះ\ក៏ ន n កំពល


ូ ែដរ
ពីកព
ំ ល
ូ មួយៃនពហុេ ណ េគRចគូសអងPតេkកំ
់េkកំពល
ូ េផgងៗេទៀតចំនន

n − 1 (េលើកែលងខ6ន
l ឯងេចញ) \ តQវI n − 1 អងPត់ ែដលែចកេចញI
ពីរែផក គឺ ៖ ២អងPត់ I ជុងរបស់ពហុេ ណែដល នកំពល
ូ េ-អម
ចំណច
n េ ះ េហើយ េ-សល់ n − 3 េទៀត IអងPត់ ទQង ។
ដូេចះ កង១
កង១កំពល
ូ េគRចគូស0នអងPត់ ទQង n − 3 េ ះកងចំេ)ម n
កំពល ួ n ( n − 3) ។ ក៏បែ៉ុ នSកងចំេ)ម
ូ េគនឹងRចគូសអងPត់ ទQង0នចំនន
អងPតV
់ ង
ំ n ( n − 3) េ ះ នអងPតដ
់ ែដលពីរៗដង (ដូចIអងPត់ AB និង
BA \គឺIអងPតែ់ តមួយ) ។

េនះេបើពហុេ ណមួយែដល ន n ជុង េ ះចំនន


ដូចេនះេបើ ួ អងPត់ ទQងែដលេគ
n ( n − 3)
Rចគូស0នគឺ
0នគឺ ។
2

កំែណេ យ ៃហ ហុន
ិ , ៃហ ចរ និងយ៉ត ពនក 232
េមេរ នទី១៧ ៖ ពហុេ ណ

16/. គណ មុក
ំ ង និងមុេំ ?ៃនឆេ ណនិយត

ឆេ ណនិយត
័ Iពហុេ ណនិយត
័ ែដល ន ជុង ៦ ប៉ន
ុ ៗLM
េគ0ន ៖

មុក
ំ ងរបស់ឆេ ណនិយត
័ គឺ ៖
( 6 − 2 )180o
= 120o ។
6
o
និង មុេំ ?ៃនឆេ ណនិយយ័ត គឺ ៖ = 60o ។
360
6
17/. គណ n

7មបំXប់ , មុេំ ?ៃនពហុេ ណនិយត


័ ែដល ន n ជុង នរ ស់ 30o
េ ចើនIងរ ស់មេុំ ?មួយៃនឆេ ណនិយត
័ , េគ0ន ៖
360o 360o
− 30o =
n 6
o
360
= 90o
n
360o
n= =4
90o
ដូចេនះ n = 4 ( េរo) តQវ0នគណ ។

កំ ន
1. The object below is built from blocks.

Which is not a top, front, or side view of the object ?

A. B. C. D.
2. Find the lateral surface area and the total surface area of the
cylinder shown
shown below to the nearest tenth of a square centimeter.

កំែណេ យ ៃហ ហុន
ិ , ៃហ ចរ និងយ៉ត ពនក 233
េមេរ នទី១៨ ៖ សូលត

ម នទ

1/. ចូររកៃផ ក ង និងៃផ ក ង


ំ អស់ៃនរូប ងេ ម៖

ប (a) ប (b) ប (c)

2/. ចូររក ឌេ ណែដល នកមស់ និង ំ តក"ង


# ករណីនមួ
ីមយួ ៗ ងេ ម៖
ក. R = 5 cm , h = 12 cm ខ. R = 3.4 cm , h = 8.9 cm
គ. R = 3.7 cm , h = 7 cm ឃ. R = 5.2 cm , h = 11cm
ង. R = 5 mm , h = 13.4 mm ច. R = 7 dm , h = 16 dm ។
3/. ចូររកៃផ ក ែស-៊ និង ឌប៊ល
ូ ក"ង
# ករណីនម
ី យ
ួ ៗ ងេ ម៖
ក. R = 3 cm ខ. D = 18 dm គ. R = 3.7 cm ឃ. D = 23.2 cm ។
4/. ចូររក ែំ ស-េ៊ /យ01ល
2 ៃ់ ផ ក ដូច ងេ ម៖
ក. S = ខ. S = គ. S = ឃ. S = 1764π cm 2 ។
36 46 12
π cm 2 π m2 π dm 2
25 16 5
5/. ចូររក ៃំ នប៊ល
ូ េ/យ01ល
2 ់ ឌ៖
ក. V = 288π cm3 ខ. V = π m3 ។
32
16
6/. េគឲ24េ ណ និងកន5ះែស-ដ
៊ ច
ូ រូប ង072ំ ។
ក. ចូររក ឌ ង
ំ អស់ៃនសូលត
ី ។
ខ. ចូររកៃផ ក ង
ំ អស់ៃនសូលត
ី ។
7/. េគឲ24ប89ត
2 ់ AB / /CD ែដល = ។
BE 1
CE 2
ៃផ ក ∆CED េស:ន
ើ ង
ឹ 16 cm2 ។ ចូររកៃផ ក ∆AEB ។

កំែណេ យ ៃហ ហុន
ិ , ៃហ ចរ និងយ៉ត ពនក 234
េមេរ នទី១៨ ៖ សូលត

8/. េតើៃផ ក េរ DEFG តូច=ងៃផ ក ABCD ប៉8


ុ ន
@2 ដង ?

(ប ប ទ ) (ប ប ទ )
9/. េគឲ24 បេលឡC ម ABCD និង I =ចំណច
D កEFល
2 ៃន AB ។ ប89ត
2 ់ BD
និង CI បសព-HI2 តង់ O ។ េតើៃផ ក ∆BIO តូច=ងៃផ ក ∆DOC

ប៉8
ុ ន
@2 ដង ?
10/. រូបថតៃនជLMង
2ំ មួយ នកមស់ 3cm និង នៃផ ក េស:ន
ើ ង
ឹ 9 cm 2 ។ េគ
ដឹងNជLMង
2ំ ិ 6 m ។ េតើO នៃផ ក
នកមស់ពត ពិត កដប៉8
ុ ន
@2 m 2 ?
11/. ៃផ ក ៃនទីQ0
R2 Sមួយ នរTUស
2 ់ 144 cm2 េVក"ង
# ែផនទី ទី១ េហើយ ន
រTUស
2 េ់ ស:ន
ើ ង
ឹ 256 cm 2 េVក"ង
# ែផនទី ទី២ ។ េ/យដឹងNវ[ តមួយៃនទីQR2
េ8ះេស:ន
ើ ង
ឹ 18 cm េVក"ង
# ែផនទី ទី១ ។ េតើវ[ តេនះេស:
តេនះេស:ន
ើ ង
ឹ ប៉8
ុ ន
@2 cm េV
ក"ង
# ែផនទី ទី២ ?
12/. ទីQម
R2 យ
ួ នៃផ ក ពិត 9 600 m 2 ។ េតើៃផ ក េនះ នប៉8
ុ ន
@2 cm 2 េV
ក"ង
# ប5ងម
់ យ
ួ ែដល ន ត/\ន
2 ?
1
2 500
13/. េគ ត់េ ណឲ24 សបនឹង ត=បីចែំ ណក
(ដូចរូប ង07)
2ំ ។ មុខ ត់ នអង_
នអង_តផ
់ ត
`ិ
េរៀងHI2 6 cm និង 10 cm ។
ក. ចូរគណ8 x ។
ខ. ចូររក ឌៃនេ ណេ8ះ ។

14/. SABCD =ពីbម


c2 ត
ី ែដល ន ត= េរ៉។ ជុងៃន េរេនះ
៉េនះ ន បែវង 60 cm ។
កមស់ SO ៃនពីbម
c2 ត
ី ន បែវង 80 cm , េគ ត់ពb
ី ម
c2 ត
ី េនះឲ24 សបនឹង ត ។
េគ ន េរe EFGH ។ េគឲ24 OO ' = x ។
ក. ចូររក ជុងៃន េរe EFGH fមតៃម5 x ។

កំែណេ យ ៃហ ហុន
ិ , ៃហ ចរ និងយ៉ត ពនក 235
េមេរ នទី១៨ ៖ សូលត

ខ. េបើ x = 30 cm គណ8 ឌៃនកំEត់ពb


ី ម
c2 ត
ី ខណgេ/យ ABCD និង
EFGH ។

15/. fមរូប ងេ
ងេ មេនះ េគឲ24 BA = 6 , SA = 8 , SA ' = SA ។ រTUស
មេនះ 2 គ
់ ត
ិ = cm ។
2
3
ក. ចូរគណ8 A ' B ' ។
ខ. ចូរគណ8ៃផ ក ៃន េរe A ' B ' C ' D ' រួចេផhងi9ត
2 N
់ ៖
2
2
ៃផ ក A ' B ' C ' D ' =   × ៃផ ក ABCD ។
3
គ. ចូរបTjញ
2 N SO ' = SO រួច ញបLMក 2 N
់ ៖
2
3
2

ឌ SA ' B ' C ' D ' =   × ឌ SABCD ។


2
3

(ប ប ទ ) (ប ប ទ )

16/. ក"ង
# រូប ងេលើ
ងេលើេនះ
េនះ េគឲ24 SM ' = SM , SO = 10 cm , OM = 6 cm ។
1
2
ក. ចូរគណ8 O ' M ' រួចេផhងi9ត
2 N
់ ៖
2

ៃផ ក Nស ( O ') =   × ៃផ ក Nស ( O ) ។
1
2
ខ. ចូរបTjញ
2 N SO ' = SO ។
1
2
គ. ចូរ 0យបLMក
2 N
់ ឌៃនេ ណៃនកំពល
ូ S និង ត O ' េស:ន
ើ ង
ឹ ឌ
3

ៃនេ ណៃនកំពល
ូ S និង ត O គុណនឹង   ។
1
2
17/. អ"កលក់ឪឡឹក ក
I2 ក
់ ណ
ំ ត់លក់ែផ5ឪឡឹកតូចពីរ ៃថ5េស:ន
ើ ង
ឹ ែផ5ឪឡឹកធំមយ
ួ ។
េ/យដឹ
េ/យដឹងN ៃំ នែផ5ឪឡឹកតូចេស:ន
ើ ង
ឹ ៃន ឪឡឹ
ំឪឡឹកធំ ។ េតើអក
" ទិញ តnវ
1
4
េ ជើសេរoសមួយEេដើមp2 ឲ
ី 42 នចំេណញ=ង ?

កំែណេ យ ៃហ ហុន
ិ , ៃហ ចរ និងយ៉ត ពនក 236
េមេរ នទី១៨ ៖ សូលត

18/. ឈ:ញ
r ក
I2 ល
់ ក់ ស
s2 ន
ុី េ កេ tវពីរ បេភទែដល នbងសុី ង
ំ ដូចHI2 ។
ែតវ[ តៃន ស
s2 ន
ុី ទីមយ
ួ េស:ន
ើ ង
ឹ ៃនវ[
ៃនវ[ ត ស
s2 ន
ុី ទីពរី ។ តៃម5ៃន ស
s2 ន
ុី
9
10
តូចេស:ន
ើ ង
ឹ ៃន ស
s2 ន
ុី ធំ ។ េតើេគ តnវេ ជើសេរoសយក ស
s2 ន
ុី បេភទE
7
10
េទើបចំេណញ=ង ?
19/. សុី ង 2 ់ ំ R និងកមស់ h េគយកេ ណែដល នរTUស
ំ មួយ នរTUស 2 ់ ំ
R និងកមស់ h /ក់កង
"# សុី ង
ំ fមរូប ។
ក. ចូរគណ8 ឌេ ណ និង ឌៃនសុី ង
ំ ។
ខ. ចូរគណ8ផលេធៀបរOង ឌៃនេ ណ និង ឌៃនសុី ង
ំ ។

(ប ប ទ ) (ប ប ទ )

20/. េ ណមួយ នរTUស


2 ់ ំ R និងកមស់ h , េVក"ង
# េ ណេ8ះ នសុី ង

មួយែដល នរTUស
2 ់ ំ តេស:ន
ើ ង
ឹ ។
R
2
ក. ចូរគណ8 ឌៃនសុី ង
ំ េ8ះ ។
ខ. ចូរគណ8ផលេធៀបរOង ឌៃនសុី ង
ំ និង ឌៃនេ ណ ។
21/. គូបមួយ នរTUស
2 ់ ជុងេស:ន
ើ ង
ឹ 2a
េVក"ង
# គូបេ8ះ នសុី ង
ំ មួយ
ែដល នរTUស
2 ់ េំ ស:ន
ើ ង
ឹ a និង
នកមស់េស:ន
ើ ង
ឹ 2a ។
ក. ចូរគណ8 ឌៃនសុី ង
ំ ។
ខ. ចូរគណ8ផលេធៀបរOង ឌ
ៃនសុី ង
ំ និង ឌៃនគូប ។

កំែណេ យ ៃហ ហុន
ិ , ៃហ ចរ និងយ៉ត ពនក 237
េមេរ នទី១៨ ៖ សូលត

22/. គូបមួយ នរTUស


2 ់ ជុង 2 cm េហើយែស-ម
៊ យ
ួ ន ំ r ។ ចូរកំណត់ ំ r
េ/យដឹងN ឌៃនគូបេស:ន
ើ ង
ឹ ពីរដងៃន ឌែស-៊ ។ េគបងvត់ π = 3 ។

23/. ៃផ ក តីេ ណសម័ងx2 ែដល ន ជុង 1cm ។ េតើ តnវ=ប៉8


ុ ន
@2 yគរយៃន
ៃផ ក ៃន េរ៉ែដល នរTUស
2 ់ ជុង1cm ?

ច មយ
1/. រកៃផ ក ង និងៃផ ក ង
ំ អស់ៃនរូប ងេ ម៖

ប (a) ប (b) ប (c)

រូប (a)
ៃផ ក ង គឺ ៖ S L = pa ែដល p = 4 ⋅15 = 60 cm និង a = 9 cm
1
2
េគ ន S L = ⋅ ( 60 cm )( 9 cm ) = 270 cm 2 ។
1
2
ៃផ ក ំ អស់ គឺ ៖ ST = S L + S B ែដល S B = (15 cm ) = 225 cm 2

2

េគ ន ST = 270 cm 2 + 225 cm2 = 495 cm 2 ។

កំែណេ យ ៃហ ហុន
ិ , ៃហ ចរ និងយ៉ត ពនក 238
េមេរ នទី១៨ ៖ សូលត

រូប (b)
ៃផ ក ង គឺ ៖ S L = pa ែដល p = 6 ⋅ 8 = 48 cm និង a = 14 cm
1
2
េគ ន S L = ⋅ ( 48 cm )(14 cm ) = 336 cm 2 ។
1
2
ៃផ ក ង
ំ អស់ គឺ ៖ ST = S L + S B
ែដល S B = 2S△ + S□ = bh + 8b = b ( h + 8)
េហើយ h = 4 cm និង
b
= 82 − h 2 = 4 3 cm
2
េគ ន ST = 336 cm 2 + 8 3 ( 4 + 8 ) cm2 = ( 336 + 96 3 ) cm 2 ។
រូប (c)
ៃផ ក ង គឺ ៖ S L = π Ra ែដល R = 8 cm និង a = 14 cm
េគ ន S L = ⋅ ( 8 cm )(14 cm ) = 352 cm 2 ។
22
7
ៃផ ក ង
ំ អស់ គឺ ៖ ST = π R ( a + R )
េគ ន ST = 3.14 ⋅ (8 cm )( 8 + 14 ) cm = 552.64 cm 2 ។
2/. រក ឌេ ណែដល នកមស់ និង ំ តក"ង
# ករណីនម
ី យ
ួ ៗ ងេ ម៖
fមរូបមន{ ឌរបស់េ ណ ៖ V = π R 2 h
1
3
ក. R = 5 cm , h = 12 cm
េគ ន ៖ V = ⋅ 3.14 ⋅ ( 5 cm ) (12 cm ) = 314 cm3 ។
1 2

3
ខ. R = 3.4 cm , h = 8.9 cm
េគ ន ៖ V = ⋅ 3.14 ⋅ ( 3.4 cm ) (8.9 cm ) = 107.68 cm3 ។
1 2

3
គ. R = 3.7 cm , h = 7 cm
េគ ន ៖ V = ⋅ 3.14 ⋅ ( 3.7 cm ) ( 7 cm ) = 100.3 cm3 ។
1 2

3
ឃ. R = 5.2 cm , h = 11cm
េគ ន ៖ V = ⋅ 3.14 ⋅ ( 5.2 cm ) (11cm ) = 311.32 cm3 ។
1 2

3
ង. R = 5 mm , h = 13.4 mm
េគ ន ៖ V = ⋅ 3.14 ⋅ ( 5 mm ) (13.4 mm ) = 70.13 mm3 ។
1 2

3
ច. R = 7 dm , h = 16 dm
េគ ន ៖ V = ⋅ 3.14 ⋅ ( 7 dm ) (16 dm ) = 820.59 dm3 ។
1 2

កំែណេ យ ៃហ ហុន
ិ , ៃហ ចរ និងយ៉ត ពនក 239
េមេរ នទី១៨ ៖ សូលត

3/. រកៃផ ក ែស-៊ និង ឌប៊ល


ូ ក"ង
# ករណីនម
ី យ
ួ ៗ ងេ ម៖
fមរូបមន{ ៖ ក ៃផែស-៊ S = 4π R 2 និង ឌប៊ល
ូ V = π R3 េគ ន ៖
4
3
ក. R = 3 cm
ក ៃផែស-៊ គឺ ៖ S = 4 ⋅ 3.14 ⋅ ( 3 cm ) = 113.04 cm 2 ។
2

ឌប៊ល
ូ គឺ ៖ V = ⋅ 3.14 ⋅ ( 3 cm ) = 113.04 cm3 ។
4 3

3
ខ. D = 18 dm េ8ះ R =
D
= 9 dm
2
ក ៃផែស-៊ គឺ ៖ S = 4 ⋅ 3.14 ⋅ ( 9 dm ) = 1017.36 dm 2 ។
2

ូ គឺ ៖ V = ⋅ 3.14 ⋅ ( 9 dm ) = 3052.08 dm3 ។


ឌប៊ល
4 3

3
គ. R = 3.7 cm
ក ៃផែស-៊ គឺ ៖ S = 4 ⋅ 3.14 ⋅ ( 3.7 cm ) = 171.95 cm 2 ។
2

ូ គឺ ៖ V = ⋅ 3.14 ⋅ ( 3.7 cm ) = 212.07 cm3 ។


ឌប៊ល
4 3

3
ឃ. D = 23.2 cm េ8ះ R =
D
= 11.6 cm
2
ក ៃផែស-៊ គឺ ៖ S = 4 ⋅ 3.14 ⋅ (11.6 cm ) = 1690.07 cm 2 ។
2

ឌប៊ល
ូ គឺ ៖ V = ⋅ 3.14 ⋅ (11.6 cm ) = 6534.95 cm3 ។
4 3

3
4/. រក ែំ ស-េ៊ /យ01ល
2 ៃ់ ផ ក ដូច ងេ ម៖

ពីរប
ូ មន{ ៃផ ក ែស-៊ S = 4π R 2 េគ ញ ន R =
S

ក. S =
36
π cm 2
25
36
π cm 2
េគ ន ៖ R = = cm = 0.6 cm ។
25 6
4π 10
ខ. S =
46
π m2
16
46
π m2
េគ ន ៖ R = m = 0.85 m ។
16 46 46
= m=
4π 64 8
គ. S =
12
π dm 2
5
12
π dm 2
េគ ន ៖ R= 5 dm = 0.77 dm ។
3
=
4π 5
ឃ. S = 1764π cm 2
1764π cm2
េគ ន ៖ R= = 21 cm ។

កំែណេ យ ៃហ ហុន
ិ , ៃហ ចរ និងយ៉ត ពនក 240
េមេរ នទី១៨ ៖ សូលត

5/. រក ៃំ នប៊ល
ូ េ/យ01ល
2 ់ ឌ៖

ពីរប
ូ មន{ ឌប៊ល
ូ V = π R3 េគ ញ ន R = 3
4 3V 3 0.75V
=
3 4π π
ក. V = 288π cm3

េគ ន ៖ R= 3
(
0.75 288π cm3 ) = 6 cm ។
π
ខ. V =
32
π m3
16
 32 
0.75  π m3 
េគ ន ៖  16  = 3 1.5 m = 1.14 m ។
3
R=
π
6/. ក. រក ឌ ង
ំ អស់ៃនសូលត

fមរូប , ឌ ង
ំ អស់ៃនសូលត
ី គឺ ឌេ ណ បូកនឹង ឌកន5ះែស-៊ ។
ឌេ ណគឺ ៖ V1 = π R 2 h = ⋅ 3.14 ⋅ ( 4.5 cm ) ( 6 cm ) = 127.17 cm3
1 1 2

3 3
៊ ឺ ៖ V2 =  π R 3  = ⋅ 3.14 ⋅ ( 4.5 cm ) = 190.75 cm3
ឌកន5ះែស-គ
1 4 2 3

23  3
ដូចេនះ ឌ ង
ំ អស់ៃនសូលត
ី គឺ ៖ V = V1 + V2 = 317.92 cm3 ។
ខ. រកៃផ ក ង
ំ អស់ៃនសូលត

fមរូប ,ៃផ ក ង
ំ អស់ៃនសូលត
ី គឺ ៃផ ងេ ណ បូកនឹងៃផ ងកន5ះែស-៊
ៃផ ង ក េ ណគឺ ៖
S1 = π Ra = 3.14 ⋅ ( 4.5 cm ) ( 6 cm ) + ( 4.5 cm ) = 106 cm2
2 2

ៃផ ក ងកន5ះែស-គ
៊ ឺ ៖ S2 =
1
2
( )
4π R 2 = 2 ⋅ 3.14 ⋅ ( 4.5 cm ) = 127.17 cm 2
2

ដូចេនះ ឌ ង
ំ អស់ៃនសូលត
ី គឺ ៖ S = S1 + S2 = 233.17 cm2 ។
7/. រកៃផ ក ∆AEB

ក"ង
# តីេ ណ AEB និង CED ន ៖
∠BAE = ∠CDE (មុ|
ំ ស
R2 ក
់ ង
"# )
∠AEB = ∠CED (មុទ
ំ ល់កព
ំ ល
ូ )
8ំឲ42 ∆ABE ∼ ∆DCE ករណីទ១
ី ម.ម
2 2

េគ នផលេធៀប ៖ ∆ABE =   =   = េ/យ S∆CED = 16 cm 2


S BE 1 1
S∆DCE  CE   2  4

8ំឲ42 េយើង ន ៖ S∆AEB = ⋅ S∆DEC = ⋅ (16 cm2 ) = 4 cm2


1 1
4 4
ដូចេនះៃផ ក ៃន តីេ ណ AEB គឺ 4 cm 2 ។

កំែណេ យ ៃហ ហុន
ិ , ៃហ ចរ និងយ៉ត ពនក 241
េមេរ នទី១៨ ៖ សូលត

8/. េតើៃផ ក េរ៉ DEFG តូច=ងៃផ ក ABCD ប៉8


ុ ន
@2 ដង ?

េ/យ េរ៉ DEFG និង េរ៉ ABCD =រូបធរណី តដូចHI2 , េគ ន ៖


2
S DEFG  DG 
=  (១)
S ABCD  DC 

េយើង ន MG / / NC fម ទឹសប
}ី ទfែលស , េគ ន ៖
DG DM 9
= =
DC DN 15
2

េ8ះ (១) ៖ DEFG =   =


S 3 9
S ABCD  5  25

ដូចេនះៃផ ក េរ៉ DEFG តូច=ងៃផ ក ABCD ចំនន


ួ ដង ។
25
9
9/. េតើៃផ ក ∆BIO តូច=ងៃផ ក ∆DOC ប៉8
ុ ន
@2 ដង ?

េគ ន , តីេ ណ ∆BIO និង ∆DOC ន៖


∠IOB = ∠COD (មុទ
ំ ល់កព
ំ ល
ូ )
និង ∠ODC = ∠OBI (មុ|
ំ ស
R2 ក
់ ង
"# )
េ8ះ ∆IOB ∼ ∆COD fមករណីដណ
ំ ច
~ ទី១ (ម.ម.)
2
S∆BIO  BI 
េគ ន ៖ =  (១)
S∆DCO  CD 

េ/យ AB = DC េហើយ I =ចំណច


D កEFល
2 ៃន AB េ8ះ IA = IB =
AB CD
=
2 2
2
 CD 
 
េគ ន (១) ៖
S ∆BIO 1
= 2  =
S ∆DCO  CD  4
 
ដូចេនះៃផ ក ∆BIO តូច=ងៃផ ក ∆DOC ចំនន
ួ ៤ដង ។

កំែណេ យ ៃហ ហុន
ិ , ៃហ ចរ និងយ៉ត ពនក 242
េមេរ នទី១៨ ៖ សូលត

10/. គណ8ៃផ
គណ8ៃផ ក ពិត កដរបស់
កដរបស់ជLMង
2ំ
េ/យរូ
េ/យរូបថតៃនជLMង
2ំ និងជLMង
2ំ =រូបធរណី តពីរដូចHI2 , េគ ន ៖
2
S h
= 
S '  h'
េ/យ h = 3 cm = 0.03 m , h ' = 6m , S = 9 cm 2 = 9 ×10−4 m2 េគ ន ៖
9 × 10−4  0.03 
2

= 
S'  6 
9 × 10−4
S'= = 36 m 2
25 × 10−6
ដូចេនះៃផ ក ពិត កដរបស់ជLMង
2ំ េ8ះគឺ 36 m2 ។
11/. រកវ[ តេ8ះ េVក"ង
# ែផនទី ទី២
េ/យែផនទី ង
ំ ពីរតំEងឲ24ទីfង
ំ ែតមួយ េ8ះែផនទី ង
ំ ពីរ=រូប
ធរណី តដូចHI2 ។
fមបំbប់ , ៃផ ក ៃនទីQ0
R2 Sមួយ នរTUស
2 ់ 144 cm2 េVក"ងែផនទី
#ងែផនទី
ទី១ េហើយ នរTU
នរTUស
2 េ់ ស:ន
ើ ង
ឹ 256 cm 2 េVក"ង
# ែផនទី ទី២ , េហើយេគដឹ
េគដឹងN
វ[ តមួយៃនទីQេ8ះេស:
R2េ8ះេស:ន
ើ ងឹ 18 cm េVក"ង
# ែផនទី ទី១ , េគ ន ៖
2
256  a 
= 
144  18 
a 16
=
18 12
16 ⋅18
a= = 24 cm
12
ដូចេនះ វ[ តេ8ះ នរTUស់
2ស់ 24 cm េVក"ង
# ែផនទីទ២
ី ។
12/. រកៃផ
រកៃផ ក ៃនទីQេVក"
R2េVក"ង
# ប5ងម
់ យ
ួ ែដល ន ត/\ន
2
1
2 500
រូបyពទីQេR2 Vក"ង
# ប5ង់ និងកែន5ងពិត =រូបធរណី តពីរដូចHI2 ។
ត/\ន
2 នន័យN1 m េVេលើបង
5 ់ តnវ= 2500 m េVេលើទី
ទf ី ង
ំ ពិត
1
2 500
េគ ន ៖
2
S  1 
= 
9 600  2500 
9600
S= m2
625 × 10 4
S = 15.36 ×10 −4 m 2
S = 15.36 cm 2
ដូចេនះៃផ
េនះៃផ ក ៃនទីQេVក"
R2េVក"ង ់េ8ះគឺ 15.36 cm2 ។
# ប5ងេ8ះគឺ

កំែណេ យ ៃហ ហុន
ិ , ៃហ ចរ និងយ៉ត ពនក 243
េមេរ នទី១៨ ៖ សូលត

13/. ក. គណ8 x

េ/យេគ
េ/យេគ ត់េ ណឲ24 សបនឹង ត=បីចែំ ណក , មុខ ត់ នអង_តផ
់ ត
`ិ
េរៀងHI2 6 cm និង 10 cm , េ8ះេគទទួល នេ ណបី(េ ណតូច , មធ24ម
និងេ ណធំ) ែដលេ ណ ង
ំ ៣ =រូបធរណី តដូចHI2 ។
េគ ន ៖
6 x
=
10 x + 4
3( x + 4) = 5x
2 x = 12
x = 6 cm
ដូចេនះ x = 6 cm តnវ នកំណត់ ។
ខ. រក ឌៃនេ ណេ8ះ
េ/យេ ណ ង
ំ បី=រូបធរណី តដូចHI2 , េគ ន ៖
3
v  h
=   ែដល v = π r 2 h
1
V H  3
េហើយ r = = 3 cm , h = x = 6 cm , H = x + 4 + 2 = 12 cm , េគ ន៖
6
2
1
v = ⋅ 3.14 ⋅ ( 3 cm ) ⋅ ( 6 cm ) = 56.52 cm3
2

3
3 3
 12 cm 
និង V = v   = 56.52 cm3 
( )
H
 = 452.16 cm
3

 h  6 cm 
ដូចេនះ ឌៃនេ ណេ8ះគឺ 452.16 cm3 តnវ នកំណត់ ។
14/. ក. រក ជុងៃន េរe EFGH fមតៃម5 x
េ/យ េគ ត់ពb
ី ម
c2 ត
ី េនះឲ24 សបនឹង ត ABCD េគ ន េរe EFGH
េ8ះេគ នពីbម
c2 ត
ី មួយតូច មួយធំ េហើយ=រូបធរណី តពីដច
ូ HI2 ។

កំែណេ យ ៃហ ហុន
ិ , ៃហ ចរ និងយ៉ត ពនក 244
េមេរ នទី១៨ ៖ សូលត

2
S ABCD  SO 
េគ ន ផលេធៀប ៖ = 
S EFGH  SO ' 
េ/យ S ABCD = ( 60 cm ) = 3600 cm 2 , SO = 80 cm , SO ' = 80 − x េគ ន ៖
2

2
3600  80 
= 
S EFGH  80 − x 
3600 ( 80 − x )
2

S EFGH =
80 2
60 ( 80 − x )  3x 
េ8ះ ជុងៃន េរe EFGH គឺ A ' B ' = S A' B 'C ' D ' = =  60 −  cm
80  4 

ដូចេនះ េរe EFGH 2 ់ ជុង  60 −  cm ។


នរTUស
3x
 4 
ខ. គណ8 ឌៃនកំEត់ពb
ី ម
c2 ត
ី ខណgេ/យ ABCD និង EFGH
នកំEត់ពb
ី ៉មត
ី ABCD − EFGH គឺ ៖ V = VSABCD − VSEFGH
េ/យ VSABCD = ⋅ S ABCD ⋅ SO និង VSEFGH = ⋅ S EFGH ⋅ SO ' ,េគ ន ៖
1 1
3 3
1 1
V = ⋅ S ABCD ⋅ SO − ⋅ S EFGH ⋅ SO '
3 3
1 2 
2
 3x 
V =  60 ⋅ 80 −  60 −  ( 80 − x ) 
3   4  

េ/យ x = 30 cm េ8ះេយើង ន ៖
1 3 ⋅ 30  
2

V =  288000 −  60 −  ( 80 − 30 ) 
3   4  

1
V= ( 288000 − 70312.5 )
3
V = 72562.5 cm3
ដូចេនះ ឌៃនកំ
ឌៃនកំEត់ពb
ី ម ី ខណgេ/យ ABCD និង EFGH គឺ
c2 ត
72562.5 cm3 តnវ នគណ8 ។

កំែណេ យ ៃហ ហុន
ិ , ៃហ ចរ និងយ៉ត ពនក 245
េមេរ នទី១៨ ៖ សូលត

15/. ក. គណ8 A ' B '

េយើង ន ៖ A ' B '/ / AB េ8ះfម ទឹសប


}ី ទfែលស េគ ន ៖
េ/យ BA = 6 cm , SA = 8 cm , SA ' = SA
A ' B ' SA ' 2
=
AB SA 3
2
SA
A' B ' 3
⇒ =
6 SA '
A ' B ' = 4 cm
ដូចេនះ A ' B ' = 4 cm តnវ នគណ8 ។
ខ. គណ8ៃផ ក ៃន េរe A ' B ' C ' D '
ៃផ ក ៃន េរe A ' B ' C ' D ' គឺ ៖ S A ' B 'C ' D ' = ( A ' B ') = ( 4 cm ) = 16 cm 2
2 2

ដូចេនះៃផ ក ៃន េរe A ' B ' C ' D ' គឺ 16 cm2 ។


2
2
រួចេផhងi9ត
2 N
់ ៃផ ក A ' B ' C ' D ' =   × ៃផ ក ABCD
3
េយើង ន ៖ េរe ABCD នៃផ ក S ABCD = AB 2 = ( 6 cm ) = 36 cm 2
2

និង េរe A ' B ' C ' D ' នៃផ ក S A ' B 'C ' D ' = 16 cm 2
2
36 9  3 
េគ ន ៖
S ABCD
= = = 
S A ' B ' C ' D ' 16 4  2 
2
2
⇒ S ABCD =   × S A ' B 'C ' D '
3
2
2
ដូចេនះ ៃផ ក A ' B ' C ' D ' =   × ៃផ ក ABCD ។
3

កំែណេ យ ៃហ ហុន
ិ , ៃហ ចរ និងយ៉ត ពនក 246
េមេរ នទី១៨ ៖ សូលត

គ. បTjញ
2 N SO ' = SO
2
3
េ/យ េរe A ' B ' C ' D ' ត់ពb
ី ម
c2 ត
ី SABCD សបនឹង ត ABCD េ8ះ
េគ នពីbម
c2 ត
ី ពីរ ដូចHI2 គឺពb
ី ម
c2 ត
ី តូច SA ' B ' C ' D ' និងពីbម
c2 ត
ី ធំ SABCD ។
2

នផលេធៀប ៖ A' B 'C ' D ' = 


SO ' 
េគ (១)
S

S ABCD  SO 
2

ក៏បែ៉ុ ន{ , fមស យ ងេលើ A' B 'C ' D ' =   (២)


S 2
S ABCD 3
fម (១) និង (២) េគ ន ៖
SO ' 2 2
= ⇒ SO ' = SO
SO 3 3
ដូចេនះ SO ' = SO តnវ នបTjញ
2 ។
2
3
2

រួច ញបLMក
2 N
់ ឌ SA ' B ' C ' D ' =   × ឌ SABCD
2
3
េ/យពីbម
c2 ត
ី SABCD និង SA ' B ' C ' D ' =រូបធរណី តពីរដូចHI2
3 3 3

ន ៖ SA' B 'C ' D ' = 


SO '   2  2
េគ  =   8ំឲ42 VSA ' B 'C ' D ' =   × VSABCD
V
VSABCD  SO   3  3
2

ដូេច"ះ ឌ SA ' B ' C ' D ' =   × ឌ SABCD ។


2
3
16/. ក. គណ8 O ' M '

េ/យNសផ`ត
ិ O' ត់េ ណ=ពីរ េហើយNសផ`ត
ិ O ' សប=មួ
សប=មួយNស
ផ`ត
ិ O េ8ះេគ ន េ ណពីរដូចHI2 ។
េគ ន O ' M '/ /OM , េ/យអនុវត{ន៍ ទឹសប
}ី ទfែលស េគ ន ៖
ែត SM ' = SM , OM = 6 cm េគ ន៖
SM ' O ' M ' 1
=
SM OM 2

កំែណេ យ ៃហ ហុន
ិ , ៃហ ចរ និងយ៉ត ពនក 247
េមេរ នទី១៨ ៖ សូលត

1
SM
2 O'M '
=
SM 6
1 O'M '
=
2 6
O ' M ' = 3 cm
ដូចេនះ O ' M ' = 3 cm តnវ នគណ8 ។
2

រួចេផhងi9ត
2 N
់ ៃផ ក Nស ( O ') =   × ៃផ ក Nស ( O )
1
2
ៃផ ក Nស O ' គឺ ៖ S ' = π ⋅ ( O ' M ')
2

ៃផ ក Nស O គឺ ៖ S = π ⋅ OM 2
េគ ន ៖
S ' π ⋅ ( O ' M ')
2

=
S π ⋅ OM 2
2
S '  O'M ' 
=
S  OM 
2
S' 1
= 
S 2
2
1
S'=  ×S
2
2

ដូេច"ះ ៃផ ក Nស ( O ') =   × ៃផ ក Nស ( O ) ។
1
2
ខ. បTjញ 2 N SO ' = SO
1
2
េយើង ន O ' M '/ /OM , fម ទឹសប
}ី ទfែលស េគ ន ៖
SO ' SM '
=
SO SM
1
SM
SO ' 2
=
SO SM
8ំឲ42 SO ' = SO
1
2
ដូចេនះ SO ' = SO តnវ នបTjញ 2 ។
1
2
គ. 0យបLMក
2 N
់ ឌៃនេ ណៃនកំពល
ូ S និង ត O ' េស:ន
ើ ង
ឹ ឌ
3

ៃនេ ណៃនកំពល
ូ S និង ត O គុណនឹង  
1
2
េ/យេ ណ ង
ំ ពីរ=រូបធរណី តដូចHI2 , េគ ន ៖
3 3
V '  SO '  1
=  8ំឲ42 V ' =   × V
V  SO  2

កំែណេ យ ៃហ ហុន
ិ , ៃហ ចរ និងយ៉ត ពនក 248
េមេរ នទី១៨ ៖ សូលត

ដូេច"ះ ឌៃនេ ណៃនកំពល


ូ S និង ត O ' េស:ន
ើ ង
ឹ ឌៃនេ ណៃន
3

កំពល
ូ S និង ត O គុណនឹង   ។
1
2
17/. េតើអក
" ទិញ តnវេ ជើសេរoសមួយEេដើមp2 ឲ
ី 42 នចំេណញ=ង ?
fង V = ឌឪឡឹកធំ និង v = ឌឪឡឹកតូច
េហើយ R = ឪ
ំ ឡឹកធំ និង r = ឪ
ំ ឡឹកតូច ែដល r = R (ស.ក.)
1
4
េ/យឪឡឹក ង
ំ េ8ះ នbង=ែស-ដ
៊ ច
ូ HI2 , េ8ះឪឡឹក ង
ំ ពីរ បេភទ
=រូបធរណី តដូចHI2 ។ េគ ន ៖
3
 
V 1R 1 R 
3

=   =  
2v 2  r  2  1 R 
4 
V
= 32 > 1
2v

⇒ V > 2v
ដូចេនះអ"កទិញ តnវេ ជើសេរo
សេរoសយកឪឡឹកធំេដើមp2 ច
ី េំ នញ=ង ។
18/. េតើេគ តnវេ ជើសេរoសយក ស
s2 ន
ុី បេភទEេទើបចំេណញ=ង ?
fង V1 , C1 = ឌ និងតៃម5ៃន ស
s2 ន
ុី ទី១
េហើយ V2 , C2 = ឌ និងតៃម5ៃន ស
s2 ន
ុី ទី២
3

fមបំbប់ , េគ ន ៖ 1 =   = 0.729 និង 1 = = 0.7


V 9 C 7
V2  10  C2 10

េ/យ 1 > 1
V C
V2 C2
ដូចេនះេគ តnវេ ជើសេរoសយក ស
s2 ន
ុី បេភទទី១ េទើបចំេនញ=ង ។
19/. ក. គណ8 ឌេ ណ និង ឌៃនសុី ង

កំែណេ យ ៃហ ហុន
ិ , ៃហ ចរ និងយ៉ត ពនក 249
េមេរ នទី១៨ ៖ សូលត

fមបំbប់ , សុី ង
ំ មួយ នរTUស
2 ់ ំ R និងកមស់ h
េគយកេ ណែដល នរTUស
2 ់ ំ R និងកមស់ h /ក់កង
"# សុី ង
ំ ។
េគ ន ៖
ឌេ ណ គឺ ៖ V = ⋅ S ⋅ h = π R 2 h
1 1
3 3
ឌសុី ង
ំ គឺ ៖ V ' = S ⋅ h = π R 2 h ។
ខ. គណ8ផលេធៀបរOង ឌៃនេ ណ និង ឌៃនសុី ង

េ/យ V = ⋅ S ⋅ h = π R 2 h និង V ' = S ⋅ h = π R 2 h េគ នផលេធៀប ៖
1 1
3 3
1
π R2h
V 1
=3 2 =
V ' πR h 3
ដូចេនះផលេធៀបរOង
េនះផលេធៀបរOង ឌៃនេ ណ និង ឌៃនសុី ង
ំ គឺេស:ន
ើ ង
ឹ 3។
20/. ក. គណ8 ឌៃនសុី ង
ំ េ8ះ

fមបំbប់ , េ ណមួយ នរTUស


2 ់ ំ R និងកមស់ h ,េVក"ង
# េ ណេ8ះ
នសុ
នសុី ង
ំ មួយែដល នរTUស
2 ់ ំ តេស:ន
ើ ង
ឹ , េគ ន ៖
R
2
R

2

ឌៃនសុី ំ គឺ ៖ V = π   h ' ែដល


ង = 2 ⇒ h ' = h (fែលស)
R h h ' 1
2 h R 2
 R h πR h
2 2
ដូចេនះ ឌៃនសុី ង
ំ គឺ V = π     = ។
 2  2 8
ខ. គណ8ផលេធៀបរOង ឌៃនសុី ង
ំ និង ឌៃនេ ណ
នៃនេ ណ ងេលើ គឺ ៖ V ' = π R 2 h
1
3
π R2h
េគ នផលេធៀប ៖
V 8 3
= =
V ' 1 π R2h 8
3

កំែណេ យ ៃហ ហុន
ិ , ៃហ ចរ និងយ៉ត ពនក 250
េមេរ នទី១៨ ៖ សូលត

ដូចេនះ ផលេធៀបរOង ឌៃនសុី ង


ំ និង ឌៃនេ ណ គឺេស:ន
ើ ង
ឹ ។
3
8
21/. ក. គណ8 ឌៃនសុី ង

fមបំbប់ , សុី ង
ំ េ8ះ នរTUស
2 ់ េំ ស:ន
ើ ង
ឹ a និង នកមស់េស:ន
ើ ង
ឹ 2a
េគ ន , ឌៃនសុី ង
ំ គឺ ៖ V = π R 2 h = π a 2 ( 2a ) = 2π a 3
ដូចេនះ ឌៃនសុី ង
ំ េ8ះគឺ 2π a 3 តnវ នកំណត់ ។
ខ. គណ8ផលេធៀបរOង ឌៃនសុី ង
ំ និង ឌៃនគូប
គូបេ8ះ ន ជុង 2a េគ ន , ឌរបស់Oគឺ ៖ V ' = ( 2a ) = 8a 3
3

V 2π a 3 π
េយើង នផលេធៀប ៖ = =
V ' 8a 3 4
π
ដូចេនះ ផលេធៀបរOង ឌៃនសុី ង
ំ និង ឌៃនគូប គឺ ។
4
22/. កំណត់ ំ r

េ/យគូ
េ/យគូប នរTUស
2 ់ ជុង 2 cm េ8ះ ឌគូប គឺ ៖ V = ( 2 cm ) = 8 cm3
3

េហើយែស-ម
៊ យ
ួ ន ំ r េ8ះ ឌែស-គ
៊ ឺ ៖ V ' = π r 3 = 4r 3 េ ƒះ π = 3
4
3
េ/យដឹងN ឌៃនគូបេស:ន
ើ ង
ឹ ពីរដងៃន ឌែស-៊ , េគ ន ៖
V = 2V '
8 = 2 ( 4r )
r = 1 cm
ដូចេនះ r = 1cm តnវ នគណ8
នគណ8 ។

កំែណេ យ ៃហ ហុន
ិ , ៃហ ចរ និងយ៉ត ពនក 251
េមេរ នទី១៨ ៖ សូលត

23/. េតើ តnវ=ប៉8


ុ ន
@2 yគរយៃន េរ៉ ?

រូបេនះ = តីេ ណសម័ងx2 ែដល នរTUស


2 ់ ជុង 1cm េគ ន a = b = c = 1cm
ក ៃផរបស់O គឺ S = ah ែតfម ទឹសប
}ី ទពីfគ័រ េគ ន ៖
1
2
2
a
b = h + 
2 2

2
2
1
h =1 − 
2 2

2
1
h = 1−
4
3
h= cm
2
េគ ន , ៃផ ក តីេ ណសម័ងx2 ែដល ន ជុង 1cm គឺ ៖
1  3  3
S= (1cm )  cm  = cm2
2  2  4
ៃផ ក េរeែដល
ែដល នរTUស
2 ់ ជុង 1cm គឺ ៖ S ' = (1cm ) = 1cm 2
2

3 2
cm
េគ នផលេធៀប ៖ =
S 4 = 0.4330 = 43.30%
S' 1 cm 2
ដូចេនះៃផ ក តីេ ណសម័ងx2 តnវ= 43.30% ៃនៃផ ក េរeែដល ន
ជុង បែវង 1cm ដូចHI
ចHI2 ។

ចប់
កំែណេ យ ៃហ ហុន
ិ , ៃហ ចរ និងយ៉ត ពនក 252
ក ង
[1]. ក ង ប យ ជន នងក ,គ ទ ទ ( !
") $%ពម()* +,-- .យ គ%/ 0 ន $%ពម( នង
1ចកផ4យ 5
[2]. ក ង ប យ ជន នងក ,គ ទ ទ ( !
" នង គ6) $%ពម()* +,,7
+,,7 .យ គ%/ 0 ន $%
ពម( នង1ចកផ4យ 5
[3]. ទ 0 នពទ8 នប 9 :, ច;ន កម1ខ= ម>ច
ង?@ជ ជន A ក1 +.-
+.- )* +,--
+,-- 5
[4]. ក 1CD ក4Eមយ Fក គ6 HG A នង
Fក គ6 I JI ភក> ( L N
M :គ
:គ ទ )5
[5]. នង
/ ក EO 1ខ= នងប ទ មយចនន ទ 5
ក ម គ ក ប នម មយ “ពន គ ”

ន ក នង យ ប ខ , , ផ (! ទ $ ព& ' (' ) )

You might also like